Physics Class 12
Physics Class 12
INDEX
TOPICS Page. No.
01. ELECTRIC CHARGES AND FIELDS 1-28
02. ELECTRIC POTENTIAL & CAPACITANCE 30-81
03. CURRENT ELECTRICITY 83-110
04. SEMICONDUCTOR ELECTRONICS 112-140
05. COMMUNICATION SYSTEM 142-155
06. MOVING CHARGES AND MAGNETISM 157-189
07. MAGNETISM AND MATTER 191-219
08. ELECTROMAGNETIC INDUCTION 221-259
09. ALTERNATING CURRENT 261-292
10. DUAL NATURE OF RADIATION AND MATTER 294-305
11. ATOMS 307-328
12. NUCLEI 330-355
13. RAY OPTICS 356-451
14. WAVE OPTICS 453-482
15. ELECTRO MAGNETIC WAVES 484-496
ELECTRIC CHARGES AND FIELDS
Page 1
ELECTRIC CHARGES AND FIELDS
charged body to neutral body(actually, electrons potential becomes zero, charge may or may
flow from low potential body to high potential not be zero.
body) till electric potential of both bodies are
If an isolated body is connected with earth then
equal. So in this case, the neutral body becomes
its potential and charge both are zero, in fact to
charged and the nature of charge developed in
this body is identical to that on initial charged make the potential zero, excess charge from the
body. body is removed to earth.
Page 3
ELECTRIC CHARGES AND FIELDS
The force of attraction between q 1 and q2
provides necessary centripetal force.
l
Fe T T
q2 q1
Fe Fe
1 q1q 2 mg mg
Hence Fe = Fc = 4 m2r
0 r2 1 q2
Fe
4 0 2l sin 2 ...... (1)
q1 q 2 2
and Time period T
4 0 mr 3 T sin Fe ...... (2)
0 mr T cos mg ...... (3)
T 4r
q1 q 2 T
Illustration 2 :
Two point charges Q and q are placed at Fe
r
distance r and respectively along a
2 mg
straight line from a third charge 4q. If q is
Q
Eliminating T and Fe, we get
in equilibrium, determine q . q2
Tan
Solution : mg 4 0l 2 sin 2
Net force on q = 0
q 2 (4mgl 2 sin2 tan ) 4 0
F1 F2 0
Illustration 4 :
r/2 r/2
Two identical balls each having density
Q F2 q F1 4q are suspended from a common point by two
insulating strings of equal length Both the
1 Qq 1 4q.q balls have equal mass and charge. In
0
4 0 (r /2)2 4 0 (r /2)2 equilibrium, each string makes an angle
with the vertical. Now both the ball are
Q immersed in a liquid. As a result, the angle
4
q does not change. The density of the liquid
Illustration 3 : is . Find the dielectric constant of the
2 Small spheres each of mass m and carrying liquid.
+q coulomb, are suspended by massless Solution :
insulating threads, each of length l. Prove
that q 2 (4mgl2 sin2 tan )40 , where l l
is angle made of strings with the vertical at
equilibrium. T T
T cos
Solution :
When the system is in equilibrium, each string F T sin F
x
makes an angle with the vertical.
W air W
Page 4
ELECTRIC CHARGES AND FIELDS
W Liquid W Illustration 6 :
Let v is the volume of each ball, then mass Atomic number of copper is 29, its atomic
of each ball is m v ; When balls are in air weight is 63.5 gm/mole. If two pieces of
T cos mg ; T sin F copper having weight 10 gm are taken and
from one of the pieces 1 electron per 1000
F mg tan vg tan ---------(1) atom is transferred to the other piece and
When balls are suspended in liquid. The there after these pieces are placed 10 cm
1 F apart, then what will be the coulomb force
coulombic force is reduced to F and
K between them?
apparent weight = weight - up thrust ;
Solution:
W 1 vg vg In 1 mole copper (63.5 gm) there are 6 × 1023
According to the problem, angle is atoms (NA = Avagadro number = 6 × 1023
uncharged-Therefore atoms)
F 1 W 1 tan vg vg tan ---(2)
6 1023 10
F Number of atoms = =9.45 × 1022
K 63.5
From (1) and (2) ;
F1 For every 1000 atoms, 1 electron is transferred
Illustration 5 : therefore total number of transferred electron is
A ring of radius R is with a uniformly
distributed charge Q on it. A charge q is now 9.45 1022
9.45 1019
placed at the centre of the ring. Find the 1000
increment in tension in the ring Therefore charge on one piece is 9.45 × 1019
Solution: e and on the other will be (9.45 × 1019e)
Consider an element of the ring. Its enlarged view
is as shown. For equilibrium of this segment, we Force when they are kept 10 cm apart
can write. 1 q1q 2 1 (9.45 1019 )2 e2
F F
4 0 r2 4 0 (10 102 )2
Q 9 109 (9.45 1019 )2 (1.6 1019 )2
d
= (10 102 )2
q d/2 d/2
T T
d
= 2 × 1014 N
Illustration 7 :
q
Consider four equal charges (q each) placed
d
F 2T sin on the corners of a square with side a.
2 Determine the magnitude and direction of
Here F is the repulsive force between q and
the resultant force on the charge on lower
elemental charge dQ
right corner.
Q Solution:
dQ 2 R Rd
The forces on the charge on lower right corner
The electric outward force on element is due to charges 1, 2, 3 are F1 = kq2/a2, F2 = kq2/
1 qdQ a2, F3 = kq2/2a2 The resultant of F1 and F2 is
F
4 0 R 2
Page 5
ELECTRIC CHARGES AND FIELDS
E E Eq
v y at t
+ m
F Eq F Eq
2 2 2E 2 q 2t 2
The body travels in a straight line path with v v vx v y u
m2
F Eq
uniform acceleration a , initial Two charges +Q each are separated by a distance
m m 'd'. The intensity of electric field at the midpoint
velocity u 0 . of the line joining the charges is zero.
At an instant of time t. Illustration10 :
Two charges +Q each are placed at the two
Eq vertices of an equilateral triangle of side a.
Its final velocity, v u at t The intensity of electric field at the third
m vertex is
1 2 1 Eq 2 Solution:
E E
Displacement s ut at t
2 2 m
Momentum, P mv Eq t a a
Kinetic energy,
+Q a +Q
1 2 1 E 2q 2 2
K .E mv t 1 2 2
E E E 2 E E cos
2 2 m 2
2 E 2 2 E 2 cos 2 E 1 cos
When a charged particle enters perpendicularly
into a uniform electric field of intensity E with a
= 2 E cos ; Here 600
velocity ‘v’ then it describes parabolic path as 2
shown in figure. 1 Q
E= 3
4 0 a 2
+ + + + + +
Illustration11 :
u
+ Two charges +Q, -Q are placed at the two
q vertices of an equilateral triangle of side ‘a’,
y then the intensity of electric field at the third
vertex is
Solution:
x
Along the horizontal direction, there is no E1 = 2E cos = E ( 1200 )
acceleration and hence x ut . 2
E
Along the vertical direction, acceleration
120°
F Eq
a (here gravitational force is not a
m m E
considered) a
1 Eq 2 +Q a –Q
Hence vertical displacement, y t
1 Q
2 m
E1 = 4 a 2
2 0
1 qE x qE 2
y x Oblique projection of charged particle in an
2 m u 2mu 2 uniform elctric field (Neglecting
At any instant of time t, horizontal component of gravitational force) : Consider a uniform
velocity, vx u electric field E in space along Y-axis. A negative
charged particle of mass ‘m’ and charge ‘q’ be
vertical component of velocity projected in the XY plane from a point ‘O’ with
Page 8
ELECTRIC CHARGES AND FIELDS
θ u 2 sin 2
X c. Range
o g
EQ
Initial velocity o f the particle is m
TIME PERIOD OF OSCILLATION OF A CHARGED BODY
u u cos iˆ u sin ˆj
The bob of a simple pendulum is given a +ve
Force acting on the particle is F qE (along- charge and it is made to oscillate in a vertically
ve Y axis) upward electric field, then the time period of
qE ˆ oscillation is
a j
m
Velocity of the particle after time ‘t’ is
T E
v u at ; v u cos iˆ u sin at ˆj
If the point of projection is taken as origin, its T 2
l
position vector after time ‘t’ is g
EQ
m
r xiˆ yjˆ where x=(ucos ) t
1 mg
y u sin t at 2
2 In the above case, if the bob is given a -ve charge
If the charged particle is projected along the x- then the time period is given by
axis, then 00
Eq ˆ T
v uiˆ tj E
m
1 Eq 2 l
T 2
Here x ut and y t EQ
2 m g
m
Direction of motion of particle after time ‘t’
makes an angle with x-axis, where mg
Eqt A sphere is given a charge of 'Q' and is
tan
mu suspended in a horizontal electric field. The
A charged particle of charge Q is projected angle made by the string with the vertical is,
with an initial velocity u in a vertically upward
EQ
electric field making an angle to the horizontal. tan 1
Then mg
If gravitational force is considered The tension in the string is EQ 2 mg 2
Net force = mg F mg Eq
Hence effective acceleration
Eq
Net acceleration = g 2
m F Eq
The negative sign is used when electric field is in geff g2
m m
upward direction where as positive sign is used
when electric field is in downward direction for Time period of oscillation is given by
positively charged projected particle.
Page 9
ELECTRIC CHARGES AND FIELDS
wall producing an acceleration The net electric field at centre will be along angle
bisector which can be calculated by integrating
F qE 1
a L at 2 dEcos within limits from / 2 to / 2
m m 2 Hence net electric field strength at centre C is
/2
2L 2mL Q
i.e., t Ec dE cos cos d
a qE 4 0 R 2
/ 2
As collision with the wall is perfectly elastic, the /2
Q
block will rebound with same speed and as now cos d
is motion is opposite to the acceleration,, it will 4 0 R 2 / 2
+ R
+
+
E Consider a rod of length L, uniformly charged
d
+
+
Page 11
ELECTRIC CHARGES AND FIELDS
rL L
Q 1 Qr 2
dx
Ep
4 0 L x r Ep
4 0 L 3/ 2
L
2
r 2
x2
Q 1 1 Q
Ep x
4 0 L r r L 4 0 r r L From the diagram tan
r
At an equatorial point : To find the electric x r tan
field due to a rod at a point P situated at a distance On differentiation; dx r sec 2 d
‘r’ from its centre on its equatorial line
Qr r sec2 d Q sec 2 d
4 0 L r 3 sec3 4 0 Lr sec3
Ep ;
P
Q Q
cos d sin
r 4 0 Lr 4 0 Lr
+ + + + + + + + 1 x 1 x
Substituting tan r sin
x2 r 2
L L
Q x 2
(a) EP
4 0 Lr x 2 r 2 L ;
2
dE dE cos
P Q 1
dE sin r
4 0 r L 2
r2
+++ + + + dx + 4
x
Q 2
Ep 2
4 0 r L 4r 2
(b) ELECTRIC FIELD DUE TO A
Consider an element of length dx at a distance UNIFORMLY CHARGED RING :
‘x’ from centre of rod as in figure (b). Charge on
THE INTENSITY OF ELECTRIC FIELD AT A DISTANCE ‘X’
Q
the element is dq dx . METERS FROM THE CENTRE ALONG THE AXIS:
L
The strength of electric field at P due to this point Consider a circular ring of radius ‘a’ having a
charge ‘q’ uniformly distributed over it as shown
charge dq is dE.
in figure. Let ‘O’ be the centre of the ring .
dq A
dE dx
4 0 r x
2 2
Page 12
ELECTRIC CHARGES AND FIELDS
1 dq dy
dE1 2
4 0 r P
The direction of dE1 is as shown in figure. The x dE
component of intensity along x-axis will be
1 dq
cos dE1 cos
4 0 r 2 Electric field strength due to a uniformly charged
The component of intensity along y-axis will disc at a distance x from its surface is given as
1 dq x
be 4 r 2 sin dE1 sin E 1 2
0 2 0 x R2
Similarly if we consider an element dx of the ring
opposite to A which lies at B, the component of
If x 0 we get E
intensity will be equal and opposite to the 2 0
component of intensity perpendicular to the axis ELECTRIC FIELD LINES
due to element at A. Hence they cancel each
It is a path such that the tangent to this path at
other. Due to symmetry of ring the component
of intensity due to all elements of the ring any point gives the direction of electric field
perpendicular to the axis will cancel. intensity at that point. Line of force start from a
So the resultant intensity is only along the axis of positive charge and end on a negative charge. It
the ring. The resultant intensity is given by can be straight or curved. They do not intersect
1 dq each other. At a point where magnitude of E is
E cos more, lines of force are more crowded. Lines of
4 0 r 2
1 qdx x force do not exist inside a conductor, because E
E
4 0 2 ar 2 r (where cos x / r )
= 0 inside a conductor.
Line of force is not the path travelled by test
1 qx 1 charge in Electric field.
E 3
dx
4 0 2 a
a x
2 2 2
E = constant
2 3/ 2
r 3 a 2 x
E = constant E = variable
1 qx 1
E 2 a
4 0 2 a a x 2 3/2
2 Number of lines of force originating from a
given charge is proportional to the magnitude
1 qx
E of charge
4 0 a x 2 3/ 2
2
At its centre x = 0
Electric field at centre is zero.
By symmetry we can say that electric field +q -q
strength at centre due to every small segment on
ring is cancelled by the electric field at centre
due to the element exactly opposite to it. As in
the figure the electric field at centre due to segment
A is cancelled by that due to segment B. Thus Electric lines of force should be to
net electric field strength at the centre of a equipotential surface.
uniformly charged ring is Ecentre 0 .
ELECTRIC FIELD STRENGTH DUE TO A
UNIFORMLY SURFACE CHARGED
DISC +q -q
Page 13
ELECTRIC CHARGES AND FIELDS
mg
(b) Tension T, electric force Fe and gravity force
Electric lines of force are always to mg act on the bob, as shown T qE mg 0
equipotential surface. Also T balances the resultant of qE and mg.
Illustration 16 : Therefore T = qE 2 mg 2
An particle is located at a point where
electric field strength is 3 × 10 4 N/C. = 2 qE (as qE = mg)
Calculate (a) the force on the -particle (b) = 2 2 10 9 2 104
its acceleration. = 5.64 × 10–5 N
Solution : Illustration 18:
Along x-axis at positions x = 1, x = 2, x = 4
(a) F qE
......... charges q are placed. What will be
F 2 1.6 10 C 3 10 N / C
19 4
electric field at x = 0 due to these charges.
What will be the value of electric field if
15 15
6 1.6 10 9.6 10 N the charges are alternately positive and
negative.
F qE 9.6 10 15 N
(b) a m m 4 1.6 10 27 kg us Solution:
By superposition theory
q q q q
1.5 1012 m /s2
x=0 x=1 x=2 x=4 x=8
Illustration 17 :
A pendulum bob has mass 4 mg and carries q 1 1 1 1
a charge 2 × 10–9 coulomb. It hangs in E 2
2 2 2 .....
4 0 1 2 4 8
equilibrium from a massless thread of
length 50 cm whose other end is fixed to a q 1 1 1 1
.....
vertical wall. A horizontal electric field of 4 0 1 4 16 64
intensity 200 V/cm exists in space. Calculate Terms in the bracket are G.P. with first term a =
(a) Angle made by the thread with 1 a
the vertical 1 and common ratio r = . Its sum S
4 1 r
(b) Tension in the thread
q 1 1 4
Solution : E q
(a) At equilibrium, qE = T sin 4 0 1 1/ 4 4 0 3
mg = T cos Therefore,
If the charges are alternately positive and negative
qE 2 109 2 10 4 q q q q
tan 1.0
mg 4 10 6 10 x=0 x=1 x=2 x=4 x=8
= 45°
q 1 1 1
E 1 4 16 64 ....
4 0
where a = 1, r = –1/4
q 1 1 4q
E = 4 5
4 0 1 ( 1/ 4) 0
Page 14
ELECTRIC CHARGES AND FIELDS
Illustration 19 : 1 q
3
A copper ball of density 8.6 gm/cm , 1 cm in
Now, E 3
rp rq
4 0 | rp rq |
diameter is immersed in oil of density 0.8 g/ Substituting the values, we have
cm3. What is the charge on the ball if it (9 109 )(1.0 106 )
remains just suspended in oil in an electric E 3/2
(ˆi 6ˆj k)
ˆ
(38)
field of intensity 36000 V cm –1 acting in the
upward direction? E (38.42iˆ 231.52jˆ 38.42k) ˆ N
Solution : C
–1 6
E = 36000 V cm = 3.6 × 10 N/C Illustration 21 :
= 3.6 × 1011 dyne/C The f i eld lines f or two point charges are
r = radius of the ball = 0.5 cm shown in f ig.
V = vo1ume of the ball
3
4 3 4 1 3
= r cm
3 3 2 6 A D B C E
U= upthrust = weight of the displaced oil =
Vdg = × 0.8 × 980 dyne i. Is the field uniform?
6
U ii. Determine the ratio qA / qB .
E
iii. What are the signs of q A and qB ?
qE
iv. If qA and qB are separated by a distance
10 cm, find the position of neutral point.
Solution:
i. No
ii. Number of lines coming from or coming to a
mg charge is proportional to magnitude of charge,
q A 12
m = mass of the ball = Vd1 = (8.6) so q 6 2
6
B
For equilibrium; U + qE = mg
iii. q A is positive and qB is negative
0.8 10 2 9.8 q 3.6 1011
6
iv. C is the other neutral point.
8.6 102 9.8 v. For neutral point E A = EB
6
–12
Solving q = 1.11 × 10 Coulomb.
1 qA 1 qB
Illustration 20: 2
4 0 l x 4 0 x 2
A charge q = 1 C is placed at point (1m,
2m, 4m). Find the electric field at point EB
P(0m, –4m, 3m). A B EA
Solution : l C
Here, rq î 2 ĵ 4k̂ 2
l x qA
and rp 4jˆ 3k ˆ 2 x 24.2 cm
x qB
rp rq î 6 ĵ k̂ ELECTRIC FLUX
or It is the measure of total number of electric
2 2 2 lines of force crossing normally the given area.
| rp rq | (1) (6) (1) 38 m
The total flux passes through the given surface is
Page 15
ELECTRIC CHARGES AND FIELDS
y
given by E . A E
E a
ds
x
ds
a
a
z
in= –a2 E and out= a2 E
EA cos total =0
where is the angle made by the normal with
E
the electric field.
total = 0
Its unit is N m2/Coulomb or volt-meter. Its R
dimension is ML3T–3A–1.
The area vector of shown surface is ds . E
acute is ve
exiting line of force R q
obtuse is ve kq q q
E 2 R 2 2
entering line of force R 2 4 0 R 2 0
This vector is perpendicular to the surface. Note : here electric field is radial
Then flux linked with this surface ELECTRIC DIPOLE
A pair of equal and opposite charges separated
E E ds E ds cos
Electric flux is also defined as the total number by a small distance is called a dipole. Its dipole
of lines of force passing normally through the moment is p q d where d is the distance
given surface. between the charges and q is magnitude of the
The value of is zero in the following
circumstances : charge. Dipole moment p is a vector and
(a) If a dipole is (or many dipoles are) enclosed by a
direction of p is from --q to +q.
closed surface
(b) Magnitude of (+ve) and (-ve) charges are equal For convenience of calculation the distance d is
inside a closed surface written as 2a in which case | p| 2a.q
(c) If no charge is enclosed by the closed surface
(d) Incoming flux (- ve) = out going flux (+ve) p (resultant due to many dipoles)
ds p1 p2 ....
E FIELD INTENSITY AT A POINT ON THE AXIS OF A
R E
ds ds DIPOLE
E
in= – R2 E and out= R2 E -q +q E2 E E
2a P 1
total = 0 r
R E 1 2 pr
Ea
4 (r a 2 ) 2
2
E
1 2pr
in= circular = – R E2 Vectorially E where r is the
40 (r 2 a 2 )2
and out= curved = R2 E distance of the point from the centre of the
total = 0
dipole. The direction of E is same as the
direction of dipole moment p . If r >> a,
1 2 p 2kp
Ea
40 r 3 r3
Page 16
ELECTRIC CHARGES AND FIELDS
+q
E
qE
P m
a
E2 2a sin E
a
r
m
qE -q
-q +q Net Force on dipole = qE – qE = 0
P Net Torque on the dipole
a a = (Either force) × (perpendicular distance
1 p kp
Ee 3
between the two parallel forces)
40 (r 2 a 2 )3 / 2
r 2
a 2 2
= qE . 2a sin = 2aq.E.sin = pE sin
Vectorically, τ = p × E
1 p
If r >> a, E e 3
kp / r 3
4π 0 r ( tries to allign p along E)
For r >> a Ea 2E e pE sin
angular acceleration
FIELD INTENSITY AT A POINT HAVING POLAR I 2ma 2
CO-ORDINATES (R, θ ) Illustration 22:
2pcos psin Calculate the electric field intensity due
E11 E
4 0 r 3 40 r3 to an electric dipole of length 10 cm having
charges of 100 µC at a point 20 cm from
1 p
2
E E11 E 2 E 3 cos 2 1 each charge on equatorial line.
4 0 r 3 Solution:
E The electric intensity on the equatorial line of
E E11
an electric dipole is
1 p
E
4 0 (d l 2 ) 3 / 2
2
r p = 2l q C-m
os
pc
= 10–5 C-m
p d2 + l2 = (20 × 10–2)2 = 4 × 10–2
-q +q
9 109 105
p sin E
Direction of E w.r.t direction of r is given by (4 102 )3/2
tan 1 tan 9 109 10 5 9
tan or tan 107
2 2 10 3 8 8
E = 1.125 × 107 N/C
E E11
Illustration23 :
P Find out the torque on dipole in N-m given :
L Electric dipole moment P 107 (5iˆ ˆj 2k)
ˆ
r
co
ulomb meter and electric field
p E 107 (iˆ ˆj k)
ˆ Vm–1 is -
-q P +q
Page 17
ELECTRIC CHARGES AND FIELDS
Solution:
d 2
ˆi ˆj kˆ
dt 2
PE = 5 1 2 This equation represents simple harmonic motion
1 1 1 (SHM). When dipole is displaced from its mean
position by small angle, then it will execute SHM.
ˆi(1 2) ˆj(2 5) k(5
ˆ 1) 3iˆ 7ˆj 4kˆ
d 2 pE
| | 8.6 N-m Eq (iii) can be written as 0
dt 2 I
Illustration24 : On comparing above equation with standard
Three points charges +q, –2q, +q are equation of SHM.
arranged on the vertices of an equilateral d 2
triangle as shown in the figure. Find the 2
2 y 0 , we have ;
dipole moment of the system. dt
-2q pE pE
2
30o 30o
I I
a a
I
T 2 , where T is the time period of
+q a +q pE
Solution:
oscillations.
Arrangement of the charges is equivalent to two
dipoles having dipole moment p each as shown Illustration 25:
above. An electric dipole of dipole moment p is kept
Net dipole moment at a distance r from an infinite long charged
Pnet = p cos 30° + p cos 30° wire of linear charge density as shown.
Pnet = 2p cos 30° = p 3 = qa 3 Find the force acting on the dipole ?
psin30o -q -q psin30
o
A
o o
+ P
30 30 +
+ r
+
2pcos
+q +q +
OSCILLATORY MOTION OF DIPOLE IN
A UNIFORM ELECTRIC FIELD Solution:
When dipole is displaced from its position of Field intensity at a distance r from the line of
equilibrium. The dipole will then experience a
charge is E
torque given by pE sin 2 0 r
For small value of , pE --------(i) dE
Where negative sign shows that torque is acting The force on the dipole is F p dr
against increasing value of
Also, I , p p
2
Where, I = moment of inertia and 2 0 r 2 0 r 2
angular acceleration. Here the net force on dipole due to the wire will
d 2 be attractive.
I 2 ---------(ii)
dt F ORCE BETWEEN TWO SHORT DIPOLES
Hence, from eqs (i) and (ii), we have Consider two short dipoles separated by a
d2 2
d pE distance r. There are two possibilities.
I 2 pE or 2 ----(iii); a) If the dipoles are parallel to each other.
dt dt I
Page 18
ELECTRIC CHARGES AND FIELDS
Illustration26 :
Six charges are placed at the vertices of a
r regular hexagon as shown in the figure. The
P1 P2
electric field on the line passing through
point O and perpendicular to the plane of
E2 E1
the figure at a distance of x a from O
1 3p1p2 is
F 4 a
0 r4
Asthe force +Q –Q
is positive, it is repulsive. Similarly
if p1 || p2 the force is attractive.
b) If the dipoles are on the same axis
+Q O –Q
P1 E2 P2 E1
r
1 6p1p 2 +Q –Q
F 4 4 Solution:
0 r
This is basically a problem of finding the electric
As the force is negative, it is attractive. field due to three dipoles. The dipole moment of
Quadrapole: We have discussed about electric
each dipole is P Q 2a
dipole with two equal and unlike point charges
separated by a small distance. But in some cases KP
Electric field due to each dipole will be E 3
the two charges are not concentrated at its ends. x
(Like in water molecule) consider a situation as The direction of electric field due to each dipole
shown in the figure. Here three charges –2q, q is as shown below:
and q are arranged as shown. It can be visualised Enet E 2 E cos 60 0 2 E
as the combination of two dipoles each of dipole
1 2Qa Qa
moment p = qd at an angle between them. 2 3
The arrangement of two electric dipoles are called 4 0 x 0 x
quadruple. As dipole moment is a vector the a
resultant dipole moment of the system is +Q –Q
p 2p cos / 2 .
|
E
q q
–q 60°
–2q
+Q E –Q
=
–q 60°
q q E
Few other quadruples are also as shown in the +Q –Q
following figures. GAUSS THEOREM
+q +q
GAUSS'S LAW
–2q
–2q +q In electrostatics, Gauss law is a powerful tool
+q which is useful in simplifying electric field
calculations where there is symmetry in
–q +q charge distribution. This law can be used to
–q 2q –q find total flux associated with a closed
surface. It can also be used to find how
+q –q electric charge is distributed itself over
Page 19
ELECTRIC CHARGES AND FIELDS
equal to times the charge enclosed. fields produced by the charges Q1, Q2, .........Qn
0
(a) PROOF OF GAUSS'S THEOREM at P
Q1
A
E q1
B P q3
q q2
Q2 Q3
Now the resultant electric field at P is given
by
Consider a point charge q inside a closed
surface (A) as shown . Imagine a sphere (B)
E E1 E 2 ... E n E11 E12 ... E1n
of small radius 'r' with charge q at the centre The flux of resultant electric field through
of the sphere. Electric field at any point on the closed surface is
1 q
the surface of the sphere is , E 4 2 .
0 r
E.d s E .ds E .d s ..... E .d s
1 2 n
Page 20
ELECTRIC CHARGES AND FIELDS
surface is along the normal to the surface.
Here E1.d s is the flux due to q1 which is TO DEDUCE COULOMB'S LAW FROM
q1 / 0 and E11.d s is the flux due to Q 1 GAUSS'S LAW
Consider a point charge q 1 at O. Let us
which is zero, as it is not enclosed by the construct a Gaussian surface in the form of
Gaussian surface. a closed sphere, having its centre at O and
Similarly the flux due to the other charges with a radius OP = r. Here P is a point on the
also can be written. surface of that sphere
dS E
Now we can write E.d s
r p
O q
q q q 1
1 2 ...... n 0 0.....
0 0 0
S
q q .... q n The electric field strength at P due to charge
1 2 q1 will be E which will be radially outwards.
0 Let us consider a small area element ds at P
qenclosed on the surface of the sphere. ds will be along
E.ds 0
outward normal to the surface and is
parallel to E. So we have
Here q enclosed is the sum of all enclosed
E.d s Eds cos 0 Eds . This condition is
charges which can be positive, negative or
applicable at every point on the surface of
zero.
The flux linked with any closed surface is the sphere.
not influenced by the charges present
E.ds E.ds E ds E4r
2
outside the surface. Now
s
But t he el ect r ic fiel d at any point is t he net qenclosed
fiel d due to al l char ges present inside as well But from Gauss's law E.ds 0
as out side t he cl osed sur face. s
(c) GAUSSIAN SURFACE q 1 q1
E4r 2 1 or E
The expression for electric field intensity 0 40 r 2
can be obtained by applying Coulomb's law This is the field intensity due to point charge
only in simple cases. In the situations, where q1 at a distance r from that charge. If we keep
Coulomb's law and principle of a charge q2 at P, the force acting between q1
superposition becomes difficult in 1 q1q2
calculating the electric field, the same is and q 2 is F Eq 2 4 2 which is
0 r
achieved easily by using Gauss's law. For
Coulomb's law in electrostatics.
this, one has to evaluate the surface integral.
To evaluate the surface integral easily, a GAUSS LAW : A FEW POINTS FOR
closed surface is chosen cleverly around the CONSIDERATION
charge distribution. The surface so chosen Gauss Theorem is applicable for all types of
is called the Gaussian surface. surfaces, whether regular or irregular, but surface
Thus, Gaussian surface around a charge must be fully closed.
distribution (may be a point charge, a line However to calculate electric field using Gauss
charge, a surfae charge or a volume charge) Theorem only symmetrical surfaces should be
is a closed surface, such that electric field considered.
intensity at all the points on the surface is qin
In the formula E.ds , E is due to entire
same and the electric flux through the 0
Page 21
ELECTRIC CHARGES AND FIELDS
Page 22
ELECTRIC CHARGES AND FIELDS
Applying Gauss’s law to the curved surface, we ELECTRIC FIELD DUE TO TWO INFINITE
λL
PLANE PARALLEL SHEETS OF
have
E dS E ( 2 πrL )
ε or E CHARGE :
0 2 0 r
Electric Field Due to a Uniformly
Charged Infinite Non Conducting Sheet:
Let us consider a thin non-conducting charged
σA σB
plane sheet, infinite in extent, and having a surface II
I III
A B
charge density (charge per unit area) C / m 2 .
Let P be a point, distance r from the sheet, at
which the electric intensity is required.
Let us choose a point P symmetrical with P, on Consider two infinite plane parallel sheets of
the other side of the sheet. Let us now draw a charge A and B, having surface charge densities
Gaussian cylinder cutting through the sheet, with equal to A and B respectively. The two sheets
its plane ends parallel to the sheet and passing divide the space of three regions namely region
through P and P . Let A be the area of each
I. Lying to the left of sheet A, region II between
end.
the sheets A and B and region III to the right of
+ + + + sheet B as shown in figure.
+ ++
++ + + +
P' ++ + + + P i) In region I : The electric fields due to both
E + + ++ + E
+
+ + ++ dS
dS +++++ + + + the sheets of charge will be from right to left
(opposite to the direction, in which distances are
By symmetry, the electric intensity at all points measured as positive). The electric field due to
on either side near the sheet will be perpendicular sheets A and B in region I will be
to the sheet, directed outward (if the sheet is
positively charged). Thus E is perpendicular to
E A A
the plane ends of the cylinder and parallel to the 2 0 2 0
curved surface. Also its magnitude will be the 1
same at P and P . Therefore, the flux through E A A .......(i)
2 0
the two plane ends is
ii) In region II: The electric field due to sheet
E E .dS E . dS E dS E dS of charge A will be from left to right (along
positive direction) and that due to sheet of charge
EA EA 2EA B will be from right to left (along negative
The flux through the curved surface of the direction). Therefore, in region II,
Gaussian cylinder is zero because E and dS are
at right angles everywhere on the curved surfaces. E A A
Hence, the total flux through the Gaussian 2 0 2 0
1
cylinder is E 2EA E A A .......(ii )
2 0
The charge enclosed by the
Gaussian surface q A iii) In region III: The electric fields due to both
the sheets of charge will be from left to right. i.e.
Applying Gauss’s law, we have along positive direction. Therefore, in region III,
A
2EA E
0 2 0
Page 23
ELECTRIC CHARGES AND FIELDS
Now resultant field at P2 : E2
E A A 2 0 2 0 0
2 0 2 0
1
E A A .......(iii ) Now resultant field at P3 : E3 0
2 0 2 0 2 0
Special case : If A and B , then it So, E1 E2
0
follows that electric field is zero in regions I and In case two infinite plane charged
III, conductors of finite thickness are placed
while in the region II, the electric field is given by parallel to each other, the equations (i), (ii)
and (iii) will modify to
1
E
2 0 E
1
A B ...in region I
0
E 1
0 E A B ...in region II
0
Thus, in case of two infinite plane sheets of charge 1
having equal and opposite surface charge E A B ...in region III
0
densities, the field is non-zero only in the space
between the two sheets and it is constant i.e,
ELECTRIC FIELD DUE TO A CHARGED
uniform in this region. Further, the field is
independent of the distance between the infinite SPHERICAL SHELL (OR
plane sheets of charge. CONDUCTING SPHERE):
ELECTRIC FIELD DUE TO INFINITE
q P
SHEET (CONDUCTING) : + + + dS
Consider an infinite conducting sheet as shown. + Rr + Charged spherical
+ +
+ O shell
When charge is given to it, it distributes itself over + +
+ +
the outer surface of the sheet. For a thin
conducting sheet, the charge distributes on both Gaussian
Surface
of its faces. So, conducting sheet is equivalent to Consider a thin spherical shell of radius R
the combination of two non-conducting sheets, and Centre O. Let +q be the charge on the
with the same charge spherical shell. Let us find electric field at
point P distant r from the centre of the
spherical shell.
P1 P3 Case (i): When point P lies outside the
P2
spherical shell: Draw the Gaussian surface
through point P. It will be a spherical shell
of radius r and centre O.
Let E be the electric field at point P due to
charge q on the spherical shell. It is
The electric field at any point is the super- position
evident that the field due to charged
of the fields due to two non -conducting charged
sheets.
spherical shell is radial and
spherically symmetric. At every point on the
surface of shell, the field has same
Now resultant field at P1 : E1
2 0 2 0 0
Page 24
ELECTRIC CHARGES AND FIELDS
magnitude and is along normal to the behaves as if the entire charge on it were
surface. Therefore, total flux through the concentrated at the centre of the shell.
Gaussian surface is given by Case (iii): For points inside the charged
spherical shell (r <R)
dS E dS E dS E4 r 2
E.
S S S Consider a concentric Gaussian surface with
Since the charge enclosed by the Gaussian radius r < R as shown. We have
surface is q, according to the Gauss's
E.ds Eds E4r
2
theorem, S S
q
E.4r 2 or q
0
+ + +
E
1 q
. 2 (for r> R) ........(i)
+ +
4 0 r +R P +
It is the same as that at distance r from a r
+
point charge q. It implies that for the + +
points outside the charged spherical shell, + +
the shell behaves as if the charge on the shell As P is inside the shell, there is no charge
were concentrated at its centre. The above enclosed by the Gaussian surface as charge
result for electric field due to a charged resides only on the outer surface of the shell.
spherical shell can also be expressed interms
of its surface density as explained below: q enclosed
From Gauss's law E.d s
0
If is uniform surface charge density of the S 0
spherical shell, then q 4R 2 E 0 inside the charged shell.
substituting for q in equation (i), we have
E
R2 2
E . 2 (for r > R) or E R r 1 r
0 r 0 r 3 1
4 0 R 2 E
Where r is position vector of the point with r2
respect to centre of the sphere.
Case (ii): When point P lies on the surface of r
R
spherical shell: The gaussian surface
through point P will just enclose the charged ELECTRIC INTENSITY DUE TO A
spherical shell. Therefore, according Gauss's UNIFORMLY CHARGED NON-
theorem, CONDUCTING SOLID SPHERE :
q Consider a charged sphere of radius R with
E.4R 2 (or) total charge q uniformly distributed
0
q
1 q on it. Here volume charge density
E . 2 (for r=R) ...... (ii) V
4 0 R
4 3
where V is R .
Since q 4R 2 , the equation (ii) becomes 3
Case (i) : For Points Outside the Sphere r R
E (for r=R) ...... (iii) (or) E ˆ
n
0 0 Consider a Gaussian surface which is
where n̂ is the unit vector normal to the concentricsphere (around the charged
surface. sphere) with radius
For points outside and on the surface of a r > R.
uniformly charged spherical shell, it
Page 25
ELECTRIC CHARGES AND FIELDS
q
From Gauss's law E.ds 0 On the surface of the sphere r = R and
S
E.d s E4r
2
where 1 q
S E
q 4 0 R 2
E.4r 2
0 In the case of a uniformly charged spherical
distrbution,
+ + + 1 q 1 q
+ + + n i) for r R, E and V
++ + + 4 0 r 2
4 0 r
r ++R
E
+ +
+ +
1 q 1 q
ii) for r R, E and V
4 0 R 2
4 0 R
1 q
E
4 0 r 2 for r > R iii) for r R, E 1 qr
and
4 0 R 3
1 q The variation of E with distance r from
and E 3
r.
4 0 r centre is as shown in the graph.
Case (ii) : For Points Inside the Sphere r R
E 1
Let us consider a concentric Gaussian E
surface of radius r < R . Here also E will be 1 q r3
1
radial everywhere but charge enclosed 4 0 R 2 E
r2
by the Gaussian surface is
R r
E
+ ++ + ELECTRIC INTENSITY DUE TO
+ ++ + + +
+R+ + + + + + CONCENTRIC CONDUCTING
+ ++ +r + + CHARGED SPHERICAL SHELLS
+
Page 26
ELECTRIC CHARGES AND FIELDS
1 1 q Illustration 29 :
E 4a 2 (q1 ) E . 21
0 4 0 a A point charge +q is located L/2 above the
The point lies on the surface of the inner centre of a square having side L. Find the
shell & inside the outer shell flux through this square.
case (iii) : If r = b (i.e, for points on the surface of Solution:
The charge q can be supposed to be situated at
center shell ); enclosed q1 q 2
the centre of a cube having side L with outward
1 1 q
E 4r 2 (q1 ) E 1
2
flux . In this cube the square is one of its face
0 4 0 r
having flux / 6 .
case (iv) :If r = b [i.e, for points on the surface of q
outer shell]; q enclosed q1 q 2 L/2
E 4b 2
1 1 q q
. 1 2 2
L
0
q1 q 2 E
4 0
O
b
case (v) : If r > b i.e, for points outside the outer q
shell; q enclosed q1 q 2 Flux through the square 6
0
1 1 q1 q 2
E 4r 2 q1 q 2 E Illustration 30 :
0 4 0 r2
Illustration 27 : A cylinder of length L and radius b has its
axis coincident with the x axis. The electric
What is the value of electric flux in SI unit
in Y-Z plane of area 2m 2, if intensity of field in this region E 200iˆ . Find the flux
through (a) the left end of cylinder (b) the
electric field is E (5iˆ 2ˆj) N/C. right end of cylinder (c) the cylinder curved
Solution: surface, (d) the closed surface area of the
N 2 cylinder.
E .dA (5iˆ 2 ˆ ˆ
j).2i = 10 m Solution:
C
From figure, then
Illustration 28 :
A point charge q is placed at a corner of a
cube with side L. Find flux through entire E SC dA
surface and flux through each face. dA
dA
Solution: E
Sa Sb
A corner of a cube can be supposed to be the
centre of a big cube made up of 8 such cubes,
(a) a E .A EA cos
therefore flux through it is q / 8 0 . The direction
of E is parallel to the three faces that pass through 2
= 200 × b cos 200b
2
this face, thus flux through these is zero.
(b) b EA cos 0 200b 2
(c) c EA cos 90 0
Aq (d) a b c 200b2 200b 2 0 0
Page 27
ELECTRIC CHARGES AND FIELDS
Illustration 31 :
A charge Q is distributed uniformly on a
ring of radius r. A sphere of equal radius r
is constructed. With its centre at the
periphery of the ring (fig.) Find the flux of
the electric field through the surface of the
sphere.
Ring A
O
O1
Sphere
B
Solution:
From the geometry of the fig. OA = OO1 and
O1A= O1O. Thus, OAO1 is an equilateral
triangle.
Hence AOO1 = 600 OR AOB = 1200
The arc AO1B of the ring subtends an angle 1200
at the centre O. Thus, one third of the ring is
inside the sphere. The charge enclosed by the
Q Q
sphere . Thus flux out of sphere 3
3 0
Page 28
Page 29
ELECTRIC POTENTIAL & CAPACITANCE
Page 30
ELECTRIC POTENTIAL & CAPACITANCE
B +q1 –q2
Two charges +Q and -Q are separated by a
distance d. The electric potential at point ‘P’ in y d
between the charges is At the point B V1 V2
Q 1 q1 1 q2
P Q
4 0 y 4 0 d y
x d x
d
q2 q
1
1 Q Q dy y
V
4 0 x d x
d q
Two charges +Q and -Q are separated by a 1 2
y q1
distance d. The electric potential at point ‘P’
out side the charges is d q2
1
y q1
Q Q P
............
d x
d
y
q2
1 Q Q 1
V q1
4 0 d x d x
Points of zero potential exists only for unlike
ZERO POTENTIAL POINT
charges
Points of zero potential exists only for unlike
Illustration-1:
charges both in between and out side the line
Two point charges - 5 C and + 3 C are
joining the charges
placed 64 cm apart. At what points on the
Two unlike charges are separated by a distance
line joining the two charges is the electric
d. Two zero potential points are obtained. One potential zero? (Assume the potential at
point between two charges and other outside infinity to be zero)
nearer to smaller charge. Solution:
+q1 A –q2 For the potential to be zero, at the point P, it
will lie either between the two given point
x (d – x) charges or on the extended position towards
the charge of smaller magnitude.
d Suppose the point P is at a distance x cm from
V1 V2 0(or )V1 V2 the charge of larger magnitude. Then from the
1 q1 1 q2 following figure potential at P is 0.
64cm
4 0 x 4 0 d x
d x q2 d q X
1 2
x q1 x q1
d –5C P +3C
d q2 x
1 q2 1 5 10 6
3 10 6
x q1 1 4 . x 10 2 64 x 102 0
q1 0
If q1 < q2 in magnitude zero potential point is 3 5 8 x 320
0 ; 0 or, x = 40 cm
obtained outside nearer to q1. 64 x x x 64 x
Page 31
ELECTRIC POTENTIAL & CAPACITANCE
x and a charge -q is fixed at each of the points
x = 2x 0, x = 4x0, x = 6x0, …. , here x0 is a
–5C +3C P positive constant. Then find the potential
at the origin due to the above system of
64cm charges.
From the above figure, we get Solution:
3 5 320 2 x Potential at origin will be given by
0 ; 0 ; x = 160 cm
x 64 x x x 64
q 1 1 1 1
V ........
Illustration -2: 4 0 x0 2 x0 3 x0 4 x0
Two electric charges q and -2q are placed
at a distance 6m apart on a horizontal plane. q 1 1 1 1
. 1 2 3 4 ........
Find the locus of point on thus plane where 4 0 x0
the potential has a value zero.
y q
ln(2)
4 0 x0
P
Illustration -4:
Infinite charges of magnitude q are placed
r1 y r2 at coordinates x = 1m, 2m, 8m........
respectively along the x-axis. Find the
+q -2q value of potential at x = 0 due to these
x
x (6 - x) 1
charges. k 4
6m 0
Solution: Solution:
If point P is at a distance r1 from point charge Resultant potential at x=0
+q and r2 from charge -2q (figure), then 1 1 1 1 1
V kq ........ kq 2 kq
1 2 4 8 1 1 / 2
1 q 2q
VP V1 V2 Illustration -5:
4 0 r1 r2 In the given figure, there are four point
charges placed at the vertices of a square
According to the problem, VP 0 , i.e., of side a = 1.4 m. If q1 = +18 nC, q2 = -24
q 2q nC, q3 = +35 nC and q4 = +16 nC, then
0 r2 2r1 ..........(I) find the electric potential at the centre P
r1 r2
of the square. Assume the potential to be
But if the charge q is assumed to be situated at zero at infinity.
q4 q3
the origin, then r12 x 2 y 2 and
2
r22 6 x y 2
P
So substituting these values of r1 and r2 in Eq.
2
(i), we get x 2 y 2 16
So the locus of the point P is a circle with radius q1 q2
4m and centre (-2, 0) Solution:
Illustration -3: The distance of the point P from each charge is
A charge +q is fixed at each of the points
x=x0, x = 3 x0, x = 5 x0, …. on the x- axis
Page 32
ELECTRIC POTENTIAL & CAPACITANCE
VA VB E x and VB VA E x
Case-II : Line AB is perpendicular to electric field.
A
y E
1 B
Wext 1 2 VA VB 0 and VA VB
V2 V1 Where
q Relation between Electrostatic Potential
2 2 (V) and Electric Intensity (E)
Wext 1 2 E.dl ; V2 V1 V E.dl Electric field at a point is equal to the negative
1 1 gradient of the electrostatic potential at that
If Wext 0, V2 V1 ; point 2 is at higher potential point.
than point 1. dV
i.e., E
dr
If Wext 0, V2 V1 ; point 2 is at same potential The negative sign indicates that the direction of
as at point 1.
E is always in the direction of decrease of
If Wext 0, V2 V1 ; point 2 is at smaller electric potential.
potential than point 1. dV
Potential difference in a uniform electric E dV Edr dV E .dr
dr
field Vf rf
B E
dV E.dr
+q Vi ri
Page 34
ELECTRIC POTENTIAL & CAPACITANCE
decreases.
For uniform electric field the relation between
E.d 0
E and V is E = V/d Where ‘d’ is distance The line integral of electrostatic field over a
between two points having Vpotential closed path in an electric field is zero.
difference.
As electrostatic force is conservative, all paths
Electric field is always in the direction of
from i to f yield the same result, when the line
decreasing potential .
integral is found.
The component of electric field in any direction
is equal to the negative of potential gradient in EQUIPOTENTIAL SURFACES :
that direction. Equipotential surface in an electric field is a
V V V
surface on which the potential is same at every
E i j k point. In other words, the locus of all points
x y z
CONSERVATIVE NATURE OF ELECTRIC which have the same electric potential is called
equipotential surface.
FIELD
An equipotential surface may be the surface of
A conservative force is that force for which the
a material body or a surface drawn in an electric
work done around a closed path is zero.
y field. The important properties of equipotential
i surfaces are as given below.
l1
a) As the potential difference between any two
ri f points on the equipotential surface is zero, no
l2
work is done in taking a charge from one point
rf to another.
x
+q b) The electric field is always perpendicular to an
equipotential surface. In other words electric
field or lines of force are perpendicular to the
z equipotential surface.
Consider a closed path i f and f i in c) No two equipotential surfaces intersect. If they
the electric field of point charge +q. Let a small intersect like that, at the point of intersection
test charge +q0 be moved over the closed field will have two different directions or at the
path. same point there will be two different potentials
f which is impossible.
w i f
Then E.d
(along l1) d) The spacing between equipotential surfaces
q0 enables to identify regions of strong and weak
i
Vf Vi dV 1
fields E . So E
i dr dr
w (if dV is constant).
and f i E.d l
(along l2)
q0 e) At any point on the equipotential surface
f
= Vi Vf component of electric field parallel to the surface
wi f wf i is zero.
(Vf Vi ) (Vi Vf ) 0 f) In uniform field , the lines of force are straight
q0 q0
and parallel and equipotential surfaces are planes
In other words work done in moving an unit perpendicular to the lines of force as shown in
positive test charge over a closed path in figure
electrostatic field is zero. It means electrostatic
field is a conservative field and electrostatic
forces are conservative.
Page 35
ELECTRIC POTENTIAL & CAPACITANCE
Solution:
As electric field E is related to potential V
equipotential through the relation
surface
dV
E
dr
dV d
Ex (4 x 2 ) 8 x 8 1 8
The equipotential surfaces are a family of dx dx
concentric spheres for a uniformly charged dV d
Ey (4 x 2 ) 0
sphere or for a point charge as shown in figure dy dy
Equipotential surfaces in electrostatics are dV d
similar to wave fronts in optics. The wave fronts And E z (4 x 2 ) 0
dz dz
in optics are the locus of all points which are in
the same phase. Light rays are normal to the So, E Ex i Ey j Ez k 8xi
wave fronts. On the other hand the equipotential
at (1m, 0m, 2m) E 8 i
surfaces are perpendicular to the lines of force.
1) In case of non-uniform electric field, the field i.e., it has magnitude 8 V/m and is directed along
lines are not straight, and in that case negative x-axis.
equipotential surfaces are curved but still Illustration -8:
perpendicular to the field. Find the potential difference VAB between
2) Electric potential and potential energy are always A (2m, 1m, 0m) and B (0m, 2m, 4m) in an
defined relative to a reference. In general we electric field, E ( xi 2 y j zk ) V/m.
take zero reference at infinity. The potential at a Solution:
point P in an electric field is V if potential at
dV E.d r
infinity is taken as zero. If potential at infinity is (2,1,0)
A
V0, the potential at P is (V–V0).
3) The potential difference is a property of two
dV
B
(0,2,4)
( xi 2 y j zk ).(dxi dy j dzk )
enables us to identify regions of strong and weak VA VB ( xdx 2 ydy zdz )
field. (0,2,4)
(2,1,0)
dV 1 x2 z2
E E VAB y2 3 volt
dr dr
2 2 (0,2,4)
Ex:
Illustration -9:
Find the potential difference between
points A and B in an electric field
EP < EQ < ER
R
E (2i 3j 4k ) NC 1
P Q
30V 20V
40V 10V
Illustration -7: where rA (i 2 j k ) m
If electric potential V at any point (x, y, z)
and rB (2i j 2k )m .
all in meters in space is given by V = 4x2
volt. Calculate the electric field at the point Solution:
(1m, 0m, 2m)
We know that dV E.d r
Page 36
ELECTRIC POTENTIAL & CAPACITANCE
A Solution:
VAB VA VB E.d r B C
B
(1, 2,1)
(1, 2,1)
0.2m
(2dx 3dy 4dz )
(2,1,2)
A 0.2m D
[2 x 3 y 4 z ] | (1, 2,1)
1 volt BPD is equipotential line. VP>VA
(2,1,2)
Electric field is perpendicular to equipotential line
Illustration -10:
and directed from higher potential to lower
The electric field in a region is given by
potential. i.e., along PA.
A
E 3 i . Write the SI unit for A. Write an dV 3 15 15
x E along PA
expression for the potential in the region dx 0.2 2 2 2
assuming the potential at infinity to be Illustration -13:
zero. The electric potential existing in space is
Solution: V(x, y, z) = B(xy + yz +zx). Find the
A expression for the electric field at point P(1,
As E , A Ex 3 ( N / C ) m 3 Nm 3 / C .
x3 1, 1) and its magnitude if B = 10 S.I. unit.
Potential in the region,
x x
Solution:
A
V E.d x 3 i .(dxi) V B ( xy yz zx )
x
x
V
x
x 2 A Ex B( y z)
3
A x dx A x
2
2 2 x because variables y and z are treated as
Illustration -11: constant
An electric field E Cxi exists in the space, Now, put the values of B, y and z. Given B =
10 and (x, y, z) are (1,1,1) respectively..
where C 10 V / m 2 . Taking the potential
at (10 m, 20 m) to be zero, find the potential V
So, E x B( y z)
at the origin. x
Solution: = - 10 (1+1) = -20N/C
x0
V
V
dV Ex dx Similarly, E y B( x z)
V 0 x 10
y
0
[now treating x and z are constants]
x2
V 0 C therefore E y 10(1 1) 20 N/C
2 10
V
V 50 C 500 volt and Ez B ( x y ) 20 N/C
z
Illustration -12:
A, B, C, D and P are points in a uniform therefore, E E x i E y j Ez k
electric field. Potentials at these points are
VA=2V, VP=VB=VD=5V. Find the electric or, E (20i 20 j 20k ) N/C
field at P.
Page 37
ELECTRIC POTENTIAL & CAPACITANCE
20 dx 30 B
dV
0
x0
y0
dy 10cm
– – – – – – – – –
V 40 60 100 volt. V
Illustration -15: qE mg or q mg
d
A positively charged oil droplet remains mgd 1.6 1014 9.8 10 103
stationary in the electric field between two V
q 1.6 1019
horizontal plates separated by a distance
of 1cm. I f the charge on the drop is 9.8 104 volt
esu and the mass of droplet is g, Illustration -17:
what is the potential difference between An infinite plane sheet of charge density
two plates? Now if the polarity of the plates 10-8 C/m2 is held in air. In this situation,
is reversed what is the instantaneous how far apart are two equipotential
acceleration of the droplet? [g = 9.8 m/s2] surfaces where potential difference is 5V.
Solution: Solution:
As the droplet is at rest, its weight W = mg will Here; 10 8 C / m 2 , dV 5V
be balanced by electric force F = qE
dV
i.e., qE = mg Since E (neglecting -ve sign with dV/dr)
dr
+ + + + + – – – – –
an d E
Fe= qE 2 0
+ (for an infinite plane charged sheet)
+
qE mg F0 = mg
– – – – – + + + + +
Page 38
ELECTRIC POTENTIAL & CAPACITANCE
dV V V 12 J / C
or As E , r 0.5 m
2 0 dr r E 24 N / C
Q
2 0 dV 2(8.85 10 12 C 2 / Nm 2 )(5V ) As V ke ,
dr r
10 8 C / m2
Vr (12 J / C )(0.5m)
8.85 103 m 8.85mm Q 0.67 109 C
ke 9 109 Nm2 / C 2
Illustration -18:
ELECTROSTATIC POTENTIAL ENERGY (OR)
A uniform field of magnitude 2000 N/C is
directed 370 below the horizontal as shown ELECTROSTATIC INTRACTION ENERGY:
in the figure. Find (a) the potential The workdone in assembling number of charges
difference between P and R. (b) If we define to a given configuration by bringing them from
infinity is called electrostatic potential energy of
the reference level of potential so that
system of charges.
potential at R is 500V, what is the potential
at P? Two charges Q1 and Q2 are separated by a
distance 'd'. The P.E. of the system of charges
1 QQ 1 2
is U 4 . d from U=W=Vq
0
d
5cm Q1 Q2
P R
37°
Three charges Q1 , Q2 , Q3 are placed at the three
vertices of an equilateral triangle of side 'a'. The
P.E. of the system of charges is
1 QQ 1 2 QQ QQ
U 2 3 3 1 or
4 0 a a a
Solution:
(a) Here 1 Q Q 1 2
0 U
E 2000 N / C , 37 , VR 500V 4 0 a
dl PR 5cm 5 102 m Q3
E.dl Edl Cos
(2000 N / C )(5 102 m) cos 37 0 80V a a
Thus, VP VR E.dl 80V
(b) As VP VR 80V and VR 500V , Q1 a Q2
Four identical charges of each magnitude Q are
VP 500V 80V 580V placed at four corners of a square of each
Illustration -19: side’d’. The electrostatic interaction energy of
Electric field intensity at point B due to a all the particles is
point charge Q kept at point A is 24 N/C Q Q
and the electric potential at point B due to
the same charge is 12J/C. Calculate the d
distance AB and the magnitude of charge
Q.
Solution: Q d
Q
Here; E 24 N / C , V 12 J / C
Page 39
ELECTRIC POTENTIAL & CAPACITANCE
Page 40
ELECTRIC POTENTIAL & CAPACITANCE
A m v0 Large separation
I
q2 a q1
II + Q Fixed
v0
B
b
q2
rmin
q1
(C) Fixed
When a charged particle is accelerated
Case-II:
by an electric field (uniform or non - uniform),
No charge is fixed and both are free to move
by Work energy theorem, i.e., KE W , we (along same line)
have Figure (a) shows a charge q1 and of mass m1 at
1 2 1 2
mv mu qV (but W = qV) rest and another charge q2 of mass m2 is
2 2 projected towards it from a large separation
2 2qV
or v u with initial speed v0. In this case as charge q2
m gets close to q1, due to mutual repulsion q1 also
And if the charged particle is initially at rest, i.e., u starts moving and because of electric force its
=0 speed increases and that of q2 decreases. As
the separation between the two charges
2qV
v decreases their interaction energy increases and
m interaction energy will be maximum when they
And if the field is uniform, i.e. E = (V/d) will be at the distance of closest approach rmin:
At this state the kinetic energy of the two
2qEd
v charges will be minimum and as already
m analyzed in the topic of linear momentum
CLOSEST DISTANCE OF APPROACH BETWEEN conservation that at the point of maximum
TWO CHARGES potential energy in collision the speeds of
Case-I: particles are equal. By conservation of energy
One Charge is kept Fixed and other is moving and linear momentum of the two moving charges
Figure (a) shows a fixed charge q1 and from a we can find the closest distance of approach as
large distance another charges particle having explained below.
mass m and charge q2 is projected towards it m2 v0 m1
Page 41
ELECTRIC POTENTIAL & CAPACITANCE
Solution:
2 Kq1q2 m1 m2
rmin WAB 100 J , q0 = 4C ; VA= -10 v, VB= V
m1m2v02
WAB
q1q2 m1 m2 Since VB VA q ,
rmin 0
2 0 m1m2 v02
100 J
V-(-10v)= 25v
Case-III: 4C
One Charge is fixed and other moving not in Or V=25 v -10v=15v
line Illustration -21:
Figure-(a) shows a fixed charge q1 and another A charge of 10 C is moved in an electric
charge q2 of mass m is projected towards it field of a fixed charge distribution from
with initial speed v0 at an impact parameter d point A to another point B slowly. The work
from large separation. Due to mutual repulsion done by external agent in doing so is 100
the path of q2 gets deviated as shown in figure- J. What is the potential difference
(b) and it moves in the curved path as shown.
The separation between the two charges is
V A VB ?
minimum during motion will be the normal Solution:
distance between charge q1 and the trajectory Wext q(VB VA ) 100 10(VB VA )
of q2 as shown. If at the point of closest
VA VB 10 Volts
approach the speed of then by conservation of
Illustration -22:
angular momentum we can write
Suppose an electron in the picture tube of
v0 d a television set is accelerated from rest
mv0 d mv f rmin ; v f ......(i )
rmin through a potential difference Vb Va Vba
By conservation of energy, we have = + 5000V
(a) What is the change in electric potential
1 2 1 2 Kq1q2
mv0 mv f ......(ii) energy of the electron?
2 2 rmin (b) What is the speed of the electron (m =
Using eq. i and ii, we can solve these for rmin 9.1 1031 kg) as a result of this
acceleration?
v0 Solution:
q The electrons, accelerated towards the positive
2 d
q1
plate, will change in potential energy by an
amount PE = q V b a . The loss in potential
a Fixed
energy will equal to its gain in kinetic energy
q2 (energy conservation)
vf
m (a) The charge on an electron is
rmin
d q = -1.6 10 19 C.
q1 Therefore its change in potential energy is
b
Illustration -20: PE = q V b a = (-1.6 10 19 C)(+5000 V)
If 100J of work has to be done in moving = -8.0 10 16 J
an electric charge of 4C from a place where The minus sign indicates that the potential energy
potential is -10V to another place where decreases. The potential difference, Vba , has a
potential is V volt, find the value of V.
Page 42
ELECTRIC POTENTIAL & CAPACITANCE
positive sign since the final potential Vb is higher between them (treat the charges as point
charges and mass of q2 is m)
than the initial potential Va . Negative electrons
Solution:
are attracted towards a positive electrode and q1 q2
repelled away from a negative electrode.
(b) The potential energy lost by the electrons r0
becomes kinetic energy KE. From conservation According law of conservation of energy
of energy, KE + PE = 0, so U1+ K1 = U2+ K2
KE = - PE 1 q1q2 1 q1q2 1 2
0 mv
1 2 4 0 r0 4 0 r 2
mv - 0 = q Vb Va = qVba
2
1 2 q q 1 1
Where the initial energy is zero since we are mv 1 2
given that the electron started from rest. We 2 4 0 r0 r
solve for v.
q1q2 1 1
2 qVba v
v 4 0 m r0 r
m
Illustration -25:
2 1.6 10 19 C 5000V A charge +q0 is fixed at a position in space.
= 31
9.1 10 kg From a large distance another charged
particle of charge -q and mass m is thrown
= 4.2 x 107m/s.
towards +q0 with an impact parameter L
Illustration -23:
as shown. The initial speed of the projected
A particle of mass m and positive charge q particle is v. Find the distance of closest
is released from point A. Its speed is found approach of the two particles?
to be v when it passes through a point B. –q V
Which of the two points is at higher
potential? What is the potential difference
between the points?
L as
Solution:
The point A is at higher potential than B as the
particle has gained kinetic energy while moving +q0
from Solution:
A to B. As -q moves towards +q0 an attractive force
The kinetic energy acquired by the particle is acts on -q towards +q0. No torque acts on -
equal to loss in potential energy. i.e., q relative to +q0 because electrostatic force
1 2 is a central force. In other words angular
K q VA VB mv q VA VB momentum of -q must remain constant.
2
When -q is closest to +q0, it will be moving
1 mv 2 perpendicularly to the line joining the two
VA VB
2 q charges as shown.
–q
Illustration -24: V
Charge q1 is fixed and another point charge
L
q2 is placed at a distance r0 from q1 on a
+q0
friction less horizontal surface. Find the r
velocity of q2 as a function of separation r
vc
Page 43
ELECTRIC POTENTIAL & CAPACITANCE
Let r be the closest separation between the closest approach for the two protons. Given
charges and vc be the velocity of -q at that
(1/ 4 0 )= 9 109 m / F ;
instant.
From conservation of angular momentum, m p 1.67 10 27 kg and e 1.6 1019
mvL = mvcr
coulomb.
From conservation of mechanical energy
Solution:
1 2 1 2 1 qq0 As here the particle at rest is free to move, when
mv mvc
2 2 4 0 r one particle approaches the other, due to
On solving the above equation, we can get r. electrostatic repulsion other will also start moving
and so the velocity of first particle will decrease
Illustration -26:
while of other will increase and at closest
A particle of mass 40 g and carrying a
approach both will move with same velocity.
charge 5 10 9 C is moving directly towards So if v is the common velocity of each particle
fixed positive point charge of magnitude at closest approach, by ‘conservation of
10-8 C. When it is at a distance of 10 cm momentum’.
from the fixed point charge it has a velocity
of 50 cm/s. At what distance from the fixed 1
mu m m i.e., u
point charge will the particle come 2
momentarily to rest? Is the acceleration And by ‘conservation of energy’
constant during motion?
1 2 1 2 1 2 1 e2
Solution: mu m m
If the particle comes to rest momentarily at a 2 2 2 4 0 r
distance r from the fixed charge, then from
4e 2 u
‘conservation of energy’ we have So, r 2 as
4 0 mu 2
1 1 Qq 1 Qq
mu 2 And hence substituting the given data,
2 4 0 a 4 0 r
Substituting the given data, we get 4 (1.6 1019 )2
r 9 109 = 10-12 m
1.67 1027 (7.45 105 )2
1 1 1
40 106 Illustration -28:
2 2 2 An electron travelling from infinity with
1 velocity ‘v’ enters into an electric field due
9 109 108 5 109 10
r to two stationary electrons separated by a
distance of 2m. If it comes to rest when it
1 5 106 100 reaches the midpoint of the line joining the
or 10 8
or ,
r 9 5 10 9 stationary electrons, find the initial
velocity ‘v’ of the electron.
1 100 190
10 m Solution:
r 9 9
i.e., r 4.7 102 m
e
1 qQ F 1
As here, F 4 r 2 ; so a 2 e e
0 m r
i.e., acceleration is not constant during motion. r
Illustration -27: From law of conservation of energy.
A proton moves with a speed of 7.45 X 105 1 1 (2e)e
m 2
m/s directly towards a free proton 2 4 0 r
originally at rest. Find the distance of
Page 44
ELECTRIC POTENTIAL & CAPACITANCE
1 Solution:
i.e., (9.1 1031 ) 2 The work required to make an arrangement of
2
charges is equal to potential energy of the system
9 109 2 (1.6 1019 ) 2
q2 3q 2
1 W 3
4 0 a 4 0 a
32 m / s.
Work done by electric field represents the
Illustration -29: decrease in potential energy. So, we have
Two identical particles of charge q each
are connected by a massless spring of force WE . Field U U i U f
constant K. They are placed over a smooth Now,
horizontal surface. They are released when q2 q2
the separation between them is r and spring Ui 3 ;U f 3
is unstretched. If maximum extension of 4 0 (a) 4 0 (2a )
the spring is r, find the value of K? (neglect 3q 2 3q 2 3q 2
gravitational effect) WE .Field
Solution: 4 0 a 8 0 a 8 0 a
From conservation of mechanical energy Illustration - 32:
Determine the interaction energy of the
1 2 1 q2 q2 point charge located at the corners of a
Kr
2 4 0 r r r or
square with the side a in the circuits shown
in figure.
1 2 q2 q2 +q +q +q –q +q +q
Kr , K
2 8 0 r 4 0 r 3
Illustration -30: +q +q –q +q –q –q
If an electron enters into a space between (a) (b) (c)
the plates of a parallel plate capacitor at Solution:
an angle with the plates and leaves at Interaction energy of any two point charges q1
an angle to the plates, find the ratio of q1q2
and q2 is given by 4 r , where r is the
its kinetic energy while entering the 0
capacitor to that while leaving. separation between the charges.
Solution: Hence, interaction energy of the system.
Let u be the velocity of electron while entering
the field and v be the velocity when it leaves the q2 q2
Ua 4 2
plates. Component of velocity parallel to the 4 0 a 4 0 ( 2a)
plates will remain unchanged.
Hence u cos v cos q 2 q2
Ub 4 2
1
4 0 a 4 0 ( 2a)
2
2 mu 2 2
and
u cos u cos
1 2 v cos
v cos mv q2 2q 2 2q 2
2 Uc 2
4 0 a 4 0 a 4 0 ( 2 a)
Illustration -31:
Calculate the work required to be done to 2q 2
make an arrangement of three particles 4 0 a
each having a charge +q such that the Illustration -33:
particles lie at the vertices of an equilateral Two protons are separated by a distance R.
triangle of side a. What work will be done What will be the speed of each proton
by electric field when the particles are when they reach infinity under their
shifted away so that the side of triangle mutual repulsion?
becomes 2a? Solution:
Let the charge on each proton be Q. Initially
they are at rest, so their kinetic energy (KE)=0
Page 45
ELECTRIC POTENTIAL & CAPACITANCE
and they have only potential energy equal to work done is final potential energy minus initial
Q2 potential energy.
. Q
4 0 R
Q R Q
Finally at infinity, they have infinite separation A A
so their potential energy will become zero.
(because R= now.). And total kinetic
1 2 Q A Q
energy is, ( mv for each proton)
2 1 3Q 2
1 Initial potential energy (for each
2 mv 2 mv 2 (where m is the mass of 4 0 A
2
each proton and v is the speed with which Q2
they are moving at infinity). Now, total initial combination, the energy is and we have
energy = total final energy. 4 0 A
Q2 three such combinations)
or, mv 2
4 0 R 3Q 2
m m Final potential energy =
v 4 0 A / 2
at inf inite separaton
3Q 2 3Q 2
Q Now, work done W
Therefore v 4 mR 4 0 A / 2 4 0 A
0
Illustration -34: 3Q 2
Therefore, W
A bullet of mass 2 gm is moving with a speed 4 0 A
of 10 m/s. If the bullet has a charge of 2
micro coulomb, through what potential it Illustration -36:
be accelerated starting from rest, to acquire Two particles have equal masses of 5 g each
the same speed? and opposite charges of 4 105 C and
Solution: 4 105 C . They are released from rest
1 2 with a separation of 1 m between them.
Use the relation qV mv ; Find the speed of particles when the
2
Here, m 2 103 kg ; v 10 m / s separation is reduced to 50 cm.
Therefore Solution:
Again in this question we have to do the
mv 2 2 10 3 (10) 2 10 1
V conservation of mechanical energy. Initially,
2q 2 2 106 2 10 6 there is only potential energy but finally there is
5 104 volt 50 kV both kinetic and potential energy.
Illustration -35: Initial state energy:
–5 –5
Three equal charges Q are at the vertices Q1 = 4×10 C Q2 = –4×10 C
of an equilateral triangle of side A. How
1m
much work is done (by an external agent)
Bodies are at rest so kinetic energy = 0
in bringing them closer to an equilateral
Q1Q2
triangle of side A/2? Potential energy = 4 R
0
Solution:
In this problem first we have to calculate the 9 109 (4 105 )(4 105 )
initial and final potential energy and then the 14.4 joule
1
Page 46
ELECTRIC POTENTIAL & CAPACITANCE
Page 47
ELECTRIC POTENTIAL & CAPACITANCE
If p is anti parallel to E then, potential energy d
F ( p.E )r
is maximum and again torque is zero, but it is in dr
unstable equilibrium. d
Work done in Rotation of a Dipole in F PE cos
dr
Electric Field
When a dipole is placed in an electric field at an If p and E are along the same direction we can
angle , the torque on it due to electric field is write
given as d dE
F ( pE cos ) or F p .
pE sin dr dr
E Illustration -37:
qE
+q An electric dipole consists of two charges
qE of equal magnitude and opposite sign
+q
separated by a distance 2a, as shown in
-q figure. The dipole is along the x-axis and
qE is centered at the origin.
qE -q y
Page 48
ELECTRIC POTENTIAL & CAPACITANCE
Solution: Work done by external agent
(W ) pE (cos 1 cos 2 )
pE sin pE
sin (Where p is the dipole moment of the dipole).
W U pE cos 2 pE cos 1 1
W pE (cos 0 cos 60) pE
W pE[cos cos 2 ] 2
(1 , 2 2 ) 1
Work done by the field W pE
2
W [cos cos 2 ] Illustration -41:
sin
An electric dipole of dipole moment p is kept
Illustration -39: at a distance r from an infinite long charged wire
A molecule of a substance has a permanent of linear charge density as shown. Find the
electric dipole moment of magnitude 10-29
force acting on the dipole?
C m. A mole of this substance is polarised
(at low temperature) by applying a strong
electrostatic field of magnitude 106 Vm-1.
The direction of the field is suddenly +
+ P
changed by an angle of 600. Estimate the
heat released by the substance in aligning +
+ r
its dipoles along the new direction of the +
field. For the simplicity, assume 100% +
polarisation of the sample.
Solution:
Here, dipole moment of each molecule = 10-29 Solution:
C m. As 1 mole of the substance contains Field intensity at a distance r from the line of
6 1023 molecules, total dipole moment of all
the molecules,
charge is E 2 r
P 6 10 23 1029 Cm 6 106 Cm 0
Initial potential energy (when 600 ), The magnitude force on the dipole is
U i 6 10 6 10 6 cos 60 0 3 J dE p p
F p ; 2;
Final potential energy, dr 2 0 r 2 0 r 2
U f pE cos Here the net force on dipole due to the wire
6 106 106 cos 00 will be attractive.
6 J ELECTRIC POTENTIAL DUE TO A
LINEAR CHARGE DISTRIBUTION
Change in potential energy = -6 J - (-3 J)= -3J
Y
So, there is loss in potential energy. This must
be the energy released by the substance in the +
form of heat in aligning its dipoles. +
Heat energy released is 3J +
+
Illustration -40:
+
An electric dipole in a uniform electric field r dS
l O+
E is turned from 0 position to 600 + P E
position. Find work done by the field. + Gaussian
+ surface
Solution: +
+
+
Y
Page 49
ELECTRIC POTENTIAL & CAPACITANCE
E
So V r Edr 2 r dr q P
0
+ + + dS
+ R r + Charged spherical
+ +
V r log e r C + O Shell
2 0 + +
+ +
Where C is constant of integration and V(r)
gives electric potential at a distance ‘r’ from the Gaussian
linear charge distribution Surface
Consider a thin spherical shell of radius R and
The electric potential due to line of charge at having charge +q on the spherical shell.
any point cannot be defined. We can define only Case (i): When point P lies outside the spherical
P.D between two points. shell.
ELECTRIC POTENTIAL DUE TO 1 q
INFINITE PLANE SHEET OF V r r R
4 0 r
CHARGED (NON CONDUCTING)
Case (ii) : When point P lies on the surface of
Consider an infinite thin plane sheet of positive spherical shell
charge having a uniform surface charge density
1 q
on both sides of the sheet. by symmetry , it V r R
4 0 R
follows that the electric filed is perpendicular to
the plane sheet of charge and directed in out Case (iii) : For points inside the charged spheri-
ward direction. cal shell (r < R), the electric field E = 0
dV
So we can write 0
The electric field intensity is E 2 dr
0
V is constant and is equal to that on the sur-
Electrostatic potential due to an infinite plane
sheet of charge at a perpendicular distance r 1 q
face.So, V 4 R for r R
from the sheet given by 0
Page 50
ELECTRIC POTENTIAL & CAPACITANCE
Case (i) : For points Outside the sphere (r > R) Potential due to a charged ring: A charge
1 q q is distributed over the circumference of ring (
V r (r > R) either uniformly or non-uniformly ) , then electric
4 0 r
po tent ial at t he cent re o f the ring is
Case (ii) : When point P lies on the surface of
spherical shell 1 q
V .
1 q 4 o R .
V r R At distance ‘r’from the centre of ring on its axis
4 0 R
Case (iii) : For points inside the sphere (r < R) 1 q
would be V 4 .
1 qr o R2 r 2
The electric field is E 4 R 3 Electric potential due of a uniformly
0
charged disc
+ E + Consider a uniformly charged circular disc
+ + + having surface charge density .
+ + +
+ + Potential a at point on its axial line at distance x
r + +
+ + +
from the centre is V 2 R x x
2 2
+
+ + o
+ +
R R
+
At the centre of disc x 0 V 2
dV E .dr Edr o
q
v r r
1 qr For x R , V 4 x
dV E dr 4
vs R R 0 R3
dr o
R
Potential on the edge of the disc is V
o
r
1 q r2 Illustration -42:
V Vs A charge Q is distributed over two concen-
4 0 R 3 2 R
tric hollow spheres of radii ‘r’ and R ( > r)
such that the surface densities are equal.
1 q 1 q r 2 R2 Find the potential at the common centre.
V
4 0 R 4 0 R3 2 2 Solution:
If q1 and q2 are the charges on spheres of radii
1 q 3 r2
V ‘r’ and R respectively, then in accordance with
4 0 R 2 2 R 2 conservation of charge
At the centre r = 0 then Potential at centre
q1 q2 Q ------(1)
1 3q 3 1 q
VC And according to given problem 1 2 ,
4 0 2 R 2 4 0 R
The variation of V with distance ‘r’ from centre q1 q2 q1 r 2
i.e., or ------(2)
is as shown in the graph. 4 r 2 4 R 2 q2 R 2
So from Eqs (1) and (2)
v
vs
Qr 2 QR 2
q1 2 q
vs r R 2 and 2 r 2 R ----(3)
vs Now as potential inside a conducting sphere is
v 1/r equal to that at its surface, so potential at the
r<R r<R common centre,
r=R r
Page 51
ELECTRIC POTENTIAL & CAPACITANCE
a) determine the charge on the outer shell,
1 q1 q2 and
V V1 V2 r R
4 0 b) find the potential of the inner shell.
Substituting the value of q1 and q2 from Eq.(3)
1 Qr QR
V 2 2 2 2
4 0 R r R r
1 QR r
4 0 R 2 r 2
Illustration -43:
Figure shows two concentric conducting
Solution:
shells of radii r1 and r2 carrying uniformly a) We know that charge on facing surfaces is equal
distributed charges q1 and q2 . respectively.. and opposite. So, if charge on inner sphere is
Find out an expression for the potential of q0 , then charge on inner surface of shell should
each shell. be q0 . Now, let charge on outer surface of
+q2
r2 shell be q2 .
As the shell is earthed. So its potential should
+q1
be zero. So,
r1
Q b a
shell will be 0.
b
b) Potential of outer surface VB potential due
to charge on A + potential due to charge on B.
VB Va ,out Vb ,both surface
1 q' 1 Q
Solution: 4 0 b 4 0 b
a) When an object is connected to earth
(grounded), its potential is reduced to zero. Let a
Q
q ' be the charge on A after it is earthed as 1 b 1 Q Q b a
shown in fig 4 0 b 4 0 b 4 0b 2
Q + q
Illustration -46:
Two circular loops of radii 0.05 and 0.09m,
–q respectively, are put such that their axes
coincide and their centres are 0.12 m apart.
Charge of 106 coulomb is spread uniformly
on each loop. Find the potential difference
between the centres of loops.
q2
q1 r2
The charge q ' on A induces q ' on inner r 1
Page 53
ELECTRIC POTENTIAL & CAPACITANCE
or v So, Vmin The point B lies outside the inner shell, the po-
2 0 m 2 0 m q1
Potential due to a group of concen- tential at B, due to inner shell is V1 4 r
0
tric shells:
q1 q2
To calculate the potential due to a group of con- So, potential at B is VB 4 r 4 b
centric shells, we can use superposition prin- 0 0
Page 54
ELECTRIC POTENTIAL & CAPACITANCE
Potential at point A: + + +
+ +
Let OA = r, such that r < a. +
The point A lies inside the outer shell, +
q2 +
+
+ +
q1 Cavity +
+
O r
a +
A +
+ +
+ +
b + +
So, potential at A, due to outer shell is Ed s 0
q2 For the dotted surface
V2 q 0 inside cavity..
4 0b
From the above result, any cavity in a conduc-
Similarly, potential at A, due to inner shell is
tor remains shielded from outside electric influ-
q1 ence: the field inside the cavity is always zero.
V1
4 0 a This is known as electrostatic shielding. The
q1 q2 effect can be made use of in protecting sensi-
So, potential at A is VA 4 a 4 b tive instruments from outside electrical influence.
0 0 When metal conductor is kept in an electric field,
ELECTRO STATICS OF CONDUCTORS:
1. Electrostatic field is zero inside a conductor.
+ + + E
2. At the surface of a charged conductor, electro- + + 0
static field must be normal to the surface at ev- +
+
ery point. + E0
3. The interior of a conductor can have no excess +
Q0
charge in the static situation: + +
0
4. Electrostatic potential is constant throughout the + +
V constant
volume of the conductor and has the same value +
(as inside) on its surface. +
+
5. Electrostatic shielding. + +
+ +
Consider a conductor with a cavity then there
1. The field inside the conductor is zero.
will be no electric field inside a charged con-
2. The charge resides only on the outer surface.
ductor and all the charge resides on its outer 3. The field is normal to every point on the sur-
surface only. Suppose that charged conductor face.
has a cavity or cavities and there are no charges 4. The potential is constant at everywhere on the
within the cavity or cavities, even then charge surface and throughout the volume of conduc-
tor.
resides on the outer surface of the conductor. CAPACITY OF AN ISOLATED CONDUC-
There will be no charge on the walls of the cav- TOR
ity or cavities. This can be verified very easily If the charge on an isolated conductor is
using Gauss’s law by enclosing the cavity with gradually increased, its potential also increases.
a Gaussian surface. At any instant, the charge Q on the
conductor is directly proportional to the
potential V of the conductor.
Q V (or) Q CV
Page 55
ELECTRIC POTENTIAL & CAPACITANCE
Dielectric
earth 40 Rr
C
Q R r
By definition of capacitance C (ii) Outer sphere is charged and inner is earthed
V
C 4 0 R
The capacity of a spherical conductor depends r
on dimensions of the conductor, nature of the
R
medium in which conductor is placed.
The capacity of a spherical conductor is
4 0 Rr 4 0 R 2
independent of charge on the conductor . C 4 0 R
R r R r
In CGS system, the capacity of a spherical
PRINCIPLE OF A CAPACITOR :
conductor is numerically equal to radius of the
In a capacitor, the maximum charge holding
conductor.
capacity of a conducting plate is increased by
For the earth, R 6.4106 m placing a neutral earthed plate near it, which
The capacity of earth is lowers its surface potential there by increasing
1 its charge holding capacity.
C 4 0 R 9
6.4106 711 F PARALLEL PLATE CAPACITOR
910
Let us consider a parallel plate capacitor which
CAPACITANCE OF A SPHERICAL CONDUCTOR:
consists of two parallel plates each of area A,
Let a spherical conductor of radius R be given
separated by a distance 'd' as shown in figure.
charge Q. Then its potential becomes
Q .Then
V
40 R
Page 56
ELECTRIC POTENTIAL & CAPACITANCE
Q 0 A
C
d V BX 2
X 1
3
Page 58
ELECTRIC POTENTIAL & CAPACITANCE
K1 t1
d
A 0 K2 t2
C
Thickness of medium
Re lative permittivity
A 0
i.e, C The capacity of capacitor is given by
t1 t2
K K ..... 0 A
1 2 C
PARALLEL PLATE CAPACITOR t1 t2
d t1 t 2 .... K K ....
PARTIALLY FILLED WITH MEDIUM 1 2
Let us consider a case of parallel plate capacitor Illustration -50:
in which a medium of dielectric constant K is A metal slab of thickness, equal to half the
partially filled as shown in figure. distance between the plates is introduced
between the plates of a parallel plate ca-
Then the field is uniform in air as well as in the pacitor as shown. Find its capacity.
medium, but fields will have different values. let q
't' be the thickness of the medium whose relative
permittivity is K. The remaining space of ( d- t) air
d
thickness be occupied by air. d/2
q
air
d
K t Solution:
When capacitor is partially filled with dielectric
0 A
capacity C
d t(1 1 )
k
Since thickness of dielectric medium is t, its
equivalent air distance is t/K. The effective For metal slab of thickness t = d/2,
distance between the plates of capacitor is (d- 0 A
C ( K for metal slab)
t) + (t/K). The capacity of capacitor is dt
0 A 0 A
C
0 A A
2 0 .
d t t / K d t(11 / K ) d d
d .
When capacitor is partially filled with dielectric 2
Page 60
ELECTRIC POTENTIAL & CAPACITANCE
Capacitance when the space is filled partly connecting wires. Now we say that capacitor
by a metal slab: is fully charged with a potential difference V and
A 0 charge Q = CV.
C {as K = } The applied voltage ‘V’ is such that the
dt
electric field between plates becomes equal to
When partly filled by metal slab, variation of E
and V as we move from +ve to –ve plate. dielectric break down then the capacitor cannot
E be further charged.
Illustration-51:
Charges Q 1 and Q 2 are given to two
identical metal plates. Find the charge
X distribution on the four surfaces of the
plates. Find the potential difference of
t such arrangement shown in the figure ?
V
Q1 Q2
X
(Area of each plate is A and separation
t between the plates is d. Ignore fringing
Capacitance when the space is completely effect)
filled by metal slab is C Solution:
If space between plates of capacitor is filled The charge on the plates is distibuted equally
by a dielectric of r = K on both sides and charges are induced on the
plates as shown.
[capacitor is charged and disconnected from Initial charges
supply] Q Q Q
1 Q1 2 2
2 2 2
E without dielectric V without dielectric 2
E with dielectric V with dielectric
capacitance (with dielectric)
K Q2 Q1 Q1
capacitance (without dielectric) + Q2 2 +2
2 2
CHARGING OF A CAPACITOR Induced charges
Consider a capacitor C connected in a circuit The charge on inner surface of positive charger
with a battery as shown. Here battery provides Q1 Q 2
plate or the charge of capacitor is
the energy and pulls the electrons from plate a 2
and send to plate b as shown. As a result positive Capacitance C
q (Q1 Q2 )
where V is
charge on plate a and negative charge on plate V 2V
b start increasing. The potential difference potential difference between plates of capacitor.
between the plates increases until it equals the
0 A
applied voltage V between the terminals of the Capacity C
d
battery. Finally plate a and the terminal of the Q1 Q 2 (Q1 Q 2 )d
battery connected to that plate will be at the V
2C 2 0 A
same potential. There will be no longer an
Illustration-52:
electric field in the conducting wires. Similarly
plate b and the terminal of the battery connected An isolated parallel plate capacitor of
to that plate also will be at the same potential capacitance ‘C’ has four surfaces with
and no electric field in the connecting wire charges q1, q2, q3 and q4 as shown. Find
between them. As there is no electric field, no the potential difference between the
further motion of electrons will be there in the plates ?
Page 61
ELECTRIC POTENTIAL & CAPACITANCE
Solution:
q3 q4 Let 1 and 2 be charges on the two plates
q2 and E be electric field between the plates due
q1
to these charges.
Solution:
q1 q 2
We know that conducting surfaces facing each
other must have equal and opposite charges
1 2 A A q1 q 2
Then, E = 2 2A 0
q2 = –q3 2 0 0
q V
V
CS ... (b) Charge comming from battery Q C
eff
C1C2
Effective capacity Ceff C C
1 2
Page 63
ELECTRIC POTENTIAL & CAPACITANCE
+Q1 –Q1
C Cs
Cs ; Cp nC ; n 2 :1
+Q2 –Q2 n Cp
+Q3 –Q3
Q 1+ Q 2 + Q 3 Q 1+ Q 2 + Q 3
C1
V
+ –
C2
As all capacitors are connected in parallel, so
the potential across any of the capacitors is Q
same . V
q1 q 2 q3 C1Q C2 Q
Q1 , Q2
i.e., V1 V2 V3 ; V C1 C2 C1 C2
C1 C2 C3
Using n number of identical capacitors,
q1 q 2 q 3 C1V C2 V C3 V
combination having different capacitance
q V C1 C2 C3
obtained = 2n – 1. While that for unidentical
q capacitors different combinations is 2n.
C1 C2 C3 ... (a)
V Illustration-56:
If a single capacitor when connected to the same
The equivalent capacity between A and B
source draws a charge q then that capacitor is in the given circuit is
said to be the effective or equivalent capacitor
8F
for the three parallel capacito rs.
A B
If the effective capactiance is Cp, 12F 12F
q
CP ... (b) 8F
V Solution:
from (a) and (b) Here 12 F and 12 F are short circuited.
C P C1 C 2 C 3 Hence they are not charged.
Take only 8 F and 8 F parallel
In general CP Cn --- (4.6) combination.
a) The resultant capacity of parallel C 8 8 16 F
combination is greater than the largest capacity Illustration-57
of the capacitors of the combination. In the net work three identical capacitors
b) In parallel, ratio of P.D. on three capacitors are connected as shown. Each of them can
is 1 : 1 :1. withstand to a maximum 100 V potential
c) The ratio of charges on three capacitors is difference. What is the maximum voltage
Q1 : Q2 : Q3 C1V : C2V : C3V C1 : C2 : C3 that can be applied across A and B so that
no capacitor gets spoiled.
d) The charge on first capacitor is
C
C1 C
Q1 Q
C1 C2 C3 B
A
similarly we can find Q2 and Q3.
C
When n identical capacitors each of capacity Solution:
C are first connected in series and next Let q max be the max-charge supplied by the
connected in parallel then the ratio of their battery between A and B so that no capacitor
effective capacities gets spoiled.
For each capacitor
q max CV0 C(100) 100 C
Page 64
ELECTRIC POTENTIAL & CAPACITANCE
For the combination q max Cequivalent Vmax each stair is a and the height is b. Find
2 the capacitance of the assembly.
100 C C Vmax Vmax 150 V
3
Among 150V, potential difference across
parallel combination is 50V and the potential
difference across the other capacitor is 100V. b
Illustration-58 d a
Calculate the capacitance of a parallel plate 3a
capacitor, with plate area A and distance
Solution:
between the plates d, when filled with a
This is equivalent to 3 capacitors in parallel
dielectic whose permittivity varies as
where area of each plate is (A/3).
d
( x ) 0 kx 0 x 0 A / 3 A / 3 A / 3
2 C1 , C2 0 , C3 0
d d db d 2b
( x) 0 k( d x ) x d Then, net capacitance C = C1 + C2 + C3
2
Solution: 0 A 1 1 1
dx =
3 d d b d 2b
Illustration-60:
x Two capacitors of capacitances 20.0 pF
and 50.0 pF are connected in series with a
battery of 20 V. Find the energy supplied
by the battery.
Solution:
In Series, net capacitance is
x=0 x=d
The given capacitor is equivalent to two C1C2 20 50
C pF 14.28pF
capacitors in series. Let C1 and C2 be their C1 C2 20 50
capacities. Then
Charge supplied by battery
1 1 l
q CV 14.281012 20 Coulomb
C dC1 dC2
2.85 10 10 C
Consider an element of width dx at a distance
Work done by battery W = Charge Supplied
x from the left plate. Then
by battery × emf of battery
0 kx A
dC1 for 0 x d = q V 2.85 1010 20 = 5.70 × 10–9
dx 2
Joule.
and dC2
0 k(d x) A for d
xd Illustration-61:
dx 2 Each capacitor shown in figure has a
on substituting these two values we get capacitance of 6.0 µF. The emf of the
battery is 50 V. How much charge will flow
1 l 2 2 Kd
n 0 through AB if the switch S is closed?
C dC KA 2 0 S
KA 2 0 Kd
C n
2 2 0
Illustration-59:
A capacitor is made of a flat plate of area
A and a second plate having a stair-like
structure as shown in figure. The width of B
A
Page 65
ELECTRIC POTENTIAL & CAPACITANCE
Solution: 2 C 2 A
On closing S, one capacitor becomes shorted,
hence charge on this capacitor becomes zero. 11V 4 4
Initially the system had 6.0 + 6.0 = 12 µF in
series with another 6 µF.
B
12 6 Solution:
Initial Capacitance C0 = 4µF
12 6 Capacitance of system
Final Capacitance C 1 = 12µF (as third 2 4
capacitor become useless) 2 4
24 16
= F
Initial Charge q 0 C 0 V 50 4 200C , Final 2 4 11
24
charge = q = 50 x 12 = 600 C. 2 4
So charge q , that flows from A to B is q – q0 Charge on system
Therefore, q = (final charge on outermost 16
q = CV = 11 × C = 16 C
plates) – (final charge on same plates) 11
= q – q0 = 600 µC – 200 µC = 400 µC Charge between A and B = Charge between
Illustration-62: 4/3 q 16
C and B = q = 4 µC
A capacitor of capacity C1 = 10 F can 4 4 3 4 4
withstand maximum voltage V 1 = 6kV P.d. between A and B =
while another capacitor of capacitance C2
= 2 µF can withstand maximum voltage Charge between A and B 4 C
= = 1 Volt
V2 = 4kV. What maximum voltage will the Capacity 4 F
system of these two capacitors withstand Equivalent Dielectric constant:
if they are connected in series ?
When the space between the plates of a parallel
Solution:
plate condenser is completely filled with two slabs
In series pd across the capacitor is inversely
of dielectric constants K1 and K2 and each slab
proportional to capacitance.
When a p.d. of 6 kV is applied across the 10 A
having area and thickness equal to distance
µF capacitor, the corresponding p.d. across the 2
of seperation d as shown in the figure.
6 10
2µF capacitor is 30 kV . But the 2µF
2
capacitor can withstand only 4 kV. Therefore
such arrangement does not work i.e., we cannot
apply maximum possible p.d. across the 10 µF
piece.
However if we apply 4 kV across the 2µF K1 K2
piece,
The equivalent circuit is as shown
Corresponding pd across other one is
2
4 kV × = 0.8 kV, which is less than the A A
10 2 2
limit of 6 kV.
Therefore Maximum p.d. across the system is d K1 K2 d
V1 + V2 = 0.8 + 4.0 = 4.8 kV.
Illustration-63:
Find the p.d. between the points A and B 0 A
in the fig. The values of capacitances are a)Capacity of the left half C1= K1
in F.. 2d
Page 66
ELECTRIC POTENTIAL & CAPACITANCE
0 A d) Effective capacity
b) Capacity of the right half C2 = K2
2d CC A 2K1K 2 2K1 K 2
C 1 2 0 C0
c) C 1 and C 2 may be supposed to be C1 C2 d K1 K 2 K1 K 2
connected in parallel then effective capacity Here C0 is the capacity of the condenser with
A K K2 air medium.
C = C1 + C2 0 1
d 2 2K1 K 2
e) Effective dielectric constant K=
K
1 K 2 K1 K 2
C = C0
where C0 is capacity of Dielectrics in series
2 t t ....... t
1 2 n
capacitor without dielectric. K eq
t1 t 2 t
..... n
k1 K 2 Kn
K1 K 2
d) Effective dielectric constant K
2
K1 K2 K3 Kn
Dielectric in parallel
K1A1 K 2 A 2 ....... K n A n
K eq t1 t2 t3 tn
A1 A 2 ....... A n Illustration-64
K1 A1
A2 A parallel plate capacitor of area A, plate
K2 separation d and capacitance C is filled
with three different dielectric materials
Kn An having dielectric constants K1, K2 and K3
When the space between the plates of a parallel as shown in fig. If a single dielectric
plate condenser is completely filled with two slabs material is to be used to have the same
of dielectric constants K1 and K2 and each slab effective capacitance as the above
d combination then its dielectric constant K
having area A and thickness equal to as shown is given by :
2
in the figure Solution
0 A
A C1 K1 2 K C
K1 Let C 0 ; d 1 ;
d
2
K2
0 A K A
C2 K 2 2 K C C3 3 o , 2K 3 C
2 ; d
d
2 2
The equivalent circuit is as shown
A/2 A/2
A
d K1 K2
K1
2 d
A K3
d
K2 A
2 The equivalent circuit as shown
A
2K1 o A 2
A
2
a) Capacity of the upper half C1 d d
d K1 K2
2 2
2K 2 o A
b) Capacity of the lower half C2
d
c) C1 and C2 may be supposed to be conencted in K3
series. A
Page 67
ELECTRIC POTENTIAL & CAPACITANCE
1 1 1 1 1 1
, KC 2K C K K C A
C C3 C1 C 2 3 1 2
1 1 1 B
K K1 K 2 2K 3 Solution:
Illustration-65
If the plates are numbered as 1,2, 3 and 4 from
A parallel plate capacitor is constructed
top to bottom, plates 1 and 3 are at same
using three different dielectric materials
as shown in the figure. seperation between potential. So they have been joined together in
the plates of the capacitor is d.Find the an equivalent diagram. Similarly plates 2 and 4,
equivalent capacitance between the plates ? being at same potential, have been joined
together. Thus, the system comprises of 3
identical capacitors in parallel. Here again, each
K
3K
0 A
Plate-1
capacitor has capacitance , Then CAB =
Plate-2 d
2K
d/2 d/2 0 A
3C 3
Solution d
Let C1 , C2 and C3 be the capacitances of the
parts with dielect rics K, 2K and 3K If alternate numbered plates are joined, then n
respectively plates will form (n–1) capacitors in parallel.
C1
A 0
CAB = n 1
d
C3 1
d
2
C2 3 B
A A 4
Let C 0
d
n
0 A / 2 K KA
C1 0 KC
d/2 d Illustration-67
0 A / 2 2K 2 0 KA Four identical metal plates are located in
C2 2KC
d/2 d air at equal distance d from one another.
The area of each plate is A. Find the
0 A
C3 3K 6KC equivalent capacitance of the system
d/2 between X and Y.
C1 and C2 are in parallel and this combination
is in series with C3. 1
1 1 1
X
So, 2
C eff C1 C 2 C3
1 1 1 2 1 1 3
C eff 3KC 6KC 6KC 2KC Y
0 A 4
C eff 2KC 2K
d Solution
Illustration-66:
Find the equivalent capacitances between Let us give numbers to the four plates. Here X
points A and B for the following fig. and Y are connected to the positive and negative
Assume that each plate has surface area A terminals of the battery (say), then the charge
and the separation between the two
consecutive plates is d. distribution will be as shown
Page 68
ELECTRIC POTENTIAL & CAPACITANCE
1 Illustration-69
+ Find equivalent capacity X and Y
2 X
1
3 X
2
4 Y
3
Here the arrangement can be represented as
the grouping of three identical capacitors each Y
4
A Solution
of capacity 0 . The arrangement will be as
d Let us give numbers to the four plates. Here X
shown and Y are connected to the positive and negative
2 1
terminals of the battery (say).
34
1
(+) (-) X
++++++ 2
23 ++++++
Now the equivalent capacitance between X and 3
(C C)C 2C 2 0 A Y
Y is CXY ++++++ 4
CCC 3 3d
Here the arrangement can be represented as
Illustration-68 the grouping of three identical capacitors each
Find equivalent capacity between X and Y A
1 of capacity 0 . The arrangement will be as
d
2 shown
X 3 Y 2 1 4 3
4 X Y
+
Solution
2 3
Let us give numbers to the four plates. Here X Now the equivalent capacitance between X and
and Y are connected to the positive and negative C C C 3
terminals of the battery (say) , Y is CXY C C C
C C 2 2
1
++++++++
2 3 3 0 A
= CXY C
2 2 d
X Y
3
WHEATSTONE BRIDGE CIRCUITS
+ + + + + + + +4
A circuit with five capacitors as shown is known
Here the arrangement can be represented as as wheatstone bridge circuit
the grouping of two identical capacitors each A
A
of capacity 0 . The arrangement will be as
C1 C2
d
shown
2
C5
1 X Y
C3 C4
X Y
+
B
4 3 C C
Now the equivalent capacitance between X and a) If 1 = 3 in this circuit, it is said to be
C2 C 4
Y is balanced. In such a case, VA – VB = 0 So,
0 A capacitor C5 will have no charge and while
C XY (C C ) 2C 2
d
simplifying the circuit, we can ignore C5.
Page 69
ELECTRIC POTENTIAL & CAPACITANCE
X Y C3 C4
(6)
C3 C4
Effective capacity between X and Ywill be X Y
C
CC CC
CXY 1 2 3 4
C1 C2 C3 C 4
Based on balanced wheatstone bridge principle, For (1) and (2)
the following circuits can be simplified
C1 C 1C 2 C 3 C 4
C0
C5 C2 C1 C 2 C 3 C 4
(1) X C3 Y
For (3) and (4) CXY = C0 + C
C4 C1 C2
b) If in the wheatstone bridge, the
C5 C3 C 4
C1 potential difference across C5 is not zero and
X C2 the circuit is not balanced. Such circuits must
(2) C3 Y be solved by circuit analysis i.e., we use
kirchhoff’s laws.
C4 Some times unbalanced bridge circuits can be
C1 analysed with reverse symmetry method. Such
X C3 C5 circuit is given below
C1 C2
(3) C C4 C2
Y C5
C Y
X
C2 C1
C C C
Let q1 and q2 be the charges on capacitor C1
(4)
and C2 when a battery is connected between
C
X and Y the charge distribution will be as
Ceq = C shown
q1 q2
C C C C
q3
C C X q2 q1 Y
C
(5)
C C C
Ceq=C Ceq=C q = q1 + q2 V
Page 70
ELECTRIC POTENTIAL & CAPACITANCE
4C 4C C C
Solution C2
Except P and 3C, remaining part is wheatstones
bridge. Y
The effective capacity of Wheatstone bridge
part is 3C. 1 1 1
P, 3C and 3C are in series. C C1 C C2
1 1 1 1 On simplification, we get
P 3C
P 3C 3C C
C2 C2 4C1C2
Illustration-71 C
2
In the given circuit if the P.D. between M
and N is zero, the capacity C is Illustration-72
16F 16F 12F Find the capacity between x and y if
M C 1F .
C C C
12F X
6F N 4F 2C 2C
Y
24 V C C
Solution Solution
If p.d. between M and N is zero then the circuit C C
is similar to wheatstones bridge. A
X
P R 8 6
i.e., C 5 F 2C 2C
Q S 12 C 4
INFINITE LADDER NETWORKS Y
B
A circuit with capacitors connected in a C C
repeated manner upto infinity can be called Here CXY = CAB = C0
as infinite ladder network. An example for such Then the given circuit can be modified as
network is as given below C
C1 C1 X
A
X
C 2C C0
C2
C2
Y Y
B C
Page 71
ELECTRIC POTENTIAL & CAPACITANCE
1 1 1 1 1 1 1
U1 : U 2 : U 3 C1V 2 : C2V 2 : C3V 2
C0 1 2 C 0 1 2 2 2
C1 : C2 : C3
1 2 C0 1 2 C0
If a battery is used to charge a capacitor, work
C0 2 C0
done by the battery is W and energy stored in
2 C0 C0 (5 2C0 ) 5C 0 2C20
the capacitor is U. Let Q be the charge delivered
2C20 4C0 2 0 by the battery to the capacitor. Then work done
on simplification we can get by the battery is
C0 1 2 1
W = VQ and energy stored is U QV
Since capacity is positive, 2
C0 CXY
2 1 F. U W2
ENERGY STORED IN A CAPACITOR The remaining energy is W – U which is equal
When a capacitor is connected to the battery, 1
the battery transfers positive charge from to QV or U will be lost as heat energy and
2
negative plate to the positive plate. Some work
is done in transferring this charge, which is light energy
stored in the capacitor in the form of ELECTROSTATIC ENERGY DENSITY
Electrostatic Energy. It is the energy per unit volume inside an electric
field. The electric field inside a parallel plate
1 1 2 Q2
Energy Stored, U QV CV = capacitor is uniform and existing in a volume
2 2 2C (A × d). Therefore
Graph between U & V or U and Q
U(P.E) Total energy CV 2
Ud
Total volume 2 Ad
Page 72
ELECTRIC POTENTIAL & CAPACITANCE
potential is given by
1 8.4 10
3
1000 2 500 103 0.1 4200
C V ~ C2 V2
2 2 8.4 103
V 1 1 10 C 0
C1 C2
Illustration-74
Here charge flow takes place if V1 V2
Parallel plate capacitor of capacitance ‘C’
In this case, the loss of energy is charged to a potential difference ‘V’.
1 C1C2 Another capacitor of capacitance ‘2C’ is
U (V1 V2 )2 charged to a potential difference 2V. The
2 C1 C2
two capacitors are connected in parallel
Redistribution of charges when two to each other in such a way that the
conductors are connected by conduting positive terminal of one is connected to
wire : negative terminal of the other. The final
Q2 energy of the configuration is
Q1 Solution
1 2Q Q
Common potential VC C C
C1 A B C2 1 2
CV 2C 2V
= =V
3C
V1 V2 1 1
C1V1 C2 V2 r1V1 r2 V2 Final energy = C1 C2 VC2 3C V 2
V 2 2
C1 C2 r1 r2
3
Effective capacity, Uf = CV2
2
Q Q11 Q12 Q11 Q12 Illustration-75
C C1 C2
V V V V A parallel plate capacitor of plate area A
Loss in energy of the system and separation ‘d’ is charged to a P.D (V)
and then battery is disconnected. A slab of
1 1 1 dielectric constant (K) is inserted to fill the
U C1V12 C2 V22 CV 2
2 2 2 space between the plates. The work done
on the system in the process of inserting
1 C1C2 the slab is
= V1 V2 2
2 C1 C2 Solution
1
1 4 0 r1r2 Initial enrgy of the system U I CV2
V1 V2 2 2
2 r1 r2 After inserting slab, C1 = KC
since charge is constant
If V1 = V2, then neither the charge flows nor the
energy is lost.
Page 73
ELECTRIC POTENTIAL & CAPACITANCE
V C1 C2
Q1 Q; KCV CV; V1 S1 S3 S2
K
2 V1 = 100 V V2 = 20 V
1 V 1 1
Final PE U f KC CV 2
2 K K 2
Work done W = Ui – Uf Solution
When S1,S2 and S3 are closed, both the
1 2 1
W CV 1 capacitors are in parallel with unlike charged
2 K
plates together. So, they attain a common
1 0 A.V 2 1
W 1 k potential.
2 d
Before closing the switches,
Illustration-76
Charge on C1 is q1 100 1 100 C
A capacitor of capacitance Co is charged Charge on C2 is q 2 20 2 40 C
to a potential Vo and then isolated. A small After closing the switches
capacitor C is then charged from C o ,
discharged and charged again, the process Common potential
being repeated n times. Due to this,
potential of the large capacitor is q q 100 40
V 1 2 20 V
decreased to V. Find the capacitance of C1 C 2 3
the small capacitor: Now final charges q11 C1V 1 20 20 C
Solution q12 C2 V 2 20 40 C
When key is closed, common potential The charge that flows through S1 is
q 100 20 80 C
C V
V1 o o charge left on large capacitor after Illustration-78
Co C
first sharing of charges Q1o Co V1 A charge ‘Q’ is given to spherical shell of
common potential after second sharing of radius R. The energy of the system is
C0 Solution
charges in V2 C C V1
0
1 Q2 Q2 Q2
C2o V0 U CV 2
V2 = 2 2C 2 4 0 R 8 0 R
( Co + C)2
n
This is same as self energy of charged con-
Co ducting sphere.
after nth sharing charges Vn = V0
C + C
o Let n identical charged liquid drops each
But Vn V having radius ‘r’ , charge ‘q’ , potential ‘v’
n
C0 and capacity ‘c’ are combined to form a
V V0 big drop.
C 0 C
For big drop
V 1/ n
C Co o 1 (i) Radius R n1/3r
V (ii) Charge Q = nq
Illustration-77 (iii) Capacitance C n1/ 3c
In the circuit shown in figure C1 1 F (iv) Charge density n1/ 30 where 0
and C2 2 F . The capacitor C1 is charged
to 100V and the capacitor C2 is charged to is charge density of small drop
20V. After charging then are connected as (v) Potential V n 2 /3v
shown. When the switches S1 , S2 and S3 are (vi) Potential Energy U n5/3u
closed, the charge flowing through S1 is
EFFECT OF DIELECTRIC
When a dielectric (K) is inserted between
Page 74
ELECTRIC POTENTIAL & CAPACITANCE
the plates of a parallel plate capacitor 2
connected to a voltage source V Q2 Q
U = 2 KC
(i) Potential difference across the capacitor 2C 0
remains same
When the plate separation between the
(ii) Capacitance becomes K-times as
capacitor plates is made n times:
A0 K Charged and source is Charged and source is
C = K C0 where C0 is capacitance
d not disconnected disconnected
with out dielectric V Same Q Same
(iii) Charge on capacitor becomes K times as
C becomes 1 times
Q = CV = K C0V = K Q0 where Q0 is n
A 0
C becomes 1 times
n
charge on capacitor with out dielectric as C
D
(iv) Electric field between the plates of capacitor
Q becomes 1 times V becomes n times
will remain same as E V n Q
as V
d as Q CV C
(v) Charge density will becomes K-times as becomes 1 times remains same
n Q
Q K Q
0 Q as s
as s A
A A A
(vi) Potential Energy U becomes K times as E becomes 1 times E remains same
n V
1 1 2 V as E
U CV 2 KC0V KU as E
d
d
2 2
(vii) Additional charge passing through battery U becomes 1 times U becomes n times
n Q2
is Q Q Q0 = Q0 K 1 1
asU CV
2 asU
2C
2
(viii) Additional work done by battery Illustration-79:
W Q0V K 1 A capacitor is composed of two plates
separated by a sheet of insulating material
When a capacitor, charged and then 3mm thick and of relative permittivity 4. The
disconnected from the source distance between the plates is increased to
and a Dielectric (K) is inserted between
allow the insertion of second sheet 5 mm
the plates of a capacitors
thick and of relative permittivity r . If the
(i) Charge (Q) on the capacitor will remain capacitance of the capacitor so formed is 1/
same. 3 of the original capacitance, find r .
(ii) Capacitance will become K times as, Solution :
A0 K Let initial capacitance final capacitance be C
C = KC0 and C/3 respectively.
d
(iii) Potential difference across the capacitor 0 A
Then, C ..... (i)
becomes 1/K times (3 / 4)
V = Q/C = Q /KC0 C 0 A
..... (ii)
1 V 3 (3/ 4) (5/ r )
(iv) Electric field will become times as E
K d 3 3 5
(v) Charge density on the plates will remain same Solving (i) & (ii), we get 3
4 4 r
Q
as r 3.33
A
1 Illustration-80:
(vi) Potential Energy U becomes times as The parallel plates of a capacitor have an
K
area 0.2 m 2 and are 10-2 m apart. The
original potential difference between them
Page 75
ELECTRIC POTENTIAL & CAPACITANCE
is 3000V, and it decreases to 1000 V when which is unchanged.
a sheet of dielectric is inserted between the
plates. Compute r2 q1 r2 q1
(a) Original capacitance C0
(b) The original charge Q on each plate r1 r1
(c) Capacitance C after insertion of the
dielectric Now electric potential of inner sphere is
(d) Dielectric constant K
(e) The original field E0 between the plates 1 q1 0 1 q1
V1
and 4 0 r1 r2 4 0 r1
(f) The electric field E after insertion of
Electric potential of outer shell is
the dielectric. ( 0 = 8.85 × 10–12 S.I. unit)
Solution : 1 q1 0 1 q1
V2
4 0 r2 r2 4 0 r2
8.85 10 12 0.2
(a) C 0 0 A F Now Potential difference between the two
d 0.01
(b) Q C0 V 1.77 3000C = 1.77 × 10–10 F q1 1 1
(c) As Q remains constant, So C0V0 = CV conductors is V1 V2 4 r r
0 1 2
C0 V0 1.77 10 10 1000
C From this we can conclude that potential
V 3000
11
= 5.90 10 F difference in this case depends on q1 only. It
does not depend on any charge on the outer
C V0 3000
(d) K =3 shell. Let us check the same as given below. If
C 0 V 1000
q2 is the charge on the outer shell, then
(e) E 0 V0 3000 3 10 5 V / m 1 q1 q2
d 0.01 V1
4 0 r1 r2
V 1000 1 q1 q2
(f) E 10 5 V / m
d 0.01 and V2 4 r
0 2
VAN DE GRAAFF GENERATOR:
Van de Graff generator is used to develop very q1 1 1
Now V1 V2 4 r r
high charges and intense electric field or very 0 1 2
high voltages of order of a few million volts. Here the potential difference ( V1 V2 ) will
The resulting large electric fields are used to remain the same for any value of q2.
accelerate charged particles like electrons, If the two spheres are connected as shown in
protons and ions to high energies. figure (b), the charge flows from higher potential
Principle:
to lower potential. Ultimately all the charge
Whenever a charge is given to a metal body it
resides on the surface of outer sphere.
will spread on the outer surface of it. If we put
a charged metal body inside the hollow metal Force Between Plates of a Capacitor
body and the two are connected by a wire, +Q -Q
Page 76
ELECTRIC POTENTIAL & CAPACITANCE
be earthed. q 3
-q3
dr
q
b At junction O, q 0
a q 1 + q2 + q3 = 0
The dotted line forms isolated system so in that
case also q1 + q2 + q3 = 0
In order to calculate the capacitance let us (Similary for currents i 0 from junction rule)
consider a coaxial cylindrical shell of thickness At a node total charge entering = total charge
dr. The surface of the imaginary cylinder is coming out
Gaussian surface. The electric flux emanating q1 q5
from charged cylinder will pass through the
q q1 + q2 + q3 = q4 + q5
E.ds Loop law :
0K
Where K is relative permittivity of the medium In a closed loop, algebraic sum of the potential
between the two cylindrical surfaces. differences across all circuit elements must be
q 1 equal to zero.
q
E 2 rL E
0K ; 2 0 KL r
p.d 0
V
q b
log e E iR 0 for resistors and batteries
2 0 KL a q
E C 0 for capacitors and batteries
Page 77
ELECTRIC POTENTIAL & CAPACITANCE
q 1µF 2µF
E C iR 0 for all the three a
C1 10V C2 b
The junction rule is an application of principle
of conservation of charge, where as loop rule Solution
is a consequence of conservation of energy. Va – Vb = 10 V
While applying loop rule, potential gain or Let the charge on each capacitor be ‘q’ (in µC)
potential fall will be calculated as mentioned q q
Va 6
10 Vb
below. 1 10 2 10 6
R +q q 1
Va Vb 10 q 6
2 10
a b a b a b
I q
q 20 10 ; q
6 40
C
Fig(b) Fall in potential 2 3
Fall in potential Po tent ial difference acro ss 1 µF is
q 40
V1 V
q C1 3
Va – Vb = + iR or or E
C Potential difference across 2µF is
R +q q 20
V2 V
a b C2 3
a b a b
I E
R +q Illustration-82
< Two capacitors A and B with capacities
a i b a b a b
E 3µF and 2 F are charged to a potential
Fig (c) Potential rise difference of 100V and 180V respectively.
q The plates of the capacitors are connected
Va – Vb = – iR or or –E with the given charge polarities shown.
C An uncharged capacitor of capacity 2µF
q
In a closed loop E 0 for eg. is connected by closing the switches.
c Calculate the final charge on the three
E1 E2 E3 capacitors
A B C
C
q1 q2 2F
C1 C2 C3
3F 2 F
A B+
F E D
In the circuit shown, q1 & q2 are the Solution
assumed charges. Then, the equation for the Let q be the charge flowing during redistribution
loop & for the loop. of charge in the circuit. From Kirchhoff’s
voltage law we can write
q1 q1 q 2
AFEBA, E1 E2 0 300 q q 360 q
C1 C2 0
3 2 2
+q -q
q 2 q1 q 2
BCDEB, –E 3 0
C3 C2 (300 q)
Illustration-81
(360 q)
In the given branch, if potential of a is 10V
higher than b, find the potential of each 4q
280 q 210C
capacitor and charge on each capacitor ? 3
Final charge on the capacitors are 90c,210c
and 150c
Page 78
ELECTRIC POTENTIAL & CAPACITANCE
the conductor, the air near these points break the battery ie H = W
down and conductor starts discharging. This is If Uf > Ui then H = (Ui – Uf) + W
known as corona discharge. W = (Uf – Ui) + H
HEAT GENERATED IN CIRCUITS Here (Uf – Ui) = H
For circuits involving capacitors and voltage W
sources, we can apply conservation of total i.e., H or (Uf – Ui) =
2
energy. The basic equation for conservation of SELF ENERGY OF A CHARGED
energy will be Ui + W = Uf + H SPHERICAL SHELL SPHERE:
Ui = Initial energy stored in all capacitors of Consider a spherical shell having charge Q and
the circuit radius R. Let the instantaneous charge on the
Uf = Final energy stored in all capacitors of the shell be q. Work done by an external agent in
circuit slowly bringing a charge from infinity and as-
W = Work done by the sources / batteries sembling on the surface of the shell is
H = Heat generated Transfer
Here W can be positive or negative R dq
Case (i) :
If battery or source supplies charge q, work q0
done will be positive
W = E q ( >0) q
E q dW Vdq dq
If a capacitor of capacitance C charged to a 4 0 R
potential V (V 0) is connected to a source of Net work done in charging the shell is
emf E work done will be positive if E >V. Q
q Q2
Work done W = E q W dq
0
4 0 R 8 0 R
Where q = (qfinal – q initial)
W = E ( CE – CV) = CE (E – V) This work done is the electrical potential
energy of self energy of the charged sphere.
Case (ii) :
If battery / source gets charge, work done by it SELF ENERGY OF A UNIFORMLY
will be negative CHARGED SPHERE:
W = E q ( < 0) Let us consider that the charge Q being brought
from infinity to form the sphere of radius R.
Let at any time, charge q has already been
E q brought to from a sphere of radius x and fur-
If a capacitor of capacitance C charged to ther a very small chrge dq is brought from in-
potential V is connected to a source of emf E finity and assembled on the surface of sphere,
which increase its radius by dx. Electrical po-
such that E < V1 work done by the battery will
tential energy of this charge dq is given by
be negative.
dU dq potential at the position of charge
Work done W = E q
dq
= where q = qfinal – q initial
or W = E ( CE – CV) = – CE ( V – E) 1 q
dU dq V dq
Case (iii) : 4 0 x
If Ui = Uf, Heat generated = work done by If Q is the total charge to be brought to form
Page 80
ELECTRIC POTENTIAL & CAPACITANCE
1 q4 3 4 2 4
dU
4 0 x 3
x
4 x 2
dx
3 0
x dx
+ +a b
q1 + q2
+ ++
+ q0
+ ++
l
(1) (2)
Page 81
Page 82
CURRENT ELECTRICITY
CURRENT ELECTRICITY
Where f is the frequency of revolution in Hertz.
SYNOPSIS is the angular frequency in rad/sec
v is linear velocity of the charge q
Strength of Electric Current r is radius of the circular path
The strength of electric current is defined as 4. If in a discharge tube n1 protons are moving
the rate of the flow of charge through any cross from left to right in t seconds and n2 electrons
section of a conductor. are moving simultaneously from right to left in t
The instantaneous current is defined by the seconds, then the net current in any cross
equation, section of the discharge tube is
Q dQ ( n n )e
I Lt I 1 2 (from left to right)
t 0 t dt t
q here e is the magnitude of charge of electron
Average current i (or) proton.
t
Ampere : If one coulomb of charge passes Current Density ( J ) : Current density at a
through a cross-section of the conductor per point is defined as a vector having magnitude
second then the current is one ampere.
equal to current per unit area.
1coulomb
1 ampere = 1sec ond I dI
J Lt nˆ
s 0 s ds
current is a scalar quantity.
Applications on electric current If the normal to the area makes an angle
1. If the current is varying with time t, then the with the direction of the current, then the current
charge flowing in a time interval from t1 to t 2 dI
is density is J , dI Jds cos
ds cos
t2 (or)
q Idt
t1 dI J .ds i.e., I J .ds
2. If n particles, each having a charge q, pass
SI unit of J is Am–2
through a given cross sectional area in time t,
nq Dimensional formula of J is [ AL2 ]
then average current is i = Current is the flux of current density.
t
3(i). If a point charge q is revolving in a circular orbit Relaxation time ( ) : It is the time interval
of radius r with speed v then its time period is between two successive collisions of electrons
with +ve ions in the metallic lattice. The
resistance of a conductor is given by
ml
R=
ne 2 A
T (2 r / v) where n = no. of electrons per unit volume of
conductor
e = magnitude of electron charge
(ii).The average current associated with this m = mass of electron ; = relaxation time.
revolving charge is DRIFT VELOCITY : Drift velocity is the
q vq average velocity acquired by free electrons
I fq q inside a metal by the application of an electric
T 2 2 r
field which results in current.
Page 83
CURRENT ELECTRICITY
J I Illustration-3:
Drift velocity v d Consider a wire of length 4m and cross-
ne Ane
sectional area 1 mm 2 carrying a current of
where, J = I/A is current density
2A. If each cubic meter of the material
n is number of free electrons per unit volume contains 10 29 free electrons, find the
e is charge of electron average time taken by an electron to cross
The drift velocity(vd) is related to relaxation time the length of the wire.
eE Solution:
( ) is vd i 2
m vd 29 6 19
ms 1
nAe 10 10 1.6 10
Note : 1.The drift velocity of electrons is of the order = 12.5 × 10–4 ms–1
of 104 ms 1 . Average time taken by an electron to cross the
2. Greater the electric field, greater will be the drift length of wire
velocity v d E l 4
t 4
3. The direction of drift velocity for electrons in a v d 1.25 104 s = 3.2 × 10 s
metal is opposite to that of electric field applied
Mobility ( ) : Mobility ( ) of a charge carrier
E (like electron) is defined as the average drift
Illustration -1: velocity resulting from the application of unit
In a Hydrogen atom, electron moves in an electric field strength.
-11
orbit of radius 5 × 10 m with a speed of
drift velocity |V |
2.2 × 106 m/s. Calculate the equivalent ; d
current. electric field E
v Mobility depends on pressure and temperature.
Solution: Current i f .e .e
OHM’S LAW : For a given conductor, at a
2 r
given temperature, the strength of electric
2.2 106 –19 current through it is directly proportional to
= 11 × 1.6 × 10
2 5 10 the potential difference applied across at its
= 1.12 × 10–3 amp = 1.12 mA. ends.
Illustration -2: V
The current through a wire depends on i.e. I V I ; V = IR
R
time as i i0 t , where i0 10 A and Where R is electrical resistance of the
conductor
4 A / s . Find the charge that crossed
through a section of the wire in 10 Ohm’s law is neither a basic law nor a derivable
seconds. one
Solution: Ohm’s law is just an empirical relation.
Microscopically, Ohm’s law is expressed as
dq
i i0 t ; but i J nev d J E where is the
dt
dq (i0 t )dt electrical conductivity of the material.
The conductors which obey Ohm’s law are
t 10 10
2
t called Ohmic conductors or ohmic devices.
q dq q i0 t
t 0 2 0 Ex : all metals
For Ohmic conductors V – i graph is a straight
= (10i0 50 ) = 300 coulomb line passing through origin (metals).
Page 84
CURRENT ELECTRICITY
T1 Factors Effecting Resistance of a
1
V V Conductor
T2
2 1. The resistance of a conductor is directly
proportional to the length (l) of the conductor
(when area of cross section is constant)
i i R1 l1
i.e. R l (or) R l
2 2
(A) Slope of the line (B) Here tan 1 tan 2
tan V / I R ; So R1 R2 i.e T1 T2 R l
For small changes in the length,
The substances which do not obey Ohm’s law R l
are called non-ohmic conductors or non-ohmic 2. The resistance of a conductor is inversely
devices. proportional to the area of cross-section (A)
Ex: Thermistor, Electronic Valve, Semi- (When length of the conductor is constant)
conductor devices, gases, crystal rectifier etc.,
The V – i graph for a non – Ohmic conductor 1 1 R1 A2 r22
is non-linear. i.e., R (or) R ; R A r2
A r2 2 1 1
(mA)
i For small changes in area (or) radius we have
i 1
X V X R A 2r
0
V R A r
(A)
i
V 3. As the temperature increases resistance of
vaccum tube semi conductors
metallic conductors increases and that of
i i semiconductors decreases.
i2 Conductance: The reciprocal of resistance
V 0 (R) is called conductance.
i1 V
i 1
V1
V
Ne Gas conductance, G .
Thermistor
With tungsten Electrode R
i The S.I unit of conductance is mho or
Siemen or ohm-1.
V 0
V Resistivity: As we know, that the resistance
i of the conductor is directly proportional to its
dilH2SO4 length and inversely proportional to its area of
(Platinum Electrode) cross section, we can write
Resistance-Definiton : The resistance of a
conductor is defined as the ratio of the potential l l
R R
difference ‘V’ across the conductor to the A A
current ‘i’ flowing through the conductor. where is specific resistance or resistivity of
V the material of the conductor.
Resistance R
i 1. Resistivity is the specific property of a material
S.I unit of resistance is ‘ohm’ . but resistance is the bulk property of a
The resistance of a conductor depends upon conductor.
1) shape (dimensions) 2) nature of material 2. Resistivity is independent of dimensions of the
3) impurities 4) Temperature conductor such as length, area of the cross
The resistance of a conductor increases with section.
impurities.
The resistance of a semi conductor decreases 3. Resistivity depends on the nature of the material
with impurities. of the conductor, temperature and impurities.
Page 85
CURRENT ELECTRICITY
4. Resistivity of any alloy is more than resistivity 5. If a wire of resistance R is stretched to ‘n’ times
of its constituent elements. its original length, its resistance becomes n2 R.
i) R alloys R conductors ii) metals alloys 6. If a wire of resistance R is stretched until its
Special Cases : 1
radius becomes th of its original radius then
1. The alternate forms of resistance is n
l2 l 2 d V m its resistance becomes n4R.
R 2 7. When a wire is stretched to increase its length
V m A d A2
by x% (where x is very small) its resistance
Where d is density of material of conductor increases by 2x %.
V is volume of the conductor
m is mass of the conductor 8. When a wire is stretched to increase its length
2. If a conductor is stretched or elongated or by x% (where x is large) its resistance increases
drawn or twisted, then the volume of the 2
conductor is constant. Hence 2x x
by 100 .
l 2
a. R R l 2 9. When a wire is stretched to reduce its radius
V
byx% (where x is very small), its resistance
V 1 1 increases by 4x%.
b. R 2 R 2 4
A A r Illustration -4:
l2 A rectangular block has dimensions 5 cm
c. Interms of mass of the wire R × 5 cm × 10 cm. Calculate the resistance
m
measured between (a) two square ends and
m m (b) the opposite rectangular ends. Specific
and R 2 4
A r resistance of the material is 3.5 105 m .
3. For small changes in the length or radius during Solution:
the stretching a) Resistance between two square ends
R l R A r
2 ; 2 4 R1
R l R A r A
4. In case of a cuboid of dimensions l b h 3.5 10 5 10 10 2
R1 1.4 103
5 5 10 4
5 cm
5 cm 10 cm
b) Resistance between the opposite rectangular
l
Resistance across AB, RAB ends R2
b h A
b 3.5 10 5 5 10 2
Resistance across CD, RCD R2 1.4 10 4
lh 5 10 10 4
Conductivity: Conductivity is the measure
h of the ability of a material to conduct electric
Resistance across EF, REF
l b current through it. It is reciprocal of resistivity.
If l b h , then 1 l
2
l h R l RA
Rmax ; Rmin max 2
bh l b Rmin h S.I unit : Siemen / m : (Sm–1)
Page 86
CURRENT ELECTRICITY
For perfect insulators 0 Temp. coefficient Variation of
Material of resistance with
For perfect conductors, is infinity.. temperature rise
resistance ()
Temperature dependence of resistance:
Metals Positive Increases
For conductors i.e metals resistance increases
Solid non-
with rise in temperature Zero independent
metal
R t R o (1 t t 2 ) for t > 300oC Semi- Negative Decreases
conductor
o
R t R o (1 t) for t < 300 C or Electrolyte Negative Decreases
R Ro Ionized
t 0 Negative Decreases
R ot / C gases
temperatures. for
sem
1 dR ico
n
At a given temperature at t0C duc
to
R dt rs
X
Graph shows the variation of resistivity with 0
Temperature
temperature for conductors, semiconductors Illustration -5:
and for alloys like manganin and constantan.
The temperature coefficient of resistance
Since the resistivity of manganin and
of platinum is 3.92 103 K 1 at 0° C.
constantan remains constant with respect to Find the temperature at which the increase
change in temperature, these materials are used in the resistance of platinum wire is 10%
for the bridge wires and resistance coils. of its value at 00C.
The resistivity of manganin and constantan is 110R1
almost independent of temperature. Solution: R2 1.1R1 ; 3.92 103 K 1
100
Two resistors having resistances R1 and R 2 R1 1.1 R1 R1
R2 at 0o C are connected in series. The t
R1 R1
condition for the effective resistance in
series is same at all temperatures
R1 (1.1 1) 0.1R1
0.1
R1 R1 3.92 10 3
R1 R2 R1' R2'
t 25.510 C ;
R1 R2 R1 (1 1t ) R2 (1 2t )
t2 t1 25.510 C t2 0 25.510 C
R11 R2 2
Variation of resistance of some materials t2 25.510 C
Page 87
CURRENT ELECTRICITY
Page 88
CURRENT ELECTRICITY
Illustration -11: Illustration -13:
A hollow cylinder of specific resistance A hollow copper cylinder is of inner radius
, inner radius R, outer radius 2R and 4cm and outer radius 5cm. Now hollow
length l is as shown in figure. What is the portion is completely filled with suitable
net resistance between the inner and copper wires. Find percentage change in
outer surfaces ? its electric resistance.
Solution : Solution :
log r R
2R
2 l
l n (2)
R (2 rl ) 2 l After filling the hollow portion, its final resistance
Illustration-12:
There are two concentric spheres of radius is ,
a and b respectively. If the space between l l k
R2 = 5 25 25
2
them is filled with medium of resistivity
, then the resistance of the inter gap
between the two spheres will be (Assume b Percentage of change = R2 R1 100
> a) R1
Solution: k k
25 9 100 64%
k
9
a
Illustration -14:
x If resistivity of the material of a
O dx
conductor of uniform area of cross-
b section varies along its length as
0 1 x .Find the resistance of the
Consider a concentric spherical shell of radius conductor , if its lengths is ‘L’ and area
x and thickness dx, its resistance is dR, of cross-section is ‘A’
dx Solutions :
dR = dx dx
L
4 x 2
dR = 0 1 x ; R dR
Total resistance A A 0
b b L
dx 1 1 x2 1
R = dR R 4 x 2 4 a b R 0 x = 0 L L2
a a A 2 0 A 2
Page 89
CURRENT ELECTRICITY
40
10 6
22.510 2
N
Solution:
N 800 Yellow Gold
6
110 Brown Green
Thermistor: A thermistor is a heat sensitive
and non-ohmic device.
This is made of semiconductor compounds as
oxides of Ni, Fe, Co etc. From Figure
This will have high +ve (or) -ve temperature Brown Yellow Green Gold
coefficient of resistance. 1 4 ×105 5%
Thermistor with -ve ‘ ’ are used as
resistance 5
R 14 105
100
14 105
themometers which can measure low
temperature of order of 10K –3and small (1.4 0.07)10 6 (1.4 0.07)M
changes of in the order of 10 K.
Some times tolerance is missing from the code
Having -ve , these are widely used in
measuring the rate of energy flow in micro and there are only three bands. Then the
wave beam. tolerance is 20%.
Thermistor can also be used to serve as Super Conductor : There are certain
thermostat. metals for which the resistance suddenly falls
Resistor Colour codes to zero below certain temperature called
Colour Number Multiplier Tolerance(%) critical temperature.
Critical temperature depends on the nature of
Black 0 × 10° material. The materials in this state are called
Brown 1 × 101 super conductors.
Without any applied emf , steady current can
Red 2 × 102
be maintained in super conductors.
Orange 3 × 103 Ex: Hg below 4.2 K or Pb below 8.2K
Yellow 4 × 104 Resistances In Series:
A B C D
Green 5 × 105
R1 R2 R3
Blue 6 × 106
I I
Violet 7 × 107
Gray 8 × 10 8
V
White 9 × 10 9 – 1. If resistors of resistances R 1, R 2, R3, ..... are
connected in series, the resultant resistance
Gold – ×10–1 5% R = R1 + R 2+ R3 + ........
Silver – ×10–2 10% 2. When resistances are connected in series, same
No color – 20% current passes through each resistor. But the
digit 2 tolerence
digit 1 multiplier Page 90
CURRENT ELECTRICITY
pot ential differences are in the ratio 2. In ‘n’ wires of equal resistances are given, the
V1 : V2 : V3 ..... = R1 : R2 : R3 ..... number of combinations that can be made to
3. When resistors are joined in series, the effective give different resistances is 2n –1 .
resistance is greater than the greatest resistance 3. If ‘n’ wires of unequal resistances are given,
in the circuit. the number of combinations that can be made
4. When two resistances are connected in series to give different resistances is 2n (If n >2).
then voltage division laws are, 4. If R s and R p be the resultant resistances of
VR1 VR2 R1 and R2 when connected in series and parallel
V1 and V2 then
R1 R2 R1 R2
1
Resistances in Parallel
R1 R s R s2 4R s R p
2
I1 R1
1
R 2 R s R s2 4R s R p
2
I2 R2 5. If a uniform wire of resistance R is,
A stretched to ‘m’ times its initial length and
bent into a regular polygon of ‘n’ sides then
I3 R3 a) Resistance of the wire after stretching is
I R1 m 2 R ( R ' l 2 )
I m2 R
V b) Resistance of each side R2
n
c) Resistance across diagonally opposite points
1. If resistors of resistance R1, R2, R 3 ……..are
connected in parallel, the resultant resistance R n
2 R2 m2 R
1 1 1 1 R0 R0
is given by R R R R ............ 2 4
1 2 3
2. If resistances R1 and R2 are connected in d) Resistance across one side
R 1R 2 (n 1) (n 1)m 2 R
parallel, the resultant resistance. R R R R3 R2
1 2 n n2
6. 12 wires each of resistance ‘r’ are connected
3. When resistors are joined in parallel the potential to form a cube. Effective resistance across
difference across each resistor is same. But the 5r
currents are in the ratio i1 : i2 : i3:............. a) Diagonally opposite corners = .
6
1 1 1 3r
=R R : : : ............ b) face diagonal .
1 2 R3 4
4. When two resistances are parallel then current 7r
division laws are c) two adjacent corners .
12
IR2 IR1 7. If two wires of resistivities 1 and 2 , lengths
I1 and I2
R1 R2 R1 R2 l1 and l2 are connected in series, the equivalent
5. When resistors are joined in parallel, the 1 l1 2 l 2
effective resistance is less than the least resistivity l l .
resistance in the circuit. 1 2
Page 91
CURRENT ELECTRICITY
2
and conductivity 1
X R2 X
.
2
9. If ‘n’ wires each of resistance ‘R’ are
B D
connected to form a closed polygon,
equivalent resistance across two adjacent Illustration -18:
n 1 For a circuit shown in fig. Find the value
corners is Reff R of resistance R2 and current I2 flowing
n
INFINITE LADDER NETWORK through R2
Some networks make a ladder (or a grid) and
extend to infinity. To reduce such networks
we use the following steps. R1 10
Step-1: Let as assume the total resistance of
the infinite network to be X (say).
Step-2: now just retain one basic repetitive unit Solution :
and we observe the remaining circuit to be the If equivalent resistance of parallel
same as the original circuit. So, resistance of combination of R1 and R2 is R, then
this left out circuit must be X. RR 10R 2
Step-3: Now the equivalent circuit, is the R 1 2
R1 R 2 10 R 2
combination of basic unit and original repetitive
circuit of resistance X, such that the net V
According to Ohm’s law, R
resistance of the entire circuit is X I
Illustration -17: 50 10R 2
R 5 5 R 10 .
Find effective resistance between points 10 10 R2 2
A and B of an infinite chain of resistors The current is equally divided into R1 and R2.
joined as shown.
R R R Hence I2 = 5A.
A
1 C 1 1
R2 R2 Illustration -19:
R2 To inifinity
Two wires of equal diameters of resistivities
B and and length x and x respectively
D 1 2 1 2
Solution :
Suppose the effective resistance between are joined in series. Find the equivalent
A and B is X. Applying the steps discussed, resistivity of the combination.
we get Solution :
R2 X
X R1 R2 || X R2 Resistance, R1 11 ;R2 2 2
R2 X A1 A2
X 2 R1 X R1 R2 0 1 x1 , 2 x2
1 As the wires are of equal diameters A1 A2 A .
X
2
R1 R1
2
4 R1 R2
Page 92
CURRENT ELECTRICITY
x1 x x
R1 , R2 2 ; R i' 100
A A A A
where x = x1 + x2; R = R1 + R2
x 1x1 2 x2 50V
; x 1x1 2 x2
A A A
(x1 x2 ) 1x1 2 x2 [ x x1 x2 ]
1 x1 2 x2 300
1.5 V
x1 x2
Illustration -20. When S1 and S2 are closed, total current is
Equivalent resistance between the points 1.5
i
A and B (in ) 100 R
300
100 R
1 1 4 1 1
A B
1.5[100 R ]
i ,
Solution : 400 R 30, 000
1 1 4 1 1 ammeter reading is
A Y Y B
R
i || i .........(2)
1 R 100
Parallel,
3 According to problem i1 i11
1
1 1 1
Solving for R, we get R 600 .
A X Y Illustration -22:
1
Find the equivalent resistance of the
1 7 1 network in Fig. between points (i) A and
Req 1 1 2
3 3 3 B and (ii) A and C.
Illustration -21: 10
In the given circuit Ammeter reading is A B C
same when both switches S1, S2 are closed
or opened. The value of resistance R is 30 7.5
100
A
S1 S2
50 Solution:
R
(i)The 10 and 30 resistors are connected
in parallel between points A and B. The
equivalent resistance between A and B is
300 1.5V
Solution : 10 30
R1 ohm = 7.5
When S1 and S2 are opened, ammeter reading 10 30
is, (ii) The resistance R1 is connected in series with
1.5 resistor of 7.5 , hence the equivalent resistance
i1 ..... (1) between points A and C is, R2=(R1+7.5) ohm
450
= (7.5+7.5) ohm = 15 .
Page 93
CURRENT ELECTRICITY
I=2.7A
8 12V, 0.5 1 1
A 9 B
I 1 1 1
Solution :
Solution : First calculate net resistance of infinite network
Between points A and B resistors of 4 , 6 1
and 8 resistances are in series and these are
in parallel to 9 resistor..
Equivalent resistance of series combination is X 1 x
R1 = (4 + 6 + 8 ) ohm = 18
If equivalent resistance between A and B is
R = 9 × 18 / (9 + 18) ohm = 6 1
Potential difference between A and B is x
x=2+ ; 2
x+1 x 2x 2 0
;
V = IR = 2.7 × 6V = 16.2V
Current through 9 resistor = 16.2/9=1.8A on solving, x 1 3 2.73
Current through 4 ,6 and 8 resistors = Total resistance = 2.73 + 0.5 = 3.23 Ω
2.7 – 1.8 = 0.9A.
12
Illustration -24: I= =3.73A
P and Q are two points on a uniform ring 3.23
of resistance R. The equivalent resistance JOULE’S LAW: According to Joule’s law,
between P and Q is the current passing through a conductor
P produces heat.
W = vit
Now, work done, W = (iR)it
O
Q 2
W = i2Rt = v t = vit
Solution : R
Resistance of section PSQ This work is converted into energy in the
P S conductor.
Thermal energy produced, Q = i2 Rt (joules)
Q
i 2 Rt
or Q = (cal)
O 4.2
As H i 2 , heating effect of current is common
T to both A.C and D.C.
R R Joule’s effect is irreversible.
R1 . r ; Resistance of section PTQ
2 r 2 Electrical Energy:
Rr 2 The electric energy consumed in a circuit is
R2 ; R 2 defined as the total workdone in maintaining
2 r R2 the current in an electric circuit for a given time.
2
As R1 and R2 are in parallel
V2t
R1 R2 R Electrical Energy = Vit = Pt = i2Rt
Req 2 2 S.I. unit of electric energy is joule
R
So, R1 R2 4
Page 94
CURRENT ELECTRICITY
1 K.W.H. = 36 × 105 J
2 r 4 sd mp r J
Electrical Power: t ;t r4
2
The rate at which work is done in maintaining I
the current in electric circuit is electrical power
i.e., in absence of radiation looses, the time in
W V2 which fuse will melt is also independent on length
P Vi i2R watt (or) joule / sec and varies with radius as r4.
t R
Heat energy produced due to the electric Illustration -26:
A fuse wire with radius of 0.2mm blows
W Pt Vit i2Rt V2t
current H = off with a current of 5 Amp. The fuse wire
J J J J RJ of same material, but of radius 0.3mm will
H mst blow off with a current of
where J is mechanical equivalent of heat. 3 5 3
Consumption of Electrical Energy: 1) 5 Amp 2) Amp
Units of electrical energy consumed by an 2 2
electrical appliance = 27
Number of watts× Number of hours 3) 5 Amp 4) 5 Amp
8 3 3
1000 Solution: i1 r1 2 0.2 2
It is in KWH. i2 r 3 ; i r 0.3
Fuse wire: 2 2
A fuse wire generally prepared from tin - lead 27
alloy (63% tin + 37% lead). It should have i2 5 Amp
high resistivity, low melting point. 8
Let R be the resistance of fuse wire.
L Illustration -27:
We know that R 2 A 1 kW heater is meant to operate at 200
r V. (a) What is its resistance ? (b) How
(L and r denote length and radius) much power will it consume , if the line
The heat produced in the fuse wire is voltage drops to 100 V ? (c) How many
i 2
L units of electrical energy will it consume
H i2R 2
r in a month (of 30 days) if it operates 10 hr
If H0 is heat loss per unit surface area of the daily at the specified voltage ?
fuse wire, then heat radiated per second is = Solution :
H 2rL At thermal equilibrium, a) The resistance of an electric appliance is given by
0
2 2
i L
2
i2 VS 200 40
H 0 2rL H
(or) 0 , R so, R
r 2 22r 3 W 1000
According to Newton’s law of cooling. b) The ‘ actual power ‘ consumed by an electric
H 0 K appliance is given by ,
2
Where is the increase in temperature of fuse V
wire and K is a constant. P A W
VS
i 2
2
22r3K so, P 100 1000 250W
Here is independent of length L of the fuse 200
wire provided i remains constant. c) The total electrical energy consumed by an
For a given material of fuse wire i2 r 3 . electric appliance in a specified time is given
If radiation losses are neglected, due to by,
heating effect of current the temperature W1h1
of fuse wire will increase continuously, and E kWh
it melt in time ‘t’ such that 1000
H ms ;
I 2 Rt
ms ( mp r ) so, E 1000 (10 30) 300kWh
J 1000
Page 95
CURRENT ELECTRICITY
Illustration -28:
Pn A lamp of 100W works at 220 volts. What
is its resistance and current capacity ?
V Solution :
Power of the lamp, P = 100W
Operating voltage, V = 220V
If bulbs (or electrical appliances) are connected Current capacity of the lamp,
in parallel, the potential difference across each
P 100
1 1 i 0.455A
resistance is same. Then P and I . V 220
R R
Resistance of the lamp, R V (220) 484
2 2
i.e. The current and power consumed will be
more in smaller resistance. P 100
When the appliances of power P1, P2 , P3 .... are Illustration -29:
in parallel, the effective power consumed(P) is A 100W – 220V bulb is connected to 110V
source. Calculate the power consumed by
P P1 P2 P3 .........
the bulb.
i.e. the effective power of various electrical Solution :
appliance is more than the power of individual Power of the bulb, P = 100W
appliance. Operating voltage, V = 200V
Page 96
CURRENT ELECTRICITY
V2 (220)2 Solution :
Resistance of the bulb, R 484
P
1
100 V2
Actual operating voltage, V = 110 V Resistance of a bulb is R where P de-
Therefore, power consumed by the bulb, P
(V1 )2 (110)2 notes the power of the bulb.
P1 25W.
R 484 Resistance of B1(R1) = V2 / 100
Resistance of B2 (R2) = V2 / 60
Illustration -30: Resistance of B3(R3) = V2 / 60
A 100W and a 500W bulbs are joined in 250 250300
series and connected to the mains. Which I1 = Current in B1 R R 8V 2
bulb will glow brighter ? 1 2
Solution : 250 250 300
Let R1 and R2 be the resistances of the two I2 = Current in B2 R R
bulbs. If each bulb is connected separately to 1 2 8V 2
the mains of voltage V, I3 = Current in B3 = I1 as B1, B2 are in series
V2 V2 output power of B1 is W1 = I12 R1
then P1 and P2
R1 R2
250300 2 V 2
W1
8V2 100
P1 R2 R1 P2 500
5
P2 R1 (or) R 2 P1 100
If the two bulbs are in series with the mains, the 250 300 2 V 2
W2 I 22 R 2 or W2
same current ‘i’ flows through each of them. 8V 2 60
Let P1 and P2 be the powers dissipated by two
bulbs, then 250300 2 V 2
W3 I22 R 3 or W3
P11 i2 R1 and P21 i2 R 2 8V 2 60
P11 R1 W1 : W2 : W3 15 : 25 : 64 or W1 < W2 < W3
5 or P11 5P21
P21 R2
Since 100 watt bulb dissipates more power, it CELLS
glows brighter Primary Cells: Voltaic, Leclanche, Daniel
Illustration -31: and Dry cells are primary cells. They convert
chemical energy into electrical energy. They
A 100 W bulb B1 and two 60 W bulbs B2
can’t be recharged. They supply small currents.
and B3, are connected to a 250V source, as
shown in the figure. Now W1 , W2 and W3 Secondary Cells (or) Storage Cells:
are the output powers of the bulbs B1, B2 Electrical energy is first converted into chemical
and B3 respectively. Then find the ratio of energy and then the stored chemical energy is
output powers of the bulbs . converted into electrical energy due to these
B1 B2 cells.
These cells can be recharged.
The internal resistance of a secondary cell is
low where as the internal resistance of a primary
cell is large.
B3 emf of a Cell: The energy supplied by the
battery to drive unit charge around the circuit is
defined as electro motive force of the cell.
EMF is also defined as the absolute potential
difference between the terminals of a source,
when no energy is drawn from it. i.e., in the
open circuit of the cell. It depends on the nature
of electrolyte used in the cell.
250V Unit :J/C (or) Volt
Page 97
CURRENT ELECTRICITY
Page 98
CURRENT ELECTRICITY
For maximum power dissipated in the device Solution :
d E R 2 i) According to maximum power transfer theorem
dP
0 0 3R 6R 4
dR dR (R r)2 R ext R int 4 R 2
9R 2
On simplification, we can get R = r
So, the power dissipated in an external ii) Pmax i2 R ext
resistance is maximum if that resistance is equal
to internal resistance of the source supplying E 2 E2
4
4 4 16
E2
the current to that device. So Pmax
4r
2
Pmax = E /4r Illustration -33:
A cell develops the same power across two
P resistances R1 and R2 separately. The in-
ternal resistance of the cell is
Solution :
R=r Let r and E are the internal resistance and emf
R of the cell. When connected across the resis-
Here the % of energy lost and energy useful tance R1 in the circuit, current passing through
are each equal to 50% the resistance is
DISTINGUISH BETWEEN E.M.F OF A E
2
E
CELL AND POTENTIAL i P1 i R1
2
R
DIFFERENCE R1 r ; R1 r 1
EMF OF A CELL
2
1. The emf is the maxdimum potential difference Similarly P2 E R 2 ; Given that P = P
betwen the electrodes of a cell, when the cell is R 2 r 1 2
Page 100
CURRENT ELECTRICITY
Solution: the circuit is from A to B through e
Current through the circuit Illustration -40:
2 2 4
i A voltmeter of resistance 500 is used to
R r 3 3.5 0.5 7
measure the emf of a cell of internal resis-
i) R 3, rA 3.5 , E 2V
tance 4 . The percentage error in the
Terminal voltage across A, VA E irA
reading of the voltmeter will be
4
2 3.5 0 volt Solution:
7
V = E – ir
ii) rB 0.5 , R 3 , E 2V
E ir
Terminal voltage across B, VB E irB Percentage error 100 100
4 E E
2 0.5 1.714 volts.
7 E
R r
Illustration -38: r
r
Two cells A and B each of 2 V are 100 100
connected in series to an external E R r
resistance R=1 ohm . The internal
4
resistance of A is rA =1.9 ohm and B is rB 100 0.8%
=0.9 ohm. Find the potential difference 500 4
between the terminals of A.
Solution: ELECTRIC CELLS IN PARALLEL
net emf When ‘n’ identical cells each of emf ‘E’ and
Total current through the circuit i internal resistance ‘r’ are connected in parallel
Total resist an ce
r = 1.9 0.9 to an external resistance ‘R’, then
total emf of the combination = E
A B
r
effective internal resistance =
R =1 n
4 4
A. r
1 1.9 0.9 3.8 total resistance in the circuit = R +
n
potential difference at A, VA ir , current through the external resistance
4 E nE
2 1.9 = 2 – 2 = 0.
3.8 i= r
nR r
R
Illustration -39: n
In the given circuit as shown below, r E
calculate the magnitude and direction of If R >> , then i = = current from one cell.
the current n R
A 2 e 2 B r nE
If R << , then i =
10V 5V n r
If two cells of emf E1 and E2 having internal
resistances r1 and r2 are connected in parallel
D C
1 to an external resistance ‘R’, then
Solution:
Effective resistance of the circuit is i1 +r1 E-1 i
Reff 2 2 1 5
V V E2
total current in the circuit is i 1 2 i2 +r -
i
Reff i 2 R
10 5
i= 1A
5
Since the cell of larger emf decides the direction
of flow of current, the direction of current in
Page 101
CURRENT ELECTRICITY
Page 102
CURRENT ELECTRICITY
I1 + I2 +(–I3) + (–I4) = 0 ; I 0 Apply the kirchhoff’s second law to the loop
Kirchhoff’s first law is in accordance with law ADCBA, then
of conservation of charge, since no charge –iR – ir2 + E2 – E1 – ir1 = 0
can accumulate at a junction and at any part of E 2 E1
the closed circuit. i(r1 + R + r2) = E2 – E1 i
r1 r2 R
(ii) Kirchhoff`s Second Law (Loop Law or 1) This law represents “conservation of energy”
Potential law): Kirchhoff’s second law 2) If there are n meshes in a circuit, the number of
states that the algebraic sum of the changes in independent equations in accordance with loop
potential around any closed loop is zero (in rule will be (n – 1).
ideal condition ) Application : This is the most general case of
(Kirchhoff’s second law) can be expressed as parallel grouping in which E and r of different
cells are different and the positive terminals of
V 0 . cells are connected as shown
In terms of potential drops and emfs, the law is E1 r1
expressed as iR E 0 i1
Sign conventions: E2 r2
(a) The change in potential in traversing a resistance i2
in the direction of current is –IR while in the i i
opposite direction +IR as shown in the figure. R
R Kirchhoff’s second law in different loops gives
A I B A I R B the following equations,
Page 103
CURRENT ELECTRICITY
r2
i2 R3 (i1 i2 ) R2 E2 0 ; i2 i1 i2 4
i V2
i1 2i2 4 (2)
+ -
A i r1 V1 B Solving equation (1) and (2) we get i1 = 0A, i2
= 2A
VA VB V1 ir1 [ V1 cell is discharg- Thus currents through R1 is 0, while through
ing] R3 and R2 are 2A each.
Illustration -43:
V1 V2
r1
or VA VB V1 Solve for current values in figure.
r1 r2 i1 8V, 1 i1
A F
V1r2 V2 r1 i S
or VA VB B
1 12V, 1.5
E
r1 r2 i3 i2 i
i3 2
Equivalent emf of the battery = V C D
V1r2 V2 r1 9
V r r
Solution :
1 2 Applying Kirchhoff’s first law at the junction B
(ii) Internal resistance of equivalent battery. r1 and we have i1 + i2 = i3 ....... (1)
r2 are in parallel. Applying Kirchhoff’s second law to loop
1 1 1 r1r2 ABEFA
or r
r r1 r2 r1 r2 – 12 + i2 x 1.5 – i1 x 1 + 8 = 0
Illustration -42: i1 – 1.5 i2 = – 4. ....... (2)
In the given circuit values are as follows From loop BCDEB
E1 2V , E2 4V , R1 1 and R2 R3 1 .
– (i2 x 1.5) – (i3 x 9) + 12 = 0
Calculate the Currents through R1,R2 and 1.5 i2 + 9i3 = 12 ....... (3)
R. on solving i1 = – 1A and i3 = 1 A
3
WHEATSTONE’S BRIDGE
B
I3
P Q
I1 I
A G C
F
I2
Solution:
R S
A -B C I4
i1 E E2 I D
1
R1 R2 R3 i
2
( )
(i1+i2)
E K
G +F D Condition for balancing of bridge :
Let i1 , i2 are currents through R1 and R3. Applying Kirchhoff`s first law at junction B and
(i1 + i2) is current through R2. D we get I1 – I3 – IG = 0 ; and I2 + IG – I4 = 0
Their direction are taken as shown Applying Kirchhoff’s second law for closed
From Kirchhoff’s second law for AGFBA loop loop ABDA, I1P IGG I2 R 0
Page 104
CURRENT ELECTRICITY
Applying Kirchhoff’s second law for closed B
loop BCDB , I3Q I4S IGG 0 P Q
The values of P, Q, R, S are adjusted such that i1
i A i2 C
I G becomes zero. At this stage the bridge is
said to be in balance condition.
R S
i.e., I n balanced condition of bridge I G = 0
In balanced condition the above equations D
respectively become
I 1 = I3 ........(1) VAB VA VB i1P i
R S P
and I2 = I4 ........(2) PQ R S
I1P I 2 R ...........(3) P Q R
VAD VA VD i2 R i
I 3Q I 4 S ............(4) PQ R S
Dividing equation (3) by equation (4) i
VB VD P Q R R S P
I1P I 2 R PQRS
I 3Q I 4S i
Using equations (1) and (2) we get
PQ R S
QR PS
P R
.......(5) if QR PS , VB VD current flows from B to
Q S
D when wire is connected beween B and D.
This is the balancing condition for Wheatstone
bridge. QR PS , VB VD current flows from D to B
APPLICATIONS OF WHEATSTONE when wire is connected beween B and D.
BRIDGE
1. We can compare two unknown resistances R QR PS , VB VD Balanced bridge
Common forms of balance Wheatstone bridge
P R
and S from Q
Q S
2. In place of resistances we can use capacitors
A G S B
to form a D.C. Wheatstone bridge with four
capacitors of capacitances C1, C2, C3 and C4. P
C1 C3
The balancing condition will be C C R
2 4 P Q
3. It has been found that the bridge has the
greatest sensitivity when the resistances are as
nearly equal as possible. A B
The bridge is most sensitive if P = Q = R = S.
4. Equivalent resistance of balanced bridge across
the ends of battery when the bridge is balanced R S
P Q R S
is given by Rtotal
PQ R S
P
Application : Direction of current in an G Q
unbalanced wheatstone’s bridge :
A
B
S
Page 105
CURRENT ELECTRICITY
P Q P R
Wheatstone Bridge
Q S
+ -
( )
Cell
R S Key
RB
R X
G J
Illustration -44: A l1 l2 B
Determine the current in each branch of H.R G
the network shown in fig. When the Meter bridge is balanced then
B R l1 l1
10 5 X l2 100 l1
A C Where l1 is the balancing length from the left
5
end.
5 1. If resistance in the left gap increases or
10
resistance in the right gap decreases, balancing
D point shifts towards right side.
2. If resistance in the left gap decreases or
10V 10 resistance in the right gap increases, balancing
B I-I1 point shifts towards left.
10 5 3. If a cm, b cm are the end corrections at A and
1
AI 5 C R l1 a
I-I1 I-2I
1 B, then X l b
2
10 2I 4
I= ; I1 A A x C B
17 5 17 Solution:
4 For null deflection of galvanometer in a meter
Current in AB branch bridge experiment,
17
Current in AD branch R1 R AC R x
or 1
10 4
=I I1 A
6 R 2 RCB R 2 100 x
17 17 17 Since R1 / R2 remains constant, x / (100-x) also
Current in DB branch remains constant. The value of x remains as
10 8 2 such.
I 2I = - = A Length of AC = x
17 17 17
Metre bridge: It works on the principle of Illustration -46:
A resistance of 2 is connected across
Page 106
CURRENT ELECTRICITY
one gap of a meter bridge (the length of length L and resistance R. The current through
the wire is 100 cm) and an unknown resis- the potentiometer wire is
tance, greater than 2 , is connected E
across the other gap. When these resis- I
tances are interchanged, the balance point r R Rs
shifts by 20 cm. Neglecting any correc- The potential of the wire decreases from the
tions, the unknown resistance is end A to the end B. The potential fall or potential
Solution: drop across a length l of the potentiometer
Refer to the diagram Apply the conditions of wire is
the balanced Wheatstone’s bridge for the two V = Current x Resistance of length l of the
cases.
R
R1 R2 potentiometer wire I l
L
G R
If the resistance per unit length of the wire ,
L
l
100-l is denoted by , the potential drop across the
wire is
2
..................................(i) V I
x 100
x 20 V
....................................(ii) is called potential drop per unit length of
2 80 l
the potentiometer wire or potential gradient of
Equations (i) and (ii) give x 3 the wire. It is given by
Potentiometer: Potentiometer is an instrument
which can measure accurately the emf of a V E R
source or the potential difference across any I
l r R RS L
part of an electric circuit without drawing any Thus, the unknown voltage V is measured when
current from the battery whose emf is to be no current is drawn from it.
measured. 1) When specific resistance (S) of potentiometer
Principle : The principle of potentiometer states wire is given then potential gradient
that when a constant current is passed through
a wire of uniform area of cross-section, the IS IS
potential drop across any portion of the wire is X 2
A r
directly proportional to the length of that portion.
The principle of potentiometer require that where A = area of cross - section of
i) potentiometer wire should be of uniform potentiometer wire r = Radius of
area of cross-section and potentiometer wire.
ii) current through the wire should remain 2) When two wires of lengths L1 and L2
constant. and resistances R1 and R2 are joined
Theory of potentiometer : The ends of the
potentiometer wire AB are connected to a together to form the potentiometer wire, then
standard cell of emf E or a source of emf E that x1 R1 L2
supplies constant current. The current through
x2 L1 R2
the potentiometer wire can be varied by means
Potential gradient depends on
of a series resistance Rs which is adjustable. a) Resistance per unit length of the
potentiometer wire ( =R/L)
b) Radius of cross-section of the potentiometer
wire, when the series resistance is included in
the circuit and cell in the primary circuit is not
ideal.
c) Current flowing through potentiometer wire.
d) emf of the cell in primary circuit
e) Series resistance in the primary circuit
Let r be the internal resistance of the cell of emf f) Total length (L) and resistance (R) of the
E connected across the potentiometer wire of potentiometer wire.
g) If cell in primary circuit is ideal and in the
Page 107
CURRENT ELECTRICITY
e, r k Rh Comparison of resistances:
( )
Let the balancing length for resistance R1 (when
J XY is connected) be l1 and let balancing length
A B
E G for resistance R1 R2 (when YZ is connected)
be l2 . keeping x constant
( )
R1 K1
Initially in secondary circuit key K 1 remains
open and balancing length l1 is obtained. Since
cell E is in open circuit so it’s emf balances on
length l1 ; i.e E xl1 .....(i)
Now key K 1 is closed so cell E comes in closed
circuit. If the process of balancing is repeated
again keeping potential gradient constant then
Then iR1 xl1 and
potential difference V balances on length l2
R2 l2 l1
i.e V xl2 .....(ii) i R1 R2 xl2
By using the formula for internal resistance R1 l1
E To determine thermo emf:
r 1 .R / e r
+ -
K Rh
V ()
Where E = emf of cell in secondary circuit R A
V = Terminal voltage i.e p.d on R1 , A HRB B
G
l l + -
r 1 2 R' E0 1 2 3
G
l2
COMPARISON OF EMF’S OF TWO CELLS Cold ice Hot sand
The value of thermo-emf in a thermocouple for
Let l1 and l2 be the balancing length with the ordinary temperature difference is very low
cell E1 and E2 respectively, then E1 xl1 106 volt . For this the potential gradient x
E1 l1 must be also very low 10 4 V / m . Hence a
and E2 xl2 E l
2 2 high resistance (R) is connected in series with
e,r K Rh the potentiometer wire in order to reduce
( ) current
in the primary circuit
A
J The potential difference across R must be equal
B
E1 G to the emf of standard cell
1 E
E2 i.e iR E0 i 0
2 R
Let E1 E2 and both are connected in series. The small thermo emf produced in the
thermocouple e xl
If balancing length is l1 when cells assist each
iR| iR ' l
other and it is l2 when they oppose each other x i e
as shown then: L L
E1 E2 E1 E2
where L = Length of potentiometer wire,
=resistance per unit length, l balancing
length of e and R| = Resistance of
potentiometer wire
Page 108
CURRENT ELECTRICITY
SENSITIVITY OF POTENTIO METER The balancing length l1 for unknown potential
1. Sensitivity of potentio meter is estimated by its difference V ' is given by (closing 2 and 3)
potential gradient.
2. Sensitivity is inversely proportional to potential E
V | xl1 0 l1
gradient so lower the potential gradient higher l0
will be the sensitivity. If the voltmeter reading is V then the error will
3. The best instrument for accurate measurement
be V V which may be +ve, -ve or zero
|
of e.m.f. of a cell is potentiometer, because it
does not draw any current from the cell. Difference between Voltmeter and
Calibration of ammeter: Checking the Potentiometer
correctness of ammeter readings with the help VOLTMETER
of potentiometer is called calibration of 1. It draws some current from source of emf.
ammeter. 2. The potential difference measured by it is lesser
+e -
K Rh than the actual potential difference.
() 3. Its sensitivity is low.
4. It is a versatile instrument.
A + B
5. It is based on deflection method.
+ - E1 1 POTENTIOMETER
2 G 1. It does not draw any currrent from the source
1 3 of unknown emf.
A
+ - 2. The potential difference measured by it is equal
K2 Rh
+ - () to actual potential difference.
For the calibration of an ammeter, 1 3. Its sensitivity is high.
resistance coil is specifically used in the 4. It measures only emf or potential diference.
secondary circuit of the potentiometer, because 5. It is based on zero deflection method.
the potential difference across 1 is equal to Illustration -47:
the current following through it i.e V i The length of a potentiometer wire is 1m
If the balancing length for the emf and its resistance is 4 . A current of 5
mA is flowing in it. An unknown source of
E0 e.m.f is balanced on 40 cm length of this
E0 is l0 then E0 xl0 x l (Process
0 wire, then find the e.m.f of the source.
of standardisation) Solution:
Let i ' current flows through 1 resistance R 5 4
x I I 20 mV
giving potential difference as V ' i ' 1 xl1 L 1
E = l x = 0. 40 x 20 = 8 mV
where l1 is the balancing length. so error can
Illustration -48:
E0 A cell of e.m.f 2 volt and internal resistance
be found as i i i ' i xl1 i l l1
0 1.5 is connected to the ends of 1m long
Here i is ammeter reading wire. The resistance of wire is 0.5 / m .
Calibration of voltmeter: Find the value of potential gradient on the
Checking the correctness of voltmeter readings wire.
with the help of potentiometer is called Solution:
calibration of voltmeter.
I R E R 2 0.5
If l0 is balancing length for E0 the emf of X =0. 5 V/m
standard cell by connecting 1 and 2 of bi- L R r L 0.5 1.5
Illustration -49:
directional key, then x E0 / l0 In a potentiometer experiment the balancing
K1 Rh length with a cell is 560 cm. When an
+ -
() external resistance of 10 is connected in
parallel to the cell, the balancing length
A + B changes by 60 cm. Find the internal
+ - E0 1 resistance of the cell.
2 G Solution:
V 3
RB
K2
+ - ()
Page 109
CURRENT ELECTRICITY
Page 110
Page 111
SEMICONDUCTOR ELECTRONICS
SEMICONDUCTOR ELECTRONICS
INTRODUCTION Different energy level with continuous energy
variation form energy bands (According to
Electronic instruments are being utilized in Pauli’s principle)
various fields like telecommunication, The energy band formed by a series of energy
entertainment, computers, nuclear physics and bands of valance electrons is valence band.
many more. Although the history started with At 0 K, electrons start filling energy level in
the adventment of vacuum tubes, however the valence band starting from the lowest one.
The highest energy level, occupied by an electron
rapid advancement in electronics which we
in the valence band at 0K is called Fermi level.
see today is due to the valuable contributions The lowest unfilled energy band formed just
of semiconductor devices. above valence band is called conduction band.
Semiconductor devices are not only small in Depending on the forbidden energy gap
size, consumes less power, have long life time between valence band and conduction band, the
and are more efficient than vacuum tubes but solids are classified as conductors, insulators and
also are of low cost. That is why these have semiconductors.
replaced vacuum tubes nearly in all applications. 1) Conductors : The energy band structure
As an example we can consider the case of a in conductors have two possibilities
computer. In early days, the vacuum tube The valance band may be completely filled and
based computers were as big as the size of a the conduction band partially filled with an
room and were capable of performming simple extremely small energy gap between them
calculations only. At present the personal Eg 0
computer (PC) that you see in laboratory or
Eg: Sodium (Na)
at your home is much smaller in size and
The valence band is completely filled and
capable of performing many operations. conduction band is empty but the two overlap
Needless to say this is possible because of each other. Eg: Zn
the advances in semiconductor technology.
We will learn the basic concept of semi
conductors. This will enable us to understand
the operation of many semiconductor devices
and then we will be discussing few
semiconductors devices like diode, transistor
along with their applications.
Electron energies
Conduction
CLASSIFICATION OF METALS, EC band
CONDUCTORS ANDSEMICONDUCTORS: Ev
Valence
Energy Band theory in solids: An isolated bond
atom has well defined energy levels and energy conductors
At room temperature
At room temperature At room temperature
3. conductivity is
Conductivity is high conductivity is nil
intermediate
They have resistivity
They possess very low or conductivity
They have high resistivity
4. resistivity intermediate to metals
1011 1019 m
102 108 m and insulators
10 5 10 6 m
V.B is almost full V.B. is completely filled
5. V.B and C.B over lap
C.B. is almost empty C.B. is completey empty
6. Ex: All metals Ex: Ge, Si Ex: Rubber, Glass, Mica
Page 113
SEMICONDUCTOR ELECTRONICS
Extrinsic semi conductor: The semi
conductor formed by adding a small quantity of
Ge Ge
impurity (tri or pentavalent) to pure semi
conductor is called as extrinsic semi conductor.
The extrinsic semi conductor formed by adding
Ge pentavalent impurity atoms is called n-type
semi conductor.
E
Ge Ge
CB
Ge Ge Ee
Excess EF
Ge
electron FB Eg Donor
levels
Concept of a Hole : Ge Ge VB
When an electron gains energy as a result of
thermal excitation, a covalent band is broken. (a) (b)
As the electron leaves its site for another one, Further, it is be noted that if
an excess positive charge appears at the vacant
site since ont he whole the crystal is neutral. N d donor atoms concnetration (i.e., density
The excess positive charge appearing at the of donor atoms),
vacant site in a srystal is called a hole ne electron density in the conduction band
CB CB
Electron
Eg Eg Hole nh hole density in the valence band, then
EF FB EF FB
(i) ne N d (ii) ne nh .... (1)
VB VB
Further, it can be sho wn t hat n-type
Intrinsic semiconductor Intrinsic semiconductor semiconductors always satisfy the equality
(at absolute zero) (at room temperature)
(a) (b) n nh ni2 .... (2)
From the above discussion, it is clear that the The extrinsic semi conductor formed by adding
charge carriers in semiconductors may be trivalent impurity atoms is called p-type semi
electrons and holes. conductor.
Further, it is obvious that the electric conductivity E Electron
of a semiconductor depends upon its carrier CB Hole
Ge Ge Eg
concentration or carrier density or number Acceptor
density (number of charge carriers per unit Electron FB levels
Ge EF
volume) vacancy Ee
An int rinsic semiconductor has equal Ge Ge VB
concentration of electrons and holes under
conditions of thermal equilibrium, i,e., (a) (b)
Further, it is to be noted that if
ne nh ni say
Note : Although an intrinsic semiconductor has equal N a acceptor atoms concentration (i.e.,
concentrations of electrons and holes under density of acceptor atoms),
conditions of thermal equilibrium, electron ne electron density in the conduction band,
conduction is predominant. This is because
electrons have a greater mobility as compared nh hole density in the valence band, then
with holes. In case of hole conduction, the
electrons are more limited (less free) to move (i) nh N a (ii) nh ne .... (3)
about than in case of electron conduction. Further, it can be sho wn t hat p-type
Doping: The process of adding impurities to semiconductors also always satisfy the equality,
a pure semi conductor in 1 : 106 ratio so as to ne nh ni2 .... (4)
improve its conductivity is called doping. Fermi level in n-type semi conductors (also
Pentavalent impurity atoms are called donor known as donor energy level) lies in forbidden
impurity atoms. energy gap and is very close to conduction band
Trivalent impurity atoms are called acceptor ( 0.01eV below conduction band)
impurity atoms. Fermi level in p-type semi conductor (also
Page 114
SEMICONDUCTOR ELECTRONICS
known as acceptor energy level) lies in Electrical conductivity of Semiconductors:
forbidden energy gap and is very close to (Formulae derivation)
valence band ( 0.01 to 0.05 eV above Consider a block of semiconductor of length l ,
valence band) area of cross-section A and having number
When ever a covalent bond breaks an electron
hole pair is formed. density of electrons and holes as ne and nh
When valency electron combines with a hole, a respectively .
covalent bond formed and at the starting point By that on applying a potential difference, say
of the electron a hole is formed. V, a current I flows through it as shown in figure.
If a conduction electron combines with a hole a
The electron current I e and the hole current
covalent bond is formed without the formation
of a hole any where. Ih
constitute the total current I flowing
In a doped (or) extrinsic semi conductor number through the semico nductor i.e .,
density of electrons in conduction band (ne),
number density of holes in the valence band I I e I h ............1
nh and number density of electrons in – + –
conduction band (or) holes in valence band in a + –
+ – +
pure semi conductor ni then they are related
l
as ne nh ni2 . I
The intrinsic concentration ni varies with T as
E g / KT
ni2 AoT 3 e (Ao is constant)
V
The fraction of electrons of valence band present
Eg If ne is the density of conduction band electrons
in conduction band is given by f e KT
in the semiconductor and ve is the drift velocity
Where K is Boltzman’s constant, Eg is forbidden of electrons, then electron current is given by
energy gap and T is temperature
I e ene Ave ........(2)
The Energy Gap: Experimentally it has been
found that the forbidden energy region Eg Also, the hole current I h enh Avh ......(3)
depends on temperature. Using the equation (2) and (3) , the equation
(1) becomes
For silicon Eg T 1.21 3.60 104 T
I ene Ave enh Avh or
For germanium
I eA ne ve nh vh ........ 4
Eg T 0.785 2.23 104 T
If is the resistivity of the material of the
At room temperature 300K for silicon semiconductor, then the resistance offered by
the semiconductor to the flow of current is given
Eg 1.1eV and for germanium Eg 0.72eV
When concentration of donor atoms exceeds by R .........(5)
the accept or concentration in P type A
semiconductor, it changes from P type to N Since V=RI, from the equations (4) and (5),
type semiconductor. we have
In semi conductors the total current I is the sum
V RI eA ne ve nh vh
of electron current Ie and holes current Ih A
I = Ie + Ih or V e ne ve nh vh .............(6)
Electrical conductivity e h If E is the electric field set up across the
ne e e nh h e V
semiconductor, then E ........(7)
For intrinsic semi conductor ne nh ni so that
From the equations (6) and (7) , we have
ni e e h
Page 115
SEMICONDUCTOR ELECTRONICS
Page 116
SEMICONDUCTOR ELECTRONICS
diffusion current equals the drift current in
magnitude and there is no transfer of charge
at any cross-section.
DEPLETION REGION(TRANSITION REGION): The
space-charge region on either side of the
p n
(a) junction together is known as depletion
region as the electrons and holes taking
p n
part in the initial movement across the
junction has depleted the region of its free
charges.
(b)
Contact potential or
BARRIER POTENTIAL (CONTACT POTENTIAL
Barrier potential
DIFFERENCE):
In P-N junction, n side is at high potential as
compared to p-side and this potential
difference tends to prevent the movement of
electron from n region into the p region. This
Depletion region or potential difference is termed as barrier
Transition region potential.
DIFFUSION CURRENT: As the diffusion of Biasing :
electrons continues, the potential difference It means the way of connecting emf source to
between the two sides goes on increasing. P-N junction diode. It is following two types
The filling of the holes in p-crystal gives rise to 1) Forward biasing : Positive terminal of the
more holes in the n-crystal and this action is battery is connected to the P-crystal and
the same as though the holes had diffused negative terminal of the battery is connected
to N-crystal
across the boundary towards the right. As the
E
n-crystal becomes more positive and p-crystal – +
Eb
more negative, electrons will be attracted back
towards the right. ultimately, an equilibrium –
–
–
–
condition would be developed in which equal –
– –
number of electrons will be crossing the
boundary in the opposite directions. P N
+ –
DRIFT CURRENT: Because of thermal collisions (i) In forward biasing width of depletion layer
occasionally, a covalent bond is broken and decreases.
the electron jumps to the conduction band. (ii) In forward biasing resistance offered
An electron-hole pair is thus created. Also RForward 10 25
occasionally a conduction electron fills up a
(iii) Forward bias opposes the potential and
vacant bond so that an electron-hole pair is
destroyed. These processes continue in every for V VB a forward current is set up across
the junction
part of the material. However, if an electron-
hole pair is created in the depletion (iv) The current is given by i is e eV / kT 1 ;
region, the electron is quickly pushed by the where is Saturation current, In the exponent
electric field towards N side and the hole
towards P-side. This makes a current from e 1.6 1019 C , k = Boltzmann’s constant
the N side to the P side. This is called the drift (v) Cut-in (Knee) voltage : The voltage at
which the current starts to increase rapidily.
current. For Ge it is 0.3V and for Si it is 0.7V.
The drift current and the diffusion current are (vi) df- diffusion, dr - drift
in opposite directions. In steady state, the (vii) It is a low resistance connection or it acts
Page 117
SEMICONDUCTOR ELECTRONICS
like a conductor and for an ideal diode the For Ge it is 25V and for Si it is 35V
resistance is zero (short circuit) Reverse voltage
(viii) The flow of current is mainly due to the Break down Reverse current
diffusion of majority charge carriers (v)
voltage Idf
(ix) The direction of current is from p to n Idr
Inet
side
(vi) The net reverse current is given by:
Forward current in mA
I resistance is infinity
dr
I rest
Knee (viii) The flow of current is mainly due to the
voltage Forward voltage
drift of minority charge carriers.
2) Reverse biasing : Positive terminal fo the
battery is connected to the N-crystal and (ix) The direction of current in it is from n to
negative terminal of the battery is connected p side
to P-crystal (x) The current in this bias is called reverse
E
saturation current or leakage current.
– +
E b
Reverse Breakdown :
If the reverse baised voltage is too high, then
– – –
– – – breakdown of P-N junction diode occurs, It
– is following two types
– – –
1) Zener breakdown : When reverse bais is
P N
increased the electric field across the junction
– + also increases. At some stage the elctric field
(i) In reverse biasing width of depletion layer becomes so high that it breaks the covalent
increases bonds creating electron-hole pairs. Thus a
(ii) In reverse baising resistance offered large number of carriers are generated. This
5 causes a large current to flow. This mechanism
RRe verse 10
is knonw as Zener breakdown.
(iii) Reverse bias supports the potential barrier 2) Avalanche breakdown : At high reverse
and no current flows across the junction due voltage, due to high electric field, the minority
to the diffusion of the majority carriers charge carriers, while crossing the junction
(A very small reverse current may exist in the acquire very high velocities. These by collision
circuit due to the drifting of minority carriers breaks down the covalent bonds generating
across the junction) more carriers. A chain reaction is established
(iv) Break down voltage : Reverse voltage at giving rise to high current. Thismechansim is
which break down of semiconductor occurs. called avalanche breakdown.
MODES OF OPERATION OF P-N JUNCTION DIODE:
W W
p n p n
2
1
2 1
3
V0 V0
Page 118
SEMICONDUCTOR ELECTRONICS
ΔVi ΔVB
Ri = 100 RB = 10 6
Δli Δl B
V-I CHARACACTERSTICS OF A P-N V-I Characteristics of P - N Junction Diode
JUNCTION DIODE: For forward bias arrangement, a milliammeter
Voltmeter(V) is used whereas in reverse bias arrangement a
microammeter is used
T HRESHOLD VOLTAGE OR CUT-IN VOLTAGE:
The voltage across the diode beyond which
p n the diode current increases exponentially is
called threshold voltage. The resistance of a
Milliammeter p-n junction diode
(mA) is of two types.
Switch
_ (A)STATIC OR DC RESISTANCE
+ (a) of a diode which is defined as rdc = V/Irdc
Experimental setup for varies widely with V as the v – I graph is non
diode in forward bias linear.
(B)DYNAMIC RESISTANCE OR AC RESISTANCE
Voltmeter (V)
V dV
R or [slopeof V versus i graph]
i di
or (Reciprocal of slope of i versus V graph)
p n Where V denotes a small change in the
Microammeter (A) applied potential difference and i denotes
corresponding small change in current.
Switch Additional points 1 : The current flows
-
+ (b) through the diode the below circuit is
Experimental setup for I
diode in reverse bias
IF (mA) R VB
Breakdown 80 E VB
Majority I
voltage 60 R rf
(zener voltage,Vz) 40 Knee carrlers
20 point V E
VR(V) –80 –60 –40–20 K
VF (V)
Reverse 0.2 0.40.6 0.8
–1 Forward
bias Reverse
–2 Where R = external resistance, r f =
Crystal current bias
breakdown Minority resistance of diode in forward bias, VB =
carriers
IR (A) barrier potential.
Page 119
SEMICONDUCTOR ELECTRONICS
Additional point 2 : Solution: The diode D1 is forward biased but
I diode D2 is reverse biased. So no current
R VB pass through the diode D2.
E Current passes through 100 resistor
6
i. Power developed across the diode = VB I 0.02A
ii. Power developed across the resistor = 150 100
E VB I . Illustration 7:
Additional point 3 : The V-I characteristic of a silicon diode
The potential barrier existing across an is shown in the Fig. Calcualtethe
resistance of the diode at a) I D =15 MA
unbiased p-n junction is VB volt.
i. The minimum kinetic energy required by a and b) VD = -10V..
hole to diffuse from the p-side to the n-side is I(mA)
30
' eVB ' . Silicon
ii. If the junction is forward biased at V volt. Then 20
15
the minimum kinetic energy required by a hole 10
to diffuse from the p-side to the n-side is -10V
e VB V 0 0.5 0.7 0.8 V(V)
1(A)
iii. If the junction is reverse biased at V volt. Solution : Considering the diode characteristics
Then the minimum kinetic energy required by as a straight line between I =10 mA to I = 20
a hole to diffuse from the p-side to then-side mA passing through the orgin,we can calcualte
is e VB V . the resistance using ohm’s las.
Illustration 5: a) From the curve, at I=20 mA, V=0.08 V,
Find voltage across resistor, across diode I=10 mA, V=0.7 V
and current through diode.
rfb V / I 0.1 /10mA 10
Si
b) From the curve at V 10V .I 1 A .
8V 2.2 k Therefore r fb 10V / 1 A 1.0 X 107
Illustration 8:
A source voltage of 8V drives the diode in
Solution: VD = 0.7V fig. through a current-limiting resistor of
VR = 8V – 0.7V = 7.3V 100 ohm. Then the magnitude of the slope
of load line on the V-I characteristics of
7.3
iR = iD = 3.31 10 3 A the diode is
2.2 10 3
Illustration 6: – 8V
If each diode forward resistance is 50 +
then find current through the 100
resistor in the given circuit. 100
Solution:
D1 150 If I is the current in the diode and V is voltage
drop across it, then for given figure,
D2 50 voltage equation is:
(I×100) - V = 8
I = (1/100)V+8/100 I=(0.01)V+ 0.08
6V 100 Thus the slope of I-V graph is equal to
1/RL = 0.01
Page 120
SEMICONDUCTOR ELECTRONICS
Illustration 9: D.C power output
The I-V characteristic of a P-N junction v) Efficiency
A.C power input
diode is shown below. The approximate
dynamic resistance of the P-N junction 0.406 RL
vi) Efficiency 100%
when a forward bias of 2volt is applied is rf RL
i(mA)
800 vii) max = 40.6% for an ideal diode.
2) Full Wave rectifier:
400 D1
RL
2 2.1 V (volt) ~ O/P (dc)
Solution:
The current at 2V is 400 mA and at 2.1 V D2
Page 121
SEMICONDUCTOR ELECTRONICS
Page 122
SEMICONDUCTOR ELECTRONICS
3) Full wave bridge rectifier: Capacitor Filter: When the voltage across
D1 D4
the capacitor is rising, it gets charged. If there
ac – is no external load, it remains charged to the
supply RL Vout peak voltage of the rectified output. When
D2 D3
+ there is a load, it gets discharged through the
load and the voltage across it begins to fall. In
the next half-cycle of rectified output it again
Input gets charged to the peak value but due to
+
ac signal + large value of time constant of capacitor,
0 – voltage across the capacitor approximately
remain constant.
Capacitor input filters use large capacitors to
output
dc signal increase the time constant of capacitor
D2D3 D4D1 D3 Capacitor filter is most widely used in power
supplies.
i) Four diodes, D1 , D2 , D3 and D4 are used in SPECIAL PURPOSE OF P-N JUNCTION
DIODE
the circuit.
ZENER DIODE:
ii) During positive half cycle D1 and D3 are
Circuit Symbol
forward biased and D2 and D4 are reverse
I-V characteristics
biased.
iii) During negative half cycle D2 and D4 are
forward biased and D1 and D3 are reverse
biased.
FILTER CIRCUIT:
The filter is a device that allows passing the
dc component of the load and blocks the ac
component of the rectifier output. Thus the
output of the filter circuit will be a steady dc
voltage. To get steady DC output from the
pulsating voltage, normally a capacitor is
connected across the output terminals (parallel Zener Diode is designed to operate under
to the load RL ) reverse bias in its breakdown region.
Formed by heavily doping both p and n sides
of the junction.
Depletion region is very thin and zener voltage
will be lesser and sharp.
More number of electron-hole pairs are created
due to the strong electric field at the junction
ac input
input filter
capacitor
Page 123
SEMICONDUCTOR ELECTRONICS
from the host atoms on the p side which are Illustration 11:
accelerated to n side. These electrons account In half wave rectifier a p–n diode with
for high current observed at the breakdown.
internal resistance 20 is used. If the
Field Ionization: The emission of electrons loadresistance of 2k is used in the
from the host atoms due to the high electric
circuit, then find the efficiency of this
field is known as field ionization or internal
field emission. half wave rectifier.
Solution:
ZENER DIODE AS VOLTAGE REGULATOR:
Zener diode is used as voltage regulator and its rf = 20
circuit diagram is RL = 2k =2000
R I IL
0.406RL
Iz Rectifier efficiency ( )
RL r f
Vi/p Vo/p
Vz 0.406 2000 812
0.402
2000 20 2020
Efficiency = 40.2%
1) I I Z I L 2) VO / P VZ I L RL Illustration 12:
3) VI / P IR VZ In the half-wave rectifier circuit shown,
4) Current thro ugh load resistance what is the form of the output across C
and D?
V V
IL 0 Z
RL RL A C
Page 124
SEMICONDUCTOR ELECTRONICS
Illustration 14: When photons of energy greater than the
From the zener diode circuit shown in energy gap of the device material are absorbed
the figure, find (1) the output voltage (2) in the device, hole-electron pairs are generated,
the voltage drop across series resistance the conductivity of the diode increases, and
and (3) the current through zener diode. the reverse current builds up..
I IL The current-voltage characteristics at
=constant are shown in figure. Here, stand
R=5k Iz
for incident luminous flux an I is the magnitude
Vi/p=220V 20k of the reverse current. When there is no light
Vz=50V
flux incident on the device, the usual reverse
leakage current I 0 , called the dark current.
Solution :
3>2
From the figure
p 2>1
R = 5k = 5 × 103 RL 1>
Input voltage = 220V n
=
3 V
Load resistance RL = 20k = 20 × 10
Connection of a Current-voltage
Zener voltage, Vz = 50V photodiode for operation characteristics of a photodiode
in the current mode in the current mode
(1) Output voltage, Vz = 50 V
h
(2) Voltage drop across, series resistance,
R = input voltage – Vz = 220 – 50 = 170 V
(3) Load current,
A
V 50 p - side n - side
IL z 3
=2.5 × 10–3A
RL 20 10
Current through R, mA
Page 125
SEMICONDUCTOR ELECTRONICS
Uses of Photodiodes:
infra red
amber
yellow
I(mA)
white
green
Photodiodes are used: As light detectors, in
blue
red
light operated switches, in reading of computer 50
VA
punched cards and tapes, in alarm systems, in 40
30
+ R
camera exposure controls, as optical fiber VA 20
10
receivers. – I
1 2 3 4 5 VF
LIGHT EMITTING DIODE(LED) separation and collection
Heavily doped p-n junction which under (i) Generation of electron-hole pairs due to light
forward bias emits spontaneous radiation. (with h E g ) in junction region,
The compound semiconductor Gallium (ii) Separation of electrons and holes due to
(Ga As1 x Px ) is used electric field of the depletion region. Electrons
A rsenide-Phosphide
for making LEDs of different colours. are swept to n-side and holes to p-side by the
junction field;
(iii) On reaching electrons at n-side and holes on
– at p-side. Thus n-side becomes negative and
+ p-side becomes positive potential and giving
rise to photovoltage.
+ –
(a) (b) LED Conventional Incandescent Lamp
Low operational voltage High operational voltage and more
When a conduction electron makes a transition and less power. power.
to the valence band to fill up a hole in P-N Fast action and no Slow action and some warm-up time
warm-up time required. required.
junction, the extra energy is emitted as a
The bandwidth of The emitted light is not
photon. If the wavelength of this photon is in emitted light is 100 Å to monochromatic.
the visible range one can see the emitted light. 500 Å or in other words
Such a P-N junction is known as light-emitting it is nearly (but not
exactly) monochromatic.
diode (LED)
Long life and ruggedness. Less life.
When an electron in the conduction band Fast on-off switching Slow on-off switching capability.
makes a transition to the valence band, to capability.
recombine with the holes, energy equal to the
band gap is released. SOLAR CELL
It is used as indicator lamps, in signal lamps Generates emf when solar radiation falls on a
and displays in computers and calculators. p-n junction.
The infra-red LED is used for optical Works on the principle of photovoltaic effect.
communication.
Semiconductors with band gap close to 1.5
eV, high optical absorption, high electrical
4 conductivity, availability in abundance and low
Light Output
Power (mW)
Page 127
SEMICONDUCTOR ELECTRONICS
(b) n-p-n transistor: MODES OF OPERATION OF JUNCTION
In this transistor, a thin layer of p-type TRANSISTOR:
semiconductor is placed between two layers A transistor can be operated in three different
of n-type semiconductors. In n-p-n transistor modes:
electrons are majority charge carriers.
a) Common base configuration: Base is
n-p-n transistor, both inside the transistor and common to both the input and output.
in the external circuit the flow of curent is due b) Common emitter configuration: Emitter is
to the flow of electrons only. common to both the input and output.
E B C c) Common collector configuration: Collector
iE
n p n
iC is common to both the input and output.
1) Common Base Configuration: Base is
iB common to both emitter and collector.
VEE VCC PNP
Ie Ic
In any transistor circuit I E I B I C and for
E C
+
change of currents I E I B IC . – Ib RL VCB=output
B –
I B I E and I C I E VEB=input
+
COMPONENTS OF JUNCTION
TRANSISTOR:
Emitter: It is of moderate size and heavily i) Input voltage I e
doped. It supplies a large number of majority
charge carriers. It supplies a large number of ii) Input voltage VEB
majority carriers for the current flow through iii) Output voltage VCB
the transistor.
Base: It is thinnest (order of 10 6 m ) in size iv) Output current I C
and lightly doped. It allows most of the With small increase in emitter-base voltage
charge carriers injected in to it to flow into VEB , the emittter current I e increase rapidly
collector wihtout getting neutralized. due to small input resistance.
Collector: It collects majority charge carriers
supplied by emitter. It is moderately doped v) Input characterstics: If VCB constant,
and larger in size as compare to emitter. curve between I e and VEB is known as input
Normally, the emitter-base contact area is characteristics. It is also known as emitter
smaller than collector-base contact area. characteristics.
SYMBOLS FOR TRANSISTOR: VCB= -10V
Collector Collector
Ie(mA) VCB= -20V VCE=0
Base Base
given by VCE=0
VCE=1V
100 VCE=2V
V 80 VCE=5V
Ri EB
I e VCB cons tan t Ib(A)
60
40
( Ri is of the order of 100 ) 20
vi) Output characteristics: Taking the emitter 0 0.2 0.4 0.6 0.8 1.0
current ie constant, the curve drawn between VBE(volt)
VBE
I C and VCB are known as output Dyamic input resistance Ri I
B VCE cons tan t
characteristics of CB configuration.
Output characteristics: Variation of collector
Ie=30mA
B current I C with VCE can be noticed for VCE
40 A
Ie=20mA
30 A B between 0 to 1 V only. The value of VCE up
IC(in mA)
Ie=10mA
20 A B
Ie=5mA to which the I C changes with VCE is called
10 A B
Ie=0 ICBO knee voltage. The transistor are operated in
the region above knee voltage.
0 -2 -4 -6 -8 -10 -12 -14
VCB (in volt) 70 IB=300A
60 IB=250A
Dynamic output resistance 50 IB=200A
40 IB=150A
V IC(mA) IB=100A
30
R0 CB 20 IB=50A
iC
iC cons tan t 10 IB=0mA
0
2) CE configurations: Emitter is common to 5 10 15 20
both base and collector. VCE(volt)
Page 129
SEMICONDUCTOR ELECTRONICS
The ratio of change in collector emitter voltage PARAMETERS OF JUNCTION
V CE to the change in collector current TRANSISTOR FOR CE CONFIGARATION :
Input resistance(ri) :
I C at constant base current I B is called V
VCE ri BE
output resistance. R0 I I B VCE
C Ib Output resistance(ro) :
Common Collector configuration: V
E ro CE
I C I B
Out put
RL I
C
ac C
I B VCE
+
Common Collector
Parameters Common Base (B) Common Emitter (CE) Common Collector (CC)
Voltage Gain V0 I C RL V0 IC RL V0 I E RL
AV AV AV
Vi I E Ri Vi I B Ri Vi I B Ri
RL RL RL
Av Av Av
Ri Ri Ri
Power Gain P0 P0 P0
Ap Ap Ap
Pi Pi Pi
2 L R RL R
Ap Ap 2 Ap 2 L
Ri Ri Ri
Phase difference Same phase (0) Opposite phase () Same phase (0)
(between output
and input)
Application For High frequency For Audible frequency For Impedance Matching
amplifier amplifier
Page 130
SEMICONDUCTOR ELECTRONICS
amplifier.”
TRANSISTOR AS A DEVICE 2) Voltage amplifier: The amplifier which is
T RANSISTOR AS A SWITCH: used to raise voltage level is known as voltage
IC amplifier.
RB C RC + Ic
B IB Ri
IB V
E – O Vi V0
IE VCC
Vi VBB
ac input RL ac output
VBE VCE
Page 131
SEMICONDUCTOR ELECTRONICS
Voltage gain = -(transconductance output circuit in correct phase to aid the oscillations
resistance) i.e. it provides positive feedback.
Oscillating frequency of oscillator is given by
TRANSISTOR AS AN OSCILLATOR:
1
A transistor with proper positive feedback f .
2 LC
can act as an oscillator. It can generate
oscillations without any external signal source.
FEEDBACK AMPLIFIER:
The circuit needs only a quick trigger signal to
start the oscillations. Once the oscillations Contains two components namely feedback
have started, no external signal source is circuit and amplifier circuit.
needed. The aim of feedback circuit is to return a
In order to get continuous undamped output fraction of the output voltage to the input of
from the circuit, the following condition the amplifier
(Barkhausen criterion) must be met:
mVAV = 1; Input
Transistor
Amplifier Output
where mV = voltage gain of amplifier without
feedback,
Feedback
AV = feedback fraction. network
T1
Mutual inductance FEEDBACK ARE OF TWO TYPES:
(Coupling through
magnetic ? eld)
3
C T2
n-p-n T2 Output
L
4
S1(Switch)
Page 132
SEMICONDUCTOR ELECTRONICS
Page 133
SEMICONDUCTOR ELECTRONICS
IB IC DIGITAL ELECTRONICS:
RB RL ANALOGUE SIGNAL DIGITAL SIGNAL
C
B VCC=VBB= 8V Analogue Digital
E
VCE Signal Analog signal is a continuous Digital signals are discrete time
VBE signal which represents signals generated by digital
physical measurements. modulation.
Waves Denoted by sine waves Denoted by square waves
Solution : Potential difference across RL
Representation Uses continuous range of Uses discrete or discontinuous
8V V CE 8V 4V 4V . Now values to represent values to represent information
information
4
IC R L 4V ; RL 3
103 1k Example Human voice in air, analog Computers, CDs, DVDs, and
4 X 10 electronic devices. other digital electronic devices.
Further for base emitter equation,
VCC 1B RB VVBE or I B RB = Potential LOGIC GATES:
difference across R B
Logic gates serve as the building blocks to
VCC VBE 8 0.6 7.4 V digital logic circuits using combinational logic.
IC 4 X 103 The table used to represent the Boolean
Again, I B 4 X 105 A expression of a logic gate function is commonly
100
called a Truth Table. A logic gate truth table
7.4 shows each possible input combination to the
RB 5
1.85 X 105 185k
4 X 10 gate or circuit with the resultant output
Illustration 21: depending upon the combination of these
In a negative feedback amplifier, the gain input(s).
without feedback is 100, feed back ratio Logic Gates follows certain logical relationships
is 1/25 and input voltage is 50m V. between input and output voltage.
Calculate
(i)gain with feedback The electronic circuits are of two types. They
(ii) feedback factor are analog and digital circuits.
(iii) output voltage Analog circuits:
(iv) feedback voltage The waveforms are continuous and a range of
(v) new input voltage so that output values of voltages are possible.
voltage with feedback equals the output
voltage without feedback Ex: amplifier, oscillator circuits.
Solution:
i) Gain with feedback
A 100 V
Af 20
1 A 1 (1/ 25) 100
1
ii)
25 t
iii) Output voltage O
V0 ' A f Vi 20 50mV 1volt
iv) Feedback voltage Digital circuits:
1 1) The waveforms are pulsated and only
V0 ' 1 0.04 volt discrete values of voltages are possible.
25
v) New increased input voltage Ex: logic gates.
Vi1 Vi 1 A
1
50 1 100 250mV
25
Page 134
SEMICONDUCTOR ELECTRONICS
AND Gate:
+V
i) An AND gate has two or more inputs with
one output.
O
Time ii) The Boolean expression is Y = A.B (Y
equals A and B).
2) in the decimal system, there are ten digits. iii) The output (Y) of AND gate is only when
They are 0,1,2,3,4,5,6,7,8,9. all the inputs are simultaneously 1.
D1
A
3) In the binary system, there are only two
digits 0 and 1. Y
4) Digital electronics is developed by B R
representing the low and high levels of voltages D2
in pulsated waveform with binary digits 0 and +5V
1 (called bits).
5) The basic building blocks of digital circuits
are called as logic gates, since they perform
A 1
B 1
logic operations.
6) Generally the level 1 or high level is at LED
4 1V and level 0 or low level is at
6V
0.2 0.2V .
TYPES OF LOGIC GATES
OR Gate:
i) An OR gate has two or more inputs with NOT Gate:
one output. i) It has a single input and a single output.
ii) The Boolean expression is Y = A + B(Y ii) The Boolean expression is Y A
equals A or B).
iii) The output of NOT gate is the inverse of
iii) The output (Y) of OR gate will be 1 when the input or it performs negation operation.
the inputs A or B or both 1.
Vcc=+5V
D1
A
R
Y = A or B
B
Y
R
D2 A
V=0
O
A 1
O
B 1
6V LED
Page 135
SEMICONDUCTOR ELECTRONICS
A B Y
AND A Y=A.B 0 0 0
B Y 0 1 0
NOR Gate: 1
1
0
1
0
1
X Y
NOT X Y Y=X 0
1
1
0
A B A B Y
A 0 0 1
NOR B Y
Y A B 0 1 0
1 0 0
1 1 0
i) It has two or more inputs and one output. A B Y
A negation (NOT operation) applied after NAND
A
Y Y A.B 0 0 1
B 0 1 1
OR gate, gives a NOT-OR gate or simple 1 0 1
1 1 0
NOR gate.
A B Y
ii) NOR gate output is inverse of OR gate A 0 0 0
XOR B Y Y A.B A.B
output. 0
1
1
0
1
1
1 1 0
The output of NOR gate is 1 only when all
the inputs are simultaneously 0. A B Y
A 0 0 1
XNOR B Y Y A..B A.B 0 1 0
iii) The Boolean expression is Y A B . 1 0 0
1 1 1
NAND Gate:
A
B
Page 136
SEMICONDUCTOR ELECTRONICS
A.(B.C) = (A.B).C = A.B.C
NAMES CIRCUIT DIAGRAM
DISTRIBUTIVE LAW:
X Z A.(B+C) = A.B + A.C
D1
A+(B.C) = (A+B).(A+C)
Y
OR D2 RL
DE MORGAN’S THEOREM:
A B A.B
+5V A.B A B
Illustration 22:
RL
Add f ollowing binary numbers
AND X
D1
Z a) 1101 and 1111 b) 11111 and 101
D2
Soluti on:
Y a) According to binary addition
+5V 1+1=10 and 10+1=11
So 1101
RL
NOT Ri + 1111
Z
X 11100
b) 11111
+ 101
100100
+5V I llustration 23:
RL
Subtract f ollowing binary numbers
NOR a) 1111 and 101 b) 1110 and 101
Z
Ri1 Soluti on:
X a) 1111
Y
Ri2 - 101
1010
+5V b) 1110
- 101
Ri1 Ri2 1001
D1 I llustration 24:
NAND X
Page 137
SEMICONDUCTOR ELECTRONICS
The output of AND gate 2 is A.B Identify the logic operation carried out
by the two circuits.
The output of OR gate is Y A.B A.B A A
Illustration 25: Y Y
B
The diagram of a logic circuit is given B
below. The output of the circuit is Solution:
represented by From fig(a). The output of NAND gate is
w connected to NOT gate ( obtained from
X NAND gate) Let Y 1 be the output of NAND
gate and the final output of the combination of
F two gates is Y. The output of a NAND gate
w
is O only when both the inputs are zero, while
Y in NOT gate, the input gets inverted. Truth
Solution: table for the arrangement
F W X .W Y A B Y' Y A B Y
0 0 1 0 0 0 0
W .W W .Y X .W X .Y
1 0 1 0 = 1 0 0
W W .Y X .W X .Y
0 1 1 0 0 1 0
W 1 Y X .W X .Y W XW X .Y 1 1 0 1 1 1 1
W 1 X X .Y W X .Y It is the truth tables of AND gate. Therefore
the given circuit acts as AND gate.
Illustration 26: b) The output of two NOT gates are
The logic circuit and its truth table are connected to NAND gate Let Y1 and Y2 be
given, what is the gate X in the diagram the outputs of the two NOT gates and the
final output of the combination of three gates
be Y. In a NOT gate. the input gets inverted,
while the output of a NAND gate is ‘O’ only
when both the inputs are zero. Truth table for
given arrangement.
A B Y
A
0 0 0
Y 1 0 1
X 0 1 1
B
Solution: 1 1 1
Illustration 28:
From the truth table we note that Y A B
i.e., it is for OR gate (or) Take X and Y input waveforms. Sketch
the output (Z) waveform obtained from
A X A B A B. A A A A 1 AND gate as per the following conditions:
For t t1 X = 0, Y = 0 Hence Z = 0
then, A X A B. A B. A
For t1 to t2 X = 0, Y = 0 Hence Z = 0
A. 1 B A.B A A.B For t2 to t3 X = 1, Y = 1 Hence Z = 1
For t3 to t4 X = 0, Y = 0 Hence Z = 0
So X A A.B , which is AND gate with
For t4 to t5 X = 0, Y = 0 Hence Z = 0
inputs as A and B
For t5 to t6 X = 1, Y = 0 Hence Z = 0
Illustration 27:
For t to t6 X = 0, Y = 1 Hence Z = 0
You are given two circuits as shown in
figure which consist of NAND gates.
Page 138
SEMICONDUCTOR ELECTRONICS
Solution: 2) NOT gate
t2 t3
yA
t1 t4 t5 t6
3) OR gate
A A
y AB
Illustration 29: B
B AB
Sketch the output Y from a OR gate AB
having inputs X and Y as per the following 4) NOR gate
conditions:
A
For t t1 X = 0, Y = 0 Hence Z = 0 y AB
For t1 to t2 X = 1, Y = 0 Hence Z = 1 B AB
For t2 to t3 X = 1, Y = 1 Hence Z = 1
For t3 to t4 X = 0, Y = 1 Hence Z = 1 5) EX – OR
For t4 to t5 X = 0, Y = 0 Hence Z = 0 A A AB p
For t5 to t6 X = 1, Y = 0 Hence Z = 1 B y pq
For t to t6 X = 0, Y = 1 Hence Z = 1 A
Solution: B B AB q
t1 t2 t3 t4 t5 t6
y pq p q
AB AB AB AB
6) EX – NOR
Illustration 30 : A A
Which represents NAND gate? AB p
B y pq y pq pq
(1)
A
B B AB q
(2)
y pq
(3) AB AB
(4)
A B . A B
Solution:
(1) as NAND is a combination of AND and
NOT gate
A B A B
CONSTRUCTION OF ALL LOGIC AA AB AB BB
GAEST USING NAND AND NOR GATES
BY USING ‘NAND’ Gate : y AB AB
1) AND gate BY USING ‘NOR’ Gate :
y AB 1) OR gate
AB
AB
Page 139
SEMICONDUCTOR ELECTRONICS
2) NOT gate INTEGRATED CIRCUITS
An integrated circuit(IC) (also termed as chip)
is a semiconductor wafer on which thousands
of tiny resistors, capacitors and transistors are
3) AND gate fabricated.
y AB The most widely used technology is the
Monolithic Integrated Circuit.
AB
IC can be used as an amplifier, oscillator,
timer, counter, computer memory.
ICs are of two types i) Analogue ICs ii)Digital
4) NAND gate
ICs.
A The linear IC’s process analogue signals which
AB y AB change smoothly and continuously over a range
B of values between a maximum and a minimum.
The digital IC’s process signals that have only
5) EX – NOR two values. They contain circuits such as logic
gates. Depending upon the level of integration
A (i.e., the number of circuit components or
A B p logic gates), the ICs are termed as follows.
y pq
Based on level of integration:
Small scale integration (logic gates 10 )
AB q Medium scale integration (logic gates 100 )
B
Large scale integration (logic gates 1000 )
Very large scale integration (logic gates >
y pq p.q 1000)
A B . A B
A B . A B
AA AB AB BA
AB AB
6) EX – OR
A
A B p
y pq
y p q
ABq
B
y pq
A B A B
A.B A.B
y AB AB
Page 140
Page 141
COMMUNICATION SYSTEMS
COMMUNICATION SYSTEMS
SYNOPSIS b) Transmission channel: The medium or
Introduction: Communication is an act of the link, which transfers message signal from
exchange of information between the sender and the transmitter to the receiver of a
the receiver. Over decades, methods have been communication system is called channel.
evolved to develop languages, codes, signals c) Receiver: The part of the communication
etc to make communication effective. system, which picks up the information sent out
Communication through electrical signals has by the transmitter is called receiver.
made things much simpler because they can be Basic Terminology Used in Electronic
transmitted over extremely large distances in Communication System
extremely short time as their speed is Some important terms needed to understand
3 108 m / s . the basic elements of communication
Modern communication has its roots in the 19th a) Information : It is nothing but, the message
and 20th century in the work of scientists like to be conveyed. The message may be a
J.C. Bose, F.B. Morse, G.Macroni and symbol, code, group of words etc. Amount of
Alexander Graham Bell. The pace of information in message is measured in “bits”
development seems to have increased b) Communication Channel : Physical
dramatically after the first half of the 20th medium through which signals propagate
century. We can hope to see many more between transmitting and receiving stations is
accomplishments in the coming decades. The called communication channel.
aim of this chapter is to introduce the concepts Transmitter: Essential components of
of communication, namely the mode of transmitter are as follows.
communication, the need for modulation, a)Transducer : Converts sound signals into
production and deduction of amplitude electric signal. The device which converts a
modulation. physical quantity (information) into electrical
Communication is basically of two signal is known as transducer.
types: b)Modulator : Mixing of audio electric signal
a) Point to point :- This takes place between with high frequency radio wave.
a transmitter and a receiver. Telephonic c)Amplifier: Boosting the power of
conversion between two persons is a good modulated signal.
example of it.
d)Antenna : Signal is radiated in the space
b) Broad cast mode :- Here, a large number
of receivers receive the information from a with the aid of an antenna.
single transmitter. Radio and television are
good examples of broadcast mode. Receiver: Basic components of receiver.
Elements of Communication a) Pickup antenna: To pick the signal
b) Demodulator: To separate out the audio
System
signal from the modulated signal
Basic units of a communication system.
c) Amplifier: To boost up the weak audio
Transmitted signal Message signal
signal
d) Transducer: To convert back audio signal
Information
source
Trans-
mitter
Channel Receiver
User
Information
in the form of electrical pulses into sound
waves.
Message Received
signal
Noise
signal Message Signal: Information converted in
a) Transmitter: The part of t he electrical form and suitable for transmission is
communication system, which sends out the called signal.
information is called transmitter.
Page 142
COMMUNICATION SYSTEMS
superposition of sinusiodal waves of frequencies
A signal is defined as a single-valued function
of time (that conveys the information) and f0 . 2 f 0 ,3 f 0 , 4 f 0 ....nf 0 where n is an integer
which, at every instant of time has a unique extending to infinity and f 0 1/ T0 . The
value.
Types of message signals fundamental f 0 , fundamental f 0 second
a) Analog signal: A signal, which is a
harmonic 2 f 0 and fundamental
continuous function of time (usually a sinusoidal
function) is called analog signal. f0 second harmonic 2 f0 third
b) Digital signal: A discrete signal
harmonic 3 f 0 , are shown in the same figure
(discontinuous function of time) which has only
two levels is called digital signal. to illustrate this fact. It is clear that to reproduce
Noise : This refers to undesired signals which the rectangular wave shape exactly we need to
superimpose all the harmonics
disturb the transmission and processing of
signals. f 0 , 2 f 0 , 3 f 0 , 4 f 0 ..... which implies an infinite
Attenuation : It is the loss of strength of a bandwidth. However, for practical purposes,
signal during propagation in a medium. the contribution from higher harmonics can be
Amplification : It is the process of increasing neglected this limiting the bandwidth. As a result,
the strength of the signal (amplitude) using an received waves are a distorted version of the
amplifier. transmitted one. If the bandwidth is large enough
Range :It is the maximum distance from a to accommodate a few harmonics, the
source upto which the signal is received with information is not lost and the rectangular signal
sufficient strength. is more or less recovered. This is so because
Repeaters : These are the devices used to the higher the harmonic. Less is its contribution
increase the range of communication system to the wave form.
Band width of signals (speech, T.V and (voltage)
-0.6
digital data) : Band width is the frequency
range over which an equipment operates. -0.4
(or) -0.2
(time)
It is the portion of the spectrum occupied by 0
unit
0.5
the signal. -0.2
0.1 0.2 0.3 0.4
Bandwidth of signals :
-0.4
In a communication system, the message
-0.6
signal may be voice, music, picture or data etc.
Each of these signals has a spread of different a) Rectangular wave
range of frequencies. Hence, the type of
communication system needed depends upon b) Fundamental f 0
the band of frequencies involved. Speech signal c) Fundamental f0 + second harmonic
requires the band width of 2800 Hz (3100 Hz
to 300 Hz). For music, a bandwidth of about 2 f0
20KHz is required (due to high frequency
produced by musical instruments). The audible d) Fundamental f 0 Second harmonic
range of frequencies extends from 20Hz to
20KHz. Video signals require band width of
2 f 0 third harmonic 3 f 0
4.2 MHz for picture transmission. However, a Bandwidth of transmission medium :
band width of 6MHz is needed for T.V signals. The most used transmission media are wire,
(as it contains both voice and picture) free space, and fibre optic cable. Different
Digital signals are in the form of rectangular transmission media offer different band width.
waves as shown in Fig. One can show that this Coaxial cable offers a band width of about 750
rectangular wave can be decomposed into a MHz. Radio wave communication through free
Page 143
COMMUNICATION SYSTEMS
space takes place over a wide range of · 2) Line communication
frequencies from 100kHz-GHz. i) Two wire transmission line
Service Frequency bands Comments ii) Coaxial cable
----------------------------------------------------------- iii) Optical fibre cable
Standard 540-1600 kHz (1)SPACE COMMUNICATION
AM broadcast
--------------------------------------------------- Propagation of EM waves in the atmosphere
FM broadcast 88-108 MHz
--------------------------------------------------- The communication process utilizing the physical
Television 54-72 MHz VHF(Very high space around the earth is termed as space
frequencies) communication. Electromagnetic waves which
76-88 MHz TV
174-216 MHz UHF(ultra are used in Radio, Television and other
high communication system are radio waves and
frequencies) microwaves.
420-890 MHz TV
--------------------------------------------------- The velocity of electromagnetic waves of
Cellular Mobile 896-901 MHz Mobile to base different frequency in a medium is different. It
station is more for red light and less for violet light.
840-935 MHz Base station to
Electromagnetic waves are of transverse nature.
mobile
--------------------------------------------------- Ground wave propagation : In this method,
Satellite 5.925-6.425 GHz the radio waves are guided along the surface.
Uplink The wave induces charges on the earth.
Communication 3.7 - 4.2 GHz These charges travel with the wave and this
Downlink forms a current. Now the earth behaves like
--------------------------------------------------- a leaky capacitor in carrying the induced
Optical communication using fibres is current. The wave loses some energy, as
performed in the frequency range of 1 THz to energy is spent due to flow of charge through
1000 THz (microwaves to ultraviolet). the earth’s resistance. The wave also looses
An optical fibre can offer a transmission band energy due to diffraction as it glides along the
width in excess of 100 GHz. ground. The loss of energy increases as the
COMMUNICATION CHANNELS: frequency increases. Thus ground
propagation is suitable upto 2MHz. As they
The medium or the link, which transfers loose energy they cannot go to long distances
message signal from the transmitter to the on the ground. Maximum range of the ground
receiver of a communication system is called wave can be increased by increasing the
Communication channel. power of the transmitter.
(i) Space communication Sky wave propagation : Above 2MHz and
(i) Ground wave propagation upto 30MHz, long distance communication
takes place through ionosphere. The ionosphere
(ii) Space wave propagation. (Tropospheric reflects the radio waves back to the earth. This
wave propagatio n. Surface wave method is called sky wave propagation. It is
propagation.) used for shortwave broad casting services.
Ionosphere is a thick blanket of 65 km to 400
(iii) Sky wave propagation: A new dimension km above the earth’s surface. UV rays and
recently added to space communication is other higher energy radiation coming from
space results in the ionization of air molecules.
satellite communication.
The ionosphere is further divided into sereval
layers as shown in table below. It should be
Page 144
COMMUNICATION SYSTEMS
understood that degree of ionization changes radio wave enters the ionosphere from the
with height. This is because the density of underlying unionized medium. Since the
atmosphere decreases with height. At great refractive index of ionosphere decreases from
heights, the radiation is intense, but the D layer to F2 layer, consequently, the incident
molecules available are few. On the other hand, ray will move away from the normal drawn at
near the earth’s surface the molecular the point of incidence following the ordinary
concentration is high but the intensity of radiation laws of refraction
is low and thus again the ionization is low.
Logically, the peak of ionization density occurs
Ionosphere
at some intermediate heights. The ionosphere
acts as a mirror (reflector) for frequencies of
3-30 MHz. Electromagnetic waves of
frequences greater than 30 MHz pass through
Unionized medium
the atmosphere and skip.
Radio wave
The process of bending of EM waves is similar
to total internal reflection in optics. The bending During the propagation in ionosphere the angle
of waves can be easily explained on the basis of refraction gradually increases and the ray
of variation of refractive index of the ionosphere goes on bending more and more till at some
with change in electron density. Suppose that a
Page 145
COMMUNICATION SYSTEMS
point, the angle of refraction becomes 900 and direct waves get blocked, at some point due to
the wave travels parallel to the earth surface. the curvature of the earth as shown in the figure.
This point is called point of reflection. Then the
ray tends to move in the down ward direction
dm
and comes back to earth because of symmetry.
dT dR
Super high frequency (SHF) waves propagate
as sky waves taking reflection at satellite.
Ionosphere
hT hR
Page 146
COMMUNICATION SYSTEMS
= (Area covered by the tower) Population As h R we can ignore h 2
density.
d 2 2 Rh and d 2 Rh
No . of persons d 2 covered Range of TV transmission depends upon the
population density (Here d = radius of the area height of the transmission antenna. Broadcasts
covered by single transmitting tower of height are made from tall transmitting antenna.
hT ) Satellite Communication: Long distance
Television broadcast, microwave and satellite communication beyond 10 to 20 MHz was not
communications are a few examples of possible before 1960 because all the three
communication systems that use space wave modes of communication discussed above failed
propagation. The figure below illustrates the (ground waves due to conduction losses, space
various modes of wave propagation. wave due to limited line of sight and sky wave
due to the penetration of the ionosphere by the
high frequencies beyond fc ).
Satellite communication made this possible. The
basic principle of satellite communication is
shown in figure. A communication satellite is a
spacecraft placed in an orbit around the earth.
The frequencies used in satellite communication
lie in UHF/ microwave regions. These waves
can cross the ionosphere and reach the satellite.
Satellite-1
Range of TV transmission : Geostationary
orbit
As the frequency range of TV signals is 100-
200 MHz, such signal transmission via ground
waves is not possible. In such situations, we Earth
use line of sight transmission.
P Satellite-3 Satellite-2
A
MODULATION AND ITS NECESSITY :
Message signals are also called base band
signals. Which essentially designate the band of
O
frequencies representing the original signal, as
Let CP be the TV tower on the earth’s surface. delivered by the source of information. No
It’s antenna is at P. Let PC = h. When TV signal, is a single frequency sinusoid, but it
broadcast is made, the signal can reach the earth spreads over a range of frequencies called the
upto A to B. There will be no reception of the signal bandwidth to transmit an electric signal
signal beyond A and B. Arc length CA and CB frequency less than 20 kHz) over a long distance
is the range of TV transmission. If O is the directly. It is clear that low frequency waves,
centre of the earth, OA = OB = R is the radius can not travel long distances. Hence, to transmit
of the earth, from right angled triangle OAP low frequency wave over long distance, we take
OP 2 OA2 PA2 the help of high frequency waves called carrier
2 wave. The low frequency wave is superposed
h R R 2 PA2 over high frequency carrier wave. This process
PA PB d is called the modulation. The low frequency wave
2 is called the modulating wave and the high
h R R2 d 2 frequency wave is called the carrier wave, and
h2 R 2 2Rh R 2 d 2
Page 147
COMMUNICATION SYSTEMS
the resultant wave is called modulated wave.
In this section we will discuss in detail about Source of
Modulator
modulation. What is it? What is the need of Original Information Baseband Modulated
signal signal signal
modulation or how modulation is done etc. Carrier
wave
No signal in general, is a single frequency but it
spreads over a range of frequencies called the Oscillator
signal bandwidth. Suppose we with to transmit
an electronic signal in the audio-frequency TYPES OF MODULATION:
(20Hz-20kHz) range over a long distance. Can The carrier wave may be continuous (sinusoidal)
we do it? No it cannot because of the following or in the form of pulses as shown in figure(2).
problems.
Size of antenna : For transmitting a signal we Time period T
2
T
need an antenna. This antenna should have a Amplitude
time
size comparable to the wavelength of the signal. Pulse Pulse (a)
For an electromagnetic wave of frequency rise duration Pulse fall
20kHz, wave length is 15km. Obviously such Pulse
Amplitude
a long antenna is not possible and hence direct
transmission of such signal is not practical. (b)
The linear size of the antenna must be the order Therefore depending upon the specific
of the wave length and for effective transmission characteristic of carrier wave which is being
varied in accordance with the message signal,
its length must be h = modulation can basically be differentiated as
4
so that antenna properly senses the time (i) continuous wave modulation; and
variation of the signal. (ii) pulse wave modulation.
Example1: For an electromagnetic wave of According to the type of modulation
f = 20 kHz, = 15 km Obviously, such a long For sinusoidal continuous carrier waves
antenna is not possible to construct and operate. i) Amplitude Modulation (AM)
ii) Frequency Modulation (FM)
Hence direct transmission of such baseband
signals is not practical. iii) Phase Modulation
For pulsed carrier waves
Example2: If f = 1 MHz, then = 300 m
i) Pulse Amplitude Modulation (PAM)
h = 75 m ii) Pulse Time Modulation (PTM)
Therefore, there is a need of translating the a) Pulse Position Modulation (PPM)
information contained in our original low b) Pulse Width Modulation (PWM) or Pulse
frequency baseband signal into high or radio Duration Modulation (PDM)
frequencies before transmission. iii) Pulse Code Modulation (PCM)
Effective power radiated by an antenna
Power radiated by an antenna is proportional MODULATION
l
to 2 . Where l is length of the antenna . Continuous wave Pulse wave
modulation modulation
For a good transmission, high powers are
required, hence low wavelength i.e high Amplitude Frequency Phase
frequency transmissions are needed. modulation modulation modulation
Page 148
COMMUNICATION SYSTEMS
I) Continuous Wave Modulation I. Continuous Wave Modulation:
1) Amplitude Modulation:
Equation representing sinusoidal carrier wave
can The method in which the amplitude of carrier is
c t Ac sin c t ----(1) varied in accordance with the modulating signal
where c(t) is the signal strength (voltage or keeping the frequency and phase of carrier
current), wave constant is called amplitude modulation
Ac is the amplitude (AM).
t is called argument of Phase angle of
c Here we explain amplitude modulation process
the carrier wave using a sinusoidal signal as the modulating signal.
c 2 f c is the angular frequency
Let m t Em sin m t represent the message
is the initial phase of the carrier wave. During
or the modulating or base band signal. Here
the process of modulation, any of the two
parameters, viz amplitude or phase angle, of m 2 f m is the angular frequency of the
the carrier wave can be controlled by the message signal.
message or information signal. This results in
+
two types of modulations:
i) Amplitude modulation (AM) 0
ii) Angle modulation _
Angle modulation again can be of two types. A-F Signal
They are
i) Frequency modulation (FM) c t Ec sin ct represent carrier wave. Here
ii) Phase modulation (PM) c 2 f c is the angular freqency of the carrier
As shown in figure. signal
1
c(t) 0 (a)
-1 Wave
0 0.5 1 1.5 2 2.5 3 + One
1 Cycle One
Current, or Power
Relative Voltage
m(t) 0 (b)
Cycle
-1
0 0.5 1 1.5 2 2.5 3
2 Peak
cm(t) for AM 0 (c) 0 to peak
-2 Amplitude
0 0.5 1 1.5 2 2.5 3
1
cm(t) for FM 0 (d) _ Time Time Axis
-1 Starting point
0 0.5 1 1.5 2 2.5 3
1
cm(t) for PM 0 (e)
-1
0 0.5 1 1.5 2 2.5 time 3 The modulated signal cm t can be written as
II) Pulse Wave Modulation.
The significant characteristics of a pulse are: i) cm t Ec Em sin mt sin ct ----(1)
Pulse amplitude E
ii) Pulse duration or pulse width cm t Ac 1 m sinmt sinct -----(2)
iii) pulse position (denoting the time of rise or Ec
fall of the pulse amplitude) as shown in figure
cm t Ec sin ct Ec sin mt sin c t --(3)
(3).
TYPES OF PULSE MODULATION: Using the trigonometric relation
a) pulse amplitude modulation (PAM), 1
b) pulse duration modulation (PDM) or pulse sin A sin B
2
cos A B cos A B ,
width modulation (PWM)
c) pulse position modulation (PPM). we can write cm t of equation (3) as
Page 149
COMMUNICATION SYSTEMS
As long as the broadcast frequencies (carrier
Ac Ac
cm t Ac sinct cos c m t cos c m t - waves) are sufficiently spaced out so that
2 2 sidebands do not overlap, different stations can
----(4) Here operate without interfering with each other.
c m 2 f c f m = Lower side band Special cases of Amplitude modulation:
frequency (LSB) CaseI: In the absence of signal.
A A
c m 2 f c f m =Upper side band + No Signal =
A
frequency (USB) Carrier Signal A.M. Wave
Here (m or Em / Ec ) is the modulation index;
O
(or) modulating factor. Modulation factor ma x100 0%
A
In practice, is kept 1 to avoid distortion.
Case-II: When the signal amplitude is equal to CW
Am wave.
Depth of modulation = A 100 100
C Amplitude varies from 2A to zero.
Depth of modulation interms of Emax and Emin
2A
A A
+ =
mEC A
0 .5 A
Modulation factor = 0 .5
The Band width of AM wave is " 2fm " A
The modulated signal now consists of the carrier 50 %
Case-IV: When the amplitude of signal is 1.5 times
wave of frequency c plus two sinusoidal
that of the CW.
waves each with a frequency slightly different Amplitude of the modulated wave changes from
fromc , known as side bands. The frequency 2.5 A to A
spectrum of the amplitude modulated signal is 2.5A
shown in A
+
1.5A
=
Page 150
COMMUNICATION SYSTEMS
Note:A carrier wave is modulated by a number of 2
sine waves with modulation indices m1, m2 and Ps ma 1
2
m3. The total modulation index of the wave is PT 2 ma 3 and
m m12 m22 m32
Pc 2 2
Power out put in AM wave 2
PT 2 ma 3
Pt Pc Ps
where Pt is power transmitted m2
Transmission Efficiency
Pc is power of carrier wave 2 m2
PRODUCTION OF AMPLITUDE
Ps is total power of side bands
MODULATED WAVE
The equation of a carrier wave Amplitude modulation can be produced by a
Yc Ac sin wc t variety of methods. A conceptually simple
Power of carrier wave method is shown in the block diagram of Fig.
2 BANDPASS AM
Ac m(t) y(t) SQUARE y(t) FILTER Wave
+
Pc
Arms 2
2 A
c
2 Am sin m t
(Modulating
LAWDEVICE CENTRED
AT c
2
R R 2R Signal) c(t) Bx(t)+Cx(t)
Ac sin ct
The power of side bands = The power of lower (carrier)
side band + the power of upper side band
2
Here the modulating signal Am sin mt is added
mAc Ac
2
Page 151
COMMUNICATION SYSTEMS
retains the frequencies c , c m and the power goes waste in the transmitted
carrier, which contains no useful information.
c m . The output of the band pass filter 3) Due to a large number of side bands,
therefore is of the same form as equation(4) FM transmission can be used for the
and is therefore an AM wave. stereo sound transmission.
It is to be mentioned that the modulated signal Disadvantage of FM:
cannot be transmitted as such. The modulator 1) The bandwidth in FM transmission is
is to be followed by a power amplifier which about 10 times as large as that needed in
provides the necessary power and then the AM transmission.As a result, much wider
modulated signal is fed to an antenna of frequency channel is required in FM
appropriate size for radiation as shown in transmission.
figure(7). 2) FM reception is limited to line of sight.
Due to this, area of reception for FM is
Transmitting
Antenna
much smaller than that for FM.
3) FM transmitting and receiving
m(t)
Amplitude Power equipments are very complex as
Message
Modulator Amplifier compared to those employed in AM
signal
transmission.
Carrier
Detection of amplitude modulated
Figure(7)
wave
A-M Transmitter. The transmitted message gets attenuated in
In the block diagram of the AM transmitter the propagating through the channel. The receiving
r-f section consists of an oscillator feeding a antenna is therefore to be followed by an
buffer, which in turn feeds a system of frequency amplifier and a detector. In addition, to facilitate
multipliers and/or intermediate power amplifiers. further processing, the carrier frequency is
If frequency multiplication is unnecessary, the usually changed to a lower frequency by what
buffer feeds directly into the intermediate power is called an intermediate frequency (IF) stage
amplifiers which, in turn, drive the final power preceding the detection. The detected signal
amplifier. The input to the antenna is taken from may not be strong enough to be made use of
the final power amplifier. and hence is required to be amplified. A block
diagram of a typical receiver is shown in figure.
Antenna
Int Receiving
Power Antenna
CSC Buffer Power
Amp
Amp
If Output
Amplifier Detector Amplifier
Audio A-M Received stage
Amp Mod Signal
Mike
Illustration-1:How many AM broadcast stations
Amplitude-Modulated Transmitter
can be accommodated in a 100 kHZ
Advantages of FM over AM bandwidth if the highest modulating
1) FM reception is quite immune to noise as frequency of carrier is 5 kHZ?
compared to AM reception. In FM receivers, Sol. Any station being modulated by a 5 kHz signal
the noise can be reduced by increasing the will produce an upper side frequency 5 kHz
frequency deviation. above its carrier and a lower side frequency 5
2) FM transmission is highly efficient as kHz below its carrier, thereby requiring a
compared to AM transmission. In FM bandwidth of 10 kHz. Thus, Number of stations
transmission, all the transmitted power is accommodated
useful, whereas in AM transmitted most of
Page 152
COMMUNICATION SYSTEMS
making height of the tower four times to 640m.
Total bandwidth 100
= =10 So, height of the tower should be increased by
Bandwidth per station 10 480 m.
Illustration-2: How many 500 kHz waves can Illustration -5: An audio signal given by
be on a 10km transm ission line
es 15sin 2 200t modulates a carrier
simultaneously?
Sol: Let be the wavelength of 500 kHz signal. wave given by es 60sin 2 100, 000t . If
c 3.0 108 calculate
Then, 5
m 600m a) Percent modulation
f 5.0 10
b) Frequency spectrum of the modulated
The number of waves on the line can be found wave.
from,
Sol: a) Signal Amplitude, B 15
d 10 103 Carrier amplitude, A 60
n 16.67
600 B 15
Illustration-3: A two wire transmission line has m 0.25
A 60
a capacitance of 20 pF/m and a
Percentage modulation
characteristic impedance of 50
0.25 100 25%
a) What is the inductance per meter of this b) By comparing the given equations of signal
cable? and carrier with their standard form
b) Determine the impedance of an
infinitely long section of such cable. es Es sin s t Es sin 2 f s t and
Sol: a) The characteristic impedance. Z L / C ec Ec sin c t Ec sin 2 f ct
2
L Z 2 C 50 20 1012 H 0.05H / m we have signal frequency f s 2000 Hz and
b) The characteristic impedance of a carrier frequency f c 100, 000 Hz
transmission line is the impedance that an infinite The frequencies present in modulated wave
length of line would present to a power supply i) f c 100, 000 Hz 100kHz
at the input end o f t he line. Thus,
ii) f c f s 100, 000 2000 98kHz
Z Z 0 50
Illustration -4: T.V. transmission tower at a iii) f c f s 100kHz 2kHz 102kHz
particular station has a height of 160m. Therefore, frequency spectrum of modulated
a) What is the coverage range? wave extends from 98kHz to 102 kHz is called
b) How much population is covered by band width.
transmission, if the average population Illustration-6: The antenna current of an AM
density around the tower is 1200 per km 2 ? transmitter is 8A when only the carrier is
c) What should be the height of tower to sent but it increases to 8.93A when the
double the coverage range carrier is modulated. Find percent
modulation.
Sol: a) Coverage range d 2 Rh Sol. The modulated or total power carried by AM
2 6400 103 160m 45.254km m2
P
wave T PC 1
b) Population covered
2 . If R is load
population density area covered
resistance. I m is the current when carrier is
1200 d 2
77.17 lac modulated and I c t he current when
c) Coverage range h unmodulated, then
Therefore coverage range can be doubled by
Page 153
COMMUNICATION SYSTEMS
1/ 2
PT I m2 R m 2 I m2 R 0.6 2
; I 2 I m 6 1 6 1.086 6.52 Amp
PC I c2 R 2 Ic R 2
Given I m 8.93 A, I c 8 A Illustration -9: A carrier wave of 1000 W is
8.93 2 subjected to 100% modulation. Calculate
2
m 2 1 m 0.7 (i) Power of modulated wave, (ii) power is
8.0 USB, (iii) power is LSB
Therefore, percentage modulation = 70% Sol: i) Total power of modulated wave
Illustration-7: A sinusoidal carrier voltage of m2
80 volts amplitude and 1 MHz frequency PT PC 1 ;
is amplitude modulated by a sinusoidal 2
voltage of frequency 5kHz producing 50%
modulation. Calculate the amplitude and 12
1000 1 1500watt
frequency of lower and upper side bands. 2
Sol: Amplitude of both LSB and USB are equal
1
and given by ii) Power in USB = PSB
2
mEc 0.5 80
20volts Where power carried by side bands is given
2 2 by amplitude modulation and detection
Now frequency of LSB = f c f s
m2 12
PSB PC ; 1000 500 watt
1000 5 kHz 995kHz 2 2
Frequency of USB = f c f s 1 1
PUSB PSB 500 250 watt
1000 5 kHz 1005kHz 2 2
iii) Since power in LSB = Power in USB
Illustration -8: The load current in the
transmitting antenna of an unmodulated PLSB PUSB 250watt
AM transmitter is 6 Amp. What will be the Illustration-10: A transmitting antenna at the
antenna current when modulation is 60%. top of a tower has a height 32m and the
Sol: Total power carried by AM wave height of the receiving antenna is 50m.
m2 What is the maximum distance between
PT PC 1 ...... 1 them for satisfactory communication in
2
LOS mode? Given radius of earth
Where Pc is the power of carrier component 6.4 106 m .
and m is the modulation factor. If R is the Sol:
resistance, I m the antenna load current when d 2 64 105 32 2 64 10 5 50 m
m
modulation is 60% and I c is the antenna load
64 102 10 8 103 10m
current when un modulated, then
144 10 2 10m 45.5km
PT I m2 R m 2 I m2
2 ,1 2 using (1)
PC I c R 2 Ic Illustration-11: A message signal of frequency
10 kHz and peak voltage of 10 volts is used
m 2
to modulate a carrier of frequency 1 MHz
or Im Ic 1 and peak voltage of 20 volts. Determine
2
a) modulation index
Given I c 6 Amp, m 0.6
Page 154
COMMUNICATION SYSTEMS
Page 155
Page 156
MOVING CHARGES & MAGNETISM
B
i7
i5
dl
ii) Ampere’s right hand thumb rule : i6
i2
When a straight conductor carrying current is i1
i4
i3
held in the right hand, such that the thumb is
pointing along the direction of current, then the Consider a closed plane curve as shown in
direction in which fingers curl round it, gives figure. dl is a small length element on the curve.
the direction of magnetic lines of force.
Page 157
MOVING CHARGES & MAGNETISM
Let B be the resultant magnetic field at the
page. Find the B dl along the loop.
position of dl . If the scalar product B dl is Sol: According to Ampere’s circuital law (integrating
in clockwise manner)
integrated by varying dl on the closed curve, it
is called line integral of B along the curve and it B.dl 0ienclosed
is represented by B dl. ; B.dl 0 i8 i5 i2 i4
The rule for deciding whether an enclosed
current is positive or negative : The fingers 5
of the right hand are to be taken in the direction 2 4
of integration around the path, keeping thumb 8
1 6
perpendicular to curling fingers. If a current 3
pierces the membrane stretched across the area 7
in the direction of the thumb, then it is positive since, all the wires carry same current of i0 ,
current. If the current pierces the membrane in
the opposite direction, then it is negative. For we have B.dl 2 0 i0 .
the above closed path Intensity of magnetic induction (B)
B.dl (i i
0 1 2 i3 i4 i6 ) near a long straight conductor :
Points to remember regarding Ampere’s Consider an infinitely long wire carrying current
Law: i as shown in figure. P is a point at a
a) The line integral does not depend on the shape perpendicular distance r from the conductor.
of the closed path or on the position of the The magnetic induction field produced by the
current carrying wire in the loop. conductor is radially symmetric i.e., magnetic
b) If a conductor carrying current, is outside the lines of force are concentric circles centered at
closed path, the line integral of B due to that the conductor. The tangent drawn to the line
conductor is zero i.e., we need not consider of force at any point gives the direction of
the currents that do not pierce the area of the
magnetic induction field B at that point. dl is a
closed path.
c) Ampere’s circuital law is always true, no matter small element on the circle of radius r and angle
how distorted the path or how complicated may between B and dl is 00 every where on this
be the magnetic field. In most cases even path for anticlockwise integration.
though Ampere’s circuital law is true, it is From Ampere’s circuital law
inconvenient, because it is impossible to perform
the path integral. However in few special
symmetric cases it is easy to perform path dl
integral using Ampere’s law. p
r
d) Ampere’s circuital law is applicable for
conductors carrying steady current. B
e) Ampere’s circuital law is analogous to Gauss
B.dl 0i Bdl cos 0
0
law in electrostatics. 0i
f) Ampere’s circuital law is not independent of
Biot-Savart’s law. It can be derived from Biot- B dl 0i
Savart’s law. Its relation with Biot-Savart’s Law
is similar to the relation between Gauss Law 0i
B (2 r ) 0 i B
and Coulomb’s Law in electrostatics. 2 r
Illustration -1: Eight wires cut the page Here r must be much less than the length of
perpendicular to the points shown. Each conductor.
wire carries current i0 . Odd currents are
out of the page and even currents into the
Page 158
MOVING CHARGES & MAGNETISM
i0
i Where i . (r 2 a 2 )
Here i ' J . r 2 , Where J (b a 2 )
2
R2
i0 (r 2 a 2 )
B
b2 a 2
dl
0i0 (r 2 a 2 )
then B arb
R 2 (b 2 a 2 )r
Page 159
MOVING CHARGES & MAGNETISM
Page 160
MOVING CHARGES & MAGNETISM
tan y / d 1
B
d
y d tan dy d (sec 2 ) d d
0 i
900 , 00 ; B
4 d i
= 90
0
d
vi)a) Magnetic induction at the centre of current
carrying wire bent in the form of square of side
‘a’ is
where 90 1 and 90 2
i
Bnet 4 Bside
Page 161
MOVING CHARGES & MAGNETISM
450
Sol: Bnet 4 Bside
Bnet 3B eachside
i
0 i 0 I
Bnet 3 (sin 600 sin 60 0 ) Bnet 4 (sin 450 sin 450 )
4 r 4 L / 2
a
where r 4
0
I
( 2)
0 8 2 2
2 3 4 L/2 4 1 0 1
0 i 1 0 7 1 6 2
B 18
4 a 1 0 1
c) Magnetic induction at the centre of current
16 2T
carrying wire bent in the form of hexagon of
side ‘a’ is given by Illustration-4: If a thin uniform wire of length
1m is bent into an equilateral triangle and
carries a current of 3A in anticlockwise
direction, find the net magnetic induction
at the centroid.
Bnet 6 B eachside I 0 0
30 30 o oSol: Bnet 3 0 (sin 60 sin 60 )
4 r
0i
Here 300 ; B4 3
4 a
Illustration -2:Find the magnetic induction
due to a straight conductor of length 16cm
carrying current of 5A at a distance of 6cm
from the midpoint of conductor.
a 0 I
I 8 4
Sol: B 0 (sin sin ) but sin r 3 (2 sin 60 0 )
4 r 10 5 2 3 4 r
I (2 3) 3 I
10cm 3 0 2 18 0
8cm 4 a 2 4 a
6cm
P 3
B 18 10 7 54 3 10 7 T
8cm 10cm
1/ 3
Illustration -5: A large straight current
5 4 40 carrying conductor is bent in the form of
B 10 7 2 T
2 L shape. Find B at P..
6 10 5 3
Page 162
MOVING CHARGES & MAGNETISM
y
0 I I ln 4
ln 2k 0 k
x 2 3 a 4 3a
1
a P
1
Illustration-7: Find the magnetic field at P due
a
2 2 to the arrangement shown
Sol: Let us divide the conductor into two semi infinite
segments 1 and 2. Then, induction at P is
y 900
x
1
a P
d
1
450
a
2 2
P
0 I
B B1 B 2 ..i Sol: Bnet 2 sin sin
4 r 2 4
0i 0 0
B1 (sin(90 1 ) sin 90 )k ..ii
4 a d I 1
here r ; Bnet 0 1
i 2
B2 0 (sin(900 2 ) sin 900 )k ..iii 2 d 2
4 a Illustration -8: A pair of stationary and
i
then B 0
(cos 1 cos 2 2) k , infinitely long bent wires are placed in the
4 a x-y plane as shown in figure. The wires
1 carry current of 10 ampere each as shown.
where cos 1 cos 2
2 The segment L and M are along the x-axis.
0ik The segment P and Q are parallel to the Y-
Hence, B (2 2) axis such that OS=OR=0.02m. Find the
4 a
Illustration -6: Infinite number of straight magnitude and direction of the magnetic
wires each carrying current I are equally induction at the origin O.
Q
placed as shown in the figure. Adjacent
wires have current in opposite direction.
I
Find net magnetic field at point P ?
I R O
L M
a S I
y I
a 0
30
P 0 x
30 2 3 4 5 P
z Sol: Since point O is along the length of segment L
and M the field at O due to these two segments
will be zero
Sol: Magnetic field at O is due to QS and RP..
0 I 1 1 1 1
Bnet (sin 300 sin 30 0 )k .....
4 d 2d 3d 4d 0 I 10
BSQ 10 7
3a 4 OS 0.02
Where d a cos 30
2 0 I 10
BRP 10 7
0 I 1 1 1 4 OR 0.02
Bnet k 1 .....
2 3 a 2 3 4 10
B0 BSQ BRP 10 7 2 10 4 T
0.02
Page 163
MOVING CHARGES & MAGNETISM
z
Illustration-9: An equilateral triangle of side y
length l is formed from a piece of wire of
uniform resistance. The current I is as
shown in figure. Find the magnitude of the
magnetic field at its centre O.
x
Page 164
MOVING CHARGES & MAGNETISM
same direction, the net field is also in this idl r idl
dB 0 0 2
direction. It can, therefore, be obtained by 4 r 3 4 r
integrating above equation under proper limits.
The direction of dB is perpendicular to the
0i
Thus, B dB dl
plane formed by r and dl .
4 R 2
If the coil has N turns dl 2 RN In case of a point P on the axis of circular coil,
for every current element ‘idl’ there is a
i
0 0 i iN
0 symmetrically situated opposite element. The
B 4 R 2 dl 4 R 2 2 RN 2 R component of the field dB perpendicular to the
a) If the current is in clock wise direction, then the axis cancel each other, while components of
magnetic field produced is normally inwards and the field dB along the axis add up and contribute
the face of the coil behaves as south pole. to the net magnetic field.
0 idl
i.e., B dB sin sin
4 r 2
Here r is same for all elements and also
b) If the current is in anti clock wise direction, then sin ( R / r ) so,
the magnetic field produced is normally
0 idl idl R
outwards and the face of the coil behaves as B 2
sin 0 2
4 r 4 r r
iR
0 3 dl
north pole. 4 r
for a loop dl 2 R and as
Field at an axial point of a circular
loop: r 3 ( x 2 R 2 )3/ 2 ;
0 2 iR 2 0iR 2
B
4 ( x 2 R 2 )3 / 2 2( x 2 R 2 )3/ 2
The direction of magnetic field B is along the
axis of the loop.
i) The magnetic field B varies non linearly with
distance x from centre as shown in figure.
Consider a circular poop of radius R, carrying
current in yz plane with centre at origin O. Let
P be a point on the axis of the loop at a distance 0 Ni
‘x’ from the centre ‘O’ of the loop. 2R
Consider a conducting element dl of loop.
According to Biot-Savart’s law, the magnitude
of magnetic field due to the current element is
0 idl r
dB where r x 2 R 2
4 r 3 For a coil having N turns, dl 2 RN
Here the element dl is in yz plane where as the
0 NiR 2
position vector r from dl to the point P is in so, B .
2( x 2 R 2 )3/2
xy plane. So idl r idl r It is maximum when x 2 0 , i.e., at the centre
of the coil whose value is given by
Page 165
MOVING CHARGES & MAGNETISM
0 2 NI 0 Ni magneton ( ) .
B
4 R 2R From Bohr’s second postulate, for an electron
revolving in first orbit of hydrogen atom
P P h
me vr (n 1)
B 2
B
where h = Planck’s constant, me mass of
2 NiR 2 0 2 NiA
ii) if x R ; B 0 evr e h eh
4 x3 4 x 3 electron 2 2 2 m 4 m
e e
Where A R 2 , area of the coil.
e
( ) min h
4 me
Circular current loop as magnetic 1.60 10 19 6.63 10 34
dipole:
4 3.14 9.11 10 31
0 2 NiA 9.27 1024 Am 2
From the above expression B
4 x3 This value is called the “Bohr magneton”
i) In cases of charged particle having charge q
o 2M and moving in a circle of radius R with velocity
Comparing with B (which is the
4 x3 v
magnetic field at an axial point of a short bar v, current i q f q
2 R
magnet)
but M i A where A R 2 nˆ
a) Magnetic moment of the circular current
1
carrying coil is M Ni A whereA r 2 nˆ ; M qvRnˆ
2
b) M is independent of shape of the coil ii) Angular momentum of the charged particle in
Current loop behaves like a magnetic dipole
the above situation is L mvRnˆ
with poles on either side of its face and it is
known as “magnetic shell”. q
M L
SI unit of magnetic moment (M) is A m 2 and 2m
dimensional formula is [ IL2]. i.e., the magnetic moment of a charged particle
c) Magnetic moment of a current loop is a vector moving in a circle is q/2m times of its angular
perpendicular to the plane of the loop and the momentum.
direction is given by right hand thumb rule. If the charge is negative, M is directed opposite
Magnetic dipole moment of a
to L
revolving electron :
Consider an electron revolving in a circular path M q
iii) is a constant which is called
of radius r around a nucleus with uniform speed v. L 2m
The current in the orbit is gyromagnetic ratio
e e ev Special cases :
i i) For an arc shaped conductor carrying
T 2 r / v 2 r
Magnetic dipole moment of a revolving electron current subtending an angle at the
centre.
ev evr
is iA r2
2 r 2
Magnetic dipole moment of a revolving electron
in the first orbit of hydrogen atom is called Bohr
Page 166
MOVING CHARGES & MAGNETISM
0i
i
B4
4 r
i i
R B 0 B net B1 B 2 B 3 B 4 ;
Bnet 0
2 R 2 2 r
iv) The upper and lower halves of the ring have
resistances R1 and R2 . Two straight wires are
Magnet ic induction at the cent re
connected to it as shown. The magnetic
0 i induction at the centre of the ring is
B
4 R i
B1 B3 0 B2 0 2
a) For a quadrant circular wire carrying current, 4r
R 1
i
900 . 2
2
3
r
0 i 1 0i4
Magnetic induction at the centre B i B4
8R i4 4 4r
b) Fo r semi circular wire carrying R2
current, 1800 Since R1 and R2 are parallel to each other
i
V1 V2 ; i2 R1 i4 R2 ; i2 R R R2 ;
R 1 2
i
i4 R1
0 i R1 R2
Magnetic induction at the centre B
0
4R
ii)To a circular wire, two straight wires are B net B1 B 2 B 3 B 4 ; Bnet 4r (i2 i4 )
attached as shown. When current is passed v) If B0 is magnetic induction at the centre of a
through it, then the magnetic field at the centre circular current carrying coil of radius R having
is zero.
i/2
N turns and BA is magnetic induction at a point
1
i on the axis of it at a distance x from centre
r
i
i 0 B0
2
3 4 B1 ; BA 3/ 2
i/2 4r then x2
2
1 R 2
i
0
B3 B4 0 B 2 0 Ni 0 NiR 2
2
4r B
Proof : 0 and A B
2R 2( R 2 x 2 )3/2
Bnet Zero
iii) To a circular wire, two straight wires are
attached as shown. When current is passed 0 Ni B0
BA 3/ 2
BA 3/ 2
through it, the magnetic field at the centre. x2 x2
2R 1 2 1 R 2
R
If x R , by using Binomial expansion
0i
B1 3x2
4 r BA B0 1 2
2R
vi) If a particle of charge q moves in a circular
i i path of radius r with a velocity v, then the
0 0 magnetic induction at the centre of circular loop
2 ; 2
B2 B3 ;
4R 4R
Page 167
MOVING CHARGES & MAGNETISM
B
dt
H B
q (v r ) i H B
0
Using the above equations, d B . W E
4 r 3 H B
viii) a)When a wire of length ‘l’ carrying current ‘i’ is B B
Page 168
MOVING CHARGES & MAGNETISM
i1 B 300
o
O o
60
A
i2 C
D
i i
Z
0 I Sol : Magnetic induction due to a loop at apex,
B due to the ring lying in YZ-plane is B yz
2R 0ir 2
along X-axis and B due to the ring lying in XZ- B
2(r 2 x 2 )3/2
I
plane is Bxz 0 along Y-axis. But r 2 x 2 l 2 (r 2 x 2 )3/2 l 3
2R
where‘ l ’is slant length
I I
B net 0 i j k Bnet 3 0
3
2R 2R 0i r r
B . But sin where is
Illustration-14: Two wires are wrapped over a 2r l l
wooden cylinder to form two co-axial loops half of the apex angle, same for both the loops
carrying currents i1 and i2 . If i2 8i1 then i r
B1 B2 1 1
find the value of x for B=0 at the origin O. B1 B 2 0 (given) i r 2 2
Page 169
MOVING CHARGES & MAGNETISM
equal to a=5cm and b=10cm. Find the the other end of the rod with angular
magnetic field at the centre of the coil.
frequency 104 rad/s. Find the magnetic
field at a point on the axis of rotation at a
distance of 0.8m from the centre of the
circular path.
a b
0ir 2 q
Sol: B 2 2 3/2 , i ;
2(r x ) 2
q r 2
Sol: Let n = number of turns per unit length along B 0 2
the radial of spiral. Consider a ring of radius x 4 (r x 2 )3/2
2
whose thickness is dx. 10 7 1 104 0.6
B 3/ 2
N 0.6 2 0.8 2
No.of turns in the ring = ndx. n
(b a)
3 2
Magnetic field at the centre due to the ring is 10 36 10 36 105
0 (ndx )i 0.36 0.643/ 2 1
dB ; so net field
2x 36 105 Tesla
b
0 nidx 0 ni b dx Illustration -19: Two circular coils made of
B dB
a 2x 2 a x same material having radii 20 cm & 30 cm
have turns 100 & 50 respectively. If they
0 ni b 0 Ni b are connected a) in series b) in parallel
or B ln or B ln separately across a source of emf. Find the
2 a 2(b a ) a ratio of magnetic inductions at the centre
4 10 7 100 8 10 3 10 of circles in each case
ln ;
2(10 5) 10 2 5 ni
Sol: a) B 0 ; coils are in series i is same
2r
B 6.9 10 6 T
Illustration-17: A plastic disc of radius ‘R’ has n B1 100 30
in both B ; B 50 20 3 :1
a charge ‘q’ uniformly distributed over its r 2
surface. If the disc is rotated with a b) coils are parallel potential difference is
frequency ‘f’ about its axis, then the 1 (n 2 r )
magnetic induction at the centre of the disc same i Where R ; Where A
R A
is given by is area of cross section of wire which is same
di q for both
Sol: dB 0 , dq (2 x) dx
2x R2 1 0 ni n 1 1
q R nr ; i butB0 B B 2
di (dq ) f 2 x dx f nr 2r r nr r
2
R 2
B 30 9
2 qxdx f R
B 0 2q.dx ( f ) 1
dB 0 2 0 2 R 2 B2 20 4
2 xR
Illustration-20: Two circular coils are made
0 qf qf
B 2
( R) B 0 from a uniform wire. The ratio of radii of
R R
circular coils are 2:3 and ratio of number
Illustration-18:A charge of 1C is placed at one of turns is 3:4. If they are connected in
end of a non conducting rod of length parallel across a battery.
0.6m. The rod is rotated in a vertical plane
about a horizontal axis passing through
Page 170
MOVING CHARGES & MAGNETISM
V V a b
b) M ni Acoil but i Awire
R l
V 2 V
M ( rcoil ); M rcoil
(2 rcoil ) X 2 Sol: Let us take a thin coil of thickness dr. Then the
N
M 1 r1 2 number of turns of the coil is dN .dr
ba
M 2 r2 3 .
Page 171
MOVING CHARGES & MAGNETISM
ma 2 Bend
1
0 ni
axis L . The ratio
2 2
The variation of magnetic field with distance
q a 2
along the axis is shown here.
M q
4 2
L ma 2m
2
SOLENOID: -l/2 -x o +x +l/2
A solenoid is a wire wound in the form of a
closely spaced spiral over a hollow cylindrical
non-conducting core. The wire is coated with
an insulating material, so that the adjacent turns
even though physically touch each other, but
they are electrically insulated.
dN ndx
TOROID OR ANCHOR RING:
MAGNETIC FIELD DUE TO A TOROID
A toroid or a toroidal coil or an anchor ring
consists of a wire uniformly wound on a torus
or a doughnut shaped structure. A toroid can
be considered to be a closed solenoid of small
radius.
This is shown in given Figure.
Torus L’’
r L’ L’’
Open space
Consider a coil of width dx at a distance x from
the point P on ths axis of the solenoid as shown. L’
B
x R tan dx R sec2 d
(a) (b)
Page 172
MOVING CHARGES & MAGNETISM
Current linked with the Amperian loop L = open solenoid. The magnetic field (B) within
total number of turns x current linking each the toroid is tangent to the circular field lines
turn =NI in the torus.
From Ampere law, Uses of Toroids
0 NI (I) Toroids are frequently used for the
B 2 r 0 NI or B ..........(1) laboratory measurements where a calculable
2 r
Thus, the field inside the toroid varies as 1/r field is desired. (ii) Toroids form the central
and hence is non-uniform. feature of the tokamak,a device showing
On the other hand, if the cross-sectional area promise for a fusion power reactor.
of the toroid is very-very small compared to Illustration-25: A solenoid of length 8cm has
r, we can neglect any variation in r. 100 turns in it. If radius of coil is 3cm and
Considering 2 r to be the circumference of if it is carrying a current of 2A, find the
magnetic induction at a point 4cm from
N
the toroid, will be a constant and equal the either end on the axis of the solenoid.
2 r
4cm 4cm
to the number of turns per unit length (n). In
3cm
5cm 5cm
this case, eqn. (1) takes the form, B 0 nI
which is the same as for the long solenoid.
(b) Magnetic field at a point in the open ni
Sol: B 0 (cos 1 cos 2 )
space inside the toroid 2
Consider the Amperian loop L’ passing
100
through any point in the open space inside the 4 10 7 2
2
8 10 2 4 800T
toroid. 2 5
Since total current linking the loop L’ is zero, Illustration-26: A toroid of non ferromagnetic
B.dl 0 or B 0 has core of inner radius 25cm and outer
radius 26cm. It has 3500 turns & carries a
(c) Magnetic field at a point outside the
current of 11A, then find the magnetic field
toroid
at a point
For any point outside the toroid, the net i) In the internal cavity of toroid
current threading the loop L” through that ii) At the midpoint of the windings
point is zero. This is due to the reason that iii) At a point which is at a distance of 30cm from
each turn of the winding passes twice through the centre of toroid
the area bounded by the path, carrying equal Sol: i) B = 0.
currents in opposite directions as shown in ni 3500 11
Fig.(b). Clearly, ii) B 0 2 10 7 2
2
2 r 51 10
B.dl 0 or B 0 30.196 103 T
Thus, in an ideal toroid (in which the turns of iii) B = 0. Based on magnetism for solenoid and
the wire are very closely spaced), the toroid.
magnetic field is confined entirely within its Illustration-27: A solenoid of 2m long & 3cm
core and is almost uniform. For regions diameter has 5 layers of winding of 500
exterior to the coil, B = 0. In other words, turns per metre length in each layer &
magnetic field does not come out anywhere in carries a current of 5A. Find intensity of
case of a toroid as it does at the ends of an magnetic field at the centre of the solenoid.
Page 173
MOVING CHARGES & MAGNETISM
Page 174
MOVING CHARGES & MAGNETISM
Page 175
MOVING CHARGES & MAGNETISM
O
The change in direction of -particle ( )
from figure is given by r
l
2 l 4
M N
l q l
x x x x x
Sin lB 2
x x x x x
R 2 mV (ON ) 2 (OM ) 2 (MN )2 x x x x x
3
x x x x x
1 l
Here l 0.1m, B 0.1 tesla, V 10 volt 4 x
x
x
x
x
x
x
x
x
x
T S
l 5
and m 6.4 10 27
kg r2 r l2 r l .
2 4
3.2 1019 1 If the particle comes out from face (4),
sin 0.1 0.1 4
27
2 6.4 10 10 2 5 mv 5 5 qBl
r l l (or) v .
or 300 . 4 qB 4 4 m
Illustration -31: The magnetic field (B) is 5 qBl
If velocity v , the particle will come
confined in a square region. A positive 4 m
charged particle of charge q and mass m is out from face (3).
projected as shown in fig. Find the Illustration-32: A particle of mass m and charge
limiting velocities of the particle so that it +q enters a region of magnetic field with a
may come out of face 1,2,3 and 4. velocity v, as shown in fig.
4 B
x x x x x
l/2 x x
x x x x x
m,q F
x x x x x x x
3
v x x x x x
1 l x x
x x x x x
E
x x x x x x x
2l
Sol: For the positive charge coming out from face
(1), the radius of the path in magnetic field a) Find the angle subtended by the circular
should be less than or equal to l / 4 . For arc described by it in the magnetic field.
l b) How long does the particle stay inside the
limiting case (2 r ) . magnetic field ?
2
c) If the particle enters at E, what is the
4 B
intercept EF ?
x x x x x
x x x x x Sol:
v x x x x x a) The particle circulates under the influence of
3
x x x x x
1
x x x x x
l magnetic field. As the magnetic field is uniform,
x x x x x the charge comes out symmetrically. The angle
2l subtended at the centre is (180 2 )
l mv qBl b) The length of the arc traced by the particle,
rmax vmax . Hence, if the l R ( 2 )
4 qB 4m
qBl l R ( 2 )
velocity is , the charge particle comes Time spent in the field, t and
v v
4m
out of face (1). mv m
R ; which gives t ( 2 )
We can observe from right palm rule, that the Bq Bq
particle cannot come out from face (2). 2 m
For a positive charge coming out of face (4), As time period: T Bq ,
let particle come out at point N from OMN T
hence t ( 2 )
2
Page 176
MOVING CHARGES & MAGNETISM
F E
qE qvB or v
90 B
R F
B
This condition can be used to select charged
O
particles of a particular velocity out of a beam
1802
R FB
containing charges moving with different speeds
90 (irrespective of their charge and mass). The
E crossed E and B fields, therefore, serve as a
velocity selector.
We can generalize this result. If is the angle Cyclotron:
subtended by the arc traced by the charged The cyclotron is a machine to accelerate
particle in the magnetic field, the time spent is positively charged particles or ions to high
energies using both electric and magnetic fields
t T in combination.
2
Cyclotron uses the fact that the frequency of
c) Intercept EF = 2R cos . revolution of the charged particle in a magnetic
Lorentz Force : field is independent of its energy.
When a charge enters a region where both
electric and magnetic fields exist simultaneously,
force acting on it is called ‘Lorentz force’ and
is given by F F e F m q E (v B ) .
Velocity Vector:-
We shall consider the simple case in which
electric and magnetic fields are perpendicular
to each other and also perpendicular to the
velocity of the particle, as shown in Fig. We
have,
y
E
FE Centripetal force is provided by the magnetic
mv2
x force Bqv
v r
mv
B Radius of circular path is r
Bq
FB
2 r
z Time period of charged particle is T
^ ^ ^
v
E E j , B B k ,V v i 2 m 1 Bq
T ;f cyclotron
Bq T 2 m
FE qE qEj , FB q v B q vi Bk qvBj
frequency.
K.E of charged particles is
Therefore F q E vB j 2
Thus, electric and magnetic forces are in 1 2 1 Bqr B 2q 2 r 2
K.E= mv m
opposite directions as shown in the figure. 2 2 m 2m
Suppose, we adjust the value of E and B such Force on a current carrying conductor
that magnitude of the two forces are equal. kept in uniform magnetic field :
Then, total force on the charge is zero and the i) A conductor carrying current i is placed in a
charge will move in the uniform magnetic field of induction B at an
fields undeflected. This happens when angle with the field direction. The force
Page 177
MOVING CHARGES & MAGNETISM
acting on it is given by F i (l B ) ;
F BilSin
Page 178
MOVING CHARGES & MAGNETISM
Page 179
MOVING CHARGES & MAGNETISM
mg il B sin 90 0
E 6
Here, i 0 .3 A Spring
R 20 balance
ilB 0.3 4.910 0.8
2
0.12
mg 10103 9.8 Insulating
F2 mg IaB (downwards)
k B k F 2IaB
MAGNETIC FIELD OF MOVING CHARGE
A point charge q, at rest in the observer’s inertial
i frame, produces an electric field along the radius
Sol. The forces acting on the rod are ‘mg’
vector and is given by
downwards, Fmag ilB downwards and
1 q
Fspring 2kx upwards E 3r
4 0 r
If the charge is moving relative to the observer’s
inertial frame, it produces a magnetic field in
kx B kx addition to electric field, the magnitude of which
is proportional to the speed of the charge
B relative to the observer provided v c . The
i B magnetic field vector B at the point P, at
distance r from the charge q moving with
mg
Under the action of these forces the rod is in velocity v is found to be
equilibrium. Then, Fnet 0 (or) q
mg ilB
B 0 3 v r .... 1
4 r
mg ilB 2kx (or) x
2k P
1 1
10 1 1 1 r
20
2
m 1cm
2 50 100 q
v B
Page 180
MOVING CHARGES & MAGNETISM
0 q1q2v1v2
qv sin Fm
B 0 .... 2 4 r2
4 r2
P 0 q1q2 2
for v1 v2 v ; Fm v ..... i
r
4 r 2
B In addition to the magnetic force, there is an
v electric force between them, whose magnitude
q 0 qv
is given by
r Bmax
4r 2
B 1 q1q2
Fe ..... ii
r 4 0 r 2
This force is of repulsive nature, On dividing
B
The following points should be remembered Fm 2
regarding the magnetic field: equation (i) by (ii), we have F v 0 0
e
(i). The magnetic field B is zero at all points on a
line on which charge moves; i.e; when 1 Fm v 2
As c ..... 3
00 or 1800, B 0 . 0 0 ; Fe c 2
(ii). It is maximum in the plane perpendicular to v Since v c, and so Fm Fe . As Fm Fe ,
and through the charge, as sin 1, at all so the net force between the charges is of
points in this plane: repulsive nature.
(iii) B remains unaltered in magnitude at all points Force between two parallel current
on the circumference of circle passing through carrying long straight conductors :
P and lying in a plane perpendicular to v with i) Force per unit length on each wire is given by
its centre on the velocity direction.
F 0 i1i2
Force between moving charges: . If i1 i2 1amp, r 1m, then
l 2 r
The force acting on a charge q2 , moving with
force per unit length of the conductor is
velocity v2 in a magnetic field produced by 2 107 N / m
charge q1 moving with a velocity v1 is ii) If currents in the two wires are in same
direction, then the force of attraction takes
F21 q2 v2 B1 place between them.
q iii) If currents in the two wires are in opposite
q2 v2 0 13 v1 r
4 r direction, then the force of repulsion takes
0 q1q2 place between them
4 r 3
v2 v1 r .
iv) A straight and very long wire carries current i1
v1 v2 and rectangular loop of wire carrying current
i2 is placed nearby it. The force on the loop is
F
Net force
q1 F12 F21
q2
r i1 i2 i2 l
B2 B1 i i l 1 1
F 012 a
The magnitude of force which they exert on 2 a b
each other is: b
Page 181
MOVING CHARGES & MAGNETISM
is rigidly fixed. Another wire is placed directly currents i1 , i2 (i1 i2 ) in opposite directions,
above and parallel to it carries a current i2 . r is and are separated by a distance r, then the null
the perpendicular distance of separation point is formed out side the conductors, the
between the wires and currents are in opposite distance of the null point from the conductor
directions. For the second wire to remain carrying smaller current is given by
stationary, the condition is
r i1 i2
ii x
F mg 0 1 2 mg i2
2 r 1 x
i1
r
Illustration -40: A long straight conductor
m 0i1i2 carrying a current of 2A is in parallel to
2 rg another conductor of length 5cm and
carrying a current 3A. They are separated
vi) Three long parallel conductors carry currents by a distance of 10cm. Calculate (a) B due
as shown to first conductor at the location of second
a) Resultant force per unit length on the wire conductor (b) the force on the short
conductor.
F 0 i1i2 i2i3
‘C’ is Sol. Given i1 2 A; i2 3 A
L 2 a b
A C B r 10cm 10 102 m; l2 5cm
0i1 2
i1 i2 i3 a) B 2 107 4 106 Tesla
2 r 10 102
a 0i1i2
b b) F 2
2 r
b) If the resultant force on the wire ‘C’ is zero,
7 23 2
i1i2 i2 i3 i1 i3 2 10 2
5 10 6 10 7 N
10 10
the condition is
a b a b Illustration -41: Two long straight parallel
Here, the resultant force per unit length on the current carrying conductors each of
A and B wires can be also determined in the length l and current i are placed at a
similar way. The currents can be along different distance r0 . Show that the total work done
directions. by an external agent in slowly reducing
Null points due to two current r0
their distance of separation to is
carrying parallel wires : 2
0 2
i) Two straight parallel conductors are carrying i l n 2
2
currents i1 , i2 i1 i2 in the same direction,
and are separated by a distance r, the null point
is formed in between them. The distance of the
null point from the conductor carrying smaller Sol. ; The force acting on the
current is
r i1 i2
x 0 i 0 i 2l
i2 x
conductor 2 is F = ilB il
1 2 r 2 r
i1 This force does a work dW in displacing the
r
conductor 2 by a distance dr
ii) Two straight parallel conductors are carrying
Page 182
MOVING CHARGES & MAGNETISM
Work done to displace it through a small Force between two streams of electric
distance charges :
i) If two streams of electrons or protons are
dx dW F .dx moving with velocity ‘v’ in parallel and same
0.2
0 i1i2 l directions, there will be both electric repulsive
ii l W dx
dW 0 1 2 dx ;
2 x
0.1 2 x
force and magnetic attractive force. Since
electric force predominates the magnetic force,
0i1i2l 0.2 there will be repulsion.
W log e x 0.1 ii) If they move parallel and opposite directions,
2
there will be electric repulsive force and
4 107 10 5 10 102
W log e 2 magnetic repulsive force and hence there will
2 be repulsion again.
W 0.693 106 J Torque acting on a current loop
Illustration-43: Three long straight wires are kept in uniform magnetic field :
connected parallel to each other across a i) When a coil carrying current is placed in uniform
battery of negligible internal resistance. magnetic field, the net force on it is zero but it
The ratio of their resistances are 3 : 4 : 5. may experience a torque or couple.
What is the ratio of distances of middle
wire from the others if the net force ex-
perienced by it is zero
n
Page 183
MOVING CHARGES & MAGNETISM
Angle made by the plane of the coil with magnetic field of 2T j . The loop is in xy
B plane with current in clock wise direction.
Angle made by the normal to the plane of Find the torque on the loop.
the coil with B
iv) If the plane of coil is parallel to the direction of
Sol. M B
ni A B
magnetic field max BiNA 10 10 k 2 j 2 10 Nm i
4
3
v) If the plane of coil is perpendicular to the
Illustration-45: A metallic wire is folded to form
direction of magnetic field, 0
a square loop of side ‘a’. It carries a current
vi) If current carrying coil is placed in a non- ‘i’ and is kept perpendicular to a uniform
uniform magnetic field it experiences both, force magnetic field. If the shape of the loop is
and torque. changed from square to a circle without
vii) For a given area, torque is independent of shape changing the length of the wire and
of the coil current, the amount of work done in doing
viii) Torque is directly proportional to area of the so is
coil. Sol. W= find P.E - initial P.E
Special cases :
i) When a current carrying coil is placed in uniform W M f B Mi B ; W iB Ai A f
magnetic field, net force on it
4
F = 0. But net torque may act. W iBa 2 1
ii) When a current carrying coil is placed in non-
uniform magnetic field, net force, net torque Illustration-46:A flat insulating disc of radius
both act. ‘a’ carries an excess charge on its surface
net 0 and Fnet 0 is of surface charge density C / m 2 .
Consider the disc is rotated around the
iii) If the angle made by M of the coil with B in
axis passing through its centre and
uniform magnetic field is ‘ ’, then its potential
perpendicular to its plane with angular
energy
speed rad/s. If magnetic field B is
P.E M.B ; P.E MB cos directed perpendicular to the rotation axis,
iv) If a current carrying coil is rotated in a uniform then find the torque acting on the disc.
magnetic field such that the angle made by M Sol. Suppose the disc is placed in xy-plane and is
rotated about the z-axis. Consider an annular
wit h B is changed fro m 1 to 2 .
ring of radius r and of thickness dr, the charge
W MB cos 1 cos 2 on this ring.
v) If an external field is along the direction of M , dq 2 rdr ; As the ring rotates with
then 0 . angular velocit y , so t he current
0 and P.E = – MB (min) dq 2 rdr
This position corresponds to stable equilibrium. i rdr
dt 2
vi) If external magnetic field is opposite to M then,
180 y
0 and P.E. = + MB (max)
dr
This corresponds to unstable equilibrium.
r x
Illustration-44: A circular loop of area 1cm 2 B
Page 184
MOVING CHARGES & MAGNETISM
The torque on the current loop i A B
0 i0 i sin b ii
Hence the torque on this annular ring dr 0 0 b a sin
a
d i d A B rdr r 2 B sin 900
Moving coil Galvanometer :
r 3 Bdr and i) Principle of moving coil galvanometer:
a
When a current carrying coil suspended in a
3 Ba 4 uniform magnetic field, it experiences a torque
B r dr
0 4 and hence it rotates.
ii) Poles of magnet are concave in shape, to make
Illustration-47: A loop carrying current ‘i’ is the magnetic field radial, so that at all
lying in the plane of the paper. It is present orientations, the plane of the coil is parallel to
in the field of a long straight wire with the field, and hence torque acting on it is
constant current i0 (inward) as shown in maximum. This makes the relation between
fig. Find the torque acting on the loop. current and deflection as linear.
i
iii) Soft iron cylinder is kept at the center of
magnetic field, to increase the flux.
i 0 iv) Phosphor Bronze material has
2 i i
a a) high Young’s modulus, so that the wire will not
i
b be stretched easily.
b) low rigidity modulus, so that the wire can be
Sol. The field due to current carrying wire is tangential twisted easily.
to every point on the circular portion of the loop c) small elastic after effect, so that it comes back
and hence the forces acting on these segments quickly to original position after withdrawing
are zero. current.
y
v) Small mirror is attached on the phosphor
dr dF k Bronze wire, to measure the deflection, using
l
r i l lamp and scale arrangement.
vi) If ' ' is the deflection for passage of current
i
0
i i 2 r sin ‘i’,
a Scale
i
2dr
r
dF k
B x
Now consider two small elements of length Permanent magnet
dr at a distance r from the axis symmetrically Pointer
i
dF B i dr 0 0 idr
2 r N Sp S
On element 1 it is into the page and on 2 it out Pivot
of the page, d dF 2r sin
Soft-iron
ii core
0 0 dr 2 r sin
2 r
Uniform radial
Now total torque magnetic field
Page 185
MOVING CHARGES & MAGNETISM
Page 186
MOVING CHARGES & MAGNETISM
Page 187
MOVING CHARGES & MAGNETISM
Page 188
MOVING CHARGES & MAGNETISM
and it indicates 0o 0o .
V 10
V 10V n 1 100 iii) current measured by Tangent galvanometer is
Vg 10
2rBH
R G n 1 100 100 1 9900 i Tan KTan ; r = radius of
0 n
Illustration-57: What is the value of shunt, coil, K = reduction factor; n = number of
which passes 10% of the main current turns of coil
through a galvanometer of resistance 99 iv) SI unit of reduction factor is ampere
ohm? v) Reading is more accurate, when 450 , since
Sol. As shunt is a small resistance S in parallel with di 1
a galvanometer (of resistance G) as shown in relative error and it is minimum
i sin 2
fig. I I G S IG G for 450 .
(I IG) vi) Sensitivity is maximum, when 00 since
d
S cos 2 , which is maximum for 00 .
di
G vii) Reduction factor K depends on horizontal
I IG component of earth’s magnetic field.
ig S 10 S viii) T.G gives different readings at different places
;
i G S 100 99 S for same current. Since BH changes from place
to place.
99 S 10S S 11 ix) T.G cannot be used at magnetic poles, since
TANGENT GALVANOMETER : BH 0 at magnetic poles.
x) T.G is used to measure the current of the order
of 106 A.
WIRE
BH Illustration-58:A magnetic needle is arranged
at the centre of a current carrying coil
having 50 turns with radius of coil 20cm
arranged along magnetic meridian. When
s B a current of 0.5mA is allowed to pass
Pointer through the coil the deflection is observed
to be 300. Find the horizontal component
of earth’s magnetic field
i) Tangent galvanometer works on the principle 0 ni
of Tangent law i.e., B BH Tan . Sol: B BH tan BH
2r tan
Here B = Magnetic induction at the centre of 4 10 7 50 5 10 4 3
ni B
2 2 10 1 1
H
the current carrying coil 0
2r
ii) It is a moving magnet type galvanometer. 25 3 109 T
iii) During experiment, plane of the coil should be 135.9 109 T
along the magnetic meridian [to fulfill the
1.36 107 T
requirement of tangent law] or magnetic need
in parallel to the plane of the coil or aluminium
pointer perpendicular to the plane of the coil
Page 189
Page 190
MAGNETISM AND MATTER
Page 191
MAGNETISM AND MATTER
Page 192
MAGNETISM AND MATTER
2 R 2 S2
New Magnetic Moment, M1
2 S1 N1
M 1 m 2 y m 2 sin i) When two bar magnets of moments M1 and M2
2
are joined so that their like poles touch each other
and their axes are inclined at an angle ' ' , then
2 M sin
2 M 2 m the resultant magnetic moment of the combination
M1 ‘M1’ is given by
S M1 M12 M 22 2M1M 2 cos
( angle between the directions of magnetic
a) If radians,
2
moments)
2 S2
N
i.e., if the magnet is bent in the form of a N2
quadrant of a circle, then
ii) M1
0
(180 - ) S N1
1
M2
Page 193
MAGNETISM AND MATTER
When two bar magnets of moments M1 and M1 and M2 are placed at right angles to each
M2 are joined so that their unlike poles touch other then resultant magnetic moment,
each other and their axes are inclined at an angle M1 M12 M22 90 .
0
is (1800 - )] N S
S N
N
S
S M N M
M 1
M12 M 22 2M1M 2 cos When identical magnets each of magnetic
M1 M2 moment M are arranged to form a closed
polygon like a triangle (or) square with unlike
a) poles at each corner, then resultant magnetic
S1 N1N2 S2 moment, M1 = 0.
When two bar magnets of moments M1 and In the above point , if one of the magnets is
M2 (M1>M2) are placed coaxially with like reversed pole to pole then resultant magnetic
poles in contact then resultant magnetic moment, moment, M 1 2M
M1=M1–M2
S N
( angle between the directions of magnetic
0
120
moments, 1800 ) 3M 2M
=
M M
2M 60
0
M
M1 M2 iv)
N S
b) M
S M N
S NS
1 N 1 2 When three bar magnets of equal length but
2
When two bar magnets of moments M1 and moments M, 2M and 3M are arranged to form
M2 (M1>M2) are placed coaxially with unlike an equilateral triangle with unlike poles at each
poles are in contact then resultant magnetic corner, resultant magnetic moment is given by
moment, M1=M1+M2
( angle between directions of magnetic moments, M 2M M 2 2M M cos120 3M
1 2 2 0
Page 194
MAGNETISM AND MATTER
M M M
1 2 2 N S
1 2 2
S
N S
B=0
Magnetic line of force with magnetic needle
Characteristics of lines of force :
(i) Magnetic lines of force are closed curves. (x) If the magnetic lines of force are straight,
Outside the magnet, their direction is from north parallel, and equally spaced then the magnetic
to south pole, while inside the magnet they are field is said to be uniform.
from south to north pole. Hence they have neither Magnetic Induction (or) Induction Field
origin nor end. Strength (B)
(ii) Tangent at any point to the line of force gives Magnetic induction field strength at a point
the direction of magnetic field at that point. in the magnetic field is defined as the force
(iii) Two lines of force never intersect each other. If experienced by unit north pole placed at
the two lines of force intersect, at the intersecting that point. It is denoted by ‘B’.
point the field should have two directions, which If a pole of strength ‘m’ placed at a point in a
is not possible. magnetic field experiences a force ‘F’, the
(iv) The lines of force tend to contract longitudinally magnetic induction (B) at that point is given by
or length wise, as shown in figure. Due to this
property the two unlike poles attract each other. F
B i.e., F mB
m
B is a vector quantity directed away from N-
N S
pole or towards S-pole.
B
(v) The lines of force tend to repel each other N B S
laterally. Due to this property the two similar Unit North Unit North
Pole Pole
poles repel each other. S.I. Unit of B :
Page 195
MAGNETISM AND MATTER
Page 196
MAGNETISM AND MATTER
M2 90– M1 N
N2 N1
F
r
S
S1 S2
Two magnets of moments M1 and M2 are joined
as shown in figure and the arrangement is Fr cos MBH sin
pivoted so that it is free to rotate in a horizontal
plane under the influence of magnetic field B. F
MBH tan 2m BH tan
Then net torque acting on the system is given by r r
c) In the above case, if the force is applied at
one end which is always perpendicular to length
of the magnetic needle, then
Page 197
MAGNETISM AND MATTER
N
rotated through 45 0 . Under similar
conditions another magnet of magnetic
r
F
moment M2 is rotated through 300. Then
S l find the ratio of M1 & M2 .
Sol: C MBsin
Fr sin 90 MB H sin
0
For first magnet,
MBH sin 2mBH sin C 180 45 M 1 B sin 450 ----(1)
F
r r For second magnet,
A magnet of moment 'M' is suspended in the
C 180 30 M 2 B sin 300 ----(2)
magnetic meridian with an untwisted wire. The
upper end of the wire is rotated through an angle Diving equation (1) by equation (2)
' ' to deflect the magnet by an angle ' ' from
135 M 1 M 9
magnetic meridian. Then deflecting couple acting 2 1
150 M 2 M 2 10 2
on the magnet = MBH sin
Illustration-3 : A magnetic dipole is under the
influence of two magnetic fields. The angle
N between the two field directions is 600 and
S N one of the fields has a magnitude of 1.2 ×
S
10 –2 T. If the dipole comes to stable
equilibrium at an angle of 150 with this
field, what is the magnitude of the other
Restoring couple developed in suspension wire
field? [sin150=0.2588]
= C where C is couple per unit twist
Sol. Here B1 = 1.2 × 10–2 T. Inclination of dipole
of suspension wire.
with B1 is 1 = 150. Therefore, inclination of
In equilibrium position,
MBH sin C dipole with B2 is 2 600 150 450 . As the
Illustration-1 : When a bar magnet is placed dipole is in equilibrium, therefore the torque on
o
at 90 to a uniform magnetic field, it is the dipole due to the two fields are equal and
acted upon by a couple which is maximum. opposite. If M is magnetic dipole moment of
For the couple to be half of the maximum the dipole, then
value, at what angle should the magnet be
Page 198
MAGNETISM AND MATTER
1 =150
0
When the bar magnet is anti-parallel to the
B1
2
=
45 applied field, then = 1800 and potential energy
B2 is maximum i.e. U= +MB. It is said to be
unstable equilibrium.
Illustration-5:A magnet is suspended at an angle
600 in an external magnetic field of 5 × 10–4
T. What is the work done by the magnetic field
60
0 in bringing it in its direction ? [The magnetic
MB1 sin 1 MB2 sin 2 moment = 20 A-m2]
Sol. Work done by the magnetic field,
B1 sin 1 1.2 102 sin150
B2 W=MB cos1 cos 2 Here 1 600 and
sin 2 sin 450
2 00
1.21020.2588 4.39103 T
W 20 5104 cos60 0 cos 0 0
0.707
Illustration-4 : A compass needle of magnetic 1
102 1 5 10 3 J .
moment 60A-m 2 , pointing towards 2
geographical north at a certain place Illustration-6 : A magnetic needle lying
where the horizontal component of earth’s parallel to a magnetic field requires W
magnetic field is 40 wb/m 2 experiences a units of work to turn it through 600. What
torque of 1.2×10 –3 Nm. Find the is the torque needed to maintain the
declination at that place. needle in this position?
Sol. If is the declination of the place, then the Sol. In case of a dipole in a magnetic field,
torque acting on the needle is M BH sin W MB cos 1 cos 2 and = MB sin
1.2 10 3 1 Here, 1 00 and 2 600
sin
M BH 60 40 106 2 30
0
So, W MB (1 cos ) 2 MB sin
2
Work done in rotating a magnetic dipole in a 2
magnetic field
The work done in deflecting a magnet from and , MB sin 2 MB sin cos
2 2
angular position 1 to an angular position 2
τ
So, W cot 2 , i.e τ W cot 30 3W
0
with the field is (equal to the change in PE)
given as W MB cos 1 cos 2 Illustration-7 : A bar magnet has a magnetic
The work done in deflecting a bar magnet moment 2.5 J T –1 and is placed in a
through an angle from its state of equilibrium magnetic field of 0.2 T. Calculate the work
done in turning the magnet from parallel
position in a uniform magnetic field is given by
to antiparallel position relative to field
W MB1 cos here 1 0 0 , 2 direction.
When it is released, this work done converts Sol. Work done in changing the orientation of a dipole
into rotational KE of moment M in a field B from position 1to 2
MB(1-Cos )= I 2
1 is given by W MB cos 1 cos 2
2
When a bar magnet is held at an angle with Here, 1 00 and 2 1800
the magnetic field, the potential energy So, W 2 MB 2 2.5 0.2 1 J
possessed by the magnet is U = –MB cos Illustration-8 : A bar magnet with poles 25 cm
When the bar magnet is parallel to the applied apart and pole-strength 14.4 A-m rests with
field, then = 00 and potential energy is its centre on a frictionless pivot. It is held
(-MB).It is said to be stable equilibrium. in equilibrium at 600 to a uniform magnetic
When the bar magnet is perpendicular to the field of induction 0.25 T by applying a force
applied field, then = 900 and potential energy F at right angles to its axis, 10cm from its
is zero
Page 199
MAGNETISM AND MATTER
Page 200
MAGNETISM AND MATTER
Page 201
MAGNETISM AND MATTER
W E S N
O N1
N
S
N2
W O E
Page 202
MAGNETISM AND MATTER
S 0 M
Bnet B|e BH BH 7BH
4 d 3
2
(iii) At a distance '2d' on equatorial line, the net
N
magnetic induction is given by
M
Bnet= BH B||e BH 0
BH BH 4 2d 3
BH
BH 7BH
n = BH
B 8 8
N S (iv) At a distance 'd' on axial line of the bar magnet,
B
H the net magnetic induction is given by
Bnet=Ba+BH=2Be + BH = 2BH + BH=3BH.
n
(v) If the axis of the bar magnet is rotated through
B
A single neutral point will be formed in the 900 clockwise at the same position then the net
combined field on the horizontal table. magnetic induction at the same point 'P' is
If ‘m’ is the pole strength and ‘d’ is the
Bnet= B2a B2H = 5 BH Ba 2Be 2BH
distance from the pole of the magnet where the
BH
0 m N Bnet
neutral point is formed, then BH 4 d 2
1 1
If the north pole is on the table, then the S N
P B a
neutral point is formed towards geographic
S
south side of the pole.
If the south pole is on the table, then the (vi) If the axis of the magnet is rotated through
neutral point is formed towards geographic 1800 at the same position, then net magnetic
north induction at the same point 'P' is
side of the pole. Bnet = Be + BH = 2BH
Note: A short bar magnet is kept along A short bar magnet is kept along magnetic
magnetic meridian with its north pole meridian with its south pole pointing north.
pointing north. A neutral point is formed at A neutral point is formed at a point 'P' at a
point 'P' at distance 'd' from the centre of distance 'd' from the centre of the magnet
the magnet then then
(i)at a distance 'd' on axial line of the bar magnet
B a net magnetic induction,
BH BH
BH BH BH BH d n1 P
N Ba
n2 n1 S
d d
d P 2d BH
S 2
Be
Be B1 Be B 11
e
BH
e
N
(i) At a distance 'd' on equatorial line, net magnetic
induction B net = 0 2M
0 M 0 . 3 BH
B = 0 i.e., B = B 4 d
ie Be BH . BH net a H
4 d3 d
(ii) At a distance d/2 from the centre of the (ii) At a distance on axial line of bar magnet,
magnet on equatorial line, the net magnetic 2
induction is given by net magnetic induction is given by
0 2M
Bnet B1a BH . BH 7BH
4 d 3
2
Page 203
MAGNETISM AND MATTER
(iii)At a distance '2d' on axial line of the bar magnet, At neutral point, B1 = B2
net magnetic induction is given by m m2
0 . 21 0 .
2M 4 x 4 d x 2
Bnet BH B||a BH 0
4 2d 3 d
on solving, we get x
m2
BH 7B H 1
BH m1
8 8
(iv) At a distance 'd' on equatorial line of the bar Neutral points in the combined field due to
magnet, net magnetic induction is short bar magnets :
B Two short bar magnets of magnetic moments
Bnet Be BH a BH M1 and M2 (M1 < M2) are placed at a distance
2 ‘d’ between their centers with their magnetic
B 3 axes oriented as shown in the figure, Then two
H BH B H
2 2 neutral points are formed (i) in between and (ii)
(v) If axis of the magnet is rotated through 900 outside and on the line passing through centers
clockwise at the same position, then net of the magnets. In either case, null point is
magnetic induction at the same point 'P' is always closer to magnet of weaker moment.
given by B1 B2
x S1
M1
N1 x
B1 B2
N2
M2
S2
n2 n1
5 B B
BH Be a H
d
Bnet B2e B2H
2
2 2 B B
S 1
1 N
1
x x
,then magnetic induction at the point 'P' is n 2 n 1
Bnet = Ba + BH = BH + BH = 2BH B N
2B 1 S 2 2
M M
Neutral points in the combined field due to Case i) : If the neutral point is formed in between
1 2
(ii) When two unlike magnetic poles of strengths Note:No null points are obtained when unlike poles
m1 and m2 (m1 < m2 ) are separated by a of the magnets are placed closer to each other.
distance ‘d’, then neutral point is formed outside Time period of Suspended Magnet in
and on the line passing through the poles. It the Uniform Magnetic Field
always lies closer to weaker pole. Principle : When a bar magnet is suspended
B1 B2 N d S freely in a uniform magnetic field and displaced
n X m1 m2
Page 204
MAGNETISM AND MATTER
from its equilibrium position, then it starts (iv) The time period of a thin bar magnet is T. It is
executing angular SHM. cut into 'n' equal parts by cutting it normal to its
Time period of oscillation and frequency of length. The time period of each piece when
magnet is oscillating in the same magnetic field will be
2
I m
T = 2 and n
n
T I 1
MB H I M
3 &M1
T| 12 n n
n
1 1 MBH
n
T 2 I
I1 T
where M magnetic moment, BH Horizontal T 1 2
component of earth’s magnetic induction and M 1B n
(v) The time period of a thin bar magnet is T. It is
(l 2 b 2 ) cut into 'n' equal parts by cutting it along its
I moment of inertia, I m
12 length. The time period of each piece remains
ml 2
unchanged, when oscillating in the same field.
for a thin bar magnet I
12 m 2
where m is mass, l is length and b is breadth
( M & I n I
| M |
of the magnet. n 12 n
For small percentage changes in moment of
I| I/n
T 1 I T| 2 2 T)
inertia 100 100 MB|
M
T 2 I B
n
As I increases , T increases
For small percentage changes in magnetic (vi) a) Two magnets of magnetic moments M1 and
M2 (M1>M2) are placed one over the other.
T 1 M
moment T 100 2 M 100 If T1 is the time period when like poles touch
As M increases , T decreases each other and T2 is the time period when unlike
Comparison of magnetic moments : poles touch each other, then
(i) If two magnets of magnetic moments M1 and T22 M1 M 2
T 1
M2 of same dimensions and same mass are T12 M1 M 2 M
oscillating in the same field separately, then
M1 T22 T12
T1 M2
M2 T22 T12
T2 M1 (Bar magnets of equal size)
b) If n1 and n2 are the corresponding
T 1 M 1 n12 n22
M frequencies, then
M 2 n12 n22
(ii) A magnet is oscillating in a magnetic field B and
its time period is Tsec. If another identical (vii) When same bar magnet is used at two different
magnet is placed over that magnet with similar places 1 and 2, then
poles together, then time period remain
BH1 T22 T 1
unchanged.
( I| 2I and M| = 2M, BH2 T12 BH
(viii) When two bar magnets of moments M1 and
I| 2I I
T| 2 2 2 T) M2 are placed one over the other such that (i)
M| B 2MB MB like poles together (ii) unlike poles together and
(iii) A magnet is oscillating in a magnetic field B and (iii) their axes are perpendicular to each other.
its time period is T sec. If another identical When vibrated in the same magnetic field, the
magnet is placed over that magnet with unlike ratio of their time periods respectively is
poles together, then time period becomes
1
infinite. i.e., it does not oscillate. T
M
M| M M 0; T 2 I
0 B
Page 205
MAGNETISM AND MATTER
Page 206
MAGNETISM AND MATTER
n1
B1Cos 1
ie
50
B1 Cos300
BE and axial field BA of a bar magnet at a
n2 B2Cos 2 40 B2 Cos 450 larger distance r. i.e,r>> l , where 2l is the
25 B1 3 B1 25 length of the magnet
16 B2
(or) B 8 6 Equatorial field
2 2
I1 I 2
T 2 ........... (1) Axial ?eld 2P/4πε0 r 3 μ 0 2M/4πr 3
3MB
When the polarity of one of the magnets is Torque
reversed, M2 = M ~2M = M; τ=P×E τ=M×B
I1 I 2 P.E U P.E U M .B
so T ' 2 ............ (2)
MB
Dividing Eq. (2) by (1), MAGNETISM AND GAUSS’S LAW
T'
The gauss’s law for magnetism is that the net
T
3, i.e., T ' 3 T 3 3 sec magnetic flux through any closed surface is zero.
ELECTROSTATIC ANALOG TO In electrostatics we studied Gauss’s law. If we
imagine a closed gaussian surface A around an
MAGNETOSTATICS electric dipole, the number of lines leaving the
We know that the magnetic field on the axis of surface is equal to the number of lines entering
0 2 M it. This is consistent with the fact that no net
a solenoid or a dipole is B charge is enclosed by the surface. How ever, if
4 r 3
we consider a closed surface B in the same
The torque on a dipole in uniform field is
figure, there is a net outward flux, since it does
M B and the potential energy of magnetic include a net(positive) charge
dipole in uniform field is U M .B.
Comparing the above equations with the
corresponding equations for electric dipole
suggests that magnetic field at large distances
due to a bar magnet of magnetic moment M
can be obtained from the equation for electric
field due to an electric dipole of dipole moment
P, by making the following replacements
1
E B, p M , 0
4 0 4 The situation is quite different for magnetic fields
Now, we can write down the equatorial field which are continous and from closed loops.
Imagine a closed gaussian surface in a magnetic
Page 207
MAGNETISM AND MATTER
Page 208
MAGNETISM AND MATTER
Page 209
MAGNETISM AND MATTER
It is the ratio of magnitudes of total field inside Illustration-17:A magnetic field strength (H)
the material to that of magnetising field or it is 3×103 Am–1 produces a magnetic field of
the ratio of permeability of a medium to that of induction (B) of 12T in an iron rod. Find
free space. the relative permeability of iron ?
B H B 12
r Sol. 4 10 3
B0 0 H 0 H 3 10 3
It has no units and dimensions. 4 10 3
r 7
10 4
Relation between relative permeability and 0 4 10
susceptibility : Illustration-18:An iron bar of length 10 cm and
diameter 2 cm is placed in a magnetic field
We know B 0 H I or,, of intensity 1000Am –1 with its length
B I parallel to the direction of the field.
0 1
H H Determine the magnetic moment produced
in the bar magnet, if permeability of its
B I material is 6.3 × 10–4 TmA–1.
or, 0 1 [as and ]
H H Sol. we know that, 0 1
(or) 1 r 1
1
6.3 10 4
1 = 500.6
0
0 4 10 7
This is the desired result.
Intensity of magnetisation,
Illustration-14: A m agn etising fi eld of
I H 500.6 1000 = 5 × 105 Am–1
1600Am -1 produces a magnetic flux of
magnetic moment, M = I x V I r
2
2.4 × 10–5 weber in a bar of iron of cross
section 0.2 cm 2 .Calculate permeability
5 10 5 3.14 10 2
2
10 10
2
Page 210
MAGNETISM AND MATTER
Page 211
MAGNETISM AND MATTER
magnet called atomic magnet. (vi) The intensity of magnetisation I is very small and
iii) In absence of external magnetic field atomic compared to one. It follows from the relation
magnets are randomly oriented due to the r 1 m . The variation of r or with
thermal agitation and the net magnetic moment
of the substance is zero. H is shown in figure. As is clear from the figure,
iv) When it is placed in an external magnetic field, the variation is non-linear. The large value of
the atomic magnets align in the direction of the r is due to the fact that the field B inside the
field and thermal agitation oppose them to do material is much stronger than the magnetising
so. field due to ‘pulling in’ of a large number of
v) At low fields, the total magnetic moment would lines of force by the material.
be directly proportional to the magnetic field B Y
and inversely proportional to temperature T.
Properties of Paramagnetic substances
(i) The substances which when placed in a or x
r
magnetic field, acquire feeble magnetism in the
direction of magnetising field are known as O H X
paramagnetic substances.
(vii) Shape of paramagnetic liquid in a glass crucible
Ex: Aluminium (Al), Platinum (Pt), Manganese
and kept over two magnetic poles.
(Mn), Copper chloride (CuCl2), Oxygen (O2), Paramagnetic Liquid
solutions of salts of iron etc. are examples of
paramagnetic substances.
(ii) When a bar of paramagnetic substance is placed
in a magnetic field, it tries to concentrate the
lines of force into it as shown in figure
S
N s n S N
Page 212
MAGNETISM AND MATTER
Page 213
MAGNETISM AND MATTER
4. When suspended inside the 4. They align with their 4. They align with their
magnetic field, they align length along the direction length along the direction
their length perpendicular of magnetic field. of magnetic field.
to the magnetic field.
5. Magnetic lines of force 5. Few lines pass through the 5. Almost all lines prefer to
prefer to move out of the specimen. move through the
specimen. specimen.
6. They move from stronger 6. They move from weaker to 6. They move from weaker to
part of the magnetic field stronger part of the stronger part of the
to the weaker part of the magnetic field magnetic field.
magnetic field
Page 214
MAGNETISM AND MATTER
State of These are weekly These get weekly These get strongly
magnetization magnetised flu a magnetised in the magnetised in the
direction opposite to direction of applied direction of applied
that of applied magnetic field. magnetic field
magnetic field
When the Liquid level in that limb Liquid level in that Liquid level in that
material in the gets depressed limb rises up limb rises up very
form of liquid is much
filled in the N s N s N s
U-tube and
placed between
pole pieces.
Page 215
MAGNETISM AND MATTER
Dependence of Does not depend on On cooling, these get These get converted
on temperature temperature (except converted to ferro- into paramagnetic
Bi at low temperature) magnetic materials at materials at Curie
Curie temperature temperature
T TC T
T
Relative μr 1 μ r 1
permeabilityμ r μr 1
I-H curves +I +I
H H HS
-I H
Magnetic Very low (= 0) Very low Very high
moment (M)
Examples Cu, Ag, Au, Zn, Bi, Sb, Al. Mn, Pt, Na, CuCl2, Fe, Co, Ni, Cd,
NaCI, H2O air and O2 and crown glass Fe304 etc.
diamond etc.
PERMANENT MAGNETS AND ELECTRO MAGNETS
The properties of ferromagnetic materrials, sttudied from their hystersis curves, help in the proper
selection of the material for various practical applications. Few applications are described below
a) Pemanent magnets(or) Hard magnetic materials:
The substances which retain their ferromagnetic property for a long period of time are called permanent
magnets. The permanent magnetic material must retain large residual magentism and its magnetism
should not be easily destroyed. Thus it must have large coercivity. So, a permanent magnet should
have both large retentivity and large coerivity along with high permeability.
The best suited materials are cobalt, steel, alnico,ticonol etc. Steel is preferred for making permanant
magnets because its coercivity is large. The very suitable alloy of highest coercity is vicalloy
(vandium+iron+cobalt). Permanent magnets are used in galvanometerrs, voltmeters,ammeters
micropones, loud speakers, telephones, etc
b) Temporary magnets (or) Soft magnetic materials : These have low retentivity ,low coercivity
and small hysteresis loss and are suitable for temporary magnetism. These are used for making
electromagnets, cores of transformers, motors and generators. Soft iron, mu - metal and stalloy are
examples of ferromagnetic materials of this type.
In certain applications, the material goes through an cycle of magentization for a long period. This is the
case in transformer cores and telephone diapragms. The hysteresis curve of such materials must be
narrow. The energy dissipated and the heating will consequently be small. The material must have a
Page 216
MAGNETISM AND MATTER
high resistivity to lower eddy current losses. Properties of soft iron and steel: For soft
Electromagnets are used in electric bells loud iron, the susceptibility, permeability and
speakers and telephone diaphragms. Giant retentivity are greater while coercivity and
electromagnets are used in cranes to lift hysteresis loss per cycle are smaller than those
machinery, and bulk quantities of iron and steel of steel.
Hysteresis: It is defined as the tendency of I I Hysterisis
demagnetisation to lag behind the change in curve
magnetic field applied to a ferromagnetic Hysterisis H H
material. curve Soft Magnetic Hard Magnetic
Material
The process of taking a ferromagnet through a Material
cycle of magnetisation, results in loss of energy. Comparision between soft iron and steel
This is called hysteresis loss and it appears in
the form of heat.
Area of hysteresis loop is equal to the energy
loss per cycle per unit volume.
When a bar of ferromagnetic material is
magnetized by a varying magnetic field H and
the intensity of magnetization I induced is
measured. The graph of I versus H is as
shown is figure.
I
L A
B
Retentivity
M C
} F K
O H
Coercvity
D E
Page 218
MAGNETISM AND MATTER
Page 219
Page 220
ELECTROMAGNETIC INDUCTION
ELECTROMAGNETIC INDUCTION
INTRODUCTION (or)Tesla-m2
For a longtime electricity and magnetism remained CGS Units of magnetic flux : maxwell
as independent entities. In 19th century great
1wb = 108 maxwell
scientists like Ampere, Oersted established the
Dimensional formula: ML2T 2 A1
relation between magnetism and electricity.
Nature of Quantity = Scalar
“The phenomenon in which electric current is
NOTE:-
generated by varying magnetic field is called as
(1) Magnetic field lines may exist but flux linked with
Electromagnetic Induction”.
For layman’s explanation we can say that by the surface may not exist.In this case 900
taking help of magnetic fields we are converting BA cos 900 0
mechanical energy in to electrical energy. The (2) If the number of turns increases flux linked with
energy used to vary the magnetic field is the coil also increases by N times
transferred to electrons in the conductor. Hence
following law of conservation of energy.
B. A N BNA cos
We establish this fact by using different laws in (3) Area vector is always taken outward. Due to
this chapter.
this reason, entering flux is negative 180 0 ,
MAGNETIC FLUX
The total number of magnetic lines of force leaving flux is positive 0 , flux is zero
passing normally through an area placed in a
magnetic field is equal to the magnetic flux
90 0
n
linked with that area.
nB 0
ds B
B C
n n
BA A B BA
n || B n || B
Page 221
ELECTROMAGNETIC INDUCTION
B A
1 1
Solution: NAB NAB NAB
2 2
B BA cos q1 (induced charge in the circuit)
42 106 2.5 cos530 63Wb 1 NAB
1
R 2 R
Illustration 7: 2
1/ 2 200 3010-2 0.02
A uniform magnetic field exists in the space q1
50
ˆ ˆ
B B1i B2 j B3k ˆ . Find the magnetic flux 11.3110 C 3
through an area S if the area S is in X-Y (ii) 2 change in the magnitude flux during
plane rotation from 0o to 180o
Solution: = (–NAB) – (NAB) = – 2NAB
Since the field is uniform, we can use formula q2 = induced charge
B B.S 2NAB
= = 45.2 × 10–3C
R
Now when area S is in X-Y plane, it means
FARADAY’S EXPERIMENTS:
S SKˆ Figure-1
Hence B B1iˆ B2 ˆj B3kˆ . Skˆ B3S
C1
Illustration 8:
A circular coil of 200 turns and mean
radius 30 cm is placed in a uniform
G f
magnetic field of induction 0.02 T and is
free to rotate about an axis coinciding
with its own plane, but perpendicular to Figure-2
the uniform magnetic field. The coil is in
closed circuit having a total resistance of
50 . If the plane of the coil is initially
C1
perpendicular to the field, find the charge C2
flown through the circuit, when the coil is
rotated through (i) 60o (ii) 180o.
Solution: G f
Page 223
ELECTROMAGNETIC INDUCTION
here A1 & A2 are the areas of the coil at the in- Induced current is given by
stants t1 & t2 respectively. If the plane of the coil Induced emf N d
i
Re sis tan ce in the circuit R dt
is perpendicular to magnetic field 0 0 then
Induced charge:
e BN
A2 A1 The amount of charge induced in a conductor is
t2 t1 given as follows
e 1 d
(ii) Instantaneous emf in this case is given by
We know, I or I
R R dt
dA
e BN cos
dt dq 1 d 1
or dq d
(i) Average induced emf when only ' ' is changed dt R dt R
is given by (i.e., when the coil is rotated in uniform
1 f
q d
R o
magnetic field) Induced charge,
e BAN
cos 2 cos 1
t2 t1 1
q f i or q i f
R R
Where 1 & 2 are the angles made by the nor- We are genrally inserted only in the magnitude of
mal the charge.
drawan to A with B . In general, induced charge is given by
(ii) Instantaneous emf is given by change of magnetic flux
q
d cos resis tan ce
e BAN For N turns, the idnuced charge is
dt
If the coil is rotated with constant angular veloc- N
q d
ity ' ' then t and R
Conclusions:
d cos t
e BAN BAN sin t 1) Induced emf is independent of total resistance
dt of the circuit but depends on time of change of
e BAN sin t flux
(iii) If t 900 , i.e, if the plane is parallel to the 2) Induced current depends on both time of
change of flux and resistance of circuit
magnetic field then induced emf is maximum. Then
3) Induced charge is independent of time but de-
peak emf, e0 BAN pends on the resistance of circuit
e e0 sin t
LENZ’S LAW:
This is the principle of AC generator “The direction of the induced emf is always such
The change in flux by changing the number of that it tends to produce a current which opposes
turns is practically difficult. the change in magnetic flux”
Induced Current: Induced emf can exist whether the circuit is
If the magnetic flux in a coil of resistance R opened or closed. But induced current can exist
only in the closed circuits.
changes from 1 to 2 in a time ‘dt’, then a
A metallic ring is held horizontally and a bar
e magnet is dropped through the ring with its length
current ‘i’ is induced in the coil as i along the axis of the ring, as shown in figure.
R
N 2 1 d
i e N
Rdt dt
Page 225
ELECTROMAGNETIC INDUCTION
Induced
Current N N S
A B
Capacitor
a) emf is induced
b) Induced current flows from B to A along the
In both the cases net force on the magnet is coil when A and B are connected through
Fnet mg f capacitor.
Hence net acceleration of the fall is c) Electrons flow from A to B along the coil
d) Hence plate A will become positively charged
f
a net g a net g and plate B becomes negatively charged.
m When the two magnets are moved perpendicular
where f=force exerted by the induced magnetic to plane of coil as shown, then
field of ring on the magnet.
B
When the magnet is allowed to fall through an
A
open ring (or) cut ring, then S N S N
S
mg a) emf is induced
N
b) Induced current flows from A to B along the
Cut ring coil when A and B are connected through resistor.
c) Electrons flow from B to A along the coil
a) an emf is induced d) Hence plate A will become negatively charged
b) No current is induced (since the ring is not and plate B becomes positively charged.
closed) and hence no induced magnetic field. The directions of induced current in coil for
c) No opposition to the motion of the magnet. different kinds of motion of magnets
d) Fnet mg
e) a net g ,Magnet falls with an acceleration =
Clockwise induced current
g
When a magnet is allowed to fall through two
identical metal coils at different temperatures then
magnet falls slowly through the coil at low
Clockwise induced current
temperature as its resistance is less more induced
current flows so more is the opposition.
A magnet allowed to fall through a long cylindrical
pipe then the acceleration of magnet is always
less than ‘g’ and the acceleration continuously No induced current
decreases due to induced currents. But the
velocity increases until the magnet moves with (because there is no change of flux linked with
acceleration. At a particular instant the the coil)
acceleration becomes zero and the magnet When a current carrying conductor is placed
moves downwards with uniform velocity, called beside a closed loop in its plane then the induced
terminal velocity. current direction for the following are
Page 226
ELECTROMAGNETIC INDUCTION
Average emf e 1 2
t t
I
Here 1 linked with the loop = B.A. = 0 A
2d
× ×
In this case, the flux through the loop due to
current carrying wire is out of the plane of the × ×
coil. Solution:
As current is decreasing, the outward flux d
through the coil also decreases. (i) Induced e.m.f. E N N 1 t 2
dt
Hence to oppose this, an outward flux is created 1 nBA cos 1 2 500 100 104 cos (0) = 10
by the anti-clock wise induced current. [ 1 0, since area vector and B are in same
Illustration 9: direction]
2 2 BA cos 2 0 ;
A small conducting loop of area 1 mm is
placed in the plane of a long straight wire 2 10 0
E 1 ; E 100
at a distance of 20 cm from it. Current in dt 0.1
the straight wire changes from 100 A to E 100
zero in 0.01 second. Find the average emf (ii) Induced current I 10A
R 10
induced during this period. (iii) Induced charge q It 10 0.1 1C
Page 227
ELECTROMAGNETIC INDUCTION
Page 229
ELECTROMAGNETIC INDUCTION
Page 230
ELECTROMAGNETIC INDUCTION
l
P
V
B
e Bleff v where leff is the shortest distance be-
Q
tween two ends of wire.
b
c c
b From the diagram PQ L sin
2
a
a v1 v cos
e Blv sin
For any two points ‘a’ and ‘b’ the motional emf in 2
the direction from ‘b’ to ‘a’ is 5. A rod of length ‘l’is held horizontal along east-
a west and is dropped then the instantaneous emf
e v B . dl
b
induced in the rod can be BH vl . Where BH is
In general we can say that, motional emf in wire horizontal component of earth’s magnetic field.
‘acb’ in a uniform magnetic field is the motional Where v is instantaneous velocity v gt .
emf in an imaginary wire ‘ab’, Thus
eacb e ab length of ab v B
Here V is the component o f velocity
Page 231
ELECTROMAGNETIC INDUCTION
of area generated by the motion of the conductor Induced emf e de Bvdl B l .dl
l1 l1
as shown figure (B) and hence it doesn’t cut the
magnetic lines. So l2
l2
B 1 B l22 l12
Fn
N 2 l1 2
W BH
nE When the rod is fixed at one end then l1 0 ,
S
Bv V 1
l2 l ; e Bl 2
2
+
l Q
e
(B)
-
d O
0 and e 0
dt
Page 232
ELECTROMAGNETIC INDUCTION
V
i b
l
x
emf = Bbv B1 B2 bv
i 0i 0ibv 1 1
0 bv ; ;
2x 2 x l 2 x l x
0ilbv
2x x l
Illustration 23:
A horizontal magnetic field B is produced
across a narrow gap between the two square
iron pole pieces. A closed square loop of side
a, mass m and resistance R is allowed to fall
with the top of the loop in the field. The
loop attains a terminal velocity equal to :
B
Bva
a
B2 va 2
F = Bia ; ;
R R
When the loop attains terminal velocity ‘v’
Page 233
ELECTROMAGNETIC INDUCTION
R
B Illustration 27:
Figure shows a conducting rod PQ in
contact with metal rails RP and SQ, which
are 0.25m apart in a uniform magnetic field
of flux density 0.4T acting perpendicular
to the plane of the paper. Ends R and S are
Solution: connected through a 5 resistance. What
Along inclined plane the force acting downwards is the emf induced when the rod moves to
mg sin .......(1) the right with a velocity of 5ms 1 ? What is
magnetic force acting upwards F Bil the magnitude and direction of the current
through the 5 resistance? If the rod PQ
2 2
Blv B l v moves to the left with the same speed, what
B l ; ......(2)
R R will be the new current and its direction ?
From (1) and (2)
B2l 2 v mgR sin
mg sin ; v
R B 2l 2
Illustration 25: -
A wire of length 2l is bent at mid point so that
the angle between two halves is 600. If it moves Solution:
as shown with a velocity v in a magnetic field e Blv 0.4 0.25 5 0.5V
B find the induced emf.
e 0.5V
Current, I 0.1A
R 5
o o
Applying Fleming’s right hand rule, the current
30 30 l in the rod shall flow from Q to P.
l
(b) : If the rod PQ moves to the left with the
same speed, then the current of 0.1 A will flow
P Q in the rod PQ from P to Q
l
Solution: Illustration 28:
e=Blv. Here l = Effective length = PQ A loop ABCD containing two resistors as
shown in figure is placed in a uniform
x l magnetic field B directed outwards to the
sin 30 x l sin 30
l 2 plane of page. A sliding conductor EF of
length l and of negligible resistance moves
leff 2 x 2 l l ; e B leff v Blv
2
to the right with a uniform velocity v as
shown in Fig. Determine the current in
Illustration 26: each branch.
A conductor of length 0.1m is moving with A E B
a velocity of 4m/s in a uniform magnetic field
B
of 2T as shown in the figure. Find the emf R1 l v B R2
induced?
D F C
Solution:
The magnetic field induction B, length l and the
velocity v of the conductor EF are mutually
perpendicular, hence the emf induced in it is
Solution: e=Blv (with end F of the rod at higher potential)
e Blv 2 0.1 4 0.8v The effective electric circuit can be redrawn
as shown in Fig.
Page 234
ELECTROMAGNETIC INDUCTION
Page 235
ELECTROMAGNETIC INDUCTION
Applying Kirchhoff’s law to the left loop : For the rest of sections, the induced emf is in the
10i1 5 i1 i2 4 10 3 same sense, (i.e., from N to M)
Total emf induced in the conductor is
15i1 5i2 4 103 (1) b a
3
e de Bv dx
Right loop : 15i2 5 i1 i2 2 10 b
i
20i2 5i1 2 103 (2) Substituting for B 0 , the above equation
2x
Solving (1) and (2) gives : gets changed to
18 10 0iv dx
b a iv b a
i1 103 A and i2 10 3 A e e 0 ln x b or,,
55 55
b 2 x 2
iv
Current through 5 i1 i2 e 0 ln 1 a / b
2
8 8 From Fleming’s Right Hand Rule, end N is at
10 3 A mA low potential
55 55
Illustration 32: Illustration 33:
A conducting rod MN moves with a speed v A square loop of side a is placed in the same
parallel to a long straight wire which carries plane as a l ong str ai ght wi r e car r yi ng a
a constant current i, as shown in Fig. The cur rent i. The centr e of the loop is at a
length of the rod is normal to the wire. Find
the emf induced in the total length of the distance r from the wire, where r a . The
rod. State which end will be at a lower loop is moved away from the wire with a
potential. constant velocity v. Find the induced e.m.f.
in the loop?
Solution:
The magnetic field induction due to current i is
different at different sections of the rod, because
they are at different distances from the wire. Solution:
Let us, first of all, subdivide the entire length of Magnetic field by the straight wire of current i at
the conductor MN into elementary sections. 0i
Consider a section (shown shaded in the figure a distance r is B
2r
(b)) of thickness dx at a distance x from the wire.
flux associated with t he loop is
As all the three, v, B and (dx) are mutually
i
normally to each other, so the emf induced in it is BA 0 a 2
de=Bvdx. 2 r
(from N to M by Fleming’s right hand rule)
d 0 2 d 1 0 2 1 dr
e ia ia 2
dt 2 dt r 2 r dt
dx Hence the induced emf in the loop is
x
a2 dr
M N e 0 i 2 v v
B 2 r dt
de
Page 236
ELECTROMAGNETIC INDUCTION
Page 237
ELECTROMAGNETIC INDUCTION
ENERGY CONSIDERATION
Page 238
ELECTROMAGNETIC INDUCTION
Illustration 41: B
A short - circuited coil is placed in a time
varying magnetic field. Electrical power is
dissipated due to the current induced in the FM
coil. If the number of turns were to be
quadrupled and the radius of the wire is to T
be halved, then find the electrical power mg
dissipated. i) The terminal velocity achieved by the rod.
Solution: ii) The acceleration of the mass at the instant
Current is induced in the short-circuited coil when the velocity of the rod, is half the terminal
due to the imposed time - varying magnetic velocity.
field.
Solution:
e2 d i) the velocity of rod = V
Power P ; Here e where
R dt Intensity of magnetic field = B
NBA emf induced in rod (e)=BLV V
l BlV
and R 2 where l and r are length and current induced in rod (i)
r R
radius of the wire.
2
B VL2
2
2
r d Force on the rod F BiL
P NBA or R
l dt Net force on the system = mg - T
r 2 2 2 dB
2 mg - T = ma
P N A or
l dt B 2VL2
but T F
N 2r 2 R
P=(constant)
l B 2VL2
Hence, mg ma
r1 R
when r2 then l2 4l1
2 B 2VL2
2
P 4N r l
2 or a g ...............(i)
2 mR
P1 N2 2r 4l For rod to achieve terminal velocity VT , a 0
2 2
P 16 N r l P2 1
2 2 or P 1 B 2VT L2
P1 N 4r 4l 2
1
0 g
mR
Power dissipated is the same.
Illustration 42: mgR
or Terminal velocity VT 2 2 .........(ii)
A pair of parallel horizontal conducting B L
rails of negligible resistance, shorted at one V
ii) Acceleration of mass when V T
end is fixed on a table. The distance 2
between the rails is L and a rod of resistance mgR
R can slide on the rails frictionlessly. The or V . Put this value of V in (i)
2 B 2 L2
rod is tied to a massless string which passes
over a pulley fixed to the edge of the table. B 2 L2 mgR g
a g 2 2 or a g
A mass m, tied to the other end of the string, mR 2 B L 2
hangs vertically. A constant magnetic field g
B exists perpendicular to the table. If the or a ...............(iii)
2
system is released from rest, calculate :
Page 239
ELECTROMAGNETIC INDUCTION
Illustration 43: 2 2 1 1
(or) mg B l V 0 P1 P2
Two parallel vertical metallic bars XX 1 and R1 R2
YY 1 , of negligible resistance and separated [From equation (ii) and (iii)]
by a length ‘l ’, are as shown in Fig. The P1 P2
The terminal velocity is V0
ends of the bars are joined by resistance R1 mg
and R 2 . A uniform magnetic field of Substituting for V0 in equation (ii),
induction B exists in space normal to the 2 2
plane of the bars. A horizontal metallic rod B2l 2 P1 P2 Bl P1 P2 1
P1 R1
PQ of mass m starts falling vertically, R1 mg mg P1
making contact with the bars. It is observed Similarly from equation (iii)
2
that in the steady state the powers dissipated Bl P1 P2 1
in the resistance R1 and R2 and the terminal R2
mg P2
velocity attained by the rod PQ.
Illustration 44:
R1 The loop ABCD is moving with velocity ‘v’
X Y
towards right. The magnetic field is 4T.
B The loop is connected to a resistance of 8 .
P Q If steady current of 2A flows in the loop
l then find the value of ‘v’ if loop has a
X' Y' resistance of 4 : (Given AB=30cm, AD=30
cm)
R2 C
Solution: D
Let V0 be the terminal velocity attained by the v
rod PQ (in the steady state). If i1 and i2 be the 8
currents flowing through R1 and R2 in this state, A 370 B
then current flowing through the rod PQ is
i i1 i2 (see the circuit diagram) as shown in
Fig. Solution:
The induced emf in the loop is e Blv
e B AD sin 370 v 4 0.3sin 370 v
Effective resistance of the circuit is
R 4 8 12 ;
e Blv
Hence i
R R
Applying Kirchhoff’s loop rule, yields. 4 0.3 sin 370 v 100
i1 R1 BV0 l and i2 R2 BV0 l 2 ; v m/ s
4 8 3
1 1
i1 i 2 B V 0 l .....(i) Illustration 45:
R1 R 2 A square loop of side 12cm with its sides
2 2 2
2 B V 0 l parallel to x and y-axes is moved with a
Given that, P1 i1 R1 R .....(ii)
1 velocity 8 cm/s along positive x-direction
B 2V02 l 2 in an environment containing magnetic
2
P
and 2 2 2 i R ....(iii) field along +ve z-direction. The field has a
R2
Also in the steady state, the acceleration of PQ=0 gradient of 103 tesla/m along -ve x-direction
mg B i1 i2 l (increasing along -ve x-axis) and also
decreases with time at the rate of 103 tesla/
s. The emf induced in the loop is
Page 240
ELECTROMAGNETIC INDUCTION
Solution: B 2l 2
Blv dv dv
The magnetic field in loop varies with position lB m. dt
‘x’ of loop and also with time simultaneously. R dt v mR
The rate of change of flux due to variation of ‘B’ v t
dv B 2l 2 v B 2l 2 t
d dB v v dt ; ln t
with time is A mR 0
dt dt 1 v1 mR T
The rate of change of flux due to variation B with
t
position ‘x’ is mR T
where T 2 2 v vi e
d
A
dB dB dx dB
A A v
Bl
dt dt dx dt dx The speed of the bar therefore decreases
Since both cause decrease in flux, the two effects exponentially with time under the action of
will add up
magnetic retarding force.
The net emf induced
t
d dB dB dB dB emf iR Blv i e T ; current :
e A A v A v.
dt dt dx dt dx
Blv Bl Tt
4 3
144 10 10 8 10 3
i v1e
R R
144 9 107 129.6 10 6 V Illustration 47:
Illustration 46: Figure shows a square loop of side 5 cm
A bar of mass m and length l moves on two being moved towards right at a constant
frictionless parallel rails in the presence of speed of 1 cm/s. The front edge enters the
a uniform magnetic field directed into the 20 cm wide magnetic field at t = 0. Find
plane of the paper. The bar is given an the emf induced in the loop at (a) t = 2s,
initial velocity vi to the right and released. (b) t = 10 s, (c) t = 22 s and (d) t = 30s.
Find the velocity of bar, induced emf across
the bar and the current in the circuit as a B = 0.6 T
function of time × × ×
v=1cm/s ×
× × × ×
5 cm × × × ×
× × × ×
l R vi × × × ×
20 cm
Page 241
ELECTROMAGNETIC INDUCTION
and d. x x F2 x
a b
×b × × × c x x x l v
× × × × F1
× × × × F d I c
x x x
×a × × × d
Page 242
ELECTROMAGNETIC INDUCTION
Page 243
ELECTROMAGNETIC INDUCTION
If the magnetic field changes with time, then an because this electric field is produced only by
variable magnetic field.
d
emf e is induced in the loop. The induced PERIODIC EMI
dt Suppose a rectangular coil having N turns
current thus produced implies the presence of placed initially in a magnetic field such that
an induced electric field E that must be tangential magnetic field is perpendicular to it’s plane as
to the loop in order to provide an electric force shown.
on the charge around the loop. =2v
The work done by the electric field on the loop
in moving a test charge q once around the B
loop=qe. Because the magnitude of electric n
force on the charge is qE, the work done by the
electric field can also be expressed as qE 2r ,
R
where 2r is the circumference of the loop. : Angular speed
These two expressions for the work must be v : Frequency of rotation of coil
equal; therefore, we see that R : Resistance of coil
e For uniform rotational motion with , the flux
qe qE 2r ; E linked with coil at any time t
2r
NBA cos NBA cos t
Using this result along with Faraday’s law and
0 cos t where 0 NBA = maximum flux
the fact that B BA Br 2 for a circular loop,
(1) Induced emf in coil : Induced emf also
the induced electric field can be expressed as changes in periodic manner that’s why this
phenomenon called periodic EMI
1 d B 1 d r dB d
E
2r dt
2r dt
Br 2
2 dt
e
dt
NBAsin t e e0 sint
where e0 = emf amplitude or maximum
The emf for any closed path can be expressed emf NBA0
as the line integral of E.dl over that path. Hence, (2) Induced current : At any time t,
the general form of Faraday’s law of induction is e e0
i sin t i 0 sin t
d R R
e E.dl B where i0 = current amplitude or maximum current
dt
It is important to recognize that the induced i0
e0 NBA 0
electric field E that appears in the equation is a R R R
non-conservative field that is generated by a EDDY CURRENT
changing magnetic field. When a changing magnetic flux is applied to a
bulk piece of conducting material then circulating
Points to remember about induced electric field.
currents called eddy currents are induced in
1) The induced electric field is produced only by
the material. Because the resistance of the bulk
changing magnetic field and not by charged
conductor is usually low, eddy currents often
particles. have large magnitudes and heat up the
2) One cannot define potentials w.r.t this induced conductor.
field (a) These are circulating currents like eddies in
3) The lines of induced electric field are closed water.
curves and have no starting and terminating (b) Experimental concept given by Foucault hence
points. also named as “Foucault current”.
4) As long as the magnetic field keeps on (c) The production of eddy currents in a metallic
changing, the induced electric field will be present block leads to the loss of electric energy in the
form of heat.
Page 244
ELECTROMAGNETIC INDUCTION
(d)By Lamination, slotting processes the shaft of the vehicle and it rotates according to
resistance of the path for circulation of eddy the speed of the vehicle. The magnet is mounted
current increases, resulting into weakening them in an aluminium cylinder with the help of hair
and also reducing energy loss caused by them springs. When the magnet rotates, it produces
Plane metal
eddy currents in the drum and drags it through
plate an angle, which indicates the speed of the
××
vehicle on a calibrated scale.
×× (v) Energy meter : In energy meters, the
×× ×
×× × ×
amature coil carries a metallic aluminium disc
(A) Strong eddies produced which rotates between the poles of a pair of
cause excessive electro permanent horse shoe magnets. As the armature
magnetic damping rotates, the current induced in the disc tends to
Slotted metal oppose the motion of the armature coil. Due to
plate this braking effect, deflection is proportional to
×× the energy consumed.
××
×× ×
×× × × Illustration 50:
(B) Feeble eddies A uniform magnetic field B exists in a
gradual damping cylindrical region of radius 10 cm as shown
B
Strong eddy in figure. A uniform conducting wire of
currents length 80 cm and resistance 4.0 is bent
(C) Solid metallic core into a square frame abcd and is placed
Feeble eddy currents with one side along a diameter of the
cylindrical region. If the magnetic field
increases at a constant rate of 0.010 T/s,
(D) Laminated core find the current induced in the frame.
(e)Application of eddy currents : Though a d
most of the times eddy currents are undesirable
but they find some useful applications as × ×
enumerated below:
× × ××
(i) Dead-beat galvanometer : A dead beat
× × ××
galvanometer means one whose pointer comes × ×
to resting the final equilibrium position b c
immediately without any oscillation about the Solution:
equilibrium position when a current is passed Flux linked with the square loop is only due to
in its coils. flux change in one half of the circular area. so
This is achieved by winding the coil on a 2
B.A (where A = r /2)
metallic frame the large eddy currents induced
in the frame provide electromagnetic damping. = BA cos 0º = BA
(ii) Electric-brakes : When the train is running d
its wheel is moving in air and when the train is In the loop, emf E – d = – (BA)
dt dt
to be stopped by electric breaks the wheel is
made to move in a field created by dB r 2 dB
E –A = (0.1)2 (0.01)
electromagnet. Eddy currents induced in the dt 2 dt 2
wheels due to the changing flux oppose the = – 1.57 × 10–4 volt
cause and stop the train.
(iii) Induction furnace : Joule’s heat causes E 1.57 10–4
Current in the loop I = Amp.
the melting of a metal piece placed in a rapidly R 4
changing magnetic field. = 3.9 × 10–5 Amp.
(iv) Speedometer : In the speedometer of an
automobile, a magnet is geared to the main
Page 245
ELECTROMAGNETIC INDUCTION
Page 246
ELECTROMAGNETIC INDUCTION
But LI LI 0 n 2 IAl from (1) & (2) c) Breaking of circuit of decreasing of current
Circuit is made off
2
N N2 or I decreasing
L 0 n Al Since n , L 0 A
l l
Self inductance of coil depends on
i) Geometry of the coil e
di
e L
i.e., a) Number of turns of the coil - + dt
b) The length (l ) of the solenoid,
c) The area of cross-section (A) of the solenoid, (B)
ii) Medium inside the coil (permeability) MUTUAL INDUCTION
iii) Nature of the material of the core of the solenoid. When current in one coil changes, magnetic flux
More is the permeability of the medium, more is linked with the second coil placed near by it also
the self inductance changes. The emf induced in secondary is called
mutually induced emf and the phenomenon is
An inductor will have large inductance and low
called mutual induction.
resistance.
Resistor opposes the current, inductor opposes
the change of current S P
One can have resistance without inductance
One cannot have inductance without resistance.
An ideal inductor has inductance and no
resistance. G E K
When the current in the coil either increases or
decreases at a rate, then the coil can be imagined If ‘ i p ’ is current flowing in the primary coil, ‘ S ’
di is magnetic flux linked with secondary coil, then
to be a cell of emf e L. S i p
dt
One can have self inductance without mutual
S Mi p , M S
inductance. ip
One cannot have mutual inductance without self Here ‘M’ is called coefficient of mutual induction
inductance. or mutual inductance.
The direction of induced emf for different Induced emf in secondary coil is
states of current in a coil :
a) Steady current d di e
e M p (or) M
di dt dt di p / dt
0
i dt i Mutual inductance between two coils is equal to
the magnetic flux linked in the secondary coil
when unit current passes through the primary coil
e = 0 no opposition
b) Make of circuit or increasing current (or) emf induced in one coil when current in the
Circuit is made on other coil changes at the rate of 1 Amp/second.
or I increasing S.I. units : henry or H
Dimensional formula of self inductance or mutual
inductance is ML2T 2 A2
e
eL
di The value of mutual inductance depends on
+ - dt 1) Distance between the two coils
(A) 2) Number of turns of coils
Page 247
ELECTROMAGNETIC INDUCTION
Page 248
ELECTROMAGNETIC INDUCTION
loop L
2 2 0 l2
2
L 1 2
expression for kinetic energy E mv .
ENERGY STORED IN THE INDUCTOR:- 2
Consider an ideal inductor of inductance ‘L’ It shows that L is analogous to mass ‘m’ and self
connected with a battery. Let I be the current in inductance is called electrical inertia.
the circuit at any instant ‘t ’ The self inductance of a coil is numerically equal
to twice the energy stored in it when unit current
di
e L
dt
flows through it.
i.e., When i=1A, L=2U
VL di
E Induced power P e i Li .
dt
+ -+ -
In case of solenoid L 0 n 2 Al
This induced emf is given by
Magnetic energy stored per unit volume
dI
e L 1 2
dt Li
1
-ve sign shows that ‘e’ opposes the change of UB 2 U B 0n2i 2
current I in the inductor. Al 2
To drive the current through the inductor against B2
the induced emf ‘e’, the external voltage is Hence U B
applied. Here external voltage is emf of the 20
battery = E
Page 249
ELECTROMAGNETIC INDUCTION
Page 250
ELECTROMAGNETIC INDUCTION
1 2
If the flux linkage is maximum, then
U1 2 L1 I1 L1 I1 4 3
2 2
3 N 2 2 N11
iii) U 2 1
M 12 M 21 M ; M 12 M 21 i i
L2 I 22 L2 I 2 3 4 4 1 2
2 M 2 L1 L2 ; M L1 L2
Illustration 58: This is the maximum mutual inductance when all
The network shown is a part of the closed the flux linked with one coil is also completely
circuit in which the current is changing. linked with the other.
At an instant, current in it is 5A. Potential In general, only a fraction of the total flux will be
difference between the points A and B if the linked with the coil due to the flux leakage.
current is
M K L1L2
A B
5A 1 5H Where K-coefficient of coupling K 1
15V
1) Increasing at 1A/sec For tight coupling (or) if the coils are closely
2) Decreasing at 1A/sec wound, then K=1.
Solution: M max L1 L2
1) The coil can be imagined as a cell of emf
di INDUCTORS IN SERIES :
e L 5 1 5V ; Equivalent circuit is
dt If two coils of inductances L1 and L2 are
A B connected in series then the potential divides.
5A 1
15V 5V L1 L2
VA 5 1 15 5 VB VA VB 25V
di
e L 5 1 5V ; Equivalent circuit is
e1 e2
dt
A B
5A 1 5V
15V
VA 5(1) 15 5 VB e
Hence VA VB 5 15 5 15V di di di
i.e., e e1 e2 (or) LS L1 L2
RELATION BETWEEN L1, L2 AND M : dt dt dt
1 2 di
Since in series, is same for all coils
dt
LS L1 L2
N1 N2 If n coils of inductances L1 , L2 , L3 ..........Ln are
connected in series then effective inductance of
the arrangement,
L1 L2 L L1 L2 L3 .......... Ln
The flux linked with coil 1 is (when coils are far away)
N INDUCTORS IN PARALLEL :
N11 L1i1 L1 1 1
i1 If two coils of inductances L1 and L2 are
The flux linked with coil 2 is connected in parallel then the current divides.
N L1
N 2 2 L2i2 L2 2 2 i1
i2
N11
M on 1 because of 2 ; M 12 i
2 i i2 L2
N 2 2
M on 2 because of 1 ; M 21 i e
1
Page 251
ELECTROMAGNETIC INDUCTION
di di1 di2 dI
i.e., i i1 i2 (or) Solution: e M
dt dt dt dt
e e e
1 2
20 10 3 M
6 2
LP L1 L2 (or) M = 10mH
2
However in parallel as potential difference Illustration 60:
remains same i.e, e e1 e2 , so If the coefficient of mutual induction of the
primary and secondary coils of an induction
1 1 1 L1L2 coil is 6H and a current of 5A is cut off in
(or) LP
LP L1 L2 L1 L2 1/5000 second, calculate the emf induced in
the secondary coil.
If n coil of inductances L1 , L2 , L3 .........Ln are Solution:
connected in parallel then effetive inductance of dI 5
the arrangement, e M ; e 6 V 15 10 4 V
dt 1/ 000
1 1 1 1 Illustration 61:
.........
LP L1 L2 Ln A solenoid is of length 50 cm and has a
(when coils are far away) radius of 2cm. It has 500 turns. Around its
Let two coils of inductances L1 and L2 are central section a coil of 50 turns is wound.
connected in series and M is their mutual Calculate the mutual inductance of the
inductance. The flux linked with one coil will be system.
the sum of two fluxes which exist independently. Solution:
When the flux in the two coils support each other N P 500, N S 50 ; A 0.02 0.02m 2
N11 L1i1 M 12i2 0 4 107 Hm 1 ,1 50 cm 0.5 m
di1 di 0 N P N S A
From Faraday’s law, e1 L1 M 12 2 Now, M
dt dt l
2
Similarly N 22 L2i2 M 21i1 4 10 7 500 50 0.02
H
di di 0.5
e2 L2 2 M 21 1
dt dt 789.8 107 H 78.98 H
di di di di Illustration 62:
e e1 e2 L1 1 M 12 2 L2 2 M 21 1
dt dt dt dt A solenoidal coil has 50 turns per
centimeter along its length and a cross-
In series the current i and the change in current sectional area of 4 104 m 2 . 200 turns of
di another wire is wound round the first
di is same e L1 M 21 L2 M 12 solenoid co-axially. The two coils are
dt
electrically insulated from each other.
L L1 M 21 L2 M12 L1 L2 2 M Calculate the mutual inductance between
If the two coils oppose each other, then the two coils.
Solution:
n1 50 turns per cm= 5000 turns per metre
n2l 200, A 4 10 4 m 2 ; M 0 n1 n2l A
L L1 M L2 M L1 L2 2 M
Illustration 59: 4107 5000200 4104 H 5.03 104 H
Illustration 63:
Calculate the mutual inductance between
Two circular coils, one of smaller radius r1
two coils when a current of 2A changes to
and the other of very large radius r2 are
6A in 2 seconds and induces an emf of 20 placed co-axially with centers coinciding.
mV in the secondary coil Obtain the mutual inductance of the
arrangement.
Page 252
ELECTROMAGNETIC INDUCTION
Solution: 0 8 2
Suppose a current I2 flows through the outer i.e., B1 I
4 L
circular coil. The field at the centre of the coil is
So the flux linked with smaller loop
0 8 21 2
I 2 B1 S 2 l
B2 0 2 4 L
2r2
2 l2 l2
The second co-axially placed coil has very small and hence, M 2 2 0 M
I L L
radius. So B2 may be considered constant over
Illustration 65:
its cross-sectional area.
(a) A toroidal solenoid with an air core has
2 2 I
0 2
an average radius of 0.15m, area of cross
Now, 1 r1 B2 r1 2r or section 12 104 m 2 and 1200 turns. Obtain
2
the self inductance of the toroid. Ignore
0 r12 field variation across the cross section of the
1 I2 toroid. (b) A second coil of 300 turns is
2r2
wound closely on the toroid above. If the
Comparing wit h 1 M 12 I 2 , we get ; current in the primary coil is increased from
2 zero to 2.0 A in 0.05s, obtain the induced
0 r1
M 12 emf in the secondary coil.
2r2 Solution:
0 r12 r12 NI NI
M M M (a) B 0 n1 I 0 1 0 1
Also, 21 12 l 2r
2r
2 r2
It would have been difficult to calculate the flux 0 N12 IA
Total magnetic flux, B N1 BA
through the bigger coil of the non-uniform field 2r
due to the current in the smaller coil and hence
0 N12 A
the mutual inductance M12. The equality But B LI L
2r
M 12 M 21 is helpful. Note also that mutual
inductance depends solely on the geometry. 4107 1200 1200 12 104
L H
Illustration 64: 2 0.15
A small square loop of wire of side l is placed 2.3 103 H 2.3mH
inside a large square loop of wire of side d
(b) e 2 , where 2 is the total magnetic
L l . The loops are coplanar and their dt
centers coincide. What is the mutual flux linked with the second coil.
inductance of the system ? d d NI
e N 2 BA N 2 0 1 A
Solution: dt dt 2r
Considering the large loop to be made up of four
N N A dI
rod each of length L, the field at the centre, i.e., or e 0 1 2
at a distance (L/2) from each rod, will be 2r dt
0 I 410 1200 300 12 104 2
7
B 4 sin sin or e V
4 d 2 0.15 0.05
=0.023 V
0 I Illustration 66:
i.e., B 4 4 L / 2 2sin 45 Two solenoid A and B have same area of
cross-section. The ratio of lengths of two is
l 1 : 2 and the ratio of number of turns is 2 :
L 1. Find the ratio of self inductance of A to
that of B.
Page 253
ELECTROMAGNETIC INDUCTION
Page 255
ELECTROMAGNETIC INDUCTION
Page 256
ELECTROMAGNETIC INDUCTION
Page 257
ELECTROMAGNETIC INDUCTION
Illustration 81: t
and q q0 1 e
A coil of resistance 20 and inductance
0.5 henry is switched to dc 200 volt supply. where q0 is the maximum charge.
Calculate the rate of increase of current:
Where CR , called capacitive time
a) At the instant of closing the switch and constant
b) After one time constant
1
c) Find the steady state current in the c) When t . q q0 1 0.63 q0
e
circuit. d) Thus the capacitive time constant is the time
Sol: a) This is the case of growth of current in an in which the charge on the plates of the capacitor
L - R circuit. Hence, current at time t is given by
t becomes 0.63 q0
L
i i0 l e e) Smaller the value of CR, more rapid is the
Rate of increase of current, growth of charge on the condenser.
t f) Charge on the capacitor becomes maximum
di i0 L
e ; after infinite time and it is q0 EC . Then current
dt L
in the circuit becomes zero.
di i0 E / R E II. Decay of charge :
At t 0, dt L / R L a) When the capacitor is fully charged the key is
L
Page 258
ELECTROMAGNETIC INDUCTION
In the circuit shown in figure, after short circuiting Sol: When slab is removed, the potential difference
the battery 3R and 6R are parallel, so their across capacitor increases to kV0
6R 3R 2R 1
combined resistance is . Now CV kCV e as q KCV
6 R 3R 0 0 0 0
this 2R is in series with the remaining R. 1
t t
t
Hence, e k e ; ln t ln k
k
Rnet 2 R R 3R ; c Rnet C 3RC Illustration 84:
Calculation of steady state charge q0 : 4 F capacitor and a resistance 2.5M
At t , capacitor is fully charged and no are in series with 12V battery. Find the time
current flows through it. after which the potential difference across
P.D across capacitor = P.D across 3R the capacitor is 3 times the potential
difference across the resistor. [Given
V V CV
3 R , q0 ln(2)=0.693]
9R 3 3 1
Now, let charge on the capacitor at any time t Sol: a) Charging current i V0 e RC
be q and current through it is i1 . Then R
Potential difference across R is
t
1
q q0 1 e i.e., 0 3 RC
t / c q q 1 e VR iR V0 e RC
1
Potential difference across ‘C’ is
dq q0 t / c q0 3 RC 1 1
and i1 e e ...(1)
RC
RC
dt c 3RC VC V0 VR ;V0 V0e V0 1 e
but given VC 3VR , we get
t
q0 3e t / RC or 4e t / RC
1 e RC
t
RC
t
Applying Kirchhoff’s second law in loop e 4 ln 4 t 2 RC ln 2
RC
ACDFA, we have 6iR 3i2 R V 0 t 2.5 106 4 106 2 0.693
V or t = 13.86 sec
2i i2 ...(ii) Illustration 85:
3R In a circuit inductance L and capacitance
Applying Kirchhoff’s junction law at B, we have C are connected as shown in f igure and A 1
i i1 i2 ...(iii) and A2 are ammeters. When key k is pressed
Solving Eqs. (i), (ii) and (iii), we have to complete the circuit, then just after
V 2 V 2q0 t / c closing key k, the reading of A1 and A2 will
i2 i1 e be :
9R 3 9 R 3c
A c R 1 1
t
CV V 2q0 3 RC
where 0q i
i.e., 2 e A2
3 9 R 3RC L
t
V q0 t / c V q0 3RC R2
i e e k E
9 R 3c 9 R 3RC
Illustration 83: Battery
A parallel - plate capacitor, filled with a Sol: At t = 0 capacitor offers zero resistance and acts
dielectric of dielectric constant k, is charged like a short circuit. While inductor offers infinite
to a potential V0. It is now disconnected resistance and it acts like an open circuit.
from the cell and the slab is removed. If it Therefore no current flow through inductor
now discharges, with time constant , branch and maximum current flows through
through a resistance, then find time after capacitor branch.
which the potential difference across it will Hence reading of A2 is zero and reading A1 is
be V0? E
given by R
1
Page 259
Page 260
ALTERNATING CURRENT
ALTERNATING CURRENT
INTRODUCTION +
When a resistor is connected across the
I = I0 cost
terminals of a battery, a current is established
in the circuit. The current has a unique direction, + +
I or V t
it goes from the positive terminal to the negative O –
terminal via the external resistor. The magnitude
of the current also remains almost constant. V = V0 cost
This is called direct current (dc). –
+ (b)
I or V The symbol used to represent the ac source in
t a given circuit is as shown in figure.
– ac source
If the direction of the current in a resistor or in
any other element changes alternately, the current Note: Other forms of ac
is called an alternating current (ac). Saw-tooth form of ac
(OR)
If the current in a circuit changes its direction in
T + + +
every sec ,then current is called an alternating e or i –
2 V or I – –
current(ac) t
sinusoidal form of a.c
+ T
Square form of ac
2
T t
I or V +
T
– 4
3T
e or i
4 V or I t
ALTERNATING CURRENT AND
VOLTAGE: –
Alternating current is that current whose
magnitude changes with time and direction Peak value or crest value or amplitude of
reverses for every half cycle.
current or voltage I0 or V0 :
Similarly, the voltage whose magnitude and
direction change with time and attains the same The maximum value of current or voltage in an
magnitude and direction after definite time intervals ac circuit is called its peak value or crest value
is called alternating voltage.
If the current or voltage varies periodically as or amplitude.
‘sine’ or ‘cos’ function of time, the current or Instantaneous value of current or emf( I or E)
voltage is said to be sinusoidal The value of current or emf in an ac circuit at
I = I0sin t any instant of time is called its instantaneous value.
+
I or V + + Instantaneous current,
O t I I 0 sin t (or) I I 0 sin(t )
–
– Instantaneous emf
V = V0sin t E E 0 sin t (or) E E 0 sin( t )
(a) Where ( t ) is called phase
Page 261
ALTERNATING CURRENT
Illustration 1: of alternating voltage over one complete cycle
Write the general equation for the is also zero.
instantaneous e.m.f. of a 50 Hz generator The reason is that alternating current or voltage
whose peak voltage is 250 V. during first half cycle ( 0 to T/2) is positive and
Solution: Here E0 250V and v 50 Hz during other half cycle (T/2 to T) is negative.
Therefore, we find the mean or average value of
2 v 2 50 100 alternating current or voltage only over any half
cycle.
and E E0 sin t 250 sin100 t
Mean (or) Average Value Of ac For Half Cycle
Note : Since alternating current varies continuously It is an arithmetic average of all the values of
with time , its effect is measured by defining either current in a sine wave for half cycle. (or) The
the mean value (Average value ) of ac or by
value of steady current which sends the same
defining root mean square value or virtual
value of ac amount of charge through a circuit in a certain
Note:Average value of a function f(t) from t1 to t2 is time interval as sent by an alternating current
t2 t2 through the same circuit in the same time interval
f .dt f .dt (half the time period or half cycle) is known as
mean or average value of alternating current over
t1 t1
f av half cycle. (either positive half cycle or negative
defined as
t2 t2 t1 .
half cycle)
dt
The value of current at any instant ‘t’ is given by
t1
t2 I I0 sin t .
We can also find the value of fdt graphically.. Hence the mean or average value for half cycle
t1
is given by
nT T /2 T /2
Page 262
ALTERNATING CURRENT
Page 264
ALTERNATING CURRENT
I0 Illustration 10:
2. We know, I rms An alternating current is given by
2
i 3sin t 4 cos t A . Find rms current.
20
I rms 14.14 A Solution: i 3sin t 4cos t
2 t T
Illustration 7: 2 2
Page 265
ALTERNATING CURRENT
peak value of alternating voltage BOA represents the phase angle by which
= peak value of AC current I0 leads the alternating emf E0.
RMS value of alternating voltage The phasor diagram, in a simple representation
= RMS value of AC is
ADMITTANCE(Y): Reciprocal of impedance
of a circuit is called admittance of the circuit. Y Y
1
admittance (Y) = I
Z
S.I. Units ohm-1 i.e. mho or siemen. E I
PHASE: The physical quantity which represents
t
both the instantaneous value and direction of O X O X
E
A.C. at any instant is called its phase.
Y
It is dimensionless quantity and its unit is radian
Phase Difference: The difference between the
phases of current and voltage is called Phase I
difference.
If alternat ing emf and current are
X
E E0 sin t 1 and i i0 sin t 2 O
then phase difference is 1 2 Note:If e.m.f (or voltage) in A.C. is E = E 0 sin t and
The quantity varies sinusoidally with time and can
be represented as projection of a rotating vector, the current I = I0sin ωt+φ ,
is called as phasor. Where phase difference φ is Positive if current
A diagram, representing alternating emf and
current (of same frequency) as rotating vectors leads,Negative if current lags and zero if current
(Phasors) with phase angle between then is called is in phase with the emf (or voltage).
as phasor diagram. I,E E.m.f
Current
3
2
t
O 2
2
Page 266
ALTERNATING CURRENT
; R=constt. f =constt
E=E 0 sin t I I 0 sin t slope
I0 1
I
where 2
Irms R
2
Current lags emf by /2 or emf leads current by Irms or f R
/2
A.C THROUGH A RESISTOR power factor cos cos 0 1
A pure resistor of resistance R is connected Instantaneous power P E I sin 2 t
i o o
across an alternating source of emf
Average power over time ' T ' sec =
R
E2
Pavg Erms I rms cos Erms . I rms rms
R
A.C THROUGH AN INDUCTOR
A pure inductor of inductance L is connected
~
across an alternating source of emf E
E=E0 sin t L
The instantaneous value of alternating emf is
E = E0 sin t
The instantaneous value of alternating current
E E0
is I sint I 0 sin t ~
R R E=E0 sin t
The instantaneous value of alternating emf is
E0
Peak value of current, I 0 E = E0 sin t ...........(1)
R
dI
Graphical representation of A C The induced emf across the inductor = L.
dt
I,E E.m.f which opposes the growth of current in the
Current circuit. As there is no potential drop across the
circuit, so
2
t dI dI
O
3 E L. 0 or L. E
2 2 dt dt
dI E0
sin t ;
dt L
Phasor diagram: On integrating
Y
E0
I cos t I 0 sin t .......... 2
L 2
The instantaneous value of alternating current
is
O X
I I0 sin t
2
I E
Where Peak value of current,
emf and current will be in phase 0
E E
emf and current have same frequency I0 0 0
L X L
Peak emf is more than peak current From equation 1 & 2
The value of impedance (Z) is equal to R and Phase difference between alternating voltage and
reactance (X) is zero
Apart from instantaneous value, current in the current is
circuit is independent of frequency and decreases 2
The alternating current lags behind the emf
with increase in R (similar to that in dc circuits).
by a phase angle of
2
Page 267
ALTERNATING CURRENT
Graphical representation of A C
Y
I, E
E.m.f
Current
For dc, f 0 X L 0
Phasor diagram For ac, high frequencies, X L
VL dc can flow easily through inductor..
Inductive reactance in terms of RMS value
E
is X L L rms
Irms
2
LL Power supplied to inductor
The instantaneous power supplied to the inductor
Inductive Reactance (XL) is
The opposition offered by an inductor to the flow
of ac is called an inductive reactance.
PL iv i0 sin t v0 sin t
The quantity L is analogous to resistance and is 2
called reactance of Inductor represented by XL . i0v0 cos t sin t
It allows D.C. but offers finite impedance to the
flow of A.C. i0 v0
sin 2 t
Its value depends on L and f. 2
Inductance not only causes the current to lag So, the average power over a complete
behind emf but it also limits the magnitude of cycle is
current in the circuit.
I0
E0 E
L 0 X L , Pavg Erms . I rms cos 0 90
L I0
i0 v0
sin 2 t 0
X L L 2 fL X L f ; 2
X L L curve Since the average of sin 2t over a
Y complete cycle is zero.
Thus, the average power supplied to an
inductor over one complete cycle is zero.
slope = tan = 2L
XL
X
f
N S
X L f curve
A B
Page 268
ALTERNATING CURRENT
Page 269
ALTERNATING CURRENT
Page 270
ALTERNATING CURRENT
1
the A.V. by a phase angle of . XC 5
63.69
2 5 10 2 3.14 50
1 1 Erms 100
Its value is Xc . Step 2: I rms X 63.69 1.57 A
C 2fC C
XC XC
VL VR
E
or f C
Note: Resistance, Impedance and Reactance have the ~
same units and Dimensional Formulae.
i.e. SI unit is ohm; Dimensional Formula is The potential difference across the inductor is
ML T
2 3 2
A given by, VL IXL .....(1)
Power supplied to capacitor: The potential difference across the resistor,
The instantaneous power supplied to the VR IR .....(2)
capacitor is
current I lags the Voltage VL by an angle of ,
2
Pc iv i0 cos t v0 sin t Therefore, the resultant of VL and VR is
2 2
i0 v0
sin 2 t
OC OA 2 OB2 or E VR VL
i0 v0 cos t sin t ; y
2
C
So, the average power over a complete cycle is B
zero
XL
since sin 2t 0 over a complete cycle. (or) E
VL
Pavg Vrms . I rms cos Vrms . I rms cos 90 0
no power is consumed in a purely
capacitive circuit. x
VR (or) R A
I
Illustration 16: A 50 F capacitor is connected Using equations ( 1 ) and (2), we get
to a 100 V, 50Hz ac supply. Determine the 2 2
rms value of the current in the circuit. E I2R 2 I2 X L I R 2 X L
Solution : Here where X L L is the inductive reactance.
E
C 50 F 50 10 6 F or I
2 .....(3)
5 105 F R2 XL
E
I 2 2 2 2 2
Erms 100V , v 50 Hz Z LR ; Z LR R X L R L
Step 1: Capacitive reactance The effective opposition offered by LR circuit to
1 1 ac is called the impedance of LR circuit.
XC
C C 2 v
Page 271
ALTERNATING CURRENT
Page 272
ALTERNATING CURRENT
tan 1 1.57 57o30 ' 57.5o The resultant potential difference of VC and VR is
57.5 represented by OC.
0.3194 radians Also, the emf and current are in phase when ac
180
Time lag between E0 and I0 is given by flows through the resistor. So, VR is represented
by OA along X-axis.
0.3194
t 3.194 103 s Therefore, the resultant potential difference of
2 v 2 50
VC and VR is represented by OC and is given
C-R SERIES CIRCUIT WITH
by
ALTERNATING VOLTAGE
2 2
Let an alternating source of emf E =E0 sin t is OC OA 2 OB2 or E VR VC
connected to a series combination of a pure Using equations (i) and (ii), we get
capacitor of capacitance (C) and a resistor of 2 2
E I2R 2 I2XC I R 2 XC
resistance (R) as shown in figure (a)
C R E E
or I
2 2
R XC Z CR
VC VR From the above equations of I and E, we have
1
2
2
Z CR R X C 2 R 2
~ C
E
Let I be the r.m.s value of current flowing through Where Z CR is the effective opposition offered
the circuit. The potential difference across the
capacitor, by the CR circuit to ac, which is the impedance
of CR circuit.
VC IX C .....(i)
Let be the angle made by E with X-axis
AC VC IX C
The current leads emf by an angle when ac tan
2 OA VR IR
flows through capacitor. X 1
The potential difference across the resistor, or tan C
R C R
VR = IR .....(ii)
In series CR circuit, emf lags behind the current
The emf and current are in phase when ac flows
or in other words, the current is said to lead the
through resistor.
emf by an angle given by the above equation.
Phasor diagram.
I Current in C-R series circuit is given by
V (or) R A E E
I 0 sin( t )
O R
X
Z CR Z CR
(or) I I0 .sin(t )
V E
C
(or) Note:
XC
The resultant potential difference of VC and VR is
B represented by OC.
C
-Y
Impedance of CR circuit.
In figure VC is represented by OB along negative 2 1
Z CR R 2 X C R 2
Y - axis and the current I is represented along C 2
2
1
X - axis. R2
4 f 2C 2
2
Page 273
ALTERNATING CURRENT
of ac. Z R and for very low frequency of ac, L - C SERIES CIRCUIT WITH
Z ALTERNATING VOLTAGE
Phase angle between voltage and current is given Let an alternating source of emf is connected to
by the series combination of a pure capacitor of
1 1 capacitance (C) and an inductor of inductance
tan
C R 2 fC R (L) is shown in fig.
As f increases, phase angle decreases.
L C
Illustration 20: A series circuit contains a
resistor of 20 , a capacitor and an ammeter
of negligible resistance. It is connected to a V LV C
2
X C ZCR R2 The voltage VL and VC are represented by OB
or 2 2 and OC respectively.
80 20 77.46
Y
Illustration 21: A 10 F capacitor is in sereis
with a 50 resistance and the combination B
Page 274
ALTERNATING CURRENT
Z R 3
resonance occurs.
Since is negative, the current in the
E0
At resonance Z 0 and I 0 . circuit lags the voltage across the source.
Z (c) The power dissipated in the circuit is
1
Resonant angular frequency 0 P I 2R
LC im 1 283
1 Now, I 40 A
Resonant frequecny f 0 . 2 2 5
2 LC 2
Illustration 22: When a capacitor of small Therefore, P 40 A 3 4800W
capacitance is connected in sereis with
series L-R circuit. The alternating current (d) Power factor = cos cos 53.1o 0.6
in the circuit increases. Explain why? A.C THROUGH LCR SERIES CIRCUIT
Solution : Addition of capacitor in the given A circuit containing pure inductor of inductance
(L),pure capacitor of capacitance (C) and
Page 275
ALTERNATING CURRENT
R X L X C
2 2 2
R L
emf to the LCR circuit. C
X X
sin
Z
R
VL VC VR cos
Z
R
V Illustration 24:
L C R A pure inductor of 50.0 mH is connected
to a source of 220 V. Find the inductive
reactance and rms current in the circuit if
VL VC VR the frequency of the source is 50 Hz.
Solution:
The inductive reactance.
~
X L 2fL 2 3.14 50 50 103
E
For ‘dc’ 15.7
The rms current in the circuit is
V = VR + VL + VC V IZ
Vrms 220V
(only before steady state) I rms 14.01A .
X L 15.7
For ‘ac’
Illustration 25:
2
V I R2 X L X C ; If an input of 50 mV is applied as Vin then
Vout at 100 kHz will be
2
IR 2 IX L IX C 1k
V 2 = VR 2 + VL - VC
2
10 nF Vout
2
V VR2 VL VC
Solution:
where VL IX L I L and
1 1 10 3
I XC 8 159
VC IX C Cω 10 2 π 10 5 2 π
C
and VR = IR Vin 50 159
Note: Rules to be followed for various combinations V0 XC 7.9 mV
|Z| 10002 1592
of ac circuits
Compute effective resistance of the circuit as R
Calculate the net reactance of the circuit as
1592 is neglected
Page 277
ALTERNATING CURRENT
Illustration 26: Illustration 28:
30.0 F capacitor is connected to a 220 V,, An LCR circuit contains resistance of 100
50 Hz AC source. Find the capacitive and supply of 200 V at 300 rad/sec. If
reactance and the current (rms and peak) only capacitance is taken out from the
in the circuit. If the frequency is doubled, circuit and the rest of the circuit is joined,
what happens to the capacitive reactance current lags behind the voltage by 60°. If
and the current. on the other hand, only inductor is taken
Solution: out, the current leads by 60° with applied
The capacitive reactance is voltage. Find the current flowing in the
1 circuit.
XC 106
2fC Solution : Resistance R = 100
The rms current is Applied voltage V0 200 volt
Vrms
I rms 2.08A When the capacitance is taken out, only inductor
XC
and resistor will remain in circuit.
The peak current is
XL
Im 2I rms 2.96A The phase tan ( )
R
If the frequency is doubled, the capacitive
reactance is halved and consequently, the XL
tan (60o ) X L 3R
current is doubled. R
Illustration 27: When the inductor is taken out, only capacitor
When an AC source of e.m.f. E = E0 sin and resistance will remain in circuit.
(100t) is connected across a circuit, the XC
phase difference between the e.m.f. E and Then phase tan ( )
R
the current I in the circuit is observed to
be / 4 , as shown in the figure. If the XC
tan (60o ) XC 3 R
circuit consists possibly only of R-C in R
series. What will be the relation between Impedance
the two elements of the circuit ?
Z R 2 ( X C X L ) 2 R 100
I E
[since X C X L 3 R ]
V0 200
t Current in the circuit I 0 2 A.
Z 100
Illustration 29:
Solution: In a series resonant circuit. the ac voltage
Given E = E0 sin (100 t). Comparing this with across resistance R, inductance L and
1
E = E0 sin t , we have 100 rad s . It capacitance C are VR = 4V, VL = 10V and
follows from the figure that the current leads VC = 7V respectively. Find the voltage
the e.m.f. which is true only for R-C circuit. applied to the circuit.
1 Solution:
tan ..... (1)
RC Voltage applied across circuit
Given / 4 . Also 100 rad s 1 . Using
2 2
these values in (1), we get V VR2 VC VL 42 10 7 = 5V
1 1
tan or RC
4 100 RC 100
Page 278
ALTERNATING CURRENT
Y
RESONANT FREQUENCY
Electrical resonance is said to take place in a R3>R2>R1
series LCR circuit, when the circuit allows I0
maximum current for a given frequency of
R1
alternating supply, at which capacitive reactance
R2
becomes equal to the inductive reactance. I
The current (I) in a series LCR circuit is given by R3
E E O X
I f0
Z 1
2
f
R 2 L .....(i)
C Note: Series LCR circuit at resonance admit
maximum current at particular frequencies, so
From the above equation (i), it is clear that they can be used to tune the desired frequency
current I will be maximum if the impedance (Z)
or filter unwanted frequencies. They are used
of the circuit is minimum.
in transmitters and receivers of radio, television
At low frequencies, L L 2 f is very small and telephone carrier equipment etc.
1 1
and C C 2 f is very large. RESONANCE IN L - C CIRCUIT :
At resonance ,
At high frequencies, L is very large and a) Net reactance X = 0
1
is very small. b) X L XC
C
1 c) Impedance Z = 0
For a particular frequency (f0), L i.e. E0
C d) peak value of current I0
X L X C and the impedance (Z) of LCR Z
1
circuit is minimum and is given by Z = R. e) Resonant frequency f 0
2 LC
Therefore, at the particular frequency ( f0 ), the
f) Voltage and current differ in phase by
current in LCR circuit becomes maximum. The 2
frequency ( f0 ) is known as the resonant g) Power factor cos 0
RESONANCE IN L - C - R CIRCUIT :
frequency and the phenomenon is called
At resonance,
electrical resonance.
a) Net reactance X = 0
Again, for electrical resonance (XL–Xc) = 0. b) X X
L C
i.e. XL = XC c) Impedance Z = R ( minimum )
1 1 E E
or L 2 d) peak value of current I0 0 0 ( maximum
C LC Z R
1 1 but not infinity )
or 2f 0 1
LC LC e) Resonant frequency f 0
1 2 LC
or 0 f .....(ii) f) Voltage and current will be in phase
2 LC g) power factor cos 1
This is the value of resonant frequency. h) Resonant frequency is independent of value of R.
The resonant frequency is independent of the i) A series L - C - R circuit behaves like a pure
resistance R in the circuit. However, the resistive circuit at resonance.
sharpness of resonance decreases with the
HALF POWER FREQUENCIES AND
increase in R.
BAND WIDTH.
Series LCR circuit is more selective when
The frequencies at which the power in the circuit
resistance of this circuit is small.
Page 279
ALTERNATING CURRENT
is half of the maximum power (The power at 1
resonance) are called half power frequencies. But at resonance, 0
Pmax LC
1 L 1 L
Q
LC R R C
P Pmax
P= I0
2
R = very low
1 2 3 V Q - Factor = Large
The current in the circuit at half power R = low
Q - Factor = Normal
1 R = High
frequencies (HPF) is or 0.707 or 70.7% Q - Factor = Low
2 V0 V
of maximum current (current at resonance). Resonance curve
There are two half power frequencies
1 called lower half power frequency. At (iv) Q - Factor tells the relation between voltage
this frequency the circuit is capacitive. across the inductor or capacitor and peak value
3 called upper half power frequency. It is of voltage.
greater than 2 . At this frequency the circuit is The sharpness or selectivity of a resonance
circuit is measured by Q-factor, called quality
inductive.
factor.
Band width : The difference of half The Q-factor of series resonance circuit is
power frequencies 1 and 3 is called band defined as the ratio of voltage developed across
the inductance or capacitance at resonance to
width and 3 1 .
the applied voltage (which is the voltage across
For series resonant circuit it can be proved R).
R / L Q
voltage across L or C
QUALITY FACTOR (Q) OF SERIES Applied Voltage( voltage acrossR)
RESONANT CIRCUIT : IX L X L L 1 L
(i) The characteristic of a series resonant circuit Q
IR R R LC R
is determined by the quality factor (Q - factor)
of the circuit. 1 L
(ii) It defines sharpness of i - v curve at Q=
R C
resonance. When Q - factor is large, the
sharpness of resonance curve is more and VL VC 0 L 1
viceversa. Q - factor V or V R or CR
R R 0
Re sonant frequency
(iii) Q 0 (v) The quality factor (Q) is also defined as 2
Band width times the ratio of the energy stored in L (or C)
For series resonant circuit, it can be proved to the average energy loss per period.
R max imum energy stored in circuit
that, Q 2
L energy loss per period
0 L The maximum energy stored in inductor,
Q = 0
R
Page 280
ALTERNATING CURRENT
Page 281
ALTERNATING CURRENT
E2rms R E 2rms R
1
2
R 2 L2 2
2
R L
C Case V : If the ac circuit contains C and R,
then
Average power is also known as true power.
The quantity E rms I rms is called the apparent R R 2
Erms R
cos Pav
power or virtual power. Z 1 1
R2 2 2 , R2 2 2 .
It is customary to express true power in kW C C
and apparent power in kVA.
Wattless current or idle current
cos is called the power factor of LCR circuit.
If the voltage and current differ in phase by
Its value varies from zero to 1.
/ 2 , then Power factor,, cos cos90 0 0 .
Power factor is defined as the ratio of true In this case, as there is no resistance to the flow
power to apparent power. of current, the average power consumed by the
In L – C – R series circuit, current is zero. Such a current is, therefore,
R R called wattless current. Since this current does
Power factorcos
Z 1
2 not perform any work, therefore, this current
R 2 L
C may also be called idle current. Such a current
flows only in purely inductive, purely capacitive
POWER FACTOR :
or L–C circuits.
(a) It may be defined as cosine of the angle of
The average power consumed per cycle in a
lag or lead (i.e. cos ) pure inductive or capacitive or L - C circuit is
(b) It is also defined as the ratio of resistance zero.
R Such current for zero power consumption is
and impedance (i.e. )
Z called Wattless current.
c) The ratio CHOKE COIL:
True power W kW Choke coil (or ballast) is a device having high
cos inductance and negligible resistance.
Apparent power VA kVA
It is used to control current in ac circuits and is
Special Cases : used in fluorescent tubes.
Case I : If the ac circuit contains only pure resistance, The power loss in a circuit containing choke
then 0 0 . coil is least.
E2 In a dc circuit current is reduced by means of a
Pave E rms I rms cos 00 E rms I rms rms rheostat.This results in a loss of electrical energy
R
Case II : If the ac circuit contains only pure I 2 R per sec.
inductance, then 90 0 Iron core
Pave E rms .I rms cos 90 0
Starter
Case III : If the ac circuit contains only pure Choke
Coil of Cu wire ~
capacitance, then 90 . Pave 0
0
~
coil
L, R
Application of choke coil
Case IV : If the ac circuit contains L and R, then Choke coil
R R It consists of a copper coil wound over a soft
cos
Z R 2 L2 2 iron laminated core. This coil is put in series
with the circuit in which current is to be reduced.
R Soft iron is used to improve inductance (L) of
Pave E rms I rms
R L
2 2 2
the circuit.
The inductive reactance or effective opposition
Page 282
ALTERNATING CURRENT
Illustration 31:
of the choke coil is given by X L L 2 vL A 750 Hz., 20 V source is connected to a
For an ideal choke coil r 0 , no electric energy resistance of 100 ohm, an inductance of
0.1803 henry and a capacitance of 10
is wasted, i.e., average power P = 0.
microfarad all in series. Calculate the
In actual practice choke coil is equivalent to a time in which the resistance (thermal
RL circuit. capacity 2 J/0C) will get heated by 100C.
Choke coil for different frequencies are made TC mc thermal capcity
by using different substances in their core. Solution:
For low frequency L should be large thus iron
X L L 2 fL 2 750 0.1803 849.2
core choke coil is used. For high frequency ac
and,
circuit, L Should be small, so air cored choke
1 1 1
coil is used. XC 5
21.2
The choke coil can be used only in ac circuits C 2 fC 2 750 10
not in dc circuits, because for dc frequency So, X X L X C 849.2 21.2 828
and hence,
v 0 . Hence X L 2 vL 0.
Z R 2 X 2 (100) 2 (828) 2 834
Choke coil is based on the principle of Wattless
current. But as in case of ac,
V R
E Pav Vrms I rms cos Vrms rms
The current in the circuit I Z Z
Z i.e.,
with 2 2
Vrms 20
2 2
Pav R 100 0.0575W
Z R r L . Z 834
and as,
The power loss in the choke U P t mc (TC)
pav Vrms I rms cos 0 (TC) 2 10
t 348s
r r r P 0.0575
as cos Z 2 2 2
L
0 Illustration 32:
r L
In the LCR series circuit find the voltmeter
Note: and ammeter reading in the figure shown
below. Also find the quality factor of
circuit.(Given R=50 )
400 V 400 V
V VL VC
A
Pavg Erms irms cos 100V, 500 Hz
Solution :
Pavg is due to irms cos , and there is no In the figure voltages across source, inductor
contribution of current com ponent and capacitor are given as V = 100 V, VL =
irms sin 400 V and VC = 400 V respectively.
Page 283
ALTERNATING CURRENT
Page 284
ALTERNATING CURRENT
i
I0 2
I rms 2
2 2
+ +
π C L
Voltage V 2cos ωt 2sin ωt – –
2
Page 285
ALTERNATING CURRENT
Let q be the charge on the capacitor at any oscillations
In L - C oscillations energy stored in inductor
di
time t and be the rate of change of current. is analogous to kinetic energy of the mass in
dt mechanical oscillations
Since no battery is connected in the circuit, In L - C oscillations maximum charge on ca-
q di pacitor q0 is analogous to amplitude in mechani-
L. 0
c dt cal oscillations
dq As Vmax = A in mechanical oscillations,
but i I 0 q0 0 in L- C oscillations
dt
from the above equations, we get ENERGY OF LC OSCILLATIONS:
q d2q
L 2 0 Let q0 be the initial charge on a capacitor. Let
C dt
2
the charged capacitor be connected to an
dq 1 inductor of inductance L. LC circuit will sustain
q 0 The above equation is
dt 2 LC an oscillations with frequency At an instant t,
analogous to charge q on the capacitor and the current i are
d2 r given by; q(t ) q0 cos t ; i q0 sin t
2
2 r=0 (differential equation of S.H.M)
dt Energy stored in the capacitor at time t is
Hence on comparing
1 1 q 2 q02
1 1 U E CV 2 cos2 (t )
2 ; 2 2 C 2C
LC LC Energy stored in the inductor at time t is
1 1 1
2 ; U M Li 2
LC 2 LC 2
2
The charge therefore oscillates with a frequency 1 q 1
Lq02 2 sin 2 t 0 sin 2 (t )( 2 )
1 2 2 C LC
and varies sinusoidally with time. Sum of energies
2 LC
COMPARISION OF L - C q02 2 2 q02
OSCILLATIONS WITH SHM : U E U M (cos t sin t ) \
2C 2C
The L - C oscillations can be compared to
S.H.M of a block attached to a spring As q0 and C, both are time independent, this
1 sum of energies stored in capacitor and inductor
In L - C oscillations 0 is constant in time. Note that it is equal to the
LC
K initial energy of the capacitor.
In Mechanical oscillations 0 where K i i m i
m ++++ + + + + ––––
is the spring constant C L
–––– – – – – ++++
1 V
In L - C oscillations tells us the 2 1 Li 2
C q U=
q U =0 E
m
B U =0 U =
2c
E B
m
2c
potential difference required to store a unit
charge
F 1 1
In a mechanical oscillation K tells us the U = kA , K= 0 2
U 0, K 0 1
U = 0, K = mv U 0, K 0
2
2
U = kA , K= 0
x 2 2
max 2
external force required to produce a unit dis- (a) (b) (c) (d) (e)
placement of mass Fig : The oscillations in an LC circuit are
In L - C oscillations current is the analogous analogous to the oscillation of a block at the
quantity for velocity of the mass in mechanical end of a spring. The figure depicts one-half of
oscillations a cycle.
In L - C oscillations energy stored in capacitor At t = 0, the switch is closed and the capaci-
is analogous to potential energy in mechanical tor starts to discharge. As the current in-
Page 286
ALTERNATING CURRENT
creases, it sets up a magnetic field in the d2 x
dv
inductor and thereby, some energy gets F ma m m 2
stored in the inductor in the form of magnetic dt dt
energy: UB = (1/2) Li2. As the current reaches di d 2q
its maximum value im, (at t = T/4) as in Fig. For an electrical system, e L L
dt dt 2
(c), all the energy is stored in the magnetic Comparing these two equations, we see that
field: UB = (1/2) Li2m. L is analogous to mass m: L is a measure of
You can easily check that the maximum resistance to change in current.
electrical energy equals the maximum mag- 1
netic energy. The capacitor now has no In case of LC circuit, 0 and for
charge and hence no energy. The current now LC
starts charging the capacitor, as in Fig. (d). mass on a spring, 0 k / m . So, 1/C is
This process continues till the capacitor is analogous to k. The constant k (=F/x) tells us
fully charged (at t = T/2) Fig. (e). But it is the (external) force required to produce a unit
charged with a polarity opposite to its initial displacement whereas 1/C (=V/q) tells us the
state in Fig. (a). The whole process just potential difference required to store a unit
described will now repeat itself till the system charge. Table gives the analogy between
reverts to its original state. Thus, the energy in mechanical and electrical quantities.
the system oscillates between the capacitor ANALOGIES BETWEEN MECHANICAL AND
and the inductor. ELECTRICAL QUANTITIES
The LC oscillation is similar to the mechanical
oscillation of a block attached to a spring. Mechanical System Electrical System
The lower part of each figure depicts the
corresponding stage of a mechanical system Mass m Inductance L
(a block attached to a spring). As noted Force constant k Reciprocal
earlier, for a block of a mass m oscillating capacitance l/C
with frequency 0 , the equation is Displacement x Charge q
Velocity v = dx/dt Current I = dq/dt
d2 x Mechanical energy Electromagnetic
20 x 0
dt 2 energy
1 1 2
Here, 0 k / m , and k is the spring E kx 2 mv 2 U 1 q 1 L 2
constant. So, x corresponds to q. In case of 2 2 2C 2
a mechanical system
Page 287
ALTERNATING CURRENT
6) Peak E0 E0 E0 E0
I0 I0 I0 I0
value of Z Z Z Z
current
7) Phase L 1 90o X XC
diff. tan 1 tan 1 tan 1 L
R RC R
between E
&I
8) Lead / I lags E by φ I leads E by φ If X L X C , I lags E by 90o If X L X C , I lags E by
lag o If X L X C , I leads E by
If X L X C , I leads E by 90
If X L X C , E and I are in phase If X L X C , no phase difference between E and I
TRANSFORMER
Step up transformer Step down transformer
It is a device which raises or lowers the
voltage in ac circuits through mutual induc- It increases voltage It decreases voltage
tion. and decreases current and increases current
It consists of two coils wound on the same
P S P S
core. The alternating current passing through
the primary creates a continuously changing
VS > VP VS < VP
flux through the core. This changing flux
induces an alternating emf in the secondary. NS > NP NS < NP
(a) Transformer works on ac only and never on ES > EP ES < EP
dc. iS > iP iS < iP
(b) It can increase or decrease either voltage or RS > RP RS < RP
current but not both simultaneously.
tS > tP tS < tP
(c) Transformer does not change the frequency of
input ac. k>l k<l
(d) There is no electrical connection between the (h) Efficiency of transformer ( ) : Efficiency
winding but they are linked magnetically. is defined as the ratio of output power and
(e) Effective resistance between primary and input power
secondary winding is infinite.
(f) The flux per turn of each coil must be same out P S S Vi
i.e. % P 100 V i 100
dS d in P P
i.e. S P : P For an ideal transformer Pout = Pin so
dt dt
(g) If NP = Number of turns in primary, NS = =100% (But efficiency of practical trans-
number of turns in secondary, VP = applied former lies between 70%–90%)
(input) voltage to primary, VS = Voltage For practical transformer Pin = Pout + Plosses
across secondary (load voltage or output), eP Pout Pout P P
= induced emf in primary; eS = induced emf in so 100 100 in L 100
Pin Pout PL Pin
secondary, = flux linked with primary as
(i) Losses in transformer : In transformers
well as secondary, iP = current in primary; iS = some power is always lost due to heating
current in secondary (or load current) effect, flux leakage, eddy currents, hysteresis
As in an ideal transformer there is no loss of and humming.
power i.e. Pout = Pin so VSiS - VPiP and VP (i) Cu loss (i2R) : When current flows through
eP and VS eS. Hence
the transformer windings some power is wasted
eS NS VS i P in the form of heat (H = i2Rt). To minimize this
k; [k
e P N P VP iS loss windings are made of thick Cu wires (To
= Transformation ratio (or turn ratio)] reduce resistance)
Page 288
ALTERNATING CURRENT
(ii) Eddy current loss : Some electrical (viii) Transformers are also used in impedance
power is wasted in the form of heat due to matching.
Illustration 38: A transformer has 400 primary
eddy currents, induced in core. To minimize
turns and 300 secondary turns. If the
this loss transformers core are laminated and
operating voltage for the load connected
silicon is added to the core material as it to the secondary is measured to be 300V,
increases the resistivity. The material of the what is the voltage supplied to the
core is then called silicon-iron (steel). primary?
(iii) Hysteresis loss : The alternating current
Solution : Here, N P 400, N S 300
flowing through the coils magnetises and
demagnetises the iron core again and again. ES 300, EP ?
Therefore, during each cycle of
magnetisation, some energy is lost due to ES N S
hysteresis. However, the loss of energy can Using E N
P P
be minimised by selecting the material of we get
core, which has a narrow hysteresis loop.
Therefore core of transformer is made of soft N 400 300
EP P ES 400V
iron. Now a days it is made of “Permalloy” NS 300
(Fe-22%, Ni-78%) Illustration 39: A step up transformer operates
(iv) Magnetic flux leakage : Magnetic flux on a 230 V line and a load current of 2
produced in the primary winding is not ampere. The ratio of the primary and
completely linked with secondary because secondary windings is 1 : 25. What is the
few magnetic lines of force complete their current in the primary?
path in air only. To minimize this loss second-
Solution : Using the relation
ary winding is kept inside the primary wind-
ing. NP IS N I
; IP S S
(v) Humming losses : When alternating NS I P NP
current passes, the core of transformer starts
vibrating and produces humming. Thus, some Here N P / N S 1 / 25 (or )
part (may be very small) of the electrical N S / N P 25 / 1 25 and I S 2 A
energy is wasted in the form of humming
sounds produced by the vibrating core of the Current in primary, I P 25 2 50 A
transformer. Illustration 40: A step down transformer
(j) Uses of transformer : A transformer is converts a voltage of 2200 V into 220 V in
used in almost all ac operations e.g. the transmission line. Number of turns in
(i) In voltage regulators for TV, refrigerator, primary coil is 5000. Efficiency of
computer, air conditioner etc. trnasfromer is 90% and its output power
(ii) In the induction furnaces. is 8kW. Calculate (i) number of turns in
(iii) Step down transformer is used for
secondary coil and (ii) input power.
welding purposes.
(iv) In the transmission of ac over long Output power
distance. Solution: Efficiency Input power
Transmission lines
Output power
or Input power
G Low
V
High
V
High
V
Low
V
House or
Load factory
8
Power Step up Step down 8.9kW
Station Transformer Transformer 90 / 100
(v) Step down and step up transformers are
used in electrical power distribution. N S ES ES
(vi) Audio frequency transformers are used in Again N E or N S E N P
P P P
radiography, television, radio, telephone etc.
(vii)Radio frequency transformers are used in Here, N P 5000, ES 220V , EP 2200V
radio communication. N S 500 turns
Page 289
ALTERNATING CURRENT
Illustration 41: A trnasformer having efficiency P 800 1000
90% is working on 100 V and at 2.0 kW I 200 A
power. If the current in the secondary coil E 4000
is 5A, calculate (i) the current in the (i) Line power loss =
2
primary coil and (ii) voltage across the I 2 R 200 2 15 0.5
secondary coil.
Solution: Here 600, 000W 600kW
(ii) Total power delivered by power plant
9
90% , I S 5 A, E P 100V 800 600 1400 kW
10 (iii) Voltage at sending end of line =
(i) EP I P 2kW 2000W Receiving end line voltage + voltage drop
2000 2000 in line
IP or I P 20 A
EP 100 400 IR 4000 200 2 15 0.5
Output power ES I S 4000 3000 7000V
(ii) Input power
EP I P 7000 V
Step up transformer of
or ES I S EP I P 440V is required.
SKIN EFFECT:
9 A direct current flows uniformly throughout the
2000 1800W
10 cross section of the conductor.
1800 1800 An alternating current, on the other hand, flows
ES 360 volt mainly along the surface of the conductor. This
IS 5 effect is known as skin effect.
Illustration 42: A small town with a demand When alternating current flows through a
of 800 kW of electric power at 220 V is conductor, the flux changes in the inner part of
situated 15 km away from an electric plant the conductor are higher.
generating power at 440 V. The resistance Therefore, the inductance of the inner part is
of the two line wires carrying power is 0.5 higher than that of the outer part. Higher the
frequency of alternating current, more is the skin
/ km . The town gets power from the lines effect.
through a 4000-220V step down
The depth upto which ac current flows through
transformer at a substation in the town.
a) Estimate the line power loss in the form a wire is called skin depth.
of heat
VR2
b) How much power must the plant supply, R RVR2
assuming there is negligible power loss due PR
to leakage? ( VR Rated voltage, PR Rated power)
c) Characterize the step up trnasformer at
the plant.
The working principle of metal
Solution: detector
4000/220 Metal detector works on the principle of
440V 220V resonance in ac circuits. When a person walks
R through a metal detector, in fact he walks
Town through a coil of many turns connected to a
capacitor tuned suitably. If he carries some
metal, impedance of the circuit changes, bringing
Power requirement of the town is given by significant changes in the current. This change
EI, where E is the voltage at the receiving in current is detected and the electronic circuit
end of the line and I is the line current causes the alarm.
Here P 800 1000 W , E 4000 V then
Page 290
ALTERNATING CURRENT
IIllustration 43: Solution:
A power transformer is used to step up an To prevent sparking, the capacitor must be
alternating emf of 200 V to 4 KV and to such that, it can hold all the energy contained
transmits 5 KW power. If the primary is by the inductor
of 1000 turns, calculate, assuming the
1
transformer to be ideal. Energy stored in capacitor U C CV 2
2
(i) The number of turns in the secondary (where V is voltage of capacitor)
(ii) The current rating of the secondary
1
Solution: Energy stored in inductor U L LI 2
2
e2 N2
(i) For an ideal transformer 1 1 LI2 2 22
e1 N 1 CV 2 LI2 V = 1414 V
2 2 C 4 106
4000 Ns ADDITIONAL INFORMATION
N s 2 104 turns
200 1000
(I) PARALLEL LCR CIRCUIT
(ii) P = (power to be transferred) = e1i1 In this case
P 5 103 L
i1 = 25 A
e1 200
i2 N1 103 1
then i N 2104 20 C
1 2
R
1 25
i 2 i1 20 20 1.25A or
Alternatively, e2i 2 5103 W
5103
i 2 4103 1.25A
Illustration 44: V = V0 sint
A 1µF condenser is charged to 50V. The LI L ,
charging battery is then disconnected
and a 10 mH coil is connected across d 1 d
E IRR , IRR IR
capacitor so that the LC oscillations dt R dt
occur. Find the maximum current in the dq
coil? q CV ; I C
dt
Solution:
d d 1 d
Maximum charge stored on capacitor I I L IC I R C
L dt dt R dt
q0 = CV = 1 × 10–6 × 50 = 5 × 10–5C
Maximum current in the inductor d 2 1 d 1 V
2
0 sin t
dt RC dt LC ZC
q0 5 105
I0 0.5A (ii)Parallel LC circut ( Rejector Circuit)
LC 10 103 106
A reads iRMS through mains.
Illustration 45:
AC read iC and AL reads iL
An inductor of 2H inductance carries a
current of 2A. To prevent sparking when Erms E
the circuit is broke capacitor of 4 µF is iC , iL rms
XC XL
connected across inductor. Find the
voltage rating of capacitor.
Page 291
ALTERNATING CURRENT
IL i |Z|
AL
IC
AC
I
1
A ωr =
LC -1
ωr =
LC
A.C measurement:
1) All ac meters read r.m.s value
E = E0 sint 2) All ac meters are based on heating effect
Phase relationship is as follows. current
900 , cos 0 ; iRMS iC iL 3) Deflection in hot wire meters i 2rms
D.C measurement
1 1
iRMS ERMS 1) All dc meters read average value
XC XL 2) All dc meters are based on magnetic effect of
ic current
3) Deflection in dc meters i
Let ic > iL
iL
1 1 1
Admittance Z X X
C L
( Admittance
1 1
= ,Suscep tan ce )
Im pedance Re ac tan ce
1
When XC = XL i.e., ( resonance)
LC
Z , iRMS 0
Thus an alternating signal or current of
1
frequency can not flow t hrough
LC
the parallel LC circuit so this circuit is also called
rejector circuit. It is used to filter out some
frequencies.
Page 292
Page 293
DUAL NATURE OF RADIATION & MATTER
Pb 4.25 Pt 5.56
wavelength.
Page 294
DUAL NATURE OF RADIATION & MATTER
Page 295
DUAL NATURE OF RADIATION & MATTER
current
When a graph is plotted with saturation photo
photo
current on y-axis and intensity of incident light
v3v2v1
on x-axis, it is as shown in figure.
v3 v saturation
i 2 v
1 current
photoelectric
Saturation
retarding
potential
potential
I Observations:
O intensity a) Larger the frequency of incident radiation, larger
of light is the stopping potential. So the maximum
Observations: It is observed that saturation kinetic energy of the emitted electrons depends
photocurrent is proportional to the intensity (I) on the frequency of incident light and nature of
of incident light at a given frequency
the metal plate.
EFFECT OF INTENSITY ON STOPPING
b) Maximum kinetic energy of photo electrons is
POTENTIAL AND SATURATION CURRENT
Keeping the frequency of incident light and independent of the intensity of incident light.
nature of the cathode constant, for different The saturation photocurrent is independent of
intensities of incident light photo current is the frequency of incident radiation.
measured. When a graph is plotted with VARIATION OF STOPPING POTENTIAL
photocurrent on y-axis and applied voltage on WITH FREQUENCY
x-axis. It is as shown in figure. When a graph is plotted with stopping potential
Observations: on y-axis and frequency of incident radiation
1) The value of stopping potential is independent on x-axis, keeping the metal constant, then it is
shown in the figure,stopping potential varies
of the intensity of incident light, if frequency is
linearly with frequence of incident light.
constant.
Observations:
2) The magnitude of saturation current depends on
1) Threshold frequency ( ) is a characteristic of
the intensity of light . Higher the intensity, larger 0
the saturation current. the metal plate and at this frequency, kinetic
energy of the photo electron is zero.
2) Above threshold frequency, kinetic energy of
current
3
I2 value.
I1 3) Maximum kinetic energy and stopping potential
stopping increases linearly with increasing frequency as
potential
shown in the figure
-V0 O V
anode
retarding
potential
potential
EFFECT OF FREQUENCY OF INCIDENT
RADIATION ON STOPPING POTENTIAL
Keeping the intensity of incident light and nature
Page 296
DUAL NATURE OF RADIATION & MATTER
hC 12400eVA0 1240eVnm 5
E 0
3.1eV 100 5 J
4000 A 400nm 100
Illustration 2 Number of visible photons emitted per second
While working with light and X-rays, there 5
is a useful relation between the energy of 1.41 1019
3.549 1019
a photon in electron volts (eV) and the
Illustration -5:
wavelength of the photon in angstom (A0).
Monochromatic light of wavelength 632.8nm
Suppose the wavelength of a photon is
is produced by helium-neon laser.The
A0 . Then energy of the photon is power emitted is 9.42mW.
hc (a)Find the energy and momentum of each
Sol. E hv ; Here wavelength = photon in the light beam.
(b)Howmany photons per second on the average
1010 m; h 6.62 1034 Js, c 3108 ms 1
,arrive at a target irradiated by this
E
6.62 10 3 10
34 8
beam?(Assume the beam to have uniform
cross section which is less than the target
1010
area),and
6.62 10 34
3 10 8
eV 12400 eV (c)How fast does a hydrogen atom have to
travel in order to have the same momentum
1010 1.6 1019
as that of photon?
Note : ( is taken in A and E in ev )
0
Sol. Here 632.8nm 632.8 109 m
Illustration -3:
P=9.42mW =9.42 10 3 W
A monochromatic source of light operating
(a)Energy of each photon in the light beam is
at 200 W emits 4 1020 photons per second.
Find the wavelength of the light. hc 6.625 1034 3 108
E 9 3.14 109
N 632.8 10
Sol. Power P E ; 200= 4 1020 E Momentum of each photon in the light beam is
t
h 6.625 1034
hc hc 6.62 1034 3 108 P 1.05 1027 kgms 1
E ; 632.8 10 9
E 5 1019
(b)Number of photons falling per second on the
3.972 107 m 3972 A0 target is
Illustration -4:
P 9.42 103
If 5% of the energy supplied to an incandescent n 19
3 1016 photons / sec
light bulb is radiated as visible light,how E 3.14 10
many visible light photons are emitted by (c)Momentum of hydrogen atom is equal to the
a 100 W bulb? Assume the average momentum of a photon.Therefore ,required
wavelength of all visible photons to be speed(v)of the hydrogen atom is
5600A.Given h=6.625* 10^-34 Js. Momentum of photon
v
Sol. Here 5600 A0 mH
Page 298
DUAL NATURE OF RADIATION & MATTER
Na
Plank’s constant.
In photoelectric effect, an electron absorbs a V0 V0| V0| V0| V0|
quantum of energy ( hv ) of radiation. If this O
frequency of incident (v)
absorbed energy exceeds the minimum energy radiation
(work function W of the metal), the most
loosely bound electron will emerge with According to photoelectric equation
maximum kinetic energy, more tightly bound h W
electron will emerge with kinetic energies less eV0 h W0 V0 v 0
e e
than the maximum value. 1) Slope of stopping potential, frequency of
Einstein’s photoelectric equation, h
1 light graph is
mVmax 2
h W (1) e
2 W
1 2) The y-intercept is 0
h W mVmax 2
(2) e
2 3) The x-intercept is 0
Millikan proved Einstein’s photoelectronic
Page 299
DUAL NATURE OF RADIATION & MATTER
Page 300
DUAL NATURE OF RADIATION & MATTER
maximum wavelength of the light beam so (i) Light meters in photographic cameras make
that some of the photoelectrons may go use of photocells to measure the intensity of
round the ion along a circle. light.
Sol. Electron is moving around the ion in a (ii) Photocells inserted in the street electric circuit,
are used to switch on and off automatically.
1 e2 mV 2 (iii) Photocells help in counting the persons entering
Circle of radius ‘d’. 4 2 ,
0 d d an auditorium.
1 e2 (iv) They are used for detection of traffic law
mV 2
40 d defaulters.
1 e 2 (v) It is used for giving theft warning in banks,
K .E ----------(1) ; But treasuries etc. Hence it is called an electric eye
80 d
also.
hc (vi) Photocells are used in the reproduction of
K .Emax W ----------(2)
sound in cinema and photo telegraphy.
hc 80 d (vii) They are used to control temperature of furnaces
2
e 80 dW and chemical reactions.
PHOTO ELECTRIC CELLS : Illustration-9:
A device which converts light energy into In a photocell bi chromatic light of wave
electrical energy is known as photoelectric cell. length 2480 A0 and 6000A0are incident on
It consists of a semi cylindrical photosensitive a cathode whose work function is 4.8eV. If
a uniform magnetic field of 3x10-5 T exists
metal plate (Cathode) and a wire loop (Anode)
parallel to the plate, find the radius of the
supported in an evacuated glass or quartz bulb. circular path described by the
The cathode acts as an emitter and the anode photoelectron. (mass of electron is
acts as a collector of photo electrons. It is 9 x1031 kg )
connected to an external circuit containing a high- 12400 12400
tension battery (B) and a micrometer ( A) as Sol. E1 1 2480 5 eV
shown in figure. E2
12400 12400
2.06 eV
2 6000
Incident light As E 2 < W0 and E 1 > W0, photo electric
C emission is possible only with 1 .
A Collector
Emitter Maximum K.E of emitted photo electrons
K = E1 – W0 = 0.2 eV.
Photo electrons experience magnetic force and
move along a circular path of radius
m 2 mK
r
- B + A Bq Bq r 5cm
Page 302
DUAL NATURE OF RADIATION & MATTER
(2) Let x and p be the uncertainty in the is 6 108 m , then the minimum
simultaneous measurement of the position and uncertainty in its speed is
momentum of the particle, then
Sol: p mv
h x
x p = ; where = and
2 1.054 1034
h = 6.63 x 10-34 J-s is the or v 1ms 1
mx 1.67 10 27 6 108
h Illustration-16:
Planck’s constant. ( = 1.05x10-34 J-s)
2 The correctness of velocity of an electron
A more rigorous treatment gives x. p moving with velocity 50 ms-1 is 0.005%. The
accuracy with which its position can be
h measured will be
( or ).
2 4
0.005 50
(3) If x = 0, then p and if p 0 then Sol: Here, v 0.0025ms 1
100
x 1.054 1034
i.e., if we are able to measure the exact position of x 31
4634 105 m
the particle (say an electron) then the uncertainty mv 9.110 0.0025
in the measurement of the linear momentum of DAVISSON AND GERMER EXPERIMENT
the particle is infinite. Similarly, if we are able to The wave nature of electrons was first
measure the exact linear momentum of the experimentally verified independently by C.J.
particle i.e., p 0 , then we cannot measure the Davisson and L.H.Germer in 1927 and by
exact position of the particle at that time. G.P.Thomson in 1928 while observing diffraction
effect with beam of electrons scattered by
crystals. Davisson and Thomson shared the
noble prize in 1937 for same.
The experimental arrangement is schematically
shown in figure.
– +
(a) H.T
Page 304
DUAL NATURE OF RADIATION & MATTER
cathode. A is a cylinder with fine hole along its According to de Broglie hypothesis, the wave
axis acting as an anode. N is a nickel crystal length of the wave associated with electron is
cut along cubical diagonal, D is an electron given by
detector which can be rotated on a circular 12.27 12.27 0
scale and is connected to a sensitive = V
=
54 1.66 A
galvanometer which records the current.
Thus there is an excellent agreement between
Working: From electron gun a fine beam of
the theoretical value and experimentally obtained
accelerated electrons is made to fall normally
value of de-Broglie
on the surface of nickel crystal. The atoms of
wavelength.
the crystal scatter the incident electrons in
Thus this experiment confirms the wave nature
different directions. The detector detects the
of electron.
intensity of the electron beam scattered in
particular direction by rotating the electron COMPTON EFFECT:
detector on circular scale at different positions. (1) The scattering of a photon by electron is called
The accelerating voltage varied from 44 V to Compton effect.
68 V and found that at accelerating voltage 54 (2) The energy and momentum is conserved.
v variation of intensity (l) and scattering angle (3) Scattered photon will have less energy (more
wavelength) as compared to incident photon
( ), (The angle between the incident and the
(less wavelength).
scattered beam) is of the type as shown in (4) The energy lost by the photon is taken by
figure. electron as kinetic energy.
I (5) The change in wavelength due to Compton
54V effect is called Compton shift. Compton shift
h
f - i = =
m0 c (1-cos )
o If = 00, = 0
50
Due to constructive interference of electrons h
scattered from different layers of regularly = 900, = m c = 0.24nm = 1800,
0
spaced atoms of the crystal, peak appears in a = 0.48nm (called Compton wave length)
particular direction i.e., diffraction of electrons
takes place. This establishes the wave nature
Compton scattering Recoil
of electron. electron
For the scattering angle = 500, the angle of Target
Incident electron
glancing photon
at rest
0
2 + = 180
1 i Scattered
0 0
or = (180 - ) = 65 photon
2 h
For the nickel crystal the inter atomic separation f – i = = m c(1–cos)
0
is d =0.91 A (6) Compton shift depends only on the scattering
According to Bragg’s law for first order angle. It does not depend on the incident
diffraction maxima (n = 1) wavelength or on the scattering material.
We have, 2d sin = 1 x
0 0
= 2 x 0.91 x sin 65 = 1.65A
Page 305
Page 306
ATOMS
ATOMS
INTRODUCTION (1906), proposed a classical experiment of
scattering of - particles by atoms. This
The structure of matter that shapes the world
experiment was later performed (1911) by Hans
around us has been a subject of study since
Geiger and Ernst Marsden.
long. The first contribution in this regard came
The results of the experiment led to the birth of
from Dalton, who postulated that matter is
Rutherford’s planetary model of an atom (nuclear
made of atoms, which are indivisible. J.J.
model of an atom)
Thomson proposed a structure for the atom,
According to this model, the entire positive
which was modified by Rutherford and later by
charge and most of the mass of the atom is
Niels Bohr. In this unit, we shall discuss these
concentrated in a small volume called the nucleus
models of atoms in some detail. We shall also
with electrons revolving around the nucleus just
study some fundamental information about the
as planets revolve around the sun.
atomic nuclei.
ALPHA – PARTICLE SCATTERING
J.J .Thomson experiment on electric discharge
AND RUTHERFORD’S NUCLEAR
through gases (1897), revealed that “atoms of
MODEL OF ATOM
different elements contain negatively charged
Vaccum
constituents’(electrons) that are identical for all
atoms”
Regarding the structure of atom, the first model
was proposed by J.J.Thomson called as Gold foil
Electron
α-particle
(energy E)
This model explained successfully the A beam of - particles of energy 5.5 Mev,,
phenomenon of t hermionic emission, emitted from 83214
Bi radioactive source was
photoelectric emission and ionization.
direct ed at a thin foil of gold.
This model fails to explain the scattering of -
particles and the origin of spectral lines observed (thickness 2.1107 m )[z=79]
in the spectrum of hydrogen and other atoms.
Johann Jokob Balmer (1885) obtained a simple
empirical formula which gave the wavelengths
of a group of lines emitted by atomic hydrogen
To investigate atomic structure, Ernst Rutherford
Page 307
ATOMS
i. N(1800)
If t’ is the thickness of the foil and N’ is the number
of -particles scattered in a particular direction
The graph shows the total number of - (i.e., θ = constant), it was observed that
particles scattered at different angles, in a given
interval of time. N N t
constant 1 1
The dots represent the data points t N2 t 2
The solid curve is the theoretical prediction. Distance of closest approach(Nuclear
(based on the assumption that the target atom dimension):
has a small, dense , positively charged nucleus)
The minimum distance from the nucleus up to
Many of the - particles pass through the foil
(They do not suffer any collisions) which the - particle approach, is called the
Only about 0.14% of the incident - particles distance of closest approach (r0). At this distance
scattered by more than 10 . the entire initial kinetic energy is converted into
potential energy, so
About 1 in 8000 deflected by more than 900
To deflect the - particle backwards, it must
experience a large repulsive force and this force
could be provided if “ greater part of the mass
of ato m and it s po sitive charge were
concentrated tightly at the centre.”
Page 308
ATOMS
Page 309
ATOMS
1 2Ze2
sin / 2 sin30
2
= mv ' ke ......... 2
2 s 1
N2 7 12.5
and o 4
(as KE of the particle after scattering + PE sin 60
of the particle and the nucleus at a distance
s equals initial kinetic energy of the particle ) RUTHERFORD’S ATOMIC MODEL
From equation (1) and (2) After -particles scattering experiment,
1 2 1 v 2b 2 2 Ze 2 vb following conclusions were made by Rutherford
mv m 2 ke ; asv' about atomic structure:
2 2 s s s
Page 310
ATOMS
Atom nucleus.
-15
10 m
Nucleus
e-
10-10m
Size of the nucleus =1 fermi =10-15 m
Size of the atom=1A0 =10-10 m
1) Most of the mass (at least 99.95%) and all of
Instability of atom
the positive charge of an atom is concentrated in (iv) According to this model the spectrum of atom
a very small region called atomic nucleus.
2) Nucleus is positively charged and it’s size is of must be continuous where as practically it is a
line spectrum.
the order of 1015 m 1 fermi .
(v) It did not explain the distribution of electrons
3) In an atom there is maximum empty space and
the electrons revolve around the nucleus in the outside the nucleus.
same way as the planets revolve around the sun. BOHR MODEL OF HYDROGEN ATOM
ELECTRON ORBITS The atom consists of central nucleus, containing
Rutherford’s Nuclear model of atom, involves the entire positive charge and almost all mass of
classical concepts. the atom. The electrons revolve around the
An atom is an electrically neutral sphere
consisting of a very small, massive and positively nucleus in certain discrete circular orbits. The
charged nucleus at the centre surrounded by necessary centripetal force for circular orbit is
revolving electrons in their respective provided by Coulomb’s attraction between the
dynamically stable orbits electron and nucleus. So,
The electro stat ic force of attraction
Fe between the revolving electrons and the mv 2 1 Ze e
nucleus provides the requisite centripetal r 40 r 2
force Fc to keep them in their orbits. where, m = mass of electron,
For a dynamically stable orbit in a hydrogen atom r = radius of circular orbit,v = speed of electron
the co ndit ion to be satisfied is in circular orbit,Ze = charge on nucleus,Z =
2 2
mv 1 e atomic number,e = charge on electron = –1.6 ×
r 4 0 r 2 10–19 C
The relation between orbit radius and electron STATIONARY ORBITS
e2 The allowed orbits for electrons are those in
velocity is r which the electron does not radiate energy. These
4 0 mv 2
orbits are also called stationary orbits.
FAILURE OF RUTHERFORD’S MODEL
The stationary orbits are those in which angular
Rutherford’s model failed to explain
(i) why atoms emit light of only discrete momentum of electron is an integral multiple of
wavelengths. h h
(ii) How could an atom like hydrogen, emit a ; L= mvr n , where ‘n’ being integer or
2 2
complex spectrum of specific Wavelengths?
(iii) Stability of atom: It could not explain stability the principle quantum number.
of atom because according to classical This postulate account for the stability of the
electrodynamics theory, an accelerated charged atom. This equation is known as the quantum
particle should continuously radiate energy. Thus condition and it limits the number of permessible
an electron moving in a circular path around the orbits.
nucleus should also radiate energy. As a result The nucleus is so heavy, that its motion may be
the electron should move into orbits of gradually neglected.
decreasing radius and should ultimately fall into
Page 311
ATOMS
The mass of the electron in motion is assumed to Velocity of an electron in the first orbit of
be constant. hydrogen atom V0 2.2 10 6 ms 1
BOHR’S TRANSITION RULE
z
When an electron jumps from one stationary v n 2.2 10 6 m /s
orbit to another, a photon is emitted or absorbed n
having energy equal to the difference of energies e2 cZ Z
between initial and final states and being given
v
n v n
2h0c n n
Ei E f
by Ei E f hv v e2 1
h where is the ‘fine structure
2h 0 c 137
Radius of orbit : For an electron around a stationary
constant’
nucleus the electrostatic force of attraction
provides the necessary centripetal force 1 cZ
vn 137 n i.e. velocity of electron in
L
c
r Bohr’s First Orbit is ,
137
Velocity of revolving electron is independent of
m, -e U mass of electron.
Force on the electron in an orbit:
1 Ze e mv2
i.e., .........(i)
40 r 2 r mv 2 m z 2e 4 me 2 z m 2 e 6 z 3
F 2 2 2 2 2 4 4 4 and
nh r 4h 0 n n h 0 4h 0 n
also mvr ............(ii)
2 z3
nh F
From (ii), Vn ; Putting in (i), we get n4
2mr Acceleration of the electron in an orbit:
n 2 h2 0 n2 n2 F me z
6 3 3
rn
me 2 Z
r0 r
n (0.53) Å 4 4 4 a z4
a
Z Z m 4h 0 n n
r0 0.53 A radius of first orbit of hydrogen Angular velocity, Time period and frequency of
0
1 mZ 2 e 4 2 Z2
f 3 3 2 ; f z3 En 2 where n 1, 2,3,.....
T 4n h 0 n n
Angular momentum of an electron in an For n 1, E n 13.6 eV is the total energy of
orbit. the electron in the ground state of a hydrogen
nh atom
L mv n rn ; L n (Independent of Z)
2 1
From the above KE PE and TE KE
Equivalent electric current by an electron 2
revolving in an orbit: PE : KE : TE 2 :1: 1
2 4
q emz e 2
me 4 Z 2 Z2
in en 3 3 2 ; i n z 3 Also, E ch E Rch
t 4n h 0 3
n 2
8 0 ch n
2
n2
Magnetic moment caused by an electron Rydberg’s constant
revolving in an orbit:
me 4
R 1.097 107 m 1
neh 8 02 ch 3
M n i n A n i n r 2
4m and Ry = Rydberg’s Energy
M n 0 n ; M n n if n=1 then Bohr magneton 2.17 10 18 J 13.6 eV 1Rydberg is the
eh electron energy in first orbit of H atom.
Mn 0 Ionisation energy and potential:
4m
The energy required to ionise an atom is called
Magnetic field caused by an electron ionisatio energy. It is the energy required to make
revolving in an orbit: the electron jump from the present orbit to the
infinite orbit.
0 i n n 2 z 3e 7 0 z3
Bn ; B Z2
2rn 803 n 5h 5 n
n5 0
Hence Eionisation E En 13.6
n2
Kinetic Energy of Electron (EK)
13.6 Z 2
mv 2
1 Zee
n2
eV
Since, we have r 4
0 r2 For H-atom in the ground state
2
1 2 Ze 2 1 2 Ze 2 13.61
2 mv E mv Eionisation 13.6eV n 1
8 0 r K 2 8 0 r n2
Potential Energy (U) of Electron: The potential through which an electron needs
to be accelerated so that it acquires energy
1 Ze e Ze2 equal to the ionisation energy is called
U U
40 r 4 0 r Eionisation
ionisation potential. Vionisation
Total Energy (E) of electron: e
Total Energy = K.E. + P.E. Excitation energy and potential:
When energy is given to an electron from
Ze 2 Ze 2 Ze2 external source, it jumps to higher energy level.
E 8 r 4 r E 8 r
0 0 0 This phenomenon is called excitation.
The minimum energy required to excite an
n 2 h 2 0 atom is called excitation energy of the
Further, since r particular excited state and corresponding
me 2 Z
potential is called exciting potential.
me 4 Z 2 Z2 Eexcitation
E E 13.6 eV EExcitation E Final E Initial and VExcitation
8h 2 2 n 2
0 n2 e
Page 313
ATOMS
1 1
Ei – E f 13.6 Z2 2 2 Ef
nf ni
f
410.2 nm
434.1 nm
486.1 nm
458.3 nm
1 1 1
R 2 2 , n= 6,7,8...
5 n
The limit of Pfund series is
2279 nm for n
The elements whose spectra can be represented
H H H H H by the above equations are hydrogen, singly
H α 656.3nm Red region ionised helium, doubly ionised lithium.
The above relations are empirical and do not
H 486.1nm blue green
give any reasoning why only certain frequencies
H 434.1nm Violet are observed in the hydrogen spectrum.
As the wavelength decreases, the lines appear Lym
an Bal
m
closer together and are weaker in Intensity Serie
s ser er
ies
The results obtained by Balmer were
expressed by J.R Rydberg in the following
1 1 1
Pasc
R 2 2 serie
mathematical form
2 n hen
s
2
Br
5
ck
6
1 1 n=7
v Rc 2 2
se un
2 n
ri d
Pf
es
-0.85
states
n=3 Brackett n
-1.5 Paschen series
n=2 series 2 rn nh / m n or m n rn nh / 2
-3.40
Balmer 5. Energy states are due to the wave nature of the
series electron and only resonant standing waves can
persist
n=1
Ground state Nucleus
-13.6
Binding energy of ground state of H atom = 13.6 r
eV.
DRAWBACKS OF BOHR’S ATOMIC
MODEL
1) It is valid only for one electron atoms,
e.g.: H, He , Li2 , Na 10 etc.
Illustration 3:
2) Orbits were taken as circular but according to
Sommerfied these are elliptical. The electron in a hydrogen atom makes a
3) Intensity of spectral lines could not be explained. transition n1 n2 where n1 & n2 are the
4) Nucleus was taken as stationary but it also rotates principal quantum numbers of the two
states. Assume the Bohr model to be valid.
Page 316
ATOMS
Page 317
ATOMS
Li++ from the first orbit to the third orbit is 1 3646 10 10
given by or (Z 1)2 911.5
R 4 1 10 10
hc hc
E 3 E1 or E E (Z 1) 911.5 or Z 30.2 31
30.2
3 1
Thus the atomic number of the element
12375
Å or 114Å concerned is 31.
108.8eV The element having atomic number Z = 31 is
(b) The spectral lines emitted are due to the transition Gallium.
n = 3 n = 2, n = 3 n = 1 and n = 2
n = 1, thus there will be three spectral lines. Illustration 11:An energy of 24.6eV is required
Illustration 9: to remove one of the electrons from a
First excitation potential of a hypothetical
hydrogen like atom is 15 volt. Find third neutral helium atom. The energy (in eV)
excitation potential of the atom. required to remove both the electrons from
Solution:
Let energy of ground state = E0 a neutral helium atom is
E0 Solution.After the removal of first electron
E0 = – 13.6 Z2 eV and En
n2
E0 E0 3E 0
remaining atom toll be hydrogen like atom.
n = 2, E 2 given E 0 15 15
4 4 4
So energy required to remove second electron
E0 20eV
E 22
for n = 4, E4 0 from the atom E 13.6 54.4 79eV
16 1
Third excitation energy =
Total energy required 24.6 + 54.4 = 79 eV
E 15E 0 15 20 75
E0 0 eV Illustration 12:A double charged lithium atom
16 16 16 4
is equivalent to hydrogen whose atomic
75
Third excitation potential is eV number is 3. The wavelength of required
4
radiation for emitting electron from first
Illustration 10: to third Bohr orbit in L++ will be
If the short series limit of the Balmer series (Ionisation energy of hydrogen atom is
for hydrogen is 3646 Å, calculate the atomic
13.6eV)
no. of the element which gives X-ray
wavelength down to 1.0 A. Identify the Z2
Solution. En 13.6 2 eV , Required energy for
element. n
Solution : said transition,
The short limit of the Balmer series is given by 1 1
E E 3 E1 13.6Z2 2 2
1 1 1 1 3
R 2 2 R /4
2 n 8
E 13.6 32 108.8eV
10 1
R 4 / (4 / 3646) 10 m 9
Further the wavelengths of the K series are E 108.8 1.6 10 19 J
given by the relation hc
Now E 108.8 1.6 1019 J
1 1 1
R(Z 1) 2 2 2
1 n
Illustration 13:Imagine an atom made up of a
The maximum wave number correspondence to proton and a hypothetical particle of double
n and, t herefore, we must have the mass of the electron but having the same
1 charge as the electron. Apply the Bohr’s
R(Z 1)2
atom model and consider all possible
Page 318
ATOMS
Page 319
ATOMS
1 Z2
1 Therefore 54.4 13.6 Z2
4n 2 5 n 2 11
3
4n 2 2
0.53 1
Illustration 18: rn 0.265 A
Consider a hydrogen like atom whose energy 2
in n th excited state is given by Illustration 20:
13.6Z2 . How much work must be done to pull apart
En when this excited makes a the electron and the proton that make up
n2
transition from excited state to ground state the Hydrogen atom, if the atom is initially
m energetic photons have energy E max = in the state with n = 2?
52.224 eV and energetic photons have Solution.The electrostatic RE is zero when the
energy Emin = 1.224 ev. The atom number of electron and proton are far apart from each other.
atom is Work done in pulling electron and proton far
Solution.Maximum energy is liberated for away from each other
transition E n 1 and maximum energy for 13.6
W E f E i 0 E i 2 eV
n
E n E n 1 …..(i)
13.6
E1 W 2
1.6 1019 J 3.4 1.6 1019 J
Hence 2 E1 52.224eV and 2
n
Illustration 21: Consider a hypothetical
E1 E1
1.224eV ….. (ii) annihilation of a stationary electron with
n 2 n 1 2
a stationary positron. What is the
Solving equation (i) and (ii) we get wavelength of resulting radiation?
E1 54.4eV and n = 5 Solution.From conservation of momentum, two
identical photons must travel in opposite
13.6Z2
Now E1 54.4eV . Hence Z = 2 directions with equal magnitude of momentum
12
Illustration 19:The energy level diagram for an hc
and energy
hydrogen like atom is in the figure. The
radius of its first Bohr orbit is From conservation of energy:
0 eV hc hc h
n=4 m0c 2 m0c 2
m 0c
-6.04eV n=3
(h = Planck’s constant. c = speed of light, m0=
-13.6 eV n=2 rest mass)
Illustration 22:
-54.4eV n=1 . A hydrogen atom emits a photon corresponding
Z2 to an electron transition from n = 5 to n =1. The
Solution.We know that En 13.6 2 eV and recoil speed of hydrogen atom is almost (mass
n
2
of proton 1.6 x 10-27 kg).
n o
rn 0.53 A Solution.The Hydrogen atom before the transition
Z was at rest. Therefore from conservation of
Here for n 1, E1 54.4eV momentum
Page 320
ATOMS
C
1 1 1
13.6 2 2 eV
mv p photon
E radiated
n1 n 2 B
c c 2 3
1 1
13.6 2 2 1.6 1019 A
1.6 1027 v 1 5 Solution.Let the energy in A, B and C state be EA.
3 108 EB and EC then from the figure
v 4.352m / s 4m / s C
Illustration 23: 1
. The following diagram indicates the energy B
levels of a certain atom when the system 2 3
moves from 4E level to E. A photon of
A
wavelength 1 , is emitted. The wavelength
of photon produced during it’s transition EC EB EB EA EC EA or
7 1 hc hc hc
from E level to E is 2 . The ratio will 3 1 2
3 2 1 2 3 1 2
be Illustration 25:
4E . In hydrogen atom, if the deference in the
7
E
energy of the electron in n = 2 and n = 3
3 orbits is E, the ionization energy of
hydrogen atom is
E 1 1
Solution.Transition from 4E to E Solution.Energy, E=K E K n 2 n 2
hc hc 1 2
4E E 1 ...... i
1 3E (K = constant) n1 2 and n 2 3 . So E = K
7
Transition from E to E 1 1 5
3 22 32 K 36
7 hc 3hc
E E 1 ……. (ii) For removing an electron n1 = 1 to n2 =
3 l 3E
36
l 4 Energy E1 K 1 E 7.2E
From equation (i) and (ii) : 9 5
2 Illustration 26:
Illustration 24: When the hydrogen atom emits a photon in
. Energy levels A, B, C of a certain atom going from n = 5 to n = 1 state, its recoil speed
corresponding to increasing values of is nearly
energy i.e. EA < EB < EC. If l , 2 , 3 are the Solution: Photon energy
wavelengths of radiations corresponding to
1
the transitions C to B. B to A and C to A, hf 13.6 1 eV 13eV
25
respectively, which of the following
Photon momentum = momentum of hydrogen atom
statements is correct
hf 13 1.6 1019
p 27 8
4ms 1
mc 1.67 10 3 10
Illustration 27:In Fig., E, to E6 represent some
Page 321
ATOMS
Illustration 29: Only a, satisfies the above, hence this is right choice
. The ratio of the speed of the electron in the Illustration 33:The wavelength of radiation
first Bohr orbit of hydrogen and the speed required to excite the electron from the first
of light is equal to (where e, h, and c have orbit to the third orbit in a doubly ionized
their usual meanings in cgs system) lithium atom will be
Solution. Required energy
Page 322
ATOMS
12375 o E1 1
Wavelength 113.7 A
8 13.6 i.e., E 0 1 m
Illustration 34: mH
. An electron with kinetic energy E eV Illustration 36:
collides with . a hydrogen atom in the . A hydrogen atom is in an excited state of
ground state. The collision is observed to principal quantum number n. It emits a
be elastic for photon of wavelength when it returns to
Solution.For an elastic collision to take place, there the ground state. The value of n is
must be no loss in the energy of electron. The
hc R
hydrogen atom will absorb energy from the Solution. Rhc 1 1/ n 2 or n
colliding electron only if it can go from ground R 1
state to first excited state, i.e., from
n = 1 to n = 2 state. For this, hydrogen atom
Illustration 37: A neutron moving with a speed
must absorb energy
v makes a head on collision with a hydrogen
E2 – El = -3.4 - (-13.6) = 10.2 eV
atom in ground State kept it rest. The
So, if the electron possesses energy less than
minimum kinetic energy of the neutron for
10.2 eV, it would never lose it and hence collision
with inelastic collision will take place is
would be elastic.
(assume that mass of proton is nearly equal
Illustration 35:The minimum kinetic energy
to the mass of neutron)
required for ionization of a hydrogen atom
Solution. Let v = speed of neutron before collision,
is E1 in case electron is collided as hydrogen
v1= speed of neutron after collision,
atom. It is E2 if the hydrogen ion is collided
v2 = speed of proton or hydrogen atom after
and E3 when helium ion is collided. Then
collision, and E = energy of excitation
Solution.Assuming that ionization occurs as a result
From conservation of linear momentum, Inv = mv1
of a completely inelastic collision, we can write
+ mv2
mv 0 m mH u From conservation of energy,
where m is the mass of incident particle, mH the 1 1 1
mass of hydrogen atom, vo the initial velocity of mv 2 mv12 mv 22 E
2 2 2
incident particle, and it the final common velocity From eq (i)
of the particle after collision. Prior to collision,
v 2 v12 v 22 2v1v 2
the ICE of the incident particle was
From eq (ii)
mv 20
E0 2 E
2 v 2 v12 v 22
m
The total kinetic energy after collision
2 E
2 2v1v 2
E
m mH u
2
mv 2
0 m
2 2 m mH
Page 323
ATOMS
6
more than 99% heat). 30KV
4
v) Intensity of x-rays depends on the number of
2 20KV
electrons striking the target which inturn depends
on filament current. o -1 -2 -3 -4 -5 -6 -7 -8 -9 -10
vi) Quality of x - rays (hard /soft) depends on P.D Wave length in A
0
CONTROL ON X-RAY eV
is max
There are two types of control on X-ray h
i) Intensity control f) In this spectrum intensity first increases, reaches
The intensity of X-ray depend on number of a maximum value I and then decreases.
max
electrons striking the target and number of g) Every spectrum starts with certian minimum
electron depend on temperature of filament wavelength called limiting wavelength or cut
which can be controlled by filament current. off wavelength min .
Thus intensity of X-ray depend on current
flowing through filament. h) With the increase in target potential, min and
(ii) Penetrating Power control wavelength corresponding to maximum intensity
The Penetrating power of X-ray depend on
Page 324
ATOMS
8
6
4
2
min I
o -1 -2 -3 -4 -5 -6 -7 -8 -9 -10
0
min 0 k k
Wave length in A
As target potential V is increased
a) Produced due to transition of electrons from a) 0 m in decreases
higher energy level to lower energy level in
b) Wavelength of k remains constant.
target atoms.
b) Wavelengths of these x-rays depend only on c) diffrence between and increases
k min
atomic number of the target element and
d) diffrence between k line and k line remains
independent of target potential.
c) Characteristic x-rays of an element consists of constant.
K, L,M and N series. e) Difference between k 0 increases.
d) K-series of lines are obtained when transition MOSLEY’S LAW
takes place from higher levels to k shell i) “The square root of frequency ( v ) of the spectral
line of the characteristic x-rays spectrum is
K L
L
directly proportional to the atomic number(z) of
K M M the target element.
Z or =a(Z-b)
+
K aZ ab y mX c
L
M For 0, aZ ab Z b
N
O
e) This spectrum is useful in identifying the elements
by which they are produced. v
Page 325
ATOMS
Page 326
ATOMS
Page 327
ATOMS
12.27 0
e A
Va
150 150
2
2
15000 volt
e 0.1
Minimum wavelength of X-rays
Page 328
Page 329
NUCLEI
NUCLEI
INTRODUCTION COMPOSITION OF NUCLEUS:
We have learnt that in every atom the positive Nuclei are composed of protons and neutrons.
charge and almost the entire mass are densely
The number of protons inside the nucleus is called
concentrated at the centre of the atom forming
its nucleus. the atomic number (Z). The total number of
The overall dimension of a nucleus is much nucleons (sum of protons and neutrons) is called
smaller than that of an atom. Experiments on the mass number (A). The number of neutrons
scattering of - particles demonstrated that in a nucleus is given by A-Z. The stability of a
the radius of nucleus is smaller than the radius nucleus depends on the relative number of
of an atom by a factor of about 104
protons and neutrons.
This means the volume of a nucleus is about
Nucleus is represented by the symbol ZxA. This
10 12 times the volume of the atom. In other symbol clearly explains the composition of the
words, an atom is almost empty.
nucleus.
The nucleus contains most (more than 99.9%)
of the mass of an atom. PROTON:
ATOMIC MASSES AND COMPOSITION The nucleus of hydrogen atom is called proton.
OF NUCLEUS It is an essential constituent of all nuclei.
ATOMIC MASS UNIT: It carries positive charge = 1.6 10-19 C and
mass = 1836 times the mass of an electron
The masses of atoms, nuclei, sub atomic
= 1.0072766 u
particles are very small. Hence, a small unit is Due to its positive charge, proton is not used as
used to express these masses. This unit is called projectile in nuclear fission.
atomic mass unit (amu). 1 amu is equal to one It is stable even outside the nucleus.
twelth part of the mass of carbon (6 C12 )
NEUTRON:
isotope.
Mass of 6C12 is exactly 12 amu Neutron was discovered in 1932 by James
Now, the mass of 1 gm -mole of carbon is 12 Chadwick.
It has no charge, hence it is a neutral particle.
gm and according to Avogadro’s Hypothesis it
Its mass = 1.6749 10-27 kg i.e.nearly equal to
has N A (Avogadro’s Number) atoms. Thus, that of proton.
mass of neuton = 1839 times the mass of an
the mass of one atom of carbon is (12/ N A )
electron = 1.008664 u
gm. According to the definition. Illustration1: How many electrons, protons and
1 neutrons are three in 12g of 6 C 12 and in
1amu 1u mass of one carbon atom
12 14g of C14 ? (Take Avogadro number
6
1 12 1 1 N 6 1033 )
gm gm
12 N N 6.0231023 Solution: The number of atoms in 12g of 6 C12
1.6603021024 gm Avogadro number 6 1023
The number of elect rons in 12g of
1.6603021027 Kg C12 6 6 1023 36 1023
6
The number of pro tons in 12g of
Page 330
NUCLEI
Page 331
NUCLEI
Atomic nuclei having same number of neutrons measured in fermi (or) femtometer. 1
are called isotones. fermi=10-15m
The actual size of nucleus has to be less than
Ex.: 1) 17 Cl37,19 K39 , 2) 7 N17 ,8 O18 ,9 F19 4.0 x 10-14 m.
For isotones both atomic number (Z) and mass Radius of the nucleus is in the order of 10-15 m.
Size of an atom is in the order of 10-10m.
number (A) are different but the value of Note-1: If R, S and V be the radius, surface
difference (A-Z) is same. area and volume of a nucleus with mass number
ISOBARS: A then
Atomic nuclei having same mass number but 1
1
different atomic numbers are called Isobars. A 3
1 1 ;
R
1) R A3
They have same number of nucleons. R 2 A 2
Ex.:1) 18Ar40,20 Ca40 , 2) 3 2 G e 7 6 , 3 4 S e 7 6 2
2
ISOMERS: A 3
1 1
2 S
2) S R A3
Atomic nuclei having same mass number and S 2 A 2
same atomic number but different nuclear
properties are called isomers. 3 V1 A1
3) V R A V A
Ex:- m 35 Br 80 metastable Bromine and 2 2
Note-2: If a stationary nucleus splits into two
g35 Br 80 ground state Bromine are two lighter nuclei with mass numbers A1 and A2 then
isomers with different half lives according to law of conservation of linear
ISODIAPHERS: momentum, the two lighter nuclei move in
Nuclei having different Atomic number (Z) and opposite directions with equal momenta hence
mass number (A) but with same excess number
m1v1 = m2v2
of neutrons over protons (A-2Z) are called
Ratio of velocities of the two nuclei
isodiaphers. Ex:- 11 Na 23 , 13 Al 27
v1 m 2 A 2 R 2
3
NUCLEAR CHARGE
Nucleus is positively charged due to the v2
m1
A1 R 1
m A R 3
presence of protons in it. Ratio of kinetic energy of the two nuclei
Charge of nucleus = Ze 2
Where Z=number of proton and e=charge of KE p & KE 1 when pis constant
2m m
each proton = 1.6 1019 C
NUCLEAR SIZE
KE1 m 2 A 2 R 2
3
A1 R 1
The scattering experiments conducting by
KE 2 m1
Rutherford and his co-workers gave the first
DENSITY OF THE NUCLEUS
evidence of the nuclear size.
Density stands for mass per unit volume.
The scattering experiment of Rutherford has
Consider nucleus of mass M and radius R,
shown that the estimated volume (V) of the assuming that it is almost spherical distribution.
nucleus is proportional to mass number (A). If Taking mass of each nucleon = 1 amu.
R is t he radius o f nucleus then
M=mass of A nucleons = A 1.66 1027 kg
4
V A R3 A
3 M A 1.66 1027
1
Density V 4 3
1
R A R A ; R R0 A3
3 3
3
1.2 1015 A
( Ro = 1.2 x 10-15 m)
Nuclear sizes are very small and are 2.3 1017 kg / m3
Density of nucleus is independent of mass
Page 332
NUCLEI
Page 333
NUCLEI
AVERGE BINDING ENERGY OR BINDING FRACTION: Illustration-5 :
It is the Binding energy per nucleon (or) Let m p be the mass of a proton, mn the mass
B.E. per nucleon = Binding fraction of a neutron, M 1 the mass of a 20
10 Ne
m x 931 Mev 40
Binding Energy nucleus and M 2 the mass of a 20 Ca
= = A nucleus. Then
Mass Number
Binding energy is not a measure of stability of i) M 2 2 M 1 ii) M 2 2 M 1
a nucleus. iii) M 2 2 M 1
Binding fraction gives the stability of nucleus
If binding fraction is more, then nucleus is stable iv) M 2 10(mn m p )
If binding fraction is less, then nucleus is less A) i, iv B) ii, iv
stable C) ii, iii D) iii, iv
PACKING FRACTION OF A NUCLEUS Solution: M 1 10( m p mn ) M 2 20(m p mn )
It is defined as the mass defect per nucleon.
m M A Heavier the nucleus, more is mass defect
Packing fraction =
A A [20(m p mn ) M 2 ] [10(m p mn ) M 1 ]
Packing fraction measures the stability of a
nucleus. Smaller the value of packing fraction, M 2 [ M 1 10( m p mn )] ; M 2 2 M 1
large is the stability of the nucleus. Illustration-6 :
The packing fraction maybe negative, zero or The binding energies per nucleon for
positive. deuterium and helium are 1.1 MeV and 7.0
If the packing fraction is negative then the
MeV respectively. What energy in joules
nucleus is more stable.
If the packing fraction is positive then the will be liberated when 2 deuterons take part
nucleus is unstable. in the reaction.
Packing fraction is zero for 6 C 12 Solution: 12 H 12 H 24 He Q
Transition elements or iron have lowest value Binding energy per nucleon of helium ( 24 He )
of packing fraction which shows maximum
=7 MeV
stability.
Binding energy = 4 x7 = 28 MeV
Illustration-4 :
Binding energy per nucleon of deuterium
Find the binding energy of 2656 Fe . Atomic
( 12 H )=1.1MeV
mass of Fe is 55.9349u and that of
Binding energy = 2 x 1.1 = 2.2 MeV
Hydrogen is 1.00783u and mass of neutron
Energy liberated (Q) = (28 -(2.2)2] = 23.6
is 1.00876u
Mev. i.e.,
Solution: Mass of the hydrogen atom mH =
1.00783u; Mass of neutron mn = 1.00867 u; Q=23.6x106x1.6x10-19; Q 37.76 1013 J
Atomic number of iron Z = 26; mass number Illustration-7: Calculate the mass defect,
of iron A = 56; Mass of iron atom Ma = binding energy and binding energy per
nucleon of an alpha particle? (An
55.9349u
particle is nothing but helium nucleus.
Mass defect m = [ZmH+(A-Z)mn] - Ma
Hence its symbol is 2 He 4 . It contains 2
= [26 x 1.00783+(56-26)×1.00867]u-
55.93493u protons, 2 neutrons with a mass number 4.
= 0.5287 u. Mass of hydrogen atom mH 1.007825u; ,
Page 334
NUCLEI
ma 4.00260u )
Solution: Mass defect,
m ZmH A Z mn ma
21.007825 4 21.008665 4.00260 u
Page 335
NUCLEI
nucleon stands maximum at 8.75 MeV is most 1. Conservation of mass number A.
stable and will undergo neither fission nor fusion. 2. Conservation of charge.
EXO-ERGIC REACTION : 3. Conservation of energy, linear momentum and
The reaction in which energy will be released angular momentum.
is called exo-ergic Reaction. Illustration-10: Find the Q value of the reac-
A B C D Q tion H 1 Li 7 He 4 . . Determine whether
Here A and B are called Reactants the reaction is exothermic or endothermic?
C and D are called Products The atomic mass of H 1 , He 4 and Li 7 are
Q is the amount of energy released
1.007825u, 4.002603u and 7.016004u re-
In an Exo - ergic Reaction
spectively.
Mass of reactants > Mass of products
Solution: The total mass of the initial particles
m MR MP mi = 1.007825 + 7.016004 = 8.023829 u and
M A M B M C M D the total mass of the final particles.
mf = 2x4.002603 = 8.005206 u
Energy Released Difference between initial and final mass of par-
Q m C2 joule [ m is in kg] ticles m mi m f 8.023829 8.00506
m931.5 MeV ( m is in amu) 0.018623u
If Binding energies are given then for Exo-ergic This mass is converted into energy and the re-
reactions. action is exothermic. The Q value is positive and
(B.E) Products > (B.E) Reactants . given by
Energy released Q = (B.E)P – (B.E)R Q m c 2 0.018623 931.5 17.35MeV
= [(B.E)C + (B.E)D] – [(B.E)A + (B.E)B] NUCLEAR FORCE:
ENDO-ERGIC REACTION : The attractive force which holds the nucleons
The reaction in which energy will be absorbed together in the nucleus is called nuclear force.
is called Endo-ergic Reaction. Properties of nuclear forces :
A B C D Q 1) Nuclear forces are strongest among all the
Here A and B are called Reactants forces in nature. Nuclear forces are about 1038
C and D are called Products times as strong as gravitational forces. The
Q is the amount of energy absorbed relative strengths of the gravitational, Coulomb's
In an Endo - ergic Reaction and nuclear forces are
mass of reactants < Mass of products Fg : Fe : Fn 1:1036 :1038
m M P MR MC MD MA MB 2) Nuclear forces are short range forces .
Nuclear forces between two nucleons becomes
Energy absorbed Q m C2 joule [ m is
negligible when the distance between them is >2
in kg] fermi (or ) 2 1015 m . However, the range of
m931.5 MeV ( m is in amu) nuclear force is taken as 1.5 fermi, where the
If Binding energies are given then for magnitude of nuclear force becomes maximum
Endo-ergic reaction. value.
3) Nuclear forces are basically strong
(B.E) Products < (B.E) Reactants
attractive forces, but contain a small
Energy absorbed Q = (B.E)R – (B.E)P
component of repulsive forces.
= [(B.E)A + (B.E)B] – [(B.E)C + (B.E)D]
The nuclear force is an attractive force, when
Note: A nuclear reaction can occur only if
certain conservation laws are followed. These two nucleons are at a distance of about
are : 1014 m (size of nucleus). As the distance is
Page 336
NUCLEI
decreased, the attractive force increases rapidly because coulombic repulsive force is also
and becomes maximum at a distance of about present between two protons.
6) Nuclear forces are spin-dependent.
1.5 1015 m . The force between two nucleons having parallel
4) Nuclear forces are saturated forces.
spins is greater the force between two nucleons
A nucleon inside the nucleus doesn’t experience
having anti parallel spins and hence nuclear
a nuclear force due to all the other nucleons. It
forces are spin dependant and these forces are
experiences force only due to its nearest
called tensor forces.
neighboring nucleon.
7) Nuclear forces are exchange forces.
A rough plot of the potential energy between
two nucleons as a function of distance is shown The nuclear force between two nucleons is the
in the figure. The potential energy is minimum at result o f the exchange of meso ns
a distance r0 of about 0.8 fm. This means that 0 , & between them. mesons alo
the force is attractive for distances larger than called pions.
0.8 fm and repulsive if they are separated by
0 0
distance less than 0.8 fm.
P
n, p
p , n
n
8) Nuclear forces are non-central forces.
The force between two nucleons doesn’t solely
depend on the distance between them and
Potential energy (MeV)
Page 337
NUCLEI
Page 338
NUCLEI
number (Z) increases by 1 unit, but the mass one of its own orbital atomic electron and emits
number does not change. a positron with neutrino.
TYPES OF -DECAY: Hence, Z X A 1e0 Z 1Y A 1e 0 Q
It is of three types. Ex: 4 Be7 1e0 3 Li 7 1e 0 Q
1) Negative -deacay This is similar to decay..
Page 339
NUCLEI
fields through small angles. Deflection of Radioactive radiations in
3) They can penetrate through matter. They electric and magnetic fields :
th
1 B
penetrating power is of rays and E
100
th
1
of rays.
10000
4) particles from a source are emitted with Ra
Ra
different velocities. Lead block
5) The velocity of particles ranges between
1.5 107 to 2.25 107 m/s. x B x x x
6) The ionising power of rays is 100 times
that due to rays and 10,000 times that
due to rays.
x x x x
7) They effect photographic plate.
RAYS x x x
1) They are fast moving electrons coming from x
the nucleus.
2) They in deflected and magnetic fields direction
opposite to that of rays but in large
angles. Lead block
3) They can ionize the gas through which they
1 Illustration-12:
pass. Their ionizing power is of rays
100 A radioactive element 90 X 238 decays into
1 Y 222 . The number of particles emitted
but of rays. 83
100
are
4) The velocity of - particles emitted from a Solution: Number of particles emitted
source is not constant.
238 222
5) The velocity of - particles ranges from 4 . This decreases atomic
4
0.36 to 0.98 times of velocity of light
number to 90 4 2 82 . Since atomic
6) The penetrating power of rays are about
100 times more penetrating than rays and number of 83 Y 222 is 83, this is possible if
100 times lessthan rays.
one particle is emitted.
7) They effect photographic plate.
RAYS Illustration-13:
1) rays are electromagnetic radiations of very A nucleus with Z=92 emits the following
short wavelength of the order of 1012 m . in a sequence:
2) They are not deflected by electric and mag- , , , , , , , , , , , , , . .
netic fields. The Z of the resulting nucleus is
3) rays possess no mass and carry no
Solution:
charge.
4) They travel with the velocity of light. Z Re sulting nucleus 92 8 2 4 1 2 1 78
5) They can ionize the gas through which they Illustration-14:
pass but with much lower ionizing power.
A radioactive decay chain starts
6) These are photons.
7) They effect photographic plate . from 93 Np 237 and produces 90 Th 229 by
Page 340
NUCLEI
successive emissions. The emitted external physical conditions like temperature,
particles can be pressure etc.
Let ‘N’ be the number of radioactive atoms
A A|
Solution: By using n and present at a time ‘t’and N0 is the initial number
4
of radio active nuclei. Let dN atoms disintegrate
n 2n Z Z| in time ‘dt’. According to the law of radioactive
Illustration-15: decay
The nucleus 2310Ne decays by emission. dN dN
Write down the -decay equation and dt N ; dt = – N ....... (1)
determine the maximum kinetic energy of The proportionality constant is called decay
the electrons emitted. Given that :
m (2310Ne) = 22.994466u; constant (or) disintegration constant. The
m(2311Na)=22.989770 u negative sign indicates that as time increases N
Solution: decreases.
_ _ dN
23
Ne 23
Na e v Q From eqn (1) dt .......(2)
10 11 N
For - decay, Q = [M(x) - M(y)]C2 dN
Integrating eq (2) on both sides dt
= [22.994466-22.989770]931.5 MeV N
= 0.004696 x 931.5 MeV = 4.37 MeV log e N t C .....
3
Illustration-16: How m any and - Here C is the constant of integration
At t = O, N= N0 Substituting in eqn (3),
particles are emitted when uranium nucleus
we get, log e N 0 C
214
(92 U238 ) decay to 82 Pb ?
log e N t loge N 0 ;
Solution: Let n be the number of - particles
log e N loge N 0 t
and m be the number of - particles emitted.
N
t ; N e t
U 238
Pb214
n ( He 4
) m (1 e0
) log e ;
92 82 2
N N
As mass is conserved, 238 = 214 + 4n + m (0) 0 0
= 214 + 4n ; 4n = 24; n = 6 N N 0 et .... (4)
As charge is conserved , 92= 82+ 2n + m (-1)
This shows that the number of radioactive nuclei
10 = 2(6) - m n 6 ; m = 2. decreases exponentially with time.Above
6 - particles and 2 - particles are emitted equation is known as the decay law (or) the law
LAW OF RADIOACTIVE DECAY: of radio-active decay. It is an exponential law.
Based on their experimental observations and Taking logarithm on both sides for the above
analysis of certain radioactive materials N
equation, loge N = loge No - t ; t loge 0
Rutherford and Soddy formulated a theory of N
radioactive decay. According to them, after 1 N
t ln 0
decay of a nucleus the new product (daughter) N
of nucleus has totally different physical as well
as chemical properties.
The rate of radioactive decay (or) the number
of nuclei decaying per unit time at any instant is
directly proportional to the number of nuclei (N)
present at that instant and is independent of the
Page 341
NUCLEI
N (remaining) 1 Rd = 106 decays per second.
1
1 Ci = 3.7 × 1010 decays per second.
T Note : Curie is approximately equal to the
N0
activity of one gram of pure radium.
0.368N0 DECAY CONSTANT ( ) :
It gives the ability of a nucleus to decay. The
decay constant for a given radio active
sample is defined as the reciprocal of the time
T 2T 3T 4T 5T
N during which the number of nuclei decreases to
(delayed) (1/e) times their original value.
N
N0
N0
N0
0.368N0 e
0 t
1
t
T ST
ln(N0) Larger value of corresponding to decay in
smaller time and vice versa. = 0 for stable
nuclei. Decay constant is the characteristic of
ln(N)
Page 342
NUCLEI
N0 1 n
or 2
n
; or N N
0
The mean life (or) average life of a radio active
N 2 sample is reciprocal to decay constant.
The number of nuclei remain in the sample after We know that N N 0 et ; When t ,
half of half life period (t=1/2T) is given by
1
1 n 1 x 0
1 1 2 N N0 e 0.37N0 37% of N0
N N 0 here n then N N 0 e
2 2 2
Hence average life period of a radio active sample
N0 can also be defined as “The time interval during
N taking N0 = 100, N = 50 2 = which 63% of sample decays or sample reduces
2
to 37% of its original amount”.
70.7
Page 343
NUCLEI
We know that N=N0 e t taking log on both Multiplying the equation by eBt we have
sides eBt dN B B N B dt e Bt A N 0 e At
Page 344
NUCLEI
B
N
N B A 0 e At e Bt ....... 1
B
B A A
C ; A B C
Special case: If parent nuclide A is of long life C
than B, then A B and from equation (1), Illustration-17:
we have A radioactive sample has an activity of 5.13
x107 Ci. Express its activity in ‘becquerel’
A N 0 and ‘rutherford’.
NB
0
1 e Bt .... 2
Solution: Since 1 Ci = 3.7 x 1010 decays per
After time t which is longer than half-life B but second, activity = 5.13 x 107 Ci
shorter than half life of A, then e a t 1 and =5.13 x 107 x 3.7 x 1010 Bq = 1.9 x 1018 Bq
A N A Since , 1 x 106 decay per second = 1Rd
N A N 0 and so N B 1.9 1018
0 Activity =1.9 x 1018 Bq = Rd
1106
or A N A B N B ..... 2 = 1.9 x 1012 Rd.
Successive Disintegration Illustration-18:
Let a parent radioactive nucleus A (decay A radioactive substance has 6.0 1018 uired
constant 1 ) has number of atoms N 0 at tiem for the active nuclei of the same substance
to become 1.0 1018 if its half-life is 40 s.
t 0 . After disintegration it converts into a Solution: The number of active nuclei at any instant
nucleus B (decay constant 2 ) which is N0 t N
radioactive. of time t, N e ; loge N0 t
At t 0 ,
N N
loge 0 2.303 log10 0
N B 0, N A N1 and N B N 2 N N
t
N0 0 0 t 0
In this problem, the initial number of active nuclei,
1 gl2
B C stable
A N0 = 6.0 1018 ;
N 1.01018 , T 40s,
N1 N2 0 N3 t t
0.693 0.693
dN1 1.733 102 s1 .
1 N1 N1 N 0 e t T 40
dt
6.0 1018
dN2 N
1N1 2 N2 N2 1 e1t e2t 2.303 log10
1.0 1018
dt 1 t
1.733102
dN 2
2 N 2 2.303 log10 6 2.303 0.7782
dt = 103.4 s.
2
1.73310 1.733102
1. If t1/2 1 t1/2 2 or 1 2
Illustration-19:
1 N1 2 N 2 (secular equilibrium) A radioactive sample can decay by two
different processes. The half-life for the first
2. If t1/2 t t1/2 or1 2
process is T1 and that for the second process
1 N1 2 1 N 2 transient equilibrium is T2. Find the effective half-life T of the
radioactive sample.
Parallel Radioactive Disintegration
Solution: Let N be the total number of atoms of
dN1
the radioactive sample initially. Let and
dt
Page 345
NUCLEI
dN 2 t t 25
be the initial rates of disintegrations of the n t1/ 2
dt t1/ 2 n 5 ; t1/2 5 days
radioactive sample by the two processes Illustration-22:
dN1 dN 2 The half -life period of a radioactive
respectively. Then 1 N and 2 N
dt dt substance is 20 days. What is the time taken
Where 1 and 2 are the decay constants for for 7/8th of its original mass to disintegrate?
the first and second processes respectively. Solution:
The initial rate of disintegrations of the radioactive Let the initial mass be one unit.
sample by both the processes. 7 1
Mass remaining = 1
dN dN 8 8
= 1 2 1 N 2 N 1 2 N . 1
dt dt
A mass of 1 unit becomes unit in 1 half life
If is the effective decay constant of the 2
radioactive sample , its initial rate of 1 1
unit becomes unit in 2nd half life
disintegration. 2 4
dN dN dN1 dN 2 1 1
N But unit becomes unit in 3rd half life
dt dt dt dt 4 8
N 1 N ; Time taken = 3 half lifes = 3x20 = 60 days
0.693 0.693 0.693
Illustration-23:
How many disintegrations per second will
T1 T2 T
1 1 1 TT
occur in one gram of 238 U , if its half-life
92
;T 1 2 .
T T1 T2 T1 T2 against -decay is 1.42 x 1017s?
Illustration-20: Solution: Given Half -life period (T)
Plutonium decays with a half life of 24,000 0.693
= 1.42 1017 s
years. If plutonium is stored for 72,000
years, what fraction of it remains? 0.693
17
4.88 1018
Solution: T1/2 = 24,000 years 1.42 10
Duration of time (t) = 72,000 years Avogadro number (N) = 6.023 1023 atoms
n = Number of atoms present in 1 g of
t
Number of half lifes (n) = T 238 N
1/2 92 U
A
72000
3 6.023 10 23
24000 25.30 10 20
238
1 1 2 1 3 1 dN
1g g g g n
2 4 8 Number of disintegrations =
dt
1 = 4.88 x10-18 x 25.30 x 1020
Fraction of plutonium remains = g = 1.2346 x 104 disintegrates/sec
8
Illustration-21: Illustration-24:
A certain substance decays to 1/32 of its Two radioactive nuclei A and B have
initial activity in 25 days. Calculate its half disintegration constants A and B and
-life. initially N A and N B number of nuclei of
Solution:
1 1 1 1 1
them are taken, then the time after which
1 2 3 4 5
1g g g g g g their undisintegrated nuclei are same is
2 4 8 16 32
n 5 Solution: N Ae At N B e Bt
Page 346
NUCLEI
t1
t1 t2 A A n1 ; For 67% decay,,
ln q (t1 t2 ) T ln q T
T Ap A N 1 33 1
p
n2 n2 .... 2
Illustration-27: A radioactive sample has an N0 2 100 2
acticity of 5.13 107 Ci . Express its activity t 2 33 1
n2 2 ; n n
in ‘becqueral’ and ‘rutherford’. T 1 67 2 2 1
Solution: Since 1Ci 3.7 1010 decays per 1 1 t t
1 2 1 1
second, activity 2 t t
2 2 1
T
5.13 107 Ci 5.13 107 3.7 1010 Bq T
t2 t1 T 20 minutes
1.9 1018 Bq
Page 347
NUCLEI
Illustration-31: Find the number of half lives Total decay constant,
elapsed before which, 93.75% of a radioac-
tive sample has decayed. 1 1 1
or per year
Solution: Fraction of atoms left undecayed is 1620 405 324
100 93.75% = 6.25% We know that N N 0 e t
N 6.25 1 3
i.e., N 100 16 When th part of the sample has sisintegrated,
0 4
4 4
N 1 1 N0 N0
or or, N N 0 N ; N 0 e t or e t 4
N0 2 2 4 4
n Taking logarithm on both side, we get
1
Comapring with N N 0 , yields n 4 1 2
2 t log e 4 or, t log e 22 log e 2
Illustration-32: Calculate the number of half
lives elapsed, at the end of which, the ac- 2 324 0.693 449 year
tivity of a radioactive sample decrease by Illustration-34: Two radioactive substnaces X
90%. and Y initially contain equal number of at-
Solution: The change in activity with time follows oms. Their half-lives are 1 hour and 2 hours
the same law as that of number of radioactive respectively. Calculate the ratio of their
atoms rates of disintegration after four hours.
Percentage of radioactive atoms left Solution: Rate of disintegration
undecayed 100 90% 10% dN 0.693
R N N
N 1 N 1
n dt T
i.e., N 10 from N 2 The ratio of rates of disintegration
0 0
R1 N1 T2
[where n=no. of half lives elapsed]
R2 N 2 T1
n
1 1
or, 2n 10 Here, T1 1 hour, T2 2 hours
2 10
N 0 is same for the the radio active substances.
1
or, n log 2 log10 or, n 3.3 In four hours, X completes 4 half-lives as its
0.3010 half-life is 1 hour. The remaining number of at-
Thus, at the end of 3.3 half lives, the activity oms of ,
(and hence the number of atoms) would have
N0 N 0 N0
decreased by 90%. N1 4 . Y completes 2 half-lives
Illustration-33: The mean lives of a radioac- 2n 2 16
tive substance are 1620 years and 405 year in four hours as its half-life is 2 hours.
for emission and emission respec- The number of atoms of Y remaining
tively. Find the time during whihc three- N0 N0
N2
forth of a sample will decay if it is decayed 22 4
both by emission and emission si- R1 N0 4 2 1
multaneously. R2 16 N 0 1 2
Solution: The decay constnat is the reciprocal
of the mean life .
NUCLEAR FISSION:
The process of nuclear fission first discovered
1 1 by German Scientists Otoo Hahn and Strassman
Thus, per year and per
1620 405 in 1939.
year
Page 348
NUCLEI
Nuclear Fission is a nuclear reaction in which a CHAIN REACTION:
heavy atomic nucleus like U235 splits into two If the mass of fissionable material exceeds a
approximately equal parts, emitting neutrons and critical value, chain reaction or self propagating
liberating large amount of energy. Bohr and fission reaction takes place.
Wheeler proposed liquid drop model to explain The rate of reaction increases in geometric
this fission process. Nucleus of U235 undergoes progression during uncontrolled chain reaction.
fission when it is struck by slow neutrons. This The process of continuation of nuclear fission
fission is not due to the impact of neutron. Energy which when once started continues
spontaneously without the supply of additional
of about 200 MeV is released during one fission
235 neutrons from outside is defined as chain
reaction of 92U . The most probable nuclear reaction.
fission reaction is In the fission reaction
235 1 141 92 1
U + n Ba + Kr +3 n +200 MeV
92 0 56 36 0
0 n1 92U 235 56 Ba141 36 Kr 92 30 n1 Q
235
There is no guarantee that U always breaks here, three more neutrons are produced. These
into Barium and Krypton. On an average,in the three neutrons may then bombarded with three
fission of U235, 2.5 neutrons are emitted per more U 235 , giving rise to 3 3 9 neutrons.
fission when fission occurs due to slow Again these 9 neutrons with fission gives
neutrons. U235 undergoes fission with fast 9 3 27 neutrons and so on. Thus the number
neutrons also. But this probability is minimum. of neutrons increase in geometric progression.
Fission fragments are unstable and emit Soon a stage is reached when a very large
neutrons. More than 99% of the neutrons number of nuclei are split releasing large energy
produced in the fission process are released in short time. An atom bomb is an example for
within 10 18 s to 10 15 s . They are called prompt this type of fission process.
neutrons. A small number of around 1% or less Ba
n
are emitted over a period of time after the fission n
U235
process has taken place. They are called delayed n
neutrons and will play an important role in teh Ba O Kr
n Ba
running of a nuclear reactor. n
In the fission of uranium about 0.1% of mass of U
235
U
235 n
n n
rectants is converted into energy. Mass defect Kr
n
in the fission reaction is nearly equal to 0.214 O n
Kr Ba
amu. n
U
235 n
Energy released per fission of 92 U 235 nucleus n
Kr
0.214 931 200 MeV The chain reaction can be controlled by slowing
U underoges fission with fast neutrons whose
238
down the neutrons and by absorbing neutrons.
kinetic energies are greater than 1 MeV, whereas Neutrons are slowed down using graphite,
235
U undergoes fission when bombarded with ordinary water or heavy water. These
slow neutrons (or) thermal neutrons whose substances are called moderators. For
kinetic energies are nearly 0.025 eV. absorbing neutrons boron (or) cadmium rods
The projectile that can be used to bombard a are used.
heavy nucleus need not be a neutron alone. It Reproduction factor (K): “It is the ratio of number
can be a proton or a deuteron or an particle. of neutrons in any particular generation to the
Hence fission of uranium and thorium is also number of neutrons in the preceding generation.
possible with the high speed prootons of energy Case(i): K<1, the number of neutrons in successive
6.9 MeV (or) by deuterons of energy 8 MeV generations decreases and the chain reaction
(or) particles of can’t continue (Sub critical state)
32 MeV energy.
Page 349
NUCLEI
Case (ii): K=1, the chain reaction will proceed at a fission. The commonly used moderators in the
steady rate and the chain reaction can be order of efficiency are (i) Heavy water (ii)
graphite, (iii) Berillium and Berillium Oxide
controlled (critical state). e.g: nuclear reactor.
Case (iii): K>1, the number of neutrons increases Heavy water is a best moderator A good
and the reaction is said to be uncontrolled chainmoderator should have
reaction (super critical state). e.g: atom bomb. (1) low atomic mass
Critical mass: To sustain the chain reaction the mass(2) poor absorption of neutrons
of uranium should be equal (or) more than a (3) good scattering property.
particular amount of mass called ‘critical mass’.
(4) The size of moderator atom should be nearly
1 of same size as that of the size of a prompt
Critical size: The value of is said to be critical
k neutron.
size of the substance. If the size of substanc eis
CONTROL RODS:
even slightly less than the critical size the chain
The function of a control rod is to absorb
reaction will stop. (capture) the neutrons. Cadmium, Boron and
Chain reactions are of two types. steel rods are used as control rods in a nuclear
a) Uncontrolled Chain-Reactions: In this reactor. Cadmium rods are best control rods.
chain reaction, more than one of the neutrons They regulate the net rate of neutron production
produced in a particular fission cause further and hence they control the intensity of fission
fissions so that the number of fissions increasesprocess.
very rapdily. Thus it is a very fast reaction and
COOLANT:
the whole substance is fissioned within a fraction
The function of a coolant is to keep the reactor
of a second. This is actually what happens in an temperature at a low value so that there may
Atom Bomb. not be any danger of heat damage to the reactor.
b) Controlled Chain-reaction: If only one of Air and CO2 are used as gaseous coolants.
the neutrons produced in each fission is able to Water, Organic liquids, Helium, Liquid Sodium
cause further fission, then the process is slow are used as liquid coolants. Liquid sodium is best
and the energy is released steadily. Such a chaincoolant.
reaction is called as controlled chain-reaction.
PROTECTIVE SHIELD:
The energy released in this process can be The process of preventing radioactive effect
utilized for peaceful purposes. this is actually what
around nuclear reactor is called Protective
happens in Nuclear-Reactors. Shield. During the working of a nuclear reactor
NUCLEAR REACTOR OR ATOMICPILE: dangerous radiations such as high energy
Nuclear reactor is a device in which nuclear neutrons, gamma rays and thermal radiations
fission is produced by controlled self sustainingare produced. To protect the persons working
chain reaction. And is used for the production there, the reactor is thoroughly shielded with
of nuclear power (energy). The essential parts concrete wall of several feet thick and lined with
of a nuclear reactor are (i) the fuel, (ii) moderator,
metals like lead.
(iii) control rods, (iv) coolant, (v) radiation
Thermal Neutrons
shields.
Fast neutrons can be converted into slow
THE FUEL:
neutrons by certain materials called moderator’s
The common fuels used are uranium U , 238
(Paraffin wax, heavy water, graphite) when fast
235 moving neutrons pass through a moderator, they
enriched uranium (U ) and plutonium collide with the molecules of the moderator, as
Pu and T h .
236 23 2 a result of this, the energy of moving neutron
decreases while that of the molecules of the
MODERATOR: moderator increases. After sometime they both
The function of a moderator is to slow down attains same energy. The neutrons are then in
the fast moving neutrons to increase the rate of thermal equilibrium with the molecules of the
moderator and are called thermal neutrons.
Page 350
NUCLEI
Energy of thermal neutron is about 0.025 eV (Pu239) is produced along with energy.
and speed is about 2.2 km/s. Fuel: Natural Uranium.During fission of U235,
POWER OF A NUCLEAR REACTOR: energy and secondary neutrons are produced.
In the nuclear reactor, large amount of heat will These secondary neutrons are absorbed by U238
be generated in the core. These reactors have and U239 is formed. This U239 coverts into Pu239
elaborate cooling systems that needs water. This after two beta decay. This Pu 239 can be
water absorbs the heat and produces steam. separated, its half life is 2400 years.
This steam in turn is used to run the steam turbines
2
which ultimately generate electric power. Such U 238 0 n1 92U 239
92 94 Pu 239
reactors are called power reactors. (best fuel of fission)
The power generated by a nuclear reactor is This Pu239 can be used in nuclear weapons
nE n because of its small critical size than U235)
P here be the number of fissions per
t t USES OF ATOMIC ENERGY :
second and E be the energy released in each Generation of electric power : The coolant in a
fission nuclear reactor absorbs the heat generated as a
E = 200 MeV 200106 1.61019 J result of the chain reaction and it releases the
heat to the water which is converted into high
= 3.2 x 10-11J
Note : Number of fissions per sec in a reactor pressure steam. This steam is used to drive
n P turbine and operate the electric generator.
of power 1 W is given by Production of radio isotopes : A small amount of
t E
the pure element is placed in an aluminium
1
=3.125x1010 fissions per sec container and the container is placed in the
3.21011 reactor for a few days. The element absorbs
Note : If only x% of energy released in fission
is converted into electrical energy then out put neutrons and the element becomes radioactive
isotope.
x n E
1 0 0 t
power of reactor is 127
0 n1 53 I128* . Radio iodine
53 I
obtained in this way can be used to treat the
Note: If ‘x’gm of fuel with mass number ‘A’ thyroid gland. These radio isotopes have a
completely undergo nuclear fission in time t sec number of applications in the field of medicine ,
in a reactor then its power is given by
agriculture, industry and basic research.
x Source of neutrons : A large number of neutrons
Number of moles in x gm of fuel =
A are produced in a reactor. They are used in
Number of atoms (nuclei) present in x gm of research . The effect of neutrons on biological
x
fuel n N A . Where NA is Avogadro
tissues is studied. A new branch of physics called
A Neutron Physics has come up. At o mic
number energy is used to create artificial lakes, to divert
xN A E the course of a river , to make tunnels for laying
nE
power P P new railway tracks etc. Atomic energy is used
t At
for driving automobiles, submarines and war -
USES OF NUCLEAR REACTORS: planes. Atomic energy is used in war - fare for
To generate electric power. creating destructive atom bombs and hydrogen
To produce nuclear fuel plutonium -239 and bombs.
other radioactive materials which have a wide NUCLEAR FUSION:
variety of applications in the fields of medicine, The phenomenon in which two lighter nuclei
industry and research.
combine to form a heavier nucleus of mass
BREADER REACTOR:
less than the total mass of the combining nuclei
The atomic reactor in which fresh fissionable fuel
is called nuclear fusion. This mass defect
Page 351
NUCLEI
appears as energy. On adding all these 6 equations, we get
1 4 0
Simple exampe for nuclear fusion reaction is 4 H He + 2( e )+ Q
1 2 +1
formation of a helium nucleus by fusing of four Where Q Q1Q2 Q3 Q4Q5Q6 The value of
hydrogen nuclei and releasing 25.7 MeV of
energy. Q as calculated from mass defect is 26.7 Mev..
1 4 0
41 H 2 He 21 e 25.7 MeV In the sun both proton - proton cycle and
At temperatures of about 10 K, light nuclei7 carbon- nitrogen cycles occur with equal
combine to give heavier nuclei. Hence, fusion pro babilities. In st ars, who se interior
reactions are called thermo nuclear reactions. temperatures are less than that of the sun, proton
Nuclear fusion takes place in the sun and other -proton cycle dominates the energy generation.
stars. Energy produced in a single fission of Again in stars, whose interior temperatures are
235
92
U is larger than that in a single fusion of more than that of the sun, the energy generation
Hydrogen into Helium. But fusion produces more is mainly due to carbon- nitrogen cycle.
energy than fission per nucleon. In fission, 0.09% The core temperature of heavier stars may be
of mass is converted into energy. In fusion larger than that of the sun and much larger nuclei
0.66% of mass is converted into energy. may be formed. (At the core of the sun,
Hydrogen bomb is a fission – fusion bomb. gravitational attraction produces immense
STELLAR AND SOLAR ENERGY: pressure and temperature, which can reach more
Stellar and solar energy is due to fusion.
than 27 million degrees Fahrenheit (15 million
The cycles that occur are.
degrees Celsius). Hydrogen atoms get
Proton - Proton Cycle & Carbon - Nitrogen
Cycle compressed and fuse together, creating helium)
PROTON - PROTON CYCLE: NOTE: Due to enormous energy released in Sun
The Thermonuclear reactions involved are: and Stars the atmosphere of them will be in
1 1 2
2( H )+2( H ) 2( H ) + 2( e ) + Q1
0 ionised state which is called Plasma (Which
1 1 1 +1 contains fast moving neutrons and electrons).
2 1 H 2 2 1 H 1 2 2 He3 Q2 Nuclear fusion can not be controlled.
3 3 4 2
He + He ( He ) + 2( H ) + Q3 Nuclear Fission Nuclear Fusion
2 2 2 1
On adding up these reactions, we obtain. 1) Neutron are required 1) Protons are required
1 4 0
4( H ) He + 2 e + Q for it for it
1 2 +1
Where Q Q1Q2 Q3 is the total energy 2) It is possible at normal 2) It is possible at high
pressure and temperature pressure and temperature
evolved in the fusion of 4 hydrogen nuclei
3) Energy released per 3) Energy released per
(protons) to form Helium nucleus. The value of
nucleon 0.9MeV nucleon 6MeV
Q as calculated from mass defect comes out to 4) % of mass getting 4) % of mass getting
be 26.7MeV converted into energy converted into energy
CARBON - NITROGEN CYCLE: = 0.1% = 0.7%
Proposed by bethe. It consists of following 5) Fissionable materials are 5) Fusion materials are
reactions. expensive cheap
12
1 H 1 7 N 13 Q1 ; 6) Harmful reactions are 6) Harmful reactions are not
6C
13 13 0
produced produced
N C +( e )+ Q 2
7 6 1 Illustration-35:
13 1 14
6 C 1 H 7 N Q3 ; An explosion of atomic bomb releases an
N 14 1 H 1 8 O15 Q4 energy of 7.6 x 1013 J. If 200 MeV energy is
7
15 15 0 released on fission of one 235 U atom
O N +( e )+ Q5 ; calculate (i) the number of uranium atoms
8 7 +1
7N
15
1 H 1 6 C 12 2 He 4 Q6 undergoing fission. (ii) the mass of uranium
Page 352
NUCLEI
used in the atom bomb Illustration-38:
Solution: How much 235U is consumed in a day in
E=7.6x103J; Energy released per fission an atomic power house operating at 400
= 200 MeV MW, provided the whole of mass 235U is
= 200 106 1.6 x 10-19 = 3.2 x 10-11 J converted into energy?
Number of uranium atoms (n) = Solution: Power = 400 MW = 400 x 106 W;
Total energy time = 1 day = 86, 400 s.
Energy produced , E = power time = 400
Energy per fission
106 86,400 = 3.456 1013 J.
7.6 1013 As the whole of mass is converted into energy ,
n 11
= 2.375 10 24 atoms by Einstein's mass -energy relation.
3.2 10
E = Mc2
Avogadro number (N) = 6.023×1023 atoms
Mass of uranium = E 3.456 1013
3.84 104 kg 0.384 g .
c2 (3 108 ) 2
n × 235 2.375 × 10 24 × 235
= = 92.66g Illustration-39:
N 6.023 × 10 23 Calculate the energy released by the fission
Illustration-36: 1 g of 235 U in joule, given that the energy
Calculate the energy released by fission released per fission is 200 MeV.
from 2 g of 23592U in kWh. Given that the ( Avogadro’s number = 6.023 1023 )
energy released per fission is 200 MeV. Solution:
Solution: Mass of uranium = 2g The number of atoms in 1 g of 235 U
Energy released per fission = 200 MeV Avogadro ' s number
= 200 x 106 x 1.6 x 10-19 = 3.2 x 10-11 J = Mass number
=
Number of atoms in 2 gram of uranium is
6.0231023
23 2.5631021
2 6.023 10 235
n = 5.125 10 21 atoms Energy released per fission = 200 MeV
235
Total energy released = No. of atoms x energy = 200106 1.61019 3.21011 J .
released per fission Energy released by 1 g of 235 U
= 5.125 x 1021 x 3.2 x 10-11 = 16.4 1010J = Number of atoms energy released per
16.4 1010 fission
Energy in Kwh = Kwh
36 105 = 2.5631021 3.2 1011 J 8.2021010 J
= 0.455 105 Kwh = 4.55 104 Kwh Illustration-40: A nuclear power reactor gen-
Illustration-37: erates electric power of 100 MW. How many
number of fissions occur per second if
200 Mev energy is released when one nuclear fuel used in the reactor is uranium.
nucleus of 235U undergoes fission. Find the Solution: Power of a reactor using uranium is
number of fissions per second required for n
producing a power of 1 megawatt. P 200 106 1.6 10 19 J
t
Solution: Energy released = 200MeV
= 200 x 106 x 1.6 x 10-19 = 3.2 x 10-11J n
100 106 200 1.6 10 13
P = 1 mega watt = 106 watts. t
No.of fissions per second O 100 MW
Total energy
n n 100 106
Energy per fission 3.125 1018 / s
t 200 1.6 10 13
106
n= 3.125 1016 Fissions Illustration-41: What is the power output of
3.2 10 11 235
92 U reactor if it takes 30 days to use up 2
Page 353
NUCLEI
kg of fuel and if each fission give 185 MeV
2
of usable energy? E0 m0c 2 9.1 1031 3 108
Solution: Number of atoms in 2 kg of
235 6.02 1023 2 103 8.2 10 14 J 0.51MeV
92 U 5.12 1024 Hence for pair production, the minimum energy
235
Therefore, energy released in 30 day of γ -photon must be 2 x 0.51 = 1.02 MeV. If
the energy of γ -photon is less than this, there
5.12 10 24 185MeV 1.511014 J
Energy released per second may be Compton’s effect. If energy of γ -
1.51 1014 photon is greater than E0 , then extra energy
58.4 MW will become kinetic energy of the particles. If E
30 24 60 60
Note:Antiparticles: An antiparticle is a form of mat- is the energy of γ -photon, then kinetic energy
ter that has the same mass as the particle but of each particle will be,
carries an opposite charge and/or a magnetic E 2 E0
moment that is oriented in an opposite direction K electron K positron
relative to the spin. 2
Name of the particle Antiparticle The inverse process of pair production is called
pair annihilation. According to it when electron
Electron e Positron e 1
1
and a positron come close to each other,
annihilate each other and produces minimum two
Proton P Anti Proton P γ -photons. Thus
0
Neutron n Anti Neutron n 1 β
0
1 β 2hf
Positron electron γ photon
Neutrino v Anti Neutrino v
0
+1
Positive Pi-Meson Negative Pi-Meson
Positive Mu-Meson Negative Mu-Meson 2 ( photon )
A few electrically neutral particles, like the pho-
ton and neutral meson are their own antipar- 10
ticles. A collision between a particle and an an- ADDITIONAL INFORMATION
tiparticle results in annihilation of matter. RADIO ISOTOPES AND THEIR USES:
Pair Production and Pair Annhilation (a) Radio isotopes have very short half lives and
Conservsion of Photon energy to mass (Pair hence used for various purposes.
Medical applications :
production) 27
PAIR PRODUCTION AND 0 n1 11 Na 24* 2 He 4
13 Al
ANNIHILATION Radio – sodium is used to find out how a given
medicine is circulated in the body. It is also used
When an energetic γ - photon falls on a heavy to find out circulatory disorders in blood vessels.
nucleus, it is absorbed by the nucleus and a pair 31 2 32*
of electron and position is produced. This 15 P 1 H 15 P 1 H1
phenomenon is called as pair production and Radio – phosphorus is used in the treatment of
can be represented by the following equation: skin diseases. It is also used for the treatment of
0
blood disorders.
0
hv β β 127
γ photon
1 1
53 I 0 n1 53 I128* rays
Positron electron
Radio – iodine is used in the treatment of thyroid
+1
0
glands. Radio - iodine ( I131) is used for diagnosis
and treatment of brain tumor and for the study
of pumping condition of heart.
photon 59
27 Co 0 n1 27 Co60* rays
Heavy nucleus 10 Radio – cobalt is used in the detection and
The rest mass energy of electron or positron is: treatment of cancer
Radio-iron is used to detect anemia and treat
anemia.
Page 354
NUCLEI
In Geology: Radio carbon(C14) is used to
determine the age of fossils by radio - carbon
dating Radio isotopes are used to determine
the age of rocks by the ratio of U 238 to Pb 206
In industry : Radio – isotopes are used to find the
wear and tear of machine parts Radio isotopes
are used to detect flaws in metal structures
Radio isotopes are used for treatment of alloys
such as quenching , annealing and hardening.
Radio isotopes are used in the selection of
appropriate lubricants.
In research: Radio - isotopes are used in the study
of nuclear disintegrations of elements.
In food preservation: By exposing vegetables and
other food stuffs to radiations from radio - active
isotopes, their shelf life can be increased.
In agriculture: Radio phosphorus (P32) is used to
study the uptake of phosphorus by plants using.
Radio sulphur (S34) is used to study the transport
of minerals in plants .Radio zinc is used to
develop new species of plants by causing genetic
mutation. Irradiation by - radiations of seeds
to improve yields.
In Chemistry: Radio oxygen (O18) is used to
study the mechanisms of photosynthesis and
hydrolysis of ester Radio isotopes are used in
the chemical analysis of solubility of sparingly
soluble salts such as PbSO4 and AgCl and
determination of trace amounts of elements in
industrial raw materials and products.
Page 355
RAY OPTICS PART-1
Page 357
RAY OPTICS PART-1
ROTATION OF IMAGE:
Case (1) :
When direction of incident ray is kept
i r
fixed and mirror is rotated.
See figure M1, N1 and R1 indicating the initial
position of mirror, initial normal and initial
direction of reflected light ray respectively.
M 2 , N 2 and R 2 indicate the final position of
From the above figure i r mirror, final normal and final direction of reflected
But i = r light ray respectively.
N1 N2
Hence angle of deviation in the case of reflection A
R1 D
is 2i fixed
C
incident
R2
ray
M1
B
M2
i
/2
i1 r1 i2 r2
y mx c From the figure it is clear that
It is noted that, generally anti - clock wise
ABC 2 2 or 2
deviation is taken as positive and clock wise
deviation as negative. That means when incident ray is fixed, and mirror
Normal incidence: In case light is incident rotates through the angle .
normally (i) Then reflected ray rotates through the angle 2
in the same sense as the mirror rotates.
i r 0; 180o (ii) The angular velocity and angular acceleration of
Reflected Light new reflected ray becomes twice as that of mirror
Incident Light
N
Grazing incidence: d d 2
m ; r
In case light strikes the reflecting surface dt dt
tangentially, i r 90o ; 0o or 360o d
r 2 2m
dt
1 Angular velocity of mirror
2 Angular velocity of reflected ray
2 21
Incident Case (2) :
Reflected When mirror is fixed and incident ray
Light Light rotates:
1 = Angular velocity of incident ray
N
Page 358
RAY OPTICS PART-1
= 2n+
fixed mirror Illustration 1: Two plane mirrors are inclined
to each other such that a ray of light
[negative sign shows that the direction of angular incident on the first mirror and parallel to
velocity is opposite to each other] the second is reflected from the second
mirror parallel to the first mirror. The angle
DEVIATION OF LIGHT RAY DUE TO between the two mirrors is
TWO SUCCESIVE REFLECTION: Solution: Let the angle between the two mirrors
2 be . Ray PQ is parallel to mirror M 1 and
M2 –
RS is parallel to M 2
–
M1 P
R
S
O M2
1 M1 Q
So, M 1 RS ORQ M1OM 2
0
1 180 2 Similarly M 2QP OQR M 2OM 1
2 1800 2 In ORQ , 3 1800 , 600
1 2 3600 2 Illustration-2:Two plane mirrors are placed
; angle between mirrors. perpendicular to each other. A ray strikes
Net deviation is equal to summation of on the mirror and after reflection falls on
individual deviation suffered by a light ray. the second mirror. The ray after reflection
1
from the second mirror will be
Solution:
– 1
o
90
net net – – 2 90o
2
– 3 90o
Page 359
RAY OPTICS PART-1
ui = uix i + uiy j +uiz k i.e., the image formed by a plane mirror left is
turned into right and vice versa with respect to
z
object as shown in figure (b).
i2 M
u
k N1
Page 360
RAY OPTICS PART-1
The principle of reversibility states that rays approach (or recede) at the same speed v,
retrace their path when their direction is and the relative velocity of image with respect
reversed. In accordance with the principle of to object will be 2v as shown in figure (a). If
reversibility object and image positions are the mirror moved towards (or away from) the
interchangable. The points corresponding to stationary object with speed v, the image will
object and image are called conjugate points. also move towards (or away from) the object
This is illustrated in figure. with a speed 2v, as shown figure (b).
O I I O
(a) (b) M M
A mirror whatever may be the size, it forms the When the object moves infront of stationary
complete image of the object lying infront of it. mirror, the relative speed between object and
Large mirror gives more bright image than a its image along the plane of the mirror is zero
smaller one. It is seen that the size of reflector and in perpendicular to plane of mirror relative
must be much larger than the wavelength of the speed is twice that of the object speed.
incident light otherwise the light will be scattered VIO y 0 and VIO x 2v x
in all directions.
Every object has its own field of view for the M
given mirror. The field of view is the region
between the extreme reflected rays and V Vy
y V
depends on the location of the object infornt of
the mirror. If our eye lies in the filed of view V
then only we can see the image of the object
other wise not. This is illustrated in figure.
O Vx Vx I
Field
of view
Illustration 4: A plane mirror is placed at
origin parallel to y-axis, facing the positive
x-axis. An object starts from (2m, 0, 0) with
M
a velocity of 2iˆ 2ˆj m/s. Find the
relative velocity of image with respect to
I object.
O
Page 361
RAY OPTICS PART-1
HF 2 x y H
H
MM 1 x y
2
Illustration 5: Find the minimum size of mir-
ror required to see the full image of a wall
behind a man standing at the centre of
360 room, where H is the height of wall
If is even number (say m) Number of Solution:From similar triangles concept from the
images formed n m 1, for all positions figure
of objects in between the mirrors.
360
If is odd integer (say m) number of images
formed n m , if the object is not on the
bisector of mirrors. n m 1, if the object
is on the bisector of mirrors.
360
If is a fraction (say m). The number of
images formed will be equal to its integer part
i.e., n m .
O Observer eye
Ex: If m=4.3, the total number of images
n 4.3 4 H
'
3x
MM '
H
MM x 3
Illustration 6: A point source of light S, placed
at a distance L in front of the centre of a
mirror of width d, hangs vertically on a
wall. A man walks in front of the mirror
along a line parallel to the mirror at a
distance 2L from it as shown in figure.
m Find the greatest distance over which he
can see the image of the light source in
the mirror.
To see complete image form head to toe,
minimum height of mirror is half of height of
object, the upper end of mirror should between
head and eye and lower end of mirror should S
be between eye and feet. d
H H L
x 2L
M
x x Solution: The ray diagram will be as shown in figure.
E
y y
M
y
F F
Page 362
RAY OPTICS PART-1
o h
In ABC , tan 45
60
h = 60m
IMAGES FORMED BY TWO PLANE
MIRRORS:
HI = AB =d, DS = CD = d/2
d
Since, AH = 2AD, GH = 2CD = 2 d
2
Similarly IJ = d
GJ = GH + HI +IJ =d +d + d =3d
Illustration 7:A man is 180cm tall and his eyes
are 10cm below the top of his head. In x y
order to see his entire height right from
toe to head, he uses a plane mirror kept y+d x+d
at a distance of 1m from him. The mini-
mum length of the plane mirror required
is x + 2d y + 2d
H
10 cm
y + 3d x + 3d
E
180 cm
180/2 cm
and so on.
Solution:
I1 = Image formed by the mirror M1
F
I 2 = Image formed by the mirror M2
According to the following ray diagram
I 21 Image of I 2 which will act as an object
1
length of mirror = 10 170 90cm for mirror M1
2
Illustration 8: When a plane mirror is placed I12 Image of I1 which will act as an object
horizontally on level ground at a distance for mirror M2
of 60 m from the foot of a tower, the top of Illustration 9: Figure shows a point O i.e. the
the tower and its image in the mirror object placed between two parallel mirrors.
0
subtend an angle of 90 at the eye. The Its distance from M1 is 2 cm and that from
height of the tower is M2 is 8 cm. Find the distance of images from
Solution: the two mirrors considering reflection by
A the combination.
M1 M2
h
45o o
90 2cm
B
45
o
C
O
h 8 cm
A
60 m
Solution:
Page 363
RAY OPTICS PART-1
M1 M2
2cm
O M M1 M2
8 cm
d = 10 cm
Page 364
RAY OPTICS PART-1
of convex mirror
Field of view
O
Field of view of
concave mirror
I
O
SPHERICAL MIRROR:
dy x Spherical Mirror is formed by polishing one
tan 2 cos surface of a part of sphere. Depending upon
dx L which part is shining the spherical mirror is
classified as
2 900 ; 450 a) Concave mirror: If the side towards center of
1 2 cos x / 2 x L / 3 curvature is shining and
b) Convex mirror: If the side away from the
2L 3L center of curvature is shining.
y sin / 3
BASICTERMS:
L 3L 2 L 3 L The basic terms associated with spherical
, & , mirrors are as follows :
3 3 (i) The centre of curvature, C is the centre of
Field of view of object as well as its the sphere from which the mirror is a small
image w.r.t an optical surface: segment
It is defined as region of space where diverging (ii) The vertex or pole, P is the midpoint of the
incident rays for an object and diverging reflected mirror.
or refracted rays for the image are present . (iii) The radius of curvature, R is the radius of
Field of view of image depends on shape, size the sphere from which the mirror is a small
and nature of given optical boundary as well as section.
position of the object. (iv) The principal axis is the line passing through
CONCEPT OF FIELD OF VIEW: the centre of curvature C and the vertex P. It
If our eyes are present in the field of view then extends indefinitely in both directions.
only we can see the object and image. (v) With a concave mirror, all rays parallel to the
Virtual object has no field of view hence it cannot
be seen. principal axis pass through a single point F after
reflection. Then F is called the principal focus.
The parallel rays converge at F after reflection.
however, in case of a convex mirror, the
reflected rays appear to emerge from F behind
Page 365
RAY OPTICS PART-1
the mirror. Thus, the reflected rays appear to cos 1,sin , tan .
diverge from the principal focus. This is also known as paraxial approximation.
SIGN CONVENTION:
Page 366
RAY OPTICS PART-1
3. A ray passing through or directed towards the 4. Parallel rays but not parallel to principal
centre of curvature, after reflection from the axis
mirror, retraces its path (as for it
i 0 and so r 0)
h0
R
B
1
M P
2. For paraxial rays (parallel to principal axis) B C hi F
Here is small so A
sin , cos 1, tan )
u
R R
Hence BC and BP . thus, point B is v
2 2 R
the Using the sign convention, u = -ve, R = -ve
midpoint of PC (radius of curvature) and is and v=-ve into the above equation, we get
R
defined as focus, so BP = f = .
2 1 1 2 1 1 2
R u v R u v R
3. BP is the focal length, f
2
Page 367
RAY OPTICS PART-1
Using the relation R = 2f in the above equation, dv
For small objects: ml
1 1 1 du
we obtain By differentiating mirror equation on both sides
u v f with respect to u we get,
This relation is the Mirror formula. 1 1 1 1 dv v 2 dv v 2
2 2 ml m2
v u f v du u du u 2
MAGNIFICATION:
Length of image = m 2 (length of object)
size of image l
Magnification : sizeof object li m 2 lo i m 2 ml m 2
lo
For transverse magnification, size means height For small one dimensional object:
and for longitudinal magnification size means ml (Transverse magnification)2
length of object / image. SUPERFICIAL MAGNIFICATION OR
For 1 dimensional object, there are two types AREAL MAGNIFICATION(mA):
of magnification
a) Transverse or Lateral magnification area of image
b) Longitudinal magnification. mA
area of object
a) Transverse or Lateral magnification:
M
A
h0
B P
B C hi F
A
u
v Overall magnification:
R
In case of more than one optical component,
height of image h1
m the image formed by first component will act
height of object h0
as an object for the second and image of
In the figure ABP and ABPare similar. So,
second acts as an object for third and so on,
hi v h v h v
i m i the product of all individual magnifications is
h0 u h0 u h0 u called over all magnifications.
b) Longitudinal magnification
I I I I
m0 1 2 n
O O1 O 2 On
m1 m 2 m n
1 1
A B Case (1):
P
A B When an object is placed with its plane
u1
u2 perpendicular to the principal axis.
v1
v2 In this case both dimensions height of the
If one dimensional object is placed along the object and width of the object are
principal axis then linear magnification is called perpendicular to the principal axis. Then
longitudinal magnification. h1 mh0 ; h0 height of object ;
Longitudinal magnification :
Wi mW0 ; W0 width of object
length of image v2 v1
ml Area of image :
length of object u2 u1 Aimage hi Wi mh0 mW0
Page 368
RAY OPTICS PART-1
Aimage m 2 Aobject u u Illustration 14: The focal length of concave
Case (2): mirror is 30cm. Find the position of the
When an object is placed with its plane parallel object in front of the mirror , so that the
to principal axis. The image of a rectangular image is three times the size of the object.
frame kept in front of a concave mirror as Solution: As the object is in front of the mirror it is
shown in the figure will be a rectangle. real and for real object the magnified image
formed by concave mirror can be inverted (i.e.
W 0 real) or erect (i.e virtual) , so there are two
h 0 possibilities.
f
a) If the image is inverted (i.e real) m
f u
hi mh0 (Transverse magnification)
Wi m 2 h0 (Longitudinal magnification for
small objects )
Then, area of image
Wi hi m 2W0 mh0
Area of image m 3 (area of object) 30
VOLUME MAGNIFICATION(mV): 3 u 40cm
30 u
Object must be at a distance 40cm in front of
volume of 3D image
mV the mirror (In between C and F)
volume of 3D object b) If the image is erect (i.e virtual)
For small cubical objects only, all dimensions
will be magnified equally because all dimensions f 30
are almost at the same distance from the mirror, m 3
hence final image is also a cube. f u 30 u u 20cm
Area of object is perpendicular to the principal
axis. Hence,
Area of image Ai m 2 A0
Length of image li m 2l0 then,
Page 369
RAY OPTICS PART-1
By principle of reversibility a convex mirror can form real and magnified image to a virtual object
which is within the focus and virtual images when virtual object is beyond the focus. i.e., the convex
mirror can form real and virtual images to virtual objects. A concave mirror with virtual object always
form real images.
If the given mirror breaks into pieces, each piece of that mirror has its own principal axis, but behaviour
is similar to that of main mirror with less intensity of image.
a) Concave mirror
Page 370
RAY OPTICS PART-1
b) Convex Mirror
Motion of Object in front of Spherical respect to mirror along the principal axis. Here
Mirror Along the Principal Axis negative sign indicates that the object and the
When the position of the object changes with image are always moving opposite to each
time on the principal axis relative to the mirror, other.
the image position also change with time Relation between object velocity and image
relative to the mirror. velocity given above is also valid for convex
mirror. In convex mirror speed of image slower
than the object whatever the position of the
object may be.
Above relation is not true in terms of
acceleration of object and image.
Motion of the object Transverse to the
Principal Axis:
If the object moves transverse to principal axis
then the image also moves transverse to
principal axis.
1 1 1
V0
v u f
Differentiating with respect to time, we get u
1 dv 1 du
2 . 2 . 0 (or) O h0 I
v dt u dt hi
C
v du v v
2 2
dv
. (or) iV
.V0
dt u dt u Vi
(or) Vi m 2V0 Consider the diagram. For a spherical mirror
Where Vi is velocity of the image with respect h
m i
ho
to mirror and V0 is velocity of the object with
Differentiating w.r.t time
Page 371
RAY OPTICS PART-1
dh i / dt 1 1 1
m or V1 mV0 (a) For all real image ;
dh 0 / dt v u f
Power of Curved Mirror : Every optical 1 1 1
; y=mx+c
instrument have power. It is the ability of an v u f
optical instrument to deviate the path of rays This is a straight line equation with slope -1.
incident on it. If the instrument converges the This is represented by the line AB.
rays parallel to principal axis, its power is said 1 1 1
to be positive and if it diverges its power is said (b) For virtual image, ;
to be negative. v u f
1 1 1
; y=mx+c
v u f
This is a straight line equation with slope +1.
This represents line BC.
F
(c) The graph between 1/v and 1/u to a convex
mirror as shown in figure (b).
Since convex mirror always form virtual image
Converging mirror to a real object.
P = +ve
(a) 1 1 1 1 1 1
; y=mx+c
v u f v u f
This is a straight line equation with slope +1.
U-V Graph for Curved Mirrors:
F
In case of concave mirror, the graph between
u and v is hyperbola as shown in figure.
v
Diverging mirror
P = -ve Plane mirror
(b)
For a mirror Power ‘P’ 2f Real image
1 100
P or P f
f metre f cm O
f 2f u
S.I unit of power is dioptre D m1
For concave mirror P is positive and for convex
mirror power is negative. Virtual image
1 1
, Graph for Curved Mirrors:
V U Figure (a)
1 1
The graph between and to a concave 1 1 1 1 uf
v u (a) For real image (or)
mirror is shown in figure (a) v u f v uf
uf
1 1 v
v v u f
1 1 1 1
f
A C (b) For virtual image
v u f
45° 45° 1 45° 1 1 f u fu
O 1 u or v
B u v fu f u
1 f (c) In case of convex mirror, the graph between u
f and v is a hyperbola as shown in figure (b)
Since convex mirror form only virtual images.
Figure - (a) Figure - (b)
1 1 1 1 1 1 fu
Since or v
v u f v u f uf
Page 372
RAY OPTICS PART-1
v
2f
f
O f 2f u
Figure (b)
m-v Graph for Spherical Mirrors:
1 1 1 1 1 1 1 1 1
In a spherical mirror: reduces to
v u f v u f f x2 f x1 f
v v v v Which on simplification gives x1 x2 f
1 or 1
2
u f u f
m m m (Newton’s Formula) ( f x1 x2 )
Illustration 15: The image of a real object in
1 a convex mirror is 4 cm from the mirror. If
v v the mirror has a radius of curvature of 24 cm,
v
f 1 f find the position of object and magnification.
Solution:
Fig.(a) Fig.(b) Fig.(c)
(a) Concave mirror: vf (4)(12)
If the object is real, for real image Now u 6cm
u=-ve, v=-ve, f=-ve, v f 4 12
v v The negative sign shows that the object is real
m 1or m 1 and it is placed in front of mirror.
f f
Graph as shown in figure (a) v (4) 2
For virtual image, u=-ve, v=+ve, f=-ve The magnification, m =
u 6 3
v
m 1, Graph as shown in figure (b) Illustration 16:A point light source lies on the
f principal axis of concave spherical mirror
(b) Convex mirror:
Since convex mirror always form virtual image with radius of curvature 160 cm. Its image
to a real object, u=-ve, v=+ve, f=+ve, appears to be back of the mirror at a
v distance of 70 cm from mirror. Determine
m 1, graph as shown in figure (c). the location of the light source.
f
From the above graph it is observed that for
v 0, m 1. i.e., when an object is very near 1 1 2
Solution: , Here v 70 cm ,
to pole of the mirror u 0 , then the curved u v R
mirror behaves like a plane mirror. 1 2 1
Newton’s Formula : R 160cm
u R v
In case of spherical mirror, if the object distance
1 2 1 15
x1 and image distance x2 are measured
u 160 cm 70 cm 560 cm
from focus instead of the pole of the mirror.
Then mirror formula reduces to a simple form 560
called the Newton’s formula. u cm 37.3cm
15
The image is at a distance of 37.3 cm in front
of the mirror.
Page 373
RAY OPTICS PART-1
Illustration 17: A point source of light is Solution: Since the image is virtual, erect and of a
located 20 cm in front of a convex mirror smaller size, the given mirror is ‘convex’
with f=15 cm. Determine the position and (concave mirror does not form an image with
nature of the image point. the said description).
1 1 1 R
Solution: Given R 60 cm f 30 cm
u v f 2
Transverse magnification,
Here u 20 cm, f 15 cm I 6 3 v 3
m Further m
1 1 1 1 1 35 O 10 5 u 5
v f u 15cm 20cm 300cm 3u
v
5
1 7
1 1 1 5 1 1
v 60 cm ; v 8.6 cm nearly Using ,
v u f 3u u 30
Also v is positive, the image is located behind 5 3 1
the mirror, hence the image is virtual. u 20 cm
3u 30
v 3 Thus the object is at a distance 20 cm (from
m the pole) in front of the mirror.
u 7
Illustration 20: A rod of length 10cm lies along
Since m is positive, image is erect. Also m 1 , the principal axis of a concave mirror of
focal length 10cm in such a way that the
hence the image is diminished. end closer to the pole is 20cm away from
So, a virtual, erect and diminished image is it. Find the length of the image.
formed. Solution:
Illustration 18: An object is 30.0 cm from a
spherical mirror, along the central axis.
The absolute value of lateral B A
magnification is 1/2. The image P
produced is inverted. What is the focal
length of the mirror?
Solution: Given that image is inverted, so it is real.
Hence u and v both are negative. Magnification 10 cm 20 cm
is 1/2, therefore, Since point A is centre of curvature, image of A
u is formed on A itself
v , given, u 30 cm, v 15cm Image of B :
2
1 1 1 u 30cm , f 10cm, v ?
Using the mirror formula, 1 1 1 1 1 1
v u f
1 1 1 1 v u f v 30 10
We have,
f 15 30 10 1 1 1 3 1
; v 15cm
f 10 cm v 10 30 30
Since focal length is negative the given mirror is
concave.
Illustration 19: An object of length 10 cm is A B
P
placed at right angles to the principal axis
of a mirror of radius of curvature 60 cm
such that its image is virtual, erect and has
15 cm
a length 6cm. What kind of mirror is it and 20 cm
also determine the position of the object? Length of image A ' B ' 20 15 5cm
Page 374
RAY OPTICS PART-1
Illustration 21: A concave mirror of focal
length 10cm and a convex mirror of focal
length 15cm are placed facing each other
40cm apart. A point object is placed X X
O
between the mirrors on their common axis Solution: I P
and 15cm from the concave mirror. Find
the position and nature of the image
produced by the successive reflections, first
at concave mirror and then at convex
mirror. 22.5 cm
1
f =+15cm f= -10cm
Let object be placed at distance x from plane
mirror as shown. The image is formed at
distance (22.5 x) from concave mirror..
I2 P2
I1 O P1 For concave mirror
u (22.5 x ), v (22.5 x ), f 10 cm
40cm
15cm 1 1 1 1 1 1
,
Solution: v u f (22.5 x ) (22.5 x ) 10
Image by concave mirror: The direction of
incident rays is from left to right. 45 1
2 2
, x 7.5 cm
u 15cm, f 10cm, v ? (22.5 x ) 10
Illustration 23: An object is placed infront of a
1 1 1 1 1 1
convex mirror at a distance of 50 cm. A
v u f v 15 10 plane mirror is introduced covering the
1 1 1 3 2 1 lower half of the convex mirror. If the
, distance between the object and the plane
v 10 15 30 30
mirror is 30 cm, if it is found that there is
v 30 cm no parallax between the images formed by
Image of O is formed at I1 the two mirrors. What is the radius of
Image by convex mirror : After every reflection, curvature of the convex mirror?
direction of incident rays is reversed, now it is
from right to left. Now I1 servers as an object
for convex mirror.
50 cm
u ' 10 cm, f ' 15 cm, v ' ?
1 1 1 1 1 1 20 cm
M
v' u' f ' v ' 10 15 P I F
Solution: 30 cm
1 1 1 23 1
, v ' 6 cm
v ' 15 10 30 6
30 cm
Illustration 22: A plane mirror is placed 22.5
cm in front of a concave mirror of focal As shown in figure the plane mirror will form
length 10cm. Find where an object can be erect and virtual image of same size at a distance
placed between the two mirrors, so that the of 30 cm behind it. So the distance of image
first image in both the mirrors coincides. formed by the plane mirror from convex mirror
will be PI = MI - MP. But as MI = MO,
Page 375
RAY OPTICS PART-1
PI= MO-MP = 30-20 = 10 cm. v 60
Now as this image coincides with the image v 60m m 3
u 20
formed by convex mirror, therefore for convex Shortcut solution :
mirror,
u 50 cm; v 10 cm f 30
1 1 1 m 3
50 f u (30) (20)
So , i.e., f 12.5 cm
10 50 f 4
So R 2f 212.5 25 cm v mu (3)(20) 60cm
Illustration 24: A convex mirror of focal length Since mirror is at rest (given that) :
1m and a plane mirror are facing each
other, if the distance between them is 1m, 2
60
their images are separated by VIM || (5) 45 m / s
20
Solution:
P P Q
VIG VIM VMG
1m 1m 1
m VIG 45 0 45m / s
2
Image of convex mirror by plane mirror is That means image approaches the mirror
formed at P ' . with velocity 45 m/s.
Image of plane mirror by convex mirror
1 1 1 1 Method of finding co-ordinates of
v m is formed at Q image of a point object, if its
v 1 2 2
1 coordinates of object are known:
P ' Q 1 1 2.5m Suppose coordinates of a point object
2
Illustration 25:Find the velocity of image x0 , y0 with respect to coordinate axis as
w.r.t. ground. Given velocity of object shown in figure. The coordinates of images
w.r.t. ground is 5 m/s towards the mirror.
xi , yi can be obtained using
V= 5m/s
1 1 1 1 1 1 yi xi fy0
m yi
v u f v 20 30 y0 x0 f x0
Page 376
RAY OPTICS PART-1
Illustration 26: A reflecting surface is
represented by the equation x 2 y 2 a 2 .
A ray travelling in negative x-direction is
directed towards positive y-direction after
reflection from the surface at point P.
Then, find the co-ordinates of point P.
Solution: The ray diagram is as shown.
||||||| 45°
|||||||| |||||||||||
|| ||||
45°
|
P
||||
|
||||||||||||||||||||||||
a
||||||
||
y
||||||||||||||||
45° x
x
||| ||||
||| ||
|||| ||||
||||||
|||||||||||||||||||
a a a a
x and y ; P ,
2 2 2 2
30 cm
f = 20 cm
Solution: V0 = Velocity of object = 9 iˆ 1 2 ˆj m / s
Vm = Velocity of mirror = 2iˆ m / s
f 20
m 2
f u 20 (30)
Page 377
RAY OPTICS PART – 2
Page 378
RAY OPTICS PART – 2
r . If i is changed slightly by i then
corresponding change in r is
1 cos i
r i
cos r
r
1 sin i 2 sin r1 ; 2 sin r1 3 sin r2
3 sin r2 4 sin r3 ; It is clear that air
i
1 sin i 2 sin r1 3 sin r2 4 sin r3
(or) sin i constant
When a light travels from optically Condition for no refraction :
rarer medium to optically denser (a) When an incident ray strikes normally at the
medium obliquely: point of incidence, it does not deviates from its
incident light ray path. i.e., it suffers no deviation.
In this case angle of incidence (i) and angle of
optically rarer
refraction(r)are equal and i r 0.
i medium (air)
O
Optically rarer Optically rarer
medium(air) medium(air)
optically denser r = i - r
medium(glass)
refracted
light ray
Page 379
RAY OPTICS PART – 2
It has no units and no dimensions. From snell’s law 1 sin i 2 sin r
From electromagnetic theory if 0 and 0 are the sin i1 sin i2 sin i sin i
permittivity and permeability of free space, or 1 2
and are the permittivity and permeability V1 V2
of the given medium then refractive index of Illustration 1:The refractive index of glass
the given media with respect to water is 9/8. If the
1 velocity and wavelength of light in glass
C 0 0 are 2 108 m/s and 4000 A 0 respectively,,
n r r find the velocity and wavelength of light
V 1 0 0 in water.
where r and r are the relative permittivity and g v w 9 vw
Solution: w g v 8 2 108 ;
permeability of the given medium. w g
Page 380
RAY OPTICS PART – 2
Illustration 4: A ray of light is incident at the D
glass-water interface at an angle i as shown 2 3 4
1
in figure, it emerges finally parallel to the
surface of water, then the value of A B C
g would be
Solution: Applying Snell’s law at B and C,
Air
r 4 1 2 3 4
w =
3
r
Water
Glass D
iC
iB B C
i
A
Page 381
RAY OPTICS PART – 2
AB = real path or geometrical path Illustration 9: Find the thickness of a
1 1 = optical path
AB transparent plastic plate which will
If light travels a path of length ‘d’ in a medium produce a change in optical path equal to
d
the wavelength of the light passing
at speed v, the time taken by it will be t = .
v through it normally. The refractive index
So optical path length, of the plastic plate is .
d c Solution: When light travel a distance x in a medium
x c t c d as
v v of refractive index , its optical path = x
Therefore optical path is times the Change in optical path = x x 1 x .
geometrical path. As for all media 1, optical
path length is always greater than actual path This is to be equal to . But 1 x
length.
The thickness of the plate x
If in a given time t, light has same optical path 1
length in different media, and if light travels a Illustration 10: Two parallel rays are travelling in
distance d1 in a medium of refractive index 1
4
and a distance d 2 in a medium of refractive a medium of refractive index 1 . One of
3
index 2 in same time t, then 1d1 2 d 2 .
the rays passes through a parallel glass slab of
The difference in distance travelled by light in
vacuum and in a medium in the same interval of 3
time is called optical path difference due to that thickness t and refractive index 2 . The
2
medium.
path difference between the two rays due to
x A ' B ' AB d d x 1 d the glass slab will be
A slab of thickness d and refractive index is
kept in a medium of refractive index ' . Solution: x 2 1 t 3 / 2 1 t t
If the two rays parallel to each other pass 1 4 / 3 8
through such a system with one ray passing Illustration 11:Consider slabs of three media
through the slab, then path difference between A, B and C arranged as shown in figure.
R.I. of A is 1.5 and that of C is 1.4. If the
two rays due to slab will be x 1 d . number of waves in A is equal to the
' number of waves in the combination of B
2 and C then refractive index of B is:
The optical phase change, (optical path
A
Monochromatic
difference) light
Illustration 8: The optical path of a B C
monochromatic light is the same if it goes x 2x
through 4.00 m of glass or 4.50 m of a x A xB xC
liquid. If the refractive index of glass is Solution: N A N B N C ;
1.5, what is the refractive index of the
A B C
liquid? x A A x B B xC C
Solution: When light travels a distance ‘x’ in a 0 0 0
medium of refractive index , the optical path
is x 3 x 1.5 x B 2 x 1.4 ; B 1.7
Given 1 x1 2 x2 1.5 4.00 2 4.50
Principle of Reversibility of Light
According to principle of reversibility, if a ray
1.5 4.00 of light travels from X to Z along a certain path,
2 1.333 it will follow exactly the same path, while
4.50 travelling from Z to X. In other words the path
of light is reversible.
Page 382
RAY OPTICS PART – 2
N Illustration 12: A light ray is incident normally
X Normal
on a glass slab of thickness ‘t’ and
Medium A
refractive index ‘ ’as shown in the figure.
(Rarer medium) Then find time taken by the light ray to
1 travel through the slab.
P Q
Y
Medium B 2
(Denser medium)
90 t
Solution:
Z Plane mirror
M
From the figure distance travelled by the light
1 ray through the slab is ‘t’
a b
b a distance travelled
Thus, the refractive index of medium ‘b’ with Velocity of light in glass
respect to ‘a’ is equal to the reciprocal of time (t')
refractive index of medium ‘a’ with respect to c t t
t'
medium ‘b’. t' c
In case of first medium. Illustration 13: A light ray is incident on a
1 sin i 2 sin r ; 1 sin i 2 sin i d1 plane glass slab of thickness ‘t’ at an angle
of incidence ‘i’ as shown in the figure. If
1 1 1 ‘ ’is the refractive index of glass, then
sin i d1 sin i ; i d1 sin sin i find time taken by the light ray to travel
2 2 through the slab.
d1 i sin 1 1 sin i
2 i
N O
X
1
i Solution:
Y t
r d
r d1
2
As shown in the above figure distance travelled
Z by the light ray through the slab is ‘d’. From
In case of second medium
then figure
2 sin r 1 sin i ; 2 sin i d 2 sin i
1
t t
2 cos r d
1 d cos r
sin i d 2 sin i ; i d2 sin 1 1 sin i Velocity of light in glass
2 2 Distance travelled through the glass
=
d2 i sin 1 1 sin i time(t')
2 c d d
N t'
X t' c
1
d2 i t 2t
t'
Y cos r c c 2 sin 2 i
r
Illustration 14: Light of wavelength 4500 A0 in
2
air is incident on a plane boundary
between air and another medium at an
Z
angle 300 with the plane boundary. As it
From (1) and (2) d1 d 2 deviation in two
enters from air into the other medium, it
media remains same
deviates by 150 towards the normal. Find
Page 383
RAY OPTICS PART – 2
refractive index of the medium and also Illustration 16: The x-z plane separates two
the wavelength of given light in the media A and B of refractive indices
medium. 1 1.5 and 2 2 . A ray of light travels
Solution: from A to B. Its directions in the two media
are given by unit vectors u1 aiˆ bjˆ and
i a
o u2 ciˆ djˆ . Then find the ratio of .
30 air c
-y
medium
r' 15
o
i
ai b j
(i)
Angle of incidence i 900 300 600. As x-z plane
the ray bends towards the normal, it deviates Solution: r ci d j
by an angle i r 150 (given)
r 450 Applying Snell’s law y( j)
air sin i med sin r ; a a
tan i so sin i
1 sin 600 sin 450 b a 2 b2
In terms of wavelengths, c c
and tan r ,sin r
air 4500 d c2 d 2
1.5 or med air
med 1 1.5 1 sin i 2 sin r ;
0
med 3674 A 3 a c
2
Illustration 15: Monochromatic light falls at 2 a 2 b2 2
c d
2
an angle of incidence ‘i’ on a slab of a But as aiˆ bjˆ and ciˆ djˆ are unit vectors so
transparent material. Refractive index of
this material being ‘ ’for the given light, a 2 b2 c2 d 2 1 ;
what should be the relation between i and 3 a 4
so that the reflected and the refracted Hence a 2c, so
rays are mutually perpendicular? 2 c 3
Solution: In the given figure let r is the angle of Illustration 17: A ray of light is incident on the
surface of a spherical glass paper-weight
reflection and r ' is the angle of refraction. making an angle with the normal and
According to the given condition, considering
the reflected and the refracted rays to be is refracted in the medium at an angle .
perpendicular to each other, Calculate the deviation.
Solution: Deviation means the angle through which
i r the incident ray is turned in emerging from the
medium. In Figure ifAB and DE are the incident
o
and emergent rays respectively, the deviation
r 90 will be .
C
From the figure r 90 r 180
0 0
B
D
So, r 900 r A
E
O
r 900 i [i = r, by law of reflection]
According to Snell’s law, 1 sin i sin r
sin i sin 900 i Now as at B ; i and r
sin i cos i, tan i i tan 1 So from Snell’s law, 1sin sin ....(1)
Now from geometry of figure at D, i
Page 384
RAY OPTICS PART – 2
So sin 1sin .....(2) sin i AO
Comparing Eqs. (1) and (2) ; According to Snell’ law
sin r AI
Now as in a triangle exterior angle is the sum of sin i sin i
remaining two interior angles, in BCD , 1
or 1
sin r sin r
2
AO AO 1 1 x
Illustration 18: A ray of light falls on a AI
transparent sphere with centre at C as 1 AI
shown in figure. The ray emerges from the The distance of image AI is called apparent
sphere parallel to line AB. The refractive depth or apparent distance. The apparent
index of the sphere is 1 xreal
depth xapp is given by x
i.e., app
C
1x
r The apparent shift OI AO AI x
A
o
B
60 1
Hence the apparent shift OI x 1
Solution: Deviation by a sphere is 2 (i - r)
Case(2) : If rarer medium is air 1 1
Here, deviation 600 2 (i-r) or i-r = 30 0
x
r = i 300 600 300 300 xapp real y mx
sin i sin 60 0
3 1
sin r sin 30 0 apparent shift OI 1 x
Apparent Depth:
Case(1) : Object in denser medium and Diagram shows variation of apparent depth with
observer in rarer medium. real depth of the object.
When object ‘O’ is placed at a distance ‘x’
from A in denser medium of refractive index
app depth
Page 385
RAY OPTICS PART – 2
V u x t
x 2t
air
I1 I3 I2 (O3)
O1 (x+t)
h I2 (x+2t)
O2 2 1 2 1
; R
Apparent depth of object due to composite slab V u R
1 1
x1 1 O;
V u 2t V x 2t
x2 2
V
x 2t
x3 3
O
x x x x 2t
is S 1 2 3
1 2 3
P P
If there are ‘n’ number of parallel slabs which
are may be in contact or may not with different air o
x - t
refractive indices are placed between the x
observer and the object, then the total apparent t
shift 2 1 2 1
v u R
1 1 1 1
S 1
x1 1
1
x 2 1
2
xn
n
0 R V1 x
V1 x
Where x1,x2 ---- xn are the thicknesses of the
Here, I1 behaves like object for pole P2 (Plane
slabs and 1 , 2 ....n are the corresponding
mirror): for this I 2 is the image from refraction
refractive indices. through the plane mirror.
Concept of Slab when one surface is
Silvered : Again for pole P1 , I 2 behaves as an object, then
Applying formula of spherical surface for pole apply
P1 2 1 2 1
; (for pole P1 )
V u R
1
0 R
I1 O V3 x 2t
1– x – p1 V3
x 2t
2 1 2 1
Object in rarer medium and observer
V u R in denser medium :
R and 1 1 and 2 u An object is in a rarer medium (air) at a distance
1 1 ‘y’ from boundary and an observer is near to
O; ; V x normal in a denser medium of refractive index
V x V x ‘ ’.Since 1 image distance in rarer
I2(O2) medium is more than object distance in the
(x+t) medium.
(x+t)
I2
x
Page 386
RAY OPTICS PART – 2
1
rarer
shift x 1
real
medium () Note: When convergent beam of rays passing from
rarer to denser medium as shown in the figure,
real image is formed in denser medium which is
far to boundary than that of virtual object.
shift real 1 x
rarer
A
When i and r are small Denser
AB AB
sin i tan i ,sin r tan r
AO AI O'
y I
Therefore apparent height of object (AI)
1 x
real height of object (AO) AO' x, AI =
rel
Apparent shift y AI AO Application Normal shift due to glass
slab :When an object is placed on normal line
to the boundary of a slab whose thickness is ‘t’
i.e. yapp 1 yreal
and refractive index is . On observing this
object from the other side of the slab, due to
Apparent shift y 1 1 y . refraction, the image of this object shift on the
normal line. This shift value is called normal shift.
If rarer medium air then 1 1. Apparent This shift is towards the slab, if the slab is denser
height relative to the surroundings and the shift is away
from the slab, if the slab is rarer relative to the
AI y y mx surroundings. Then the normal shift
Apparent shift 1 y 1 1
OI 1 t 1 t
Diagram shows variation of apparent height with rel
real height of the object. 1
If l 1, normal shift OI 1 t .
app height
Real height
slope tan 1
' (a) (b)
Note: When convergent beam of rays passing from (a) shifts towards the slab
denser to rarer medium as shown in the figure, (b) shift is away from the slab
real image is formed in rarer medium which is Relation between the velocities of
nearer to boundary than that of virtual object. object and image :
(a) The figure shows an object O in denser
O' medium is moving towards the plane
1x
I rarer boundary of a rarer medium. xap
rarer
medium ()
A Denser
x
AO' x, AI =
rel
Page 387
RAY OPTICS PART – 2
Differentiating the above equation with H
1V Apparent depth of point Q =
respective to time, we get Vap
Apparent distance between points P and Q = u
If the speed of the object is v. Then the speed
v1 H H h h
of the image will be . – =
To an observer in rarer medium and object in Image appears to be formed behind the
denser medium, the image appears to be closer
but moving slowly. In same direction of object. h
mirror at I at a distance of v =
1 V
If rarer medium is air 1 Vi The apparent distance between object and
rarer medium() 2h
image is x = v+u = .
(b) I Illustration 20: A concave mirror is placed at
the bottom of an empty tank with face
The figure shows an object O in rarer upwards and axis vertical. When sunlight
medium is moving towards the plane falls normally on the mirror, it is focussed
x at distance of 32cm from the mirror. If the
boundary of a denser medium. xap 1 4
upto a
tank filled with water
Differentiating the above equation with 3
V height of 20cm, then the sunlight will now
respective to time, we get Vap 1 get focussed at
(a) 16 cm above water level
To an observer in the denser medium, the object (b) 9 cm above water level
in rarer medium appears to be more distant and (c) 24 cm below water level
moving faster. In same directio of object . If (d) 9 cm below water level
Solution: Sun is at infinity i.e, u so from mirror
rarer medium is air then Vi V formula we have
Illustration 19: A plane mirror is placed at the
F F O
bottom of a tank containing a liquid of
I
refractive index . P is a small object at a 12cm
Page 388
RAY OPTICS PART – 2
the water level. The focal length of the M M'
mirror is
32cm 4cm
32cm
15cm
30cm 28cm
=1.33 25cm
33.25cm I
t = 6cm
O
Position of
Solution: Distance of object from mirror shifted mirror
Again, after reflection from plane mirror, light
33.25 rays pass through the glass slab and shifted by
15 3 39.93cm
4 2 cm to the right.
Distance of image from mirror Final image is formed at 30 cm behind the
25 1 1 1 plane mirror.
15 3 33.75 For mirror, Illustration 24: An observer can see through
4 v u f
a pin-hole the top end of a thin rod of
1 1 1 height h, placed as shown in figure. The
f 18.3cm
33.75 39.93 f beaker height 3h and its radius h. When
the beaker is filled with a liquid upto a
Illustration 22: In a tank, a 4cm thick layer of height 2h, he can see the lower end of the
rod. Find the refractive index of the liquid.
4
water floats on a 6 cm thick layer
3
of an organic liquid 1.5 . Viewing at
normal incidence, how far below the water
surface does bottom of tank appear to be? 3h
h1 h2 6 4
Solution: d AP 1.5 4 / 3 7 cm h
1 2
Illustration 23: An object is placed in front of 2h
Solution: QP = QR = 2h
a slab 1.5 of thickness 6 cm at a P
distance 28 cm from it. Other face of the
i
slab is silvered. Find the position of image. h i
Solution: S
2h
M r
Silvered face Surface 1 i
surface 2 1=1 2h R
T
Q
M N
K
O 2h
28cm
2=1.5 i 450 ; ST = RT = h = KM = MN
2
h 1
6cm KS = h 2 2h h 5 ; sin r
First, while light rays from object O passes h 5 5
through the glass slab of thickness t, its image sin i sin 450 5
1
sin r 1/ 5 2
is shifted by t 1 2cm towards left. This
Illustration 25: A person looking through a
acts as object for plane mirror which is at a telescope focuses the lens at a point on the
distance of 6+26=32 cm, whose image is edge of the bottom of an empty cylindrical
formed at 32 cm behind the mirror. vessel. Next he fills the entire vessel with
a liquid of refractive index , without
disturbing the telescope. Now, he observes
Page 389
RAY OPTICS PART – 2
the mid point of the bottom of the vessel. O
Determine the radius to depth ratio of the
vessel.
Solution:After the vessel is filled with the liquid, i O' i
A D
light ray starting from the mid point O of the
bottom of the vessel as OA, after refraction
goes along CA.
r B E
D A i i
i
r
F
H C
Illustration 27: An observer looks at an object
C B
O
R
kept at a distance 30 cm in air. If a
Applying Snell’s law, for the refraction at rectangular glass plate 1.5 is placed
sin i 1
point A, ... (i) between the observer and the object with
sin r its thickness along the line of observation,
OB R the object appears to the observer to be at
From AOB,sin i OA a distance 25 cm. Find the thickness of
R 2 H2
glass plate. Position of the glass plate is
CB 2R now shifted (i) from object towards observer
and from ACB,sin r CA
4R 2 H 2 (ii) from observer towards the object. How
From Eq(i) does it change the apparent position of the
2 R H2 2 2
4R 4H 2 object as seen by the observer?
2 2 Solution: As a glass plate is placed between the
4R 2 H 2 4R H
observer and the object, the object appears to
3H 2 be at a distance 25 cm in-stead of 30 cm. It
or 2 1
4R 2 H 2 implies that normal shift due to refraction at the
2 3H 2 2 3H 2 glass plate is 30 - 25 = 5 cm.
i.e., 1 or 1 2
4R 2 H 2 4R H 2 1
Illustration 26: A diverging beam of light from Normal shift t 1
a point source S having divergence angle
falls symmetrically on a glass slab as 1 1
shown. The angles of incidence of the two Hence t 1 5 t 1 5;
extreme rays are equal. If the thickness of 1.5
the glass slab is t and its refractive index t = 15 cm
is n, then the divergence angle of the Since the plate has given values of refractive
emergent beam is index (1.5) and thickness (15 cm), if its position
S is shifted, it will not change the values of normal
i i
1
shift t 1 . It implies that the object will
appear to be at the same distance 25 cm and
n t its apparent position remains the same.
Illustration 28: An air bubble is trapped inside
Solution: Divergence angle will remain unchanged a glass cube of edge 30 cm. Looking
because in case of a glass slab every emergent through the face ABEH, the bubble
ray is parallel to the incident ray. However, the appears to be at normal distance 12 cm
rays are displaced slightly towards outer side. from this face and when seen from the
(In the figure OA BC and OD EF ) opposite face CDGF, it appears to be at
normal distance 8 cm from CDGF. Find
Page 390
RAY OPTICS PART – 2
refractive index of glass and also the actual OM OM t
position of the bubble. But cos r , OP ....(2)
B
OP cos r cos r
C
from (1) and (2)
A D sin i r
x t
12cm 8cm
cos r
E F For small angle of i and r sin i i,
H G sin r and cos r 1 ; sin i r i r
Solution: Let the actual distance of the bubble from
the face ABEH is ‘x’ then its actual distance r
x t i r ti 1
from the face CDGF is 30 - x. i
Actual dis tan ce of bubble 1
But Apparent dis tan ce of bubble Lateral shift x 1 ti
The apparent distance of bubble from refracting
t
x x
surface ABEH ; 12 .....(1)
x
The apparent distance of bubble from refracting (Lateral shift)
30 x 30 x
surface CDGF ; 8 .....(2) O /2
i (Angle incidence)
x 30 x Fig. Plot of lateral shift versus angle of incidence
Adding Eqs. (1) and (2) 20 ,
x x Note: When i (grazing incidence)
2
From Eq. (1); 12 or =12 ; so x = 18 cm
1.5 t
This actual position of bubble is at normal xmax sin c t r c
distance 18 cm from face ABEH or at a cos r 2
distance 12 cm from face CDGF. Examples of Refraction :Visibility of two
Lateral Shift : Consider a ray AO incident on images of an object :
the slab at an angle of incidence ‘i’ and passing When an object in a glass container filled with
through a slab of thickness ‘t’. After two a liquid is viewed from outside at a level higher
refractions at the boundary, the ray emerges
parallel to the incident ray. The perpendicular than of liquid, there will be two images one due
distance between incident ray direction and to refraction through liquid and another due to
emergent ray direction is called lateral shift or refraction through glass.
lateral displacement (x) Twinkling of stars : Due to fluctuations in
Ai
O refractive index of atmosphere, the refraction
r i-r
becomes irregular and the light some times
t Q reaches the eye and some times it does not.
r This gives rise to twinkling of stars.
x
M P i Oval shape of Sun in the morning and
i
evening :- In the morning and evening, the
From the figure, the distance PQ is called lateral sun is at the horizon and refractive index in the
displacement (or) lateral shift atmosphere of earth decreases with height.
PQ Therfore unequal refracted rays diverge from
From the triangle PQO, sin i r the lower and upper ends of the sun. Rays from
OP
the lower edge have to go through a greater
PQ x OP sin i r
thickness of air than rays from the upper edge.
x = OP sin i r .....(1) So the vertical diameter appears to be dimin-
ished in size. Where as horizontal diameter re-
mains unaltered.
Page 391
RAY OPTICS PART – 2
Visibility of two images of an object:- When
an object in a glass container filled with a liquid i
is viewed from outside at a level than of liquid, A
there will be two images one due to refraction
through liquid and another due to refracrion t r i-r
through glass D
r E
Twinkling of stars:- Due to fluctuations in re-
fractive index if atomsphere, the refraction be- B C
comes irregular and the light some times reaches
the eye and some times it does not. This gives
sin i sin 600
rise to twinkling of stars By Snell’s law, , 3
Bending linear object in liquid:- If a linear sin r sin r
object is dipped in a liquid inclined to the sur- 3 1
3 ,sin r ; Hence, r 300
face of liquid. Actual depth will be different for 2 sin r 2
its different points and so apparent depth. Due t
Lateral shift sin i r
to this object appears to bending cos r
Duration of sun’s visibility:- In absence of at- sin 300 4
mosphere the sun will be visible for its posi- 4 0
4 tan 300
cos 30 3
tions from M to E as shown in figure
Critical Angle and total Internal Reflection :
(A). However, in presence of atmosphere (in Critical Angle: It is the angle of incidence in
which decreases with height) due to the phe- denser medium at which the angle of refraction
nomenon of refraction, the sun will become vis- in rarer medium is 900.
ible even when it is below the horizon and will Consider a point object O placed in an optically
remain visible for some time even when it goes denser medium as shown in the figure. Rays of
light travel from O in all possible directions.
below the horizon as shown in figure
(1) When light is refracted at the surface into
This results in increases in duration for which the rarer medium, it bends away from the
the sun is visible. It is estimated that due to this normal.
effect the period of visibility of the sun increases
by 2 minutes in the morninig an d2 minutes the Due to Snell’s law as the angle of incidence
evening. increases, the angle of refraction also increases
till for a certain angle of incidence, the angle of
E Horizon M refraction is 900 and light is refracted along the
surface separating the two media. The
(Evening) O (Morning)
corresponding angle of incidence is called the
Earth
critical angle C .
(2) When light is incidence at any point beyond
S
O >c
P, that is when the angle of incidence is greater
S
Sun than the critical angle i C , then no light is
Illustration 29: A parallel sides glass slab of refracted, and the entire incident light is reflected
thickness 4 cm is made of a material of into the same medium. This phenomenon is
refractive index 3 . When light is known as the total internal reflection.
incident on one of the parallel faces at an
angle of 600 , it emerges from the other
parallel face. Find the lateral displacement
of the emergent beam.
Solution:
Page 392
RAY OPTICS PART – 2
In case of total internal reflection as all (100%)
Expression for criticalangle C or C : incident light is reflected back into the same
According to Snell’s law, at critical angle of medium i.e. there is no loss of intensity. This is
incidence why images formed by total internal reflection
are much brighter than that formed by mirrors
and lenses.
D sin C R sin 900 or sin C R
D Image due to total internal reflection is real,
lateral and inverted with respect to object.
R VD D
sin C Deviation of light under total internal
D VR R reflection: The figure shows a light ray
1 travelling from denser to rarer medium at an
For R 1 , D
s in C angle i, less than the critical angle c .
N
Condition for total internal reflection :
For total internal reflection to take place light
must be propagating from denser to rarer rarer (air)
medium.
Ex: Ray from water to air, glass to water. r
Total internal reflection will take place only if
angle of incidence is greater than critical angle. a) Denser()
C )
i.e.
i(<
i C where C sin 1 R
D The deviation of the light ray is given by
Critical angle C depends on nature of pair of
r i
R Since sin i sin r , therefore
media. Greater ratio greater will be the
critical angle
D
sin 1 sin i i
For glass - air : This is a non linear equation. The maximum
3 2 value of ‘ ’ occurs when i c , and
D ; R 1; c sin 1 ;c 420
2 3
max c
For water - air : 2
4 3 If the light incident at an angle i c , as shown
D ; R 1; c sin 1 ; c 490
3 4 in the figure (b) then the angle of deviation is
For glass - water : given by 2i. The maximum value of
3 4 8
D ; R ; c sin 1 ;c 630
occurs when i c and is equal to
2 3 9 max 2 c .
For diamond - air :
1
D 2.5; R 1; c sin 1 ; c 24
0
2.5
For a given pair of media, critical angle depends i
on wavelength of light used i.e greater is the air
wavelength of light, lesser will be the value Medium ()
i i
1
of as and so greater will be the
critical angle. This is why critical angle is
maximum for red and minimum for violet.
Generally, the critical angle of a medium is The variation of ‘ ’ with the angle of incidence
defined for mean colour i.e yellow.
‘i’ is plotted in figure.
Page 393
RAY OPTICS PART – 2
in cold regions particularly in the vicinity of the
cold surface of sea or of a lake. Light rays
C
starting from an object S (say a ship) are curved
downward and on entering the eye the rays
appear to come from S , thus giving an
C
2
impression that the ship is floating in air.
i
C
Some exampels of total internal S'
reflection:
1) An air bubble in water shines due to TIR at its
outer surface when light propogates from wa-
ter to air medium for i c
2) The faces of a piece of diamond are so cut that E S
light entering into it suffers total internal reflec-
tion repeatedly at different faces and come our Field of vision of fish : - A fish at a depth
through one or two faces only. So these faces ‘h’ from the surface of water of refractive index
appears very glittering which explains the bril- can see the outer world through an inverted
liance of diamonds cone with
3) Mirage:
N N
c r c
Refractive index
h
decreasing
c c
Vertex angle = 2C
Radius of the circular base of the cone formed
O hot summer days, the air near the ground on surface of water is given by
becomes hotter than the air at higher levels. The h
refractive index of air decreases with its den- r h tan C
sity. Hotte air is less , and has smaller refractive 2 1
index than tha cooler air. If the air is still, then
opticla density at different layers of air increases 1 1
sin C and tan C
with height. As a result, light from a tall object 2 1
such as tree, passes the medium whose refrac-
tive index decreases towards the ground. Then 3h
a ray of light from the object successively bends For water 2C 980 and r
7
away from the normal and under goes total in- 2
ternal reflection if the angle of incidence for the h
air near the ground exceeds the ciritical angle. d) Area of the base : A 2
1
This is as shown in figure. To a distant observer, Total Internal reflection in Prisms:
the light appears to be coming from somewhere The critical angle of ordinary glass is very nearly
below the ground. Such inverted imgaes of dis-
tant tall objects cause an optical illusion to the 420 . If light is incident inside a prism at an angle
observer. This phenimenon is called mirage. greater than 420 , then the light will be totally
Looming : This effect occurs when the internally reflected. This is achieved by taking
density of air decreases much more rapidly with a right angled prism ( 900 ) so that the other
increasing height than it does under normal angles of the prism are 450 each.
conditions. This situation sometimes happens
Page 394
RAY OPTICS PART – 2
For Deviation through 900 : fibre is bent or twisted in any form. And there
is almost no loss of light through the sides of
45° the fibre or due to absorption by the core.
Light ray rotated (cladding)
through 90°
45° (core)
45°
The only condition is that angle of incidence of
45° light must be greater than the critical angle for
the fibre material w.r.t its coating.
Formula for maximum lunching angle:
For Deviation through 1800 : From the figure, from snell’s law
sin i
1 ; sin i sin r .....(1)
sinr1 1 1
Page 395
RAY OPTICS PART – 2
on the surface of the water that produces an
(rarer) angle of incidence C is a circle of radius r.
R
i r
r=90o
o A D B
90
c
r1
C C
(Denser) h
D
Solution: Figure shows incidence of a ray at the C
rarer-denser boundary such that reflected and
refracted rays are mutually perpendicular.
i.e., r 900 r1 1800 . or S
r 90 r 90 i [r=i, law of reflection]
1 0 0 r
In the DBS, tan C ;
Apply Snell’s law at the boundary, h
R sin i D sin r1 h
r h tan C; r
D 2 1
R sini D sin 900 i D cosi or tani --- (1) The area of the surface through which the light
R
passes,
1 1 2
sin c R ---- (2) 2 0.20
R D D / R D A r 2
0.1005m 2
1.5 1
Using equation (1), Illustration 33: An optical fibre is made of
1 glass fibre of refractive index 1.68. The
sin c cot i c sin 1 cot i outer coating of the glass fibre is made of
tan i
material of refractive index 1.44. What is
Illustration 31: A ray of light travelling in a the range of angles of the incident rays
transparent medium falls on a surface with the axis of the pipe for which total
separating the medium from air at an angle internal reflection inside the optical fibre
of incidence 450 . The ray undergoes total takes place?
internal reflection. If is the refractive Solution: We know that maximum launching
angle in case of optical fibre as
index of the medium with respect to air,
select the possible value(s) of . i max sin 1 12 22 1 2
Solution: For total internal reflection to occur,
i critical angle, C or sin i sin C or i max sin 1 2
1.68 1.44
2
1 1 1 i max sin 1 0.8653 590551
sin 450 or or 2 or 1.414
2 This is the maximum value of i. All light rays
Illustration 32: A liquid of refractive index 1.5 with angle of incidence between 00 and 590551
is poured into a cylindrical jar of radius will undergo total internal reflection.
20 cm upto a height of 20cm. A small bulb Illustration 34: Light is incident at an angle
is lighted at the centre of the bottom of the on one planar end of a transparent
jar. Find the area of the liquid surface cylindrical rod of refractive index so that
through which the light of the bulb passes the light entering the rod does not emerge
into air. from the curved surface of rod irrespective
Solution:Let S be the small bulb at the bottom of of the value of .
the jar. The bulb is in the denser medium. The Solution:The angle of incidence at the curved
light from the bulb is incident on the water - air surface of the cylindrical rod is given to
interface. Light emerges for incident angles less be 90 0 .
than the critical angle. The locus of all the points
Page 396
RAY OPTICS PART – 2
Light entering the rod at angle of incidence
on one planar end will not emerge from the n2 n
Where sin C or C sin 1 2
n1 n1
curved surface if angle of incidence 90
0
n1
P
S or sin max n cos C
2
90 C ; sin 90 sin C n
1 1 max sin 1 1 cos C
cos ; 1 sin 2 -------- (1) n2
By Snell’s law at plane surface n n
max sin 1 1 cos sin 1 2
sin sin n 2 n1
; sin -------- (2)
sin Illustration 36: What will be the minimum
Substituting eq. (2) in eq. (1) angle of incidence such that the total
internal reflection occurs on both the
By simplification 1 sin 2 surfaces?
But sin max 1 ; 2
1 2
Illustration 35: A rectangular glass slab ABCD
of refractive index n1 is immersed in water
2 2
of refractive index n 2 n1 n 2 . A ray of
light is incident at the surface AB of the
slab as shown. The maximum value of the 3 3
max B
3 3
Critical angle at A C1
B C
Solution:
1 2 1
A
n2
D sin C1 ; C1 450
n1 2 2 2
r2
r1 Critical angle at B C2
n
3 3
sin C 2 C2 600
B C 2 2
r1 r2 900 ; r1 900 r2 Minimum angle of incidence for total internal
0 reflection to occur on both the slabs should be
r1 max 90 r2 min and r2 min C 0
(for total internal reflection at AD) 600 . i min 60
Page 397
RAY OPTICS PART – 2
Illustration 37: A ray of light incident on the Liquid
horizontal surface of a glass slab at an
o o
angle of incidence ‘i’ just grazes the A 60 30 C
adjacent vertical surface after refraction.
Compute the critical angle and refractive
index of glass.
90o
A i B
B
r
Solution: Critical angle between glass and liquid
C 2
F D sin C
Solution: face is 3 3
2
E
Liquid
By Snell’s law of refraction, at the horizontal o o
surface A 60 30o 30 C
o
90
1 sin i sin r; sin r sin i ---(1)
Apply Snell’s law at vertical surface,
90o
1
sin c 1 sin 900 ; sin C
B
From figure in BFD Angle of incidence at face AC is 600
For the TIR to take place at the face AC, the
r C 900 1800 ; c 900 r angle of incidence at the face AC, i C
1
sin 900 r
; cos r 1 ---(2) 2 3 3
600 c or sin 600 or
3 4
From equations 1 & 2 Illustration 39: What is the value of the
2 sin 2 r 2 cos 2 r 1 sin 2 i refractive index for a 900 450 450 prism
which is used to deviate a beam
1 sin 2 i through 900 by total internal reflection?
Critical angle is determined from
1 1 45°
sin C
1 sin 2 i
Illustration 38: Light is incident normally on
face AB of a prism as shown in figure. A
90° 45°
liquid of refractive index is placed on
face AC of the prism. The prism is made of Solution: At face AB, the ray of light suffers no
deviation if the light is incident normally
glass of refractive index 3/2. The limits of
for which total internal reflection takes
place on face AC is
Page 398
RAY OPTICS PART – 2
A
Illustration 41: White light is incident on the
interface of glass and air as shown in the
figure. If green light is just totally
internally reflected then the emerging ray
in air contains:
45°
d e
Re rang w
O ello
Y
B C
Air Green
Glass
At face AC, the light ray undergoes TIR, so
angle of incidence at the face AC is greater than B
critical angle In lue
white Vi dig
1 1 ol o
; 2 et
45 0 C; sin 450 sin C; 2
Illustration 40: A beam of light consisting of Solution: Critical angle C sin 1 1
red, green and blue colours is incident on
a right angle prism. The refractive indices Wavelength increases in the sequence of
of the material of the prism for the red, VIBGYOR. According to Cauchy’s formula
green and blue wavelengths are 1.39, 1.44
and 1.47 respectively. The colour of light refractive index ( ) decreases as the
that comes out of the prism is wavelength increases. Hence, the refractive
index will increase in the sequence of
ROYGBIV. The critical angle C will thus
increases in the same order VIBGYOR. For
green light the incidence angle is just equal to
the critical angle. For yellow, orange and red
the critical angle will be greater than the incidence
45° angle. So, these colours will emerge from the
glass air interface, while for violet, indigo and
Solution: The colours for which i c , will get total blue, the critical angle is less than angle of
internal reflection i c incidence. So, they undergo TIR.
A Illustration 42: A rectangular block of glass
is placed on a printed page lying on a
horizontal surface. Find the minimum
value of the refractive index of glass for
Composite which the letters on the page are not visible
i from any of the vertical faces of the block.
Red
o
45 R
B o C i
i = 45
Blue
Green
r
R G B ; CR CG CB A
O
B
Paper
0 1 1 1 Solution: The situation is as shown in the Fig,
or sin i sin c or sin 45
2 D C
2or 1.414
Hence, the rays for which 1.414 will get R P
i
TIR. For green and blue 1.414 , so they
will suffer TIR on face AC only red comes out r
O
from this face. A Paper B
Page 399
RAY OPTICS PART – 2
Light will not emerge out from the vertical face When angle of incidence c , then the
BC if incident ray will refract and hence ‘x’ region
1 will be illuminated. So, in limiting case:
at it i C or sin i sin C ; sin i ..... (1)
But from Snell’s law at O.
1 sin sin r and OPR , C
r i 900 , r 900 i
So sin sin 900 i ; cos i sin /
2
But sin i 1 cos 2 i 1 sin / --(2)
In PQR;
Substituting the value of sin i from eq (2) in (1)
R 1 3
2 cos c sin c r
sin 1 2 Rx d 2 / 3 2
1 2
, 1 sin 2
o
c 60 , R 5cm ( given)
Now as sin max 1; 2, 2
2 2 5
cos 60 5 x 10 x 5cm.
5 x
Hence min 2 Concept of Refraction in a medium of
Illustration 43:A uniform, horizontal beam of variable refractive index :
light is incident upon a quarter cyclinder of The atmosphere becomes thinner as we move
radius R 5cm , and has a refractiv index up. When we move up from the earth refractive
2 / 3 . A patch on the table at a distance index decreases. We assumed the situation
‘x’ from the cylinder is un illuminated. Find where refractive index changes only in one
the value of ‘x’? direction, medium can be considered as a
Solution: When the angle of incidence is c , ‘x’ collection of large number of the thin layers.
region will remain unillumiated. This is the Let refractive index be a function of y i.e.,
limiting case. f (y) then the medium can be considered as to
normal be made up of large number of thin slabs placed
Q parallel to x-axis and optical normal at any
C 90 interface is parallel to y-axis. Similarly if
f(x) then slabs are parallel to y-axis and
optical normal at any interface is parallel to x-
axis.
y y
P R X R = f(y)
Page 400
RAY OPTICS PART – 2
trajectory. of the incident light are taken as negative.
y
Refraction at Spherical surfaces :
Normal 1 N
2
i
f(y) r
x O M C I
u R
air V
Page 401
RAY OPTICS PART – 2
h0 h
Therefore 1 2 i dh i
u v v hi
mt 1 dt
h i 1 v 2 u h 0 dh 0
Lateral magnification mt
h 0 2 u dt
Longitudinal magnification at 2 v
refracting curved surface: If a small Velocity of image is, Vi u V0
1
object of length ‘du’ is placed on the axis,
produces an image of length ‘dv’ along the or Vi m t V0
axis of the refracting surface, then longitudinal Principal foci :
magnification
1
dv 1 1 2
mL and 2 1 2 ...(1) 2 O
du v u R F2 F1
dv 1 v 2 f2
fig (a)
f1
f ig (b)
On differentiating (1), .
du 2 u 2 2 1 2 1
In the equation
v u R
1 v 2 If the object at infinity i.e., u
mL 2
m 2t 1
2 u 2 2 1 v 2 R
0 2 ;
Where ‘ m t ’ is transverse magnification. v R 2 1
Concept of Motion of Object 2 R
From the figure v f 2 ; f 2 ...(1)
When object moves Along the 2 1
Principlal Axis : i.e. The position of image corresponding to the
As the object position changes with time, the object at infinity, is called the second principal
image position also changes. focus of the refracting surface. This is shown in
Refraction at a spherically surface, fig (a)
2 1 2 1 Similarly if v , i.e., the object is so placed
..... (1) that the refracting rays becomes parallel to the
v u R
principal axis, then
differentiating (1) we get
2 1 2 1 R
dv 1 du u 1
22 2 0 ; 22 v1 21 v 0 0 u R 2 1
v dt u dt v u
1R
v
2
From figure, u f1 ; f1 ...(2)
Vi 1 V0 ; V , velocity of image 2 1
2 u i
i.e. The position of the object, whose image is
formed at infinity to known as the first principal
and V0 , velocity of object Vi m L V0 focus of the refracting surface. This is shown in
We use sign convention to get direction of figure(b).
motion of image. f1 1
When object moves Along Hence f .....(3)
Perpendicular ( Transverse) to the 2 2
Principal Axis: If the object moves It is easy to see that first focal length f1 for
transverse to the principle axis with the spherical refracting surface is not equal to the
speed V0 and if ‘mt’ is the magnification, then second focal length f 2 .
2 1 2 1
Further .....(4)
v u R
Page 402
RAY OPTICS PART – 2
2 R 1R 4 1 1 4 1 1 5
1 ;
v 2 1 u 2 1 3v 20 120 3v 120 20 120
160
f 2 f1 v 32 cm
1 .....(5) 5
v u
Illustration 45: A small object stuck on the
Power of Refracting Spherical
surface: surface of a glass sphere 1.5 is
Any distance divided by the refractive index of viewed from the diametrically opposite
the space in which it is measured is called a position. Find transverse magnification.
reduced distance. The refracting power of a Solution: Refraction will take place at side II of
spherical surface is defined as the reciprocal of 2 1 2 1
the reduced focal length. If f1 and f 2 are first the surface
v u R
and second principal focal length of refracting
surface.
f1 f2 I II
The ratios and
1 2 are the reduced focal
lengths.Then refracting power I O C P
1 f1 f
P 2 As 2
f1 f2 1 2
Here, 1 1.5, 2 1 and u 2R
2 1 1R
Power (P) R f1 1
1.5
1 1.5 1 0.5 1.5 0.5
2 1
v 2R R ; v R 2R 2R
Relative motion of object and image
w.r.t spherical refracting surface: or v 4R
y negative sign indicates that the image is
formed to the left of refracting surface as
(u h0) shown.
O
1 v 1.5 4R
Magnification, m u 1 2R ; m 3
(v hi)
I
2
Illustration 46: A point object is placed at the
2 1 2 1 1 v centre of a glass sphere of radius 6 cm and
m refractive index 1.5. The distance of the
v u R 2 u
virtual image from the surface is.
2 dv 1 du
0 Solution: A point object is at the centre of a glass
v 2 dt u 2 dt sphere of radius 6 cm.
dv 1 v du 1 2 du 1 1.5 1 1.5 1 3 1
mt .
dt 2 u dt 2 dt v ( R) R v 2R 2 R
Illustration 44: Locate the image of the point 1 2 1
object O. The point C is centre of curvature v R
of the spherical surface. v 2R R
=1.0 =4/3
O
P C
20cm 40cm
Solution: Here O
1 1, 2 4 / 3, u 20cm, R 40cm
2 1 2 1 4 / 3 1 4 / 3 1
;
v u R v 20 40
Page 403
RAY OPTICS PART – 2
The rays from the object fall normally on the conventions, we
surface of the sphere and emerge undeviated.
When drawn backwards, they meet at O. The 1.5 1.0 1.5 1.0
have v1
image will be formed at the centre O itself. v 2 R R
Illustration 47: An air bubble in Second: reflection
glass 1.5 is situated at a distance 3 1 1 1 2
Using with proper sign
cm from a convex surface of diameter 10 v u f R
cm as shown in figure. At what distance conventions, we
from the surface will the bubble appear?
1 1 2 R
have v R v2 2
A 2
=1 Third: refraction
=1.5
2 1 2 1
P Again using with reversed
C O I v u R
sign convention, we
1.0 1.5 1.0 1.5
3cm have v 1.5 R R orv3 2 R i.e.,
3
5cm final image is formed on the vertex of the
Solution: In case of refraction from curved surface silvered face.
2 1 2 1 Illustration 49: A sphere of radius R made of
v u R material of refractive index 2 . Where
Here, 1 1.5, 2 1;R 5cm should an object be placed so that a real
image is formed equidistant from the
1 1.5 1 1.5 sphere?
and u 3cm ; So v 3 5 ,
1
1
i.e., v 2.5 cm
0 P1 P2 1
Illustration 48: A solid glass sphere with ra-
dius R and an index of refraction 1.5. is sil-
vered over one hemisphere. A small object X 2R X
is located on the axis of the sphere at a dis- A: Let object be placed at a distance x from
tance 2R to the left of the vertex of the
unsilvered hemisphere. find the position of the pole P1 of the sphere, then the ray must
final image ofter all refractions and reflec-
tion have taken place. pass symmetrically through the sphere, as
Solution: The ray of light first gets refracted then shown in the figure Applying the equation at
reflected and then again refracted. For first the first surface, we get
refraction and then reflection the ray of light
travels from left to right while for the last 2 1 2 1 1
or x R
refraction it travels from right to left. Hence, x R 2 1
the sign convention will change accordingly. Note that the real image is formed only when
the refracive index of the sphere is more than
I2 that of the surronding. i.e. 2 1 .
0 I3
2R
Illustration 50: Consider the situation shown
in figure. The refractive index of medium
is 3/2 and its radius of curvature is 30cm,
1.5R R PO1 = PO2 =10cm. Find distance between
2 O1 and O2 and see by (a) O1 and (b) O2 .
First: refraction
1
Using 2 1 2 with proper sign
v u R
Page 404
RAY OPTICS PART – 2
distance 10 cm from the surface and inside
=3/2 glass when seen from the other medium.
O1 P O2 Find the actual location of air bubble.
Air 1
Medium Solution: 2 1 2
u R
C O1 P I O2 O I P C
Page 405
RAY OPTICS PART – 2
1 1.0, 2 3/ 2, u 20cm, R 5cm, v ? I 1v I 1 (32)
m ;
2 1 2 1 O 2u 1 (4 / 3) (40)
v u R
I 0.6cm ; The image is erect.
3/ 2 1 3 / 2 1 3 1 1
; Illustration 54: A transparent thin film of
v 20 5 2v 20 10
uniform thickness and refractive index
3 1 1 1 n1 1.4 is coated on the convex spherical
v 30cm
2v 10 20 20 surface of radius R at one end of a long
v is +ve i.e, image is formed right of P.. solid glass cylinder of refractive index
I 1v n 2 1.5 , as shown in figure. Rays of light
m parallel to the axis of the cylinder
O 2u
traversing through the film from air to
I 1 30 glass get focused at distance f1 from the
I 0.5 mm
0.5 3 (20) film, while rays of light traversing from
2 glass to air get focused at distance f 2 from
I is –ve i.e, image is inverted.
the film. Then, the magnitudes of f1 , f 2 are
Illustration 53: Locate the image and find its
height. n1
=1 4
μ=
3
Air
1 cm
C P
n2
20cm
Solution: For air to glass
40cm
n1
Solution:
1 1, 2 4 / 3, u 40cm, R 20cm, v ?
Air R
2 1 2 1 F2 F1
f2
v u R f1
n2
4
1
4/3 1 3
v 40 20 1.5 1.4 1 1.5 1.4
f1 3R
f1 R R
4 1 1 2 3 1
3v 60 40 120 24 For glass to air
v 32cm 1 1 .4 1 .5 1 1 .4
; f2 2R
The magnifincation f2 R R
Page 406
RAY OPTICS PART – 2
Illustration 55: A spherical solid glass paper
weight of diameter 6 cm has a small air
bubble at a distance of 1.5 cm from the
the side to which it is nearest along the
line joining the bubble and the centre, find
3
where will it appear. (Given glass )
2
Solution: Fig shows the relevant situation. O is the
air bubble which acts as the object and P is the
pole. The light ray travelling from the object
passes from glass to air.
Air
Glass
1 = 3/2 2 = 1
C O P
1.5 cm
1 3 / 2 and 2 1
3
Also, u cm, R 3cm and v ?
2
2 1 2 1
Using Eq.
v u R
1 3/ 2 1 3/ 2
We have, v 3/ 2cm 3cm
1 3/ 2 1 3/ 2
or, v 3/ 2cm 3cm
1 1 1
or, v 6cm 1cm or, v 1.2 cm
Page 407
RAY OPTICS PART – 3
(a) (b)
Second focal point ( F2 ) is an image point on
the principal axis for which object lie at infinity.
(c) (d)
Page 408
RAY OPTICS PART – 3
The distance between optical centre of a lens those measured downward are considered as
and the point where the parallel beam of light negative.
converges or appears to converge. i.e., second (e) Angles measured clockwise, beginning at the
principal focal point ( F2 ), is called focal length optic axis, are negative. Angels measured
f. counterclockwise are positive.
To a lens, if the media on the two sides is same, Transverse distance measured above the
then first principal focal distance is equal to principal axis are taken to be positive while
those below it to be negative.
second principal focal distance. i.e., f1 f 2 .
Longitudinal distances are measured from
Focal plane: It is a plane passing through optical centre and are taken to be positive if in
the principal focus and perpendicular to the the direction of light propagation and negative
principal axis. if opposite to it.
Aperture(D): In reference to a lens, aperture While using the sign convention it must be kept
is the effective diameter of its light transmitting in mind that, to calculate an unknown quantity
area. The intensity of image formed by a lens the known quantities are substituted with sign
depends on square of aperture. i.e., I D2 in a given formula.
Incident Incident
SIGN CONVENTION light
light
R1 R2
R1 R2
C2 O F C1 C1 O C2
(a)
(b)
For a convex lens as shown in fig.(a)
(a) The diagrams are drawn with the incident light
travelling from left to right R1 OC1 is +ve; R 2 OC2 is -ve
(b) The distances are measured by taking the optic
center P as the origin. f OF is +ve
(c) The distances measured in the direction of the For concave lens as shown in fig. (b)
incident light are considered positive while those R1 OC1 is -ve; R 2 OC2 is +ve
measured in the direction opposite to the
incident light are taken as negative. f OF is -ve
(All quantities measured to the right of P are
positive and all those to its left are negative).
(d) Heights measured upward and perpendicular
to the principal axis are taken as positive while
Page 409
RAY OPTICS PART – 3
POSITION AND NATURE OF IMAGES :
a) Convexlens and Formation of Image by a Convex Lens:
A Real, inverted
2. Beyond 2F1 Between
F2 B ' 2F2 F2 and 2F2 and diminished
B 2F1 F1
A'
A Real, inverted
At 2F1 and of same
3. At 2F1 B F2 2F2
B' size as the
2F1 F1
object
A'
A
4. Between Between 2F2 Real, inverted
F2 2F2
F1 and 2F1 and highly
2F1 B F1 B'
magnified
A'
Page 410
RAY OPTICS PART – 3
b) Concave lens 2 1 2 1
v u R
Virtual, For first surface
F erect,
diminished 2 1 2 1
I P .... (1)
at F v1 u R1
The image I1 acts as object to second surface,
and form final image I2
Virtual, For second surface
In front
of lens erect, 1 2 1 2
F diminished .... (2)
between O IP v v1 R2
between
P and F and P So adding (1) and (2) equation, we have
1 1 1 1
From the ray diagrams it is clear that 1 2 1
Regarding convex lens: A convex lens will v u R1 R 2
form a real image for a real object when the 1 1 2 1 1
object is placed beyond focus. or v u 1 R R
When the object comes with in the focus, then 1 1 2
a virtual image is formed for the real object.
The real image formed is always inverted while 1 1 1 1
r 1
virtual image is always erected. v u
Regarding concave lens:A concave lens R1 R 2
always form virtual image for a real object . 2 L
The image of a real object formed by a concave with r or
1 M
lens is always erected and diminished in size.
A concave lens can form a real image as well If object is at infinity, image will be formed at
as virtual image if the object is virtual. the focus
Lens Maker’s formula and Lens formula : i.e. for u , v f , so that above equation
In case of image formation by a lens, the
incident ray is refracted at first and second 1 1 1
surface respectively. The image formed by the r 1
f R1 R 2
first surface acts as object for the second.
Consider an object O placed at a distance u Which is known as Lens-maker’s formula and
from a convex lens as shown in figure. Let its 1 1 1
image is I1 after refraction through first surface. for a lens it becomes which is
v u f
So from the formula for refraction at curved known as the “lens - formula” or “Gauss’s
surface. formula” for a lens.
Lens formula :Lens formula is a relation
connecting focal length of the lens with the
object distance and image distance,
1 1 1
f u
The above formula is valid for convex as well
as concave lenses and it is independent of
nature of the image (real or virtual)
uf vf vu
v ; u ; f
uf f v uv
Page 411
RAY OPTICS PART – 3
Page 412
RAY OPTICS PART – 3
I
u
(b) Concave lens: The graph between v and
u is hyperbola to concave lens as shown in figure d
(b). If the distance ‘d’ between an object and screen
is greater than four times the focal length of a
convex lens, then there are two positions of
lens between the object and screen. This
method is called displacement method and is
used in the laboratory to determine the focal
length of convex lens.
If the object is at a distance u from the lens, the
distance of image from the lens, v d u ,
Newton’s Formula:
1 1 1
In case of thin convex lens if an object is placed From lens formula
v u f
at a distance x1 from first focus and its image
1 1 1
is formed at a distance x 2 from the second or u 2 du df 0
focus. du u f
d d d 4f
h1 or u
2
O F1 h2 h2
Now there are three possibilities.
x1 f1
f2 x2 i) If d 4f , then u will be imaginary, so
From properties of triangles, to the left of the physically no position of lens is possible.
lens d
h1 h 2 h1 h 2 ii) If d 4f , then u 2f so only one
2
x1 f1 and to the right of the lens f 2 x 2 . position is possible and in this v 2f . That is
x1 f 2 why the minimum separation between the real
f x or x1x 2 f1f 2 ,if f1 f 2 =f, object and real image is 4f.
1 2
d d d 4f
x1 x 2 f 2 iii) If d 4f then u1 and
is called Newton’s formula or lens user formula. 2
Lens with Different Media on either d d d 4f
side: u2 for two positions of the
2
Consider a lens made of a material with object for two real images
refractive index with a liquid a on the left (a) For u u1 ,
and a liquid b on the right as shown in figure
d d d 4f
v d u1 u2
2
(b) For u u 2 ,
d d d 4f
v2 d u 2 u1
The governing equation for this system is 2
b a a b
v u R1 R2
Page 413
RAY OPTICS PART – 3
i.e., for two different positions of the lens the Again this acts as object for lens and final image
dist ances of object and image are I2 is formed at 6 cm infront of the lens.
interchangeable as shown in the figure. The final image is real and at a distance of
Lens positions 16 cm from the mirror.
(1) (2)
(d-u) Illustration 2: A bi-convex lens is formed with
two thin plano convex lenses as shown in
the figure. Refractive index ‘n’ of the first
I lens is 1.5 and that of the second lens is
1.2. Both the curved surfaces are of the
x same radius of curvature R=14 cm. For this
(d-u) bi-convex lens, for an object distance of
The magnification for both the positions of the 40 cm, find the position of the image.
I I n = 1.5
object are related as m1 1 , m2 2
O O n = 1.2
1
m1
m2
I1 I 2 I1I 2
i.e m1m 2 1 or m1m 2 . 1
O O O2
Therefore O I1I2 where I1 and I 2 are the R = 14 cm
Solution:
sizes of the images for two positions of the
object and O is the size of the object. 1 1
PT 1.5 1 0 1.2 1 0
If ‘x’ is the distance between the two positions 14 14
x
of the lens. Then f m m 0.5 0.2 1
PT
1 2 14 14 20
Illustration 1: A biconvex lens of focal length
15 cm is in front of a plane mirror. The 1 1 1
f 20 cm ;
distance between the lens and the mirror v 40 20
is 10 cm. A small object is kept at a 1 1 1 1
distance of 30 cm from the lens. Find the ; v 40 cm
location of final image. v 20 40 40
Solution: Illustration 3: An object is 5 m to the left of a
flat screen. A converging lens for which
the focal length is 0.8 m is placed between
object and screen. (a) Show that for two
positions of lens form images on the screen
and determine how far these positions are
O I1 I2 from the object? (b) How do the two images
differ from each other?
Solution:
10cm
6cm
For the first time refraction through the lens:
1 1 1
f v1 u v1 30
This acts as object for plane mirror with
u 20 .
Hence it forms image at v2 20 .
Page 414
RAY OPTICS PART – 3
v 5 4
(b) m ; m1 4 0.25 and
u h h
F
m2
5 1 4.00 O I
1
Hence, both the images are real and inverted, f
the first has magnification -0.25 and the second (a) (b)
-4.00. h 1
Illustration 4: A point object is placed at a tan ; if h 1, tan
f f
distance of 12 cm on the axis of a convex
lens of focal length 10 cm. On the other 1
side of the lens, a convex mirror is placed As per definition, power (P) = tan
f
at a distance of 10 cm from the lens such If lens is placed in a medium other than air of
that the image formed by the combination
coincides with the object itself. What is the refractive index . Then power P ;
focal length of convex mirror? f
1 1 1 1
Solution: For convex lens, In air P 1
v 12 10 f
The S.I unit of power is diopter (D) and
i.e., v 60 cm ; i.e., in the absence of convex
1D 1m1
mirror, convex lens will form the image I1 at a
1 100
distance of 60 cm behind the lens. Since, the i.e. P f in m f in cm D
mirror is at a distance of 10 cm from the lens,
I1 will be at a distance of 60 10 50 cm A convex lens converge the incident rays. Due
to this reason, the power of a convex lens is
from the mirror, i.e., MI1 50 cm taken as positive. On the other hand, a concave
lens diverge the incident rays. Therefore its
power is taken as negative.
Lens with one Silvered surface
When the back surface of a lens is silvered.
The rays are first refracted by lens, then
reflected from the silvered surface and finally
refracted by lens, so that we get two refractions
and one reflection.
Page 415
RAY OPTICS PART – 3
1 2 1 2
=
O O I1
2
2 1 2 f R
e
In the diagram if fl and f m are the respective R R R 2
focal lengths and Pl and pm are the powers Since fe is negative, the system behaves as a
of lens and mirror. Then effective power R
Pe Pl Pm Pl 2 Pl Pm concave mirror with focal length
2
1 1 1 1
When the curved surface of an
f e fl f m fl equiconvex lens is silvered
1 R
with Pm f where f m 2 and
m 2 = + +
1 1 1
Pl 1
fl R1 R2 1 1 2
Use sign convention in the formula of focal Pe Pm 2Pl ; f f f
length. e m l
So the system will behave as a curved mirror 2 1 2
1 Pl Pm
R R
of effective focal length, f e P
1 R 2 4 1
When the plane surface of plano
e
Pm where f m ; Pe
fm 2 R R
convex lens is silvered.
2 4 4 4 2 2 2 1
R R R
R
fe
2 21
Since fe is negative, the system behaves as a
Then, the focal power of the given lens is R
concave mirror with focal length 2 2 1
( Pm 0 )
1 1 2 Illustration 5: A pin is placed 10 cm in front
Pe Pm 2 Pl ; f of a convex lens of focal length 20cm made
e f m fl of material having refractive index 1.5. The
1 2 1 surface of the lens farther away from the
Pe 2. 0 pin is silvered and has a radius of curvature
R R 22cm. Determine the position of the final
1 R image. Is the image real or virtual?
fe Solution: As radius of curvature of silvered surface
Pe 2 1 is 22 cm, so
Since 1 , fe is negative, the system behaves R 22
as a concave mirror. f M 11cm and hence,
2 2
Page 416
RAY OPTICS PART – 3
x 12.5cm
1 1 1
PM D Hence, the object should be placed at a
fM 0.11 0.11 distance
Further as the focal length of lens is 20cm, 12.5cm in front of the silvered lens.
i.e., 0.20 m, its power will be given by: Illustration 7: A convex lens and convex
1 1 mirror are seperated by distance d as
PL D shown in the figure. What should be the
f L 0.20
value of d so that image is formed on the
object itself
f=10cm
f=10cm
I
O
10 cm O
15cm
d
11 cm
Page 417
RAY OPTICS PART – 3
C1 O C2
formula
1 1 1 1 6 1 1
R1 R2
m g 1 or 1
f R1 R2 f 7 R R
For concave lens R1 is -ve and R2 is +ve so
the lens makers formula is 1 1 2 7R
or or f 3.5 R
1 1 1 f 7 R 2
1 Hence, the given lens in medium behaves like
f R1 R2 convergent lens of focal length 3.5R
1 2 Illustration 10: Show that “A hollow equi con-
For equiconcave lens 1 vex lens of glass will behave like a glass
f R
plate”.
3) For converging meniscus (or)
Solution: Hollow convex lens is as shown in figure
concavo-convex
1 1 1
1 1 1 g 1 0
1 , if R R f1 R1 R2
f R1 R2
1 2
1 R2
f R Hollow glass lens
Page 418
RAY OPTICS PART – 3
The refractive index of the lens is 2 Solving eqs. (1) and (2) , we get x 6cm
Solution: Illustration 14: An object placed at A
3 1 1 2 2 3 1 2 OA f . Here, f is the focal length of
3 2 the lens. The image is formed at B. A
v u 2R R 2 perpendicular is erected at O and C is
1 2 2 3 2 2 1 2 2 3 chosen such that BCA 900 .
Illustration 12: The magnification of an ob- Let OA a, OB b and OC c . Then find
ject placed in front of a convex lens of fo-
cal length 20cm is +2. To obtain a magni- the value of f in terms of a,b and c.
fication of -2, the object will have to be 1 1 1 1 1 1
moved by a distance equal to d. Find the Solution: From we have
value of d. v u f b a f
Solution: When magnification is +2 then the image ab
is virtual. Both the image and the object are on or f 1
the same side of the lens. a b
Further AC 2 BC 2 AB 2
u x; v 2 x; f 20
2
1 1 1 1 1 1 or a 2 c 2 b 2 c 2 a b
Using we have or
v u f 2 x x 20 or a 2 b 2 2c 2 a 2 b 2 2ab ; ab c 2
x 10cm .
To have a magnification of -2 the image must o
90
be real.
u y, v 2 y and f 20 c
1 1 1
or y 30cm d y x 20cm
2 y y 20 A O B
Illustration 13: Two point sources S1 and S 2
are 24cm apart. Where should a convex
lens of focal length 9 cm be placed in be- a b
tween them so that the images of both c2
sources are formed at same place? Substituting this in Eq. (1) we get f
ab
Solution: In this case, one of the images will be Illustration 15:Convex lens has a focal length
real and other virtual. Let us assume that image
of 10cm.
of S1 is real and that of S 2 is virtual. a) Where should the object be placed if the
f=9 cm image is to be formed at 30cm from the lens
on the same side as the object?
b) What will be the magnification?
S1 I S2
I
y
x O FO F
Solution:
24-x
1 1 1 30cm
Applying
v u f a) In case of magnifying lens, the lens is
convergent and the image is erect, enlarged,
1 1 1 virtual , between infinity and object and on
For S1 : 1
y x 9 the same side of lens as shown in figure. So
here f 10cm and v 30cm and hence
1 1 1
for S 2 : 2 from lens-formula,
y 24 x 9
Page 419
RAY OPTICS PART – 3
1 1 1 1
Hence b 45cm
2 3 2b b 30
Thus the object has to be moved through a dis-
tance of 45 15 30cm away from the lens.
Illustration 18:The distance between the ob-
ject and the real image formed by a convex
Solution: For refraction at first surface, lens is d. If the linear magnification is m,
find the focal length of the lens in terms
2 1 2 1 of d and m.
i
v1 R Solution: The convex lens formula for a real image
For refraction at 2nd surface is
3 2 3 2 1 1 1
ii i
v2 v1 R v u f
Where no sign conventions are to be used. Mul-
3 u u
tiplying by u we get 1
1 2
v f
2 1 1 u u f
or 1
m f f
1 m f
or m u f f or u ii
v1 m
v2
Multiplying (i) by v we get
Adding equations (i) and (ii) we get
v v v
3 3 1 3 R 1 or 1 m or v f 1 m iii
or v2 u f f
v2 R 3 1
Now u v d . Using (ii) and (iii) we have
Therefore, focal length of the given lens system
3 R
d
1 m f f 1 m
is m
3 1
Page 420
RAY OPTICS PART – 3
Illustration 19: A concave lens of focal length If a lens made of two or more materials and
f forms an image which is n times the size are placed side by side as shown in below, then
of the object. What is the distance of the there will be one focal length and hence one
object from the lens in terms of f and n? image
1 1 1
Solution: Using lens formula is Thus
v u f
u u 1 u v
1 or 1 n
v f n f u
1 n
or u f
n
Some important points regarding lens:
Every part of a lens forms complete image. If a
portion of lens is obstructed, full image will be
formed but the intensity will be reduced. Lens immersed in a liquid:
The focal length of a lens depends on its Consider a lens made of a material of refrac-
1 tive index lens is immersed in a liquid of re-
refractive index i.e 1 , so the focal
f fractive index liquid ,
length of a given lens is different for different
wave lengths and maximum for red and If f a is the focal length of the lens placed in air,,
minimum for violet whatever the nature of the then
lens as shown in figure.
1 1 1
white light
white light lens 1 1
fa R1 R2
If fl is the focal length of the lens when im-
mersed in a liquid then
fv
fR 1 lens 1 1
1 2
Filling up of a lens:
fl liquid
R1 R2
If a lens made of a number of layers of different
refractive indices as shown in figure, for a given 1 fl
lens 1
wave length of light it will have many focal lengths 2 f a lens
1 1
1
as,
f liquid
Hence it will form many images. According to Depending upon the values of lens and liquid ,
given diagram number of images formed by lens we have three cases
is 4.
lens liquid
lens liquid lens liquid
liquid liquid
liquid
liquid
liquid liquid
L
Page 421
RAY OPTICS PART – 3
lens >liquid lens=liquid lens<liquid A 1 2
; R1 10cm
liquid liquid liquid liquid liquid
Initially, G
3
2
3
liquid
and R2 10cm
1 2 1 1 2
So 1 ie
(a) (b) ( c) f 3 10 10 30
Case(a): If lens liquid , then fl and f a are of f 15cm
same sign and f l f a i.e. the nature of lens re- ie. the air lens in glass behaves as divergent
mains unchanged, but it’s focal length increases lens of focal length 15cm.
and hence power of lens decreases. When the liquid of 2 is filled in the air cavity,,
Case(b): If lens liquid then fl ,ie. the lens 2 4
L
behaves as a plane glass plate and becomes M 1.5 3
invisible in the medium.
1 4 1 1 2
Case(c): If lens liquid , then fl and f a have op- So that now 1
posite sign and the nature of lens changes ie. a f ' 3 10 10 30
convex lens diverges the light rays and con- f ' 15cm ie. the liquid lens in glass will be-
cave lens converge the light rays. have as a convergent lens of focal length 15cm.
Illustration 20: A glass convex lens of refrac-
tive index (3/2) has a focal length equal to If the two radii of curvature of a thin lens are
0.3m. Find the focal length of the lens if it not equal, the focal length remains unchanged,
is immersed in water of refractive index where the light is incident on either of two
(4/3)? surfaces.
Solution: As according to lens-makers’ formula Cutting of a lens:
3 If an equi convex lens of focal length ‘f’ is cut
fw g 1 f w 1 into two equal parts along its principal axis, as
2
; f 1.2 m
w
fa g 0.3 3 shown in figure , as none of , R1 and R2 will
1 2 1
w change, the focal length of each part will be
4
3 equal that of initial lens, but intensity of the image
Illustration 21: As shown in figure, a spheri- formed by each part will be reduced.
R1=R2=R3
cal air lens of radii R1 R2 10cm is
formed by cutting in a glass 1.5 cyl-
f
inder. Determine the focal length and na- f
ture of air lens. If a liquid of refractive in-
dex 2 is filled in the lens, what will hap-
pen to its focal length and nature?
f
= 1.5
= 1.5 = 1.5
(a) (b) ( c)
A) =1 =2
= 1.5
If an equi convex lens of focal length ‘f’ is cut
into two equal parts transverse to principal axis,
as shown in figure, the focal length of each part
Solution: According to lens-maker’s formula, will become double that of initial value, but
intensity of image remains same.
1 1 1
1 with L
f R1 R2 M
Page 422
RAY OPTICS PART – 3
R1=R2=R3
1 1 1 1 2
,
Fnet 2 f 2 f Fnet 2 f and
f 2f 1
Fnet f , Pnet
f
If an equi convex lens of focal length ‘f’ is
divided into two equal parts along its principal
axis as shown in figure, the focal length of each
part is f . If these two parts are put in contact
in different combinations as shown in figure (a)
(a) (b) ( c) f (ve) f (ve) f (ve)
For original lens
1 1 1 2( 1)
( 1) (1)
f R1 R2 R
For each part of cut lens
1 1 1 ( 1) f (ve)
( 1) (2) (a) (b)
f R R
1 1 1
From (1) and (2) we get f ' 2 f For first combined lens, f f f
On removing a part of lens with out disturbing net
f
(a) V0
(b)
If the equi convex lens of focal length ‘f’ is
divided into two equal parts transverse to the
principal axis as shown in figure, the focal length I
of each part is 2f. If these two parts are put in O u
Vi
contact in different combinations as shown in
figure
2f 2f 2f 2f 2f 2f v
1 1 1
For a lens
v u f
(or) (or)
On differentiating above equation, we get
1 dv 1 du du dv
2
, 2 0; V0 , Vi
v dt u dt dt dt
Page 423
RAY OPTICS PART – 3
2
v
(or) Vi V0 where Vi velocity of im-
u
age with respect to lens, V0 =velocity of object
with respect to lens. ;
2
2 f
i.e. Vi m .V0 .V0
u f
If an object is moved at constant speed to-
wards a convex lens from infinity to focus, the
image will move other side of the lens slower in
the beginning and faster later on away from the
lens. If the object moves from F to optical cen-
tre, the image moves with greater speed on the u cm
cm
same side of object from infinity to towards So each part will form a real image of the point
lens.
object at 60cm from the lens as shown in fig-
A point object is kept at the first focus of a
convex lens. If the lens starts moving towards ure. As there are two pieces, two images are
right with constant velocity. The image first will fo rmed. No w in similar triangles
move towards left and then move towards right.
OI1 I 2 and OL1L2
V
I1 I 2 OP u v
F
L1 L2 OQ u
Illustration 22: A point object O is placed at a
distance of 30 cm from a convex lens of 90
ie I1 I 2 2 0.05 0.3cm
focal length 20cm cut into two halves each 30
of which is displaced by 0.05cm as shown
in figure. Find the position of the image? So the two images formed are 0.3cm apart.
If more than one image is formed, find Rotation of Lenses:
their number and distance between them?
P
C
O O
O
20.05cm
When a thin lens is rotated in a uniform
30cm
medium by an angle ‘ ’, the principal axes also
rotates by an angle ' ' in the same direction.
All calculations to be performed now according
f =20cm
to the new system of principal axis.
Solution: Considering each part as separate lens Combination of Lenses :
with u 30cm and f 20cm , from lens 1) Consider two thin lenses kept in contact as
1 1 1 1 1 1 shown in figure. Let a point object ‘O’ is placed
formula we have on the axis of the combination.
v u f v 30 20
ie. v 60cm
Page 424
RAY OPTICS PART – 3
d
f1 f2 f1 f2
O
I I1
u v
v1
First lens of focal length f1 form the
image I1 of the object ‘O’ at a distance v 1
L1 L2
from it. Then
1 1 1 i) The distance of equivalent lens from second
1
v 1 u f1 fd
lens L2 is f towards the object if the value is
Now the image I1 will act as an object for sec- 1
ond lens and the second lens forms image I at positive and away from the object if the value
a distance ‘v’ from it, then is negative
ii) The distance of equivalent lens from the first
1 1 1
2 fd
v v1 f 2 lens L1 is f away from the object, if the value
So adding (1) and (2) equations we have 2
is positive and towards the object if the value is
1 1 1 1 1 1 1
or negative. It is noted that f, f1 and f 2 are to be
v u f1 f 2 v u F
substituted according to sign convention.
1 1 1 4) If the medium on either side of the lens is air
so f f f and the medium between the lens is having
1 2
refractive index , we can imagine that the
i.e., the combination behaves as a single lens of
rays emerging from the first lens are incident on
equivalent focal length “F” given by
the second lens as if they have traversed a thick-
1 1 1
d 1 1 1 d /
f f1 f 2 ness in air. Hence
F f1 f 2 f1 f 2
This derivation is valid for any number of thin
lenses in contact co-axially. d
P p1 p2 P1 P2
1 1 1 1 1
......... 5) A plane glass plate is constructed by combin-
f f1 f 2 f 3 fn
ing a plano-convex lens and a plano-concave
In terms of power Pnet P1 P2 P3 .....Pn lens of different materials as shown in figure.
Here focal length values are to be substituted ( C is the refractive index of convergence lens,
with sign.
2) If two thin lenses of equal focal length but of D is the refractive index of divergent lens and
opposite nature are paired in contact, the re- R is the radius of curvature of common inter-
sultant focal length of the combination will be face).
1 1 1
0 i.e. F and P 0
F f f
3) If two thin lens are separated by a distance d ,
1 1 1 d
then f f f f f and
1 2 1 2
Pe P1 P2 dP1P2
Page 425
RAY OPTICS PART – 3
By lens maker’s formula and the whole arrangement is placed in a liquid
1 1 1 C 1 of refractive index liquid then this is equivalent
C 1 1 and
fC R R to combination of two lenses kept in contact in
1 1 1 D 1 a medium. Now
D 1 2
fD R R 1 1 1
Now as the lenses are in contact,
f f1 f 2
1 1 1 C D R
;f 1 1 1 1
f fC f D R C D where f 1
R1 R2
As C D , the system will act as lens. The 1 liquid
system will behave as convergent lens if 1
1 1
C D (as its focal length will be positive) and 1 1
f 2 m liquid R ' R '
as divergent lens if C D (as f will be nega- 1 2
R1 R2 R1 R2
air air
1 2
1 1 1
liquid 1
fliquid R2 R1 '
1 1 1
2 1
f2 R1 ' R2 '
If the effective focal length f of the combination
is +ve then the combination behaves like con-
verging lens, if f is -ve then the combination
behaves like diverging lens. Illustration 23:Two thin lenses, when in con-
7) If two convex lenses made of materials of re- tact, produce a combination of power +10
fractive indices 1 and 2 are kept in contact diopter. When they are 0.25m part, the
Page 426
RAY OPTICS PART – 3
w R
But by lens-maker’s formula 1 2 ; substituting the values we
60
1 3 1 1 1 4 15 1
1 get 1 2
f1 2 20 40 3 60 3
1 4 1 1 5 Illustration 26: A converging lens of focal
1
f 2 3 20 30 180 length 5.0cm is placed in contact with a
diverging lens of focal length 10.0cm. Find
the combined focal length of the system.
Page 427
RAY OPTICS PART – 3
second refracting
1 2 edge
first refracting
F 10.0cm ie. the combination behaves
surface
surface
as a converging lens of focal length 10.0cm.
Illustration 27: Two thin converging lenses are
placed on a common axis, so that the cen- Incident A
ray emergent
tre of one of them coincides with the focus ray
of the other. An object is placed at a dis-
Principal section
tance twice the focal length from the left- of prism
side lens. Where will its image be formed? The plane surfaces on which light incident and
What is the lateral magnification? The fo- emerges are called refracting faces.
cal length of each lens is f. The angle between the faces on which light is
Solution: incident and from which it emerges is called
refracting angle or apex angle or angle of
the prism (A).
The two refracting surfaces meet each other in
a line called refracting edge.
A section of the prism by a plane perpendicular
to the refracting edge is called principal section
O
2f f Angle of deviation means the angle between
emergent ray and incident ray. While measuring
the deviation value in anticlockwise direction is
taken as positive and clockwise direction is
taken negative.
The image formed by first lens will be at dis-
tance 2f with lateral magnification m1 1 .For A
second lens this image will behave as a virtual
1 1 1
object. Using the lens formula, we Incident emergent
v u f ray ray
1 1 1 = deviation angle
have, If refractive index of the material of the prism is
v f f
equal to that of surroundings, no refraction at
f v f / 2 1 its surfaces will takes place and light will pass
v ; m2 2
2 u2 f 2 through it undeviated ie. 0 .
Therefore, final image (real) is formed at a dis- vii) Generally we use equilateral or right angled
tance f/2 right side of the second lens with total or Isosceles prism.
lateral magnification,
Determination of Refrative index of
1 1 material of the prism for minimum
m m1 m2 1 deviation:
2 2
Refraction through Prism:
Prism is a transparent medium bounded by any
number of surfaces in such a way that the
surface on which light is incident and the surface
from which the light emerges are plane and non-
parallel as shown in figure.
Page 428
RAY OPTICS PART – 3
X
A vi) We know that r1 r2 A, r r A, r A/ 2
N1 N2
also i1 i2 A, min i i A i1 i2 i
min A
2i min A; i
P 2
S
Y Z sin i sin i1 sin i2
Angle of deviation through a prism By Snell’s law sin r sin r sin r
1 2
i1 i2 A and A r1 r2
A min A min
Minimum Deviation sin p
sin
2 2
From the equation i1 i2 A , the angle sin
A m sin
A
of deviation depends upon angle of incidence 2 2
Page 429
RAY OPTICS PART – 3
B C
Apply Snell’s law at face AB,
1 sin 900 sin r1
1 1
r2 C 1 ie. r1 sin ; or r1 C 2
But as in a prism r1 r2 A; r1 A r2
From eq. (1) and (2) ; r1 r2 2C 3
So r1 A C ie. r1 A C 2
Now from Snell’s law at face XY, we have But r1 r2 A 4
1sin i1 sin r1
Page 430
RAY OPTICS PART – 3
A 2C 5 then
A
A A A 1
C or sin sin C sin
2 2 2 c c
A
i.e. cos ec 6
2
i.e., A ray of light will not emerge out of a prism
(what ever be the angle of incidence) if
A (i) i1 i2 900 (ii) r1 r2 C
A 2 C , if cos ec
2
(iii)Angle of prism A 2 C
(or) cot A / 2 1
2
(iv) Deviation d 180 2C 180 A
Limiting Angle : In order to have an emer-
Prism formula A r1 r2
gent ray, the maximum angle of the prism is 2C ,
i1 i2 A are not always applicable
where C is the critical angle of the prism w.r.t
the surrounding medium2C is called the limit- i1 A
ing angle of the prism. r1
If the angle of incidence at first surface i is such
that r2 i2
a) If i sin 1 sin A C , then the ray
grazes at the other surface. In this figure A r2 r1 and i1 i2 A
1
b) If i sin sin A C , then the ray Image formation by a prism:
emerges out of a prism from the other surface. A small angled prism forms a virtual image for
1
small objects.
c) If i sin sin A C , then the ray un-
der go TIR at the other surface. I
Normal incidence- grazing emer- h
gence: If the incident ray falls normally on
O
the prism and grazes from the second sur-
face, then a
h
tan ; h a
a
‘h’ height of the image formed by the prism.
Illustration 28: An equilateral glass prism is
c made of a material of refractive index 1.5.
Find its angle of minimum deviation.
(Given sin 48035' 0.75 )
Page 431
RAY OPTICS PART – 3
0 600 min 0 r2 A; r2 A
1.5sin 30 sin
2 When the emergent light grazes the second sur-
face , r2 becomes the critical angle C
60 0 min
sin 1.5 0.5000 0.75 1 1
2 i.e. C A and
sin C sin A
600 min 0 Illustration 32: A ray of light passing through
48 35'
2 a prism having 2 suffers minimum
0 0 0
60 min 97 10 ' min 37 10 ' deviation. It is found that the angle of in-
cidence is double the angle of refraction
Illustration 29:A prism of refracting angle 40 within the prism. What is the angle of
is made of a material of refractive index prism.
1.652, Find its angle of minimum devia- Solution: As the prism is in ‘The position of mini-
tion.
mum deviation’, m 2i A with r A / 2
Solution: A 40 , 1.652, ?
According to given problem,
0
A 1 4 1.652 1 i 2r A as r A / 2
0 0
4 0.652 2.608 m 2 A A A and hence from
Illustration 30: A ray of light is incident nor-
mally on one of the faces of a prism of apex sin A / 2 sin A
i.e 2 or
angles 300 and refractive index 2 . The sin A / 2 sin A / 2
angle of deviation of the ray is......degree
A A A
Solution: Apply Snell’s law of refraction at P 2 sin 2sin cos
2 2 2
A 1
i.e, cos or A 450 ie. A 900
2 2 2
Illustration 33: A ray of light is incident at an
angle of 600 on one face of prism of angle
300
300 . The ray emerging out of the prism
makes an angle of 300 with the incident
ray. Show that the emergent ray is perpen-
sin 300 1 dicular to the face through which it
emerges and calculate the refractive index
sin r 2
of the material of the prism.
1 1
or sin r 2 sin 450 or r 450 Solution: According to given problem,
2 2
A 300 , i 600 and 300 and as in prism ,
r 300 450 300 150
Deviation of ray =150 i1 i2 A ; 300 600 i2 300 ;i.e i2 00
Illustration 31: A ray of light is incident nor-
mally on one of the refracting surfaces of So the emergent ray is perpendicular to the face
a prism of refracting angle A. The emer- from which it emerges.
gent ray grazes the other refracting sur- Now as i2 0, r2 0;
face. Find the refractive index of the ma-
terial of prism.
But as r1 r2 A, r1 A 300
Solution: For normal incidence on one of the re-
fracting faces of the prism, i1 0 and r1 0 . So at first face 1sin 600 sin 300 ie 3
But r1 r2 A , when light undergoes refrac-
tion through a prism.
Page 432
RAY OPTICS PART – 3
Illustration 34: A 600 prism has a refractive Solution: Since the emergent light slides along the
index of 1.5. Calculate (a) the angle of in- face AC , angle of emergence is 900 , as shown
cidence for minimum deviation, (b) the . It implies that angle of incidence ray of the ray
angle of emergence of light at maximum that falls on face AC is equal to the critical angle
deviation. (Given sin 1 0.75 490 , C r2 C 1
From the prism theory, we know
2
1
sin 420 and sin 1 0.465 280 .
3
Solution: (a) As the prism is in the position of mini-
mum deviation, r A / 2 60 0 / 2 30 0 ,
so that at either face sin i 1.5sin 300 0.75 0
r1 r2 A 900 r2 90 r1 2
or i sin 1 0.75 490
From the equations (1) and (2) 900 r1 C
Note: In this situation angle of emergence is equal
to angle of incidence = 490 and deviation sin 90 0 r1 sin C or cos r1 sin C
m 2i A 2 49 60 380 1 1
But sin C cos r1
b) For maximum deviation, i1 900 so that Applying Snell’s law at the boundary
2 0 1
r1 C sin 1 420 , But as in a prism AB,1sin 45 sin r1 1
2
3
1
r1 r2 A so r2 A r1 600 420 180 2 1 or 2 3 / 2 1.5 1.5
2
Now applying Snell’s law at the second face, Illustration 36: The refractive index of a prism
is 2. This prism can have what maximum
3
sin r2 sin i2 , i.e., sin180 sin i2 refracting angle, such that light emerges
2 from the prism?
Soluton: Critical angle
ie. i2 sin 1 1.5 0.31 sin 1 0.465 280
1 1
Illustration 35: Monochromatic light falls on a C sin 1 sin 1 30 0
2
right angled prism at an angle of incidence
450 . The emergent light is found to slide If A 2 C the ray does not emerge from the
along the face AC. Find the refractive index prism. So, maximum refracting angle can be
of material of prism. 600 .
Illustration 37: For an equilateral prism, it is
observed that when a ray strikes grazing
N at one face it emerges grazing at the other.
Then find it’s refractive index.
A
Solution: i1 i2 900 , r1 r2 300
2
Page 433
RAY OPTICS PART – 3
A Dispersive Power :
R v Screen
White light y
Dispersive power indicates the ability of the ma-
R
O terial of the prism to disperse the light rays. It is
Y
G the ratio of angular dispersion of two extreme
B
I colours to their mean deviation
V
Angular dispersion
In visible spectrum the deviation and the re-
Mean deviation
fractive index for the yellow ray are taken as
the mean values. If the dispersion in a medium v R
takes place in the order given by “VIBGYOR”
it is called normal dispersion. If however, the v R ... (i)
dispersion does no t follo w the rule 2
“VIBGYOR”, it is said to be anomalous dis- But the mean colour of red and violet is yellow,
persion. A medium which brings about disper-
sion is called dispersive medium. v R
so y ... (ii)
Prism that separated light accordance to wave- 2
length are known as dispersive prisms. Disper-
v R
sive prism are mainly used in spectrometers to
... (iii)
separate closely adjacent spectral lines. Prisms y y
Page 434
RAY OPTICS PART – 3
Fraunhofer’s lines in the solar spectrum. The materials and specified angles such that rays of
C,D and F lines lie in the red , yellow and blue white light may pass through the combination
regions of the spectrum respectively and their without dispersion, though it may suffer devia-
0 0 tion. Such a combination is called achromatic
wavelengths are 6563 A , 5893 A and combination.
0
4861 A respectively. Then the dispersive i.e 0 and 0
power may be expressed as,
v R A 'v 'R A' 0
F C F C
D 1 where D
2 v R A 'v 'R A
y 1 ' 1 0
y
It is noted that a single prism produces both y 1 ' 1
y
Page 435
RAY OPTICS PART – 3
=0
A R
Y
White light V
A
y 1
v R A The glass slabs forming the prism are very thin
v R and permit the rays to pass undeviated.
' y 1
'v ' R A ' 0
... (ii) Hence a hollow prism gives no spectrum.
'v ' R Illustration 39:White light is passed through
a prism of angle 50 . If the refractive indi-
C f ces for red and blue colours are 1.641 and
ie. 0 ... (iii) 1.659 respectively. Calculate the angle of
C f
dispersion between them.
In this case the dispersion produced by flint
Solution: As for small angle of prism 1 A
glass prism is opposite to crown glass prism.
Therefore the net angular dispersion b 1.659 1 50 3.2950 and
C f
r 1.641 1 50 3.2050 so
V R A 'V 'R A ' or
b r 3.2950 3.2050 0.0900
V R A 'V 'R A ' Illustration 40:The refractive indices of flint
y 1 ' 1
y
y 1 ' y 1 glass prism for C,D and F lines are 1.790,
1.795 and 1.805 respectively. Find the dis-
C C f f persive power of the flint glass prism.
Solution:
Illustration 38:A beam of white light passing C 1.790, D 1.795 and F 1.805
through a hollow prism gives no spectrum
F C 1.805 1.790 0.015
why? 0.1887
D 1 1.795 1 0.795
Solution: Light travels from air to air in case of
hollow prism. No refraction and no disper-
sion occur.
Page 436
RAY OPTICS PART – 3
Illustration 41: A thin prism P1 with angle 40 are now added as shown in figure. The ray
and made from glass of refractive index 1.54 will suffer
is combined with another prism P2 made Q
1.524 1.514 80 0.080 40 1and 2 be the refractive indices for the D
A' line of flint and crown glasses respectively. (a)
1.665 1.645 0.02
If 1 and 2 be the angles of deviations due to
1.514 1.524 the flint and crown glass prisms respectively,
For crown glass 1.519
2 then for no deviation of D line, condition is
1.645 1.665 1 2 0; A1 1 1 A2 2 1 0
For flint glass 1.655
2
A1 1
The net deviation 2
A2 1 1
' 1 A ' 1 A '
The negative sign indicates that A1 and A2 are
= 0.159 80 0.655 40 1.530 oppositely directed.
Illustration 43: A given ray of light suffers
minimum deviation in an equilateral A1 1.530 1 0 0.530
0
; A1 6 40
prism P. Additional prism Q and R of iden- 6 1.795 1 0.795
tical shape and of the same material as P b) Angular dispersion due to the flint glass prism
Page 437
RAY OPTICS PART – 3
Page 438
RAY OPTICS PART – 3
Page 439
RAY OPTICS PART – 3
This defect is rectified by using spectacles However, when eye is suffering from some
having convergent lens(i.e convex lens) which other defect along with astigmatism, the spectacle
forms the image of near objects at the near point lens surface is made toroidal. The two radii of
of the defected eye (which is more than 25cm) curvature are chosen as to compensate for that
1 1 1 defect and the astigmatism at the same time
P
N .P. dis tan ce of object f Illustration 45: A person cannot see distinctly
N.P.= Near Point of defective eye. any object placed beyond 40cm from his
If the object is placed at D=25cm=0.25m eye. Find the power of lens which will en-
able him to see distant stars clearly is?.
1 1 1 Solution: The person cannot see objects clearly
P
f 0.25 N .P. beyond 0.4m.
Presbyopia: The power of accommodation so his far point = 0.4m , distance of object =
of eyelens may change due to the decreasing .
effectiveness of ciliary muscles. So, far point is He should use lens which forms image of dis-
lesser than infinity and near point is greater than tant object u at a distance of 40cm
25cm and both near and far objects are not
clearly visible. This defect is called infront of it.
presbyopia.This defect is rectified by using 1 1 1
bifocal lens. p;
0.40 f
ASTIGMATISM
10
P 2.5D
It is yet another common defect of vision, which 4
occurs when cornea has unequal curvatures in Illustration 46: A far sighted person cannot
different directions. For example cornea may focus distinctly on objects closer than 1m.
have larger curvature in the vertical plane than in What is the power of lens that will permit
the horizontal plane or vice – versa. Therefore, him to read from a distance of 40cm?
such a person is not able to focus simultaneously Solution: As near point is 1m and distance of ob-
on horizontal and vertical lines. If horizontal lines jects is 0.40m
are well focused, the vertical lines appear both in front of lens.
distorted or vice – versa.
1 1 1 1 1
From the distribution of the intensity variation, P P 1.5D
f v u 1 0.40
an optometrist deduces the variations in the
curvature of the cornea. Astigmatism is corrected Simple Microscope :
by using a cylindrical lens of suitable radius of To view an object with naked eye, the object
curvature and suitable axis. To correct must be placed between D and infinity. The
astigmatism only, one surface of each lens will maximum angle is subtended when it is placed
be flat and the other surface of each lens will be at D.
a section of circular cylinder, with the axis of Magnifying power of simple
cylinder horizontal. Rays passing through the lens microscope:
in a horizontal plane are unaffected, and rays in The magnifying power or angular magnification
vertical plane are converged by the circular of a simple microscope is defined as the ratio
surface. The extra convergence is such as to of visual angle with instrument to the maximum
compensate for the lack of curvature of the visual angle for unaided eye when the object is
cornea in that direction. at least distance of distinct vision.
The corrected astigmatic eye is shown in fig.
Cylindrical h'
lens
2F h
I O F
u
Image formed v D to
on retina Eye with instrument
Page 440
RAY OPTICS PART – 3
Page 441
RAY OPTICS PART – 3
1 1 25 D
P diopter 20 D i.e MP min 5 M
5 10 m 0.05
2 5 f
and MP will be maximum when
b) For relaxed eye, MP is minimum and will be
u min 25 / 6 cm
D 25
MP 5 25 D
f 5 MP max 6 1
25 / 6 f
While for strained eye, MP is maximum and
Illustration 50: a) The near point of a hyper-
will be
metropic person is 75 cm from the eye. What
D is the power of the lens required to enable
MP 1 1 5 6
f the person to read clearly a book held 25
Illustration 49: A man with normal near point cm from the eye?
(25cm) reads a book with small print us- b) In what way does the corrective lens help
ing a magnifying glass, a thin convex lens the above person? Does the lens magnify
objects held near the eye?
of focal length 5cm.
c) The above person prefers to remove the
a) What is the closest and farthest distance
spectacles while looking at the sky. Ex-
at which he can read the book when view-
plain why?
ing through the magnifying glass?
Solution: a) u=0.25 m, v=0.45m, F=?
b) What is the maximum and minimum
magnifying power possible using the above 1 1 1
; power
simple microscope? F v u
Solution: a) As for normal eye far and near points 1 1 100 100 4
are and 25cm respectively, so for magni- P 4
0.75 0.25 75 25 3
fier vmax and vmin 25cm. . However , P = 2 . 6 7 D
b) The corrective lens produces a virtual image
1 1 1 (at 75 cm) of an object at 25 cm. The angular
for a lens as size of this image is the same as the object. IN
v u f
this sense, the lens does not magnify the object
f but merely brings the object to the near point
ie u f / v 1 so u will be minimum when of the hypermetropic eye which then gets fo-
cused on the retina.
vmin 25cm c) A hypermetropic eye may have normal far
point i.e. if may have enough converging power
5 25 to focus paralllel rays from infinity on the retina
ie. u min 5 / 25 1 6 4.17cm of the shortened eyeball. Wearing spectacles
of converging lenses (used for near vision) will
and u will be maximum when vmax amount to more convering power than needed
for parallel rays. Therefore , distant object may
5
ie u max 5 / 1 5cm get focused in front of the retina, appearing
blurred. Hence the person prefers not to use
so the closest and farthest distances of the book the spectacles for distant objects.
from the magnifier (or eye) for clear viewing
are 4.17cm and 5cm respectively. (b) As in case Illustration 51: A far sighted person can-
of simple magnifier MP=(D/u). So MP will be not focus distinctly on objects closer
minimum when than 1m. What is the power of lens
umax 5cm that will permit him to read from a
distance of 40 cm?
Page 442
RAY OPTICS PART – 3
Solution: As nearpoint is 1m and distnace Note: When the object is very close to the principal
of object is 0.40m both in front of lens. focus F0 of the objective, the image due to the
objective becomes very close to the eyepiece.
1 1 1 1 1
P P 1.5D Then replace u with f0 and v 0 with L so the
f v u 1 0.40 expression for magnifying power.
Compound Microscope
A simple magnifying lens is not useful where L D
M
large magnification is required. A highly mag- f 0 fe
nified image must be produced in two stages.
Case-ii: If the final image is formed at D (Near
A compound microscope is used for that pur- point):
pose.
In this case, for eye piece v e D, ueis ve
Magnifying power:
1 1 1
L D ue f e
u v ue 1 1 D
h0
O
F0 Fe i.e u D 1 f ; m mo me
F0 IM hi e e
v D fD
MD 1 with LD v e
I u fe fe D
f eyelens f objective In this situation as ue is minimum magnifying
power is maximum and eye is most strained.
diameter eyelens diameter objective When the object is very close to the principal
intermidiate image is real, inverted focus F0 of the objective, the image due to the
Final image is magnified and inverted objective becomes very close to the eyepiece.
visual angle with instrument Then replace u with f0 and v with L so the ex-
M
max. visual angle for unaided eye 0 pression for magnifying power.
L D
v D MD 1
M f0 fe
u ue
Some important points regarding
Where u is the object distance for the objec- compound microscope:
tive lens, v is image distance for the objective As magnifying power of a compound
lens, u e is the object distance for the eye piece. microscope is negative, the image seen is always
truly inverted.
i.e M m0 me For a microscope magnifying power is minimum
when final image is at and maximum when
The length of the tube L v ue final image is at least distance of distinct vision
Case(i): If the final image is formed at infinity
(far point): v D
D, and M max 1
u fe
In this case ue f e
For a given microscope magnifying power for
v D normal setting remain practically unchanged if
M with L v f field and eye lens are interchanged as
u fe e
LD
A microscope is usually considered to operate M
in this mode unless stated other wise. In this f0 fe
mode ue is maximum and hence magnifying In an actual compound microscope each of the
objective and eye piece consists of a
power is minimum. combination of several lenses instead of a single
Page 443
RAY OPTICS PART – 3
lens to eliminate the aberrations and to increase sion (25cm)
the field of view. Solution: The magnifying power of a compound
In low power microscopes, the magnifying microscope when the final image forms at the
power is about 20 to 40, while in high power least distance of distinct vision,
microscopes, the magnifying power is about
500 to 2000. v0 D
Illustration 52: A microscope consists of two m 1
u fe
convex lenses of focal lengths 2cm and 5cm
placed 20cm apart. Where must the object To find v0 ; power of the objective
be placed so that the final virtual image is
p0 100 D . Focal length of the objective,
at a distance of 25cm from the eye?
Solution: For t he eyepiece, fo cal length 1 1 100
f f0 m cm 1cm
f f e 5cm;v ve 25cm,u ue ?sub- p0 100 100
stituting in u u0 1.1cm; v v0 ?
1 1 1 1 1 1
; 1 1 1
f v u 5 25 ue For a lens,
f v u
1 1 1 6
1 1 1 1 1 1 0.1
ue 25 5 25 ;
1 v0 1.1 v0 1 1.1 1.1
25 v0 11cm
ue cm
6
Power of the eyepiece, pe 16 D ;focal length
object for the eyepiece is to be at a distance of
of the eye piece.
25
cm to its left. 1 1 100
6 fe m cm 6.25cm
pe 16 16
25
But v0 ue 20cm where ue cm Least distance of distinct vision, D 25cm
6
11 25
25 95 m 1 10 5 50
v0 20 ue 20 cm 1.1 6.25
6 6
Illustration 54: In a compound microscope,
95
For the objective, v v0 cm the object is 1cm from the objective lens.
6 The lenses are 30 cm apart and the inter-
f f 0 2cm; u ? mediate image is 5cm from the eye piece.
1 1 1 1 6 1 What magnification is produced?
;
f v u 2 95 u Solution: As the lenses are 30cm apart and inter-
mediate image is formed 5cm in front of eye
1 6 1 83 190
;u 2.29cm lens,
u 95 2 190 83
The object is to be placed at a distance of ue 5cm and v L ue 30 5 25cm
2.29cm to the left side of the objective. Now as in case of compound microscope,
Illustration 53: Find the magnifying power of
a compound microscope whose objective v D
M m0 m e
has a focal power of 100D and eye piece u ue
has a focal power of 16D when the object here u 1cm and D 25cm
is placed at a distance of 1.1cm from the
objective. Assume that the final image is 25 25
formed at the least distance of distinct vi- So M 125
1 5
Page 444
RAY OPTICS PART – 3
Negative sign implies that final image is inverted. 1 1 1 1 1
ie F ' 2.5cm
Illustration 55: A compound microscope has F ' F f ' 2 10
a magnifying power 30. The focal length This lens will form the image of same object at
of its eyepiece is 5cm. Assuming the final a distance v2 such that
image to be at the least distance of distinct
vision (25cm), calculate the magnification 1 1 1
produced by objective. v2 3 2.5 ie v2 =15cm
Solution: In case of compound microscope, So to refocus the image, eyepiece must be
moved by the same distance through which the
M m0 me 1 image formed by the objective has shifted ie.
And in case of final image at least distance of 15-6=9cm away from the objective.
distinct vision, Illustration 57: The focal lengths of the ob-
jective and the eyepiece of a compound mi-
D croscope are 2.0cm and 3.0cm respectively.
me 1 2
fe The distance between the objective and the
eyepiece is 15.0cm. The final image formed
by the eyepiece is at infinity. Find the dis-
D
so, from eqs. 1 and 2 , M m0 1 tance of object and image produced by the
f e objective, from the objective lens.
Solution: As final image is at infinity, the distance
Here M 30; D 25cm and f e 5cm
of intermediate image from eye lens ue will be
25 30 given by
So, 30 mo 1 ie m0 5
5 6 1 1 1
Negative sign implies that image formed by ob- ue f e ie ue f e 3cm
jective is inverted. and as the distance between the lenses is
Illustration 56: A compound microscope is 15.0cm , the distance of intermediate image
used to enlarge an object kept at a distance (formed by objective) from the objective will
be
0.03m from its objective which consists of
several convex lenses in contact and has v L ue L f e 15 3 12cm
focal length 0.02m. If a lens of focal length and if u is the distance of object from objec-
0.1m is removed from the objective, find tive,
out the distance by which the eyepiece of 1 1 1
the microscope must be moved to refocus ie u 2.4cm
12 u 2
the image? So object is at a distance of 2.4cm in front of
Solution: If initially the objective forms the image objective.
1 1 1 Telescopes : A microscope is used to view the
at distance v1 . ie v 6cm objects placed close to it. To look at distant
v1 3 2 1
objects such as star, a planet or a cliff etc, we
Now as in case of lenses in contact use another optical instrument called telescope,
which increases the visual angle of distant ob-
1 1 1 1 1 1 ject.
........... or
F f1 f 2 F f1 F ' The telescope that uses a lens as an objective
1 1 1
is called refracting telescope. However in many
with F ' f ........ telescopes we use curved mirrors as objec-
2 f3
tives such telescopes are known as reflecting
So if one of the lenses is removed, the focal
telescopes. There are three types of refracting
length of the remaining lens system
telescopes in use.
Page 445
RAY OPTICS PART – 3
i) Astronomical telescope f0 fe
MD 1
ii) Terrestrial telescope f e D
iii) Galilean telescope In this case length of the tube
Astronomical Telescope : fD
Magnifying power (M): LD f 0 e
fe D
Magnifying power of a telescope is given by
In this situation ue is minimum , hence magnify-
A fo ue ing power is maximum while the length of the
tube is minimum and eye is most strained.
Some important points regarding
B astronomical telescope:
B'
O Q E In case of telescope if object and final image
are at infinity and total light entering the telescope
A' leaves it and parallel to its axis.
P f 0 A0
ue D to
magnifying power
Visual angle with instrument f e Ae
M
Visual angle for unaided eye 0 where A0 and Ae are the apertures of objec-
tives and eyepiece respetively.
h
From the above figure, 0 As magnifying power is negative, the image
f0 seen in astronomical telescope is truly inverted
i.e left is turned right with upside down
h simultaneously. However as most of the
u astronomical objects are symmetrical this
and h ; M e f 0 inversion does not effect the observations.
ue 0 h ue
For given telescope, magnifying power is
f0 minimum when final image is at infinity (Far
point) and maximum when it is at least distance
The length of the tube L f 0 ue of distinct vision (Near point) ie.
Case-i: If the final image is at infinity f f f
(far point):In this case, for eyepiece M min 0 and M max 0 1 e
fe fe D
1 1 1
v e , ue ve ; As magnifying power for normal setting as
ue f e
f0
Hence ue f e to have large magnifying power f0 must
fe
f0
Hence M and L f 0 fe be as large as practically possible and fe is
fe small. This is why in a telescope, objective is of
Usually a telescope is operated in this mode large focal length while eyepiece of smaller focal
unless stated other wise. In this mode ue is maxi- length.
mum, hence magnifying power is minimum, Larger aperture of objective helps in improving
while length of tube is maximum. the brightness of image by gathering more light
This case is also called normal adjustment be- from the distant object. However it increase
cause in this case eye is least strained or re- aberrations particularly spherical.
laxed. If a fly is sitting on the objective of a telescope
and we take a photograph of distant
Case-ii:If the final image is at D (Near point): astronomical object through it, the fly will not
In this situation for eyepiece v e D be seen but the intensity of the image will be
slightly reduced as the fly will act as obstruction
1 1 1 1 1 fe to light and will reduce the aperture of the
ie 1 D
D ue f e ue f e objective.
A telescope produces angular magnification
Page 446
RAY OPTICS PART – 3
whereas a microscope produces linear
magnification. The image due to a telescope
appears to be near to the eye increasing the is diverging lens.
visual angle. f0 fe
2) Magnification: mD f 1 D and
e
S. Microscope Telescope
no f
m 0
1 It is used to see It is used to see fe
very distant objects
small objects 3) Length : LD f 0 ue and L f 0 f e
2 Its objective is of Its objective is Illustration 58: An astronomical telescope has
small focal length of large focal an angular magnification of magnitude 5
and of small length and of
aperture. large aperture for distant objects. The separation between
3 It produces linear It produces the objective and eye piece is 36cm and the
magnification angular final image is formed at infinity. Determine
and size of the magnification
the focal length of objective and eye piece.
image is larger and the image is
than that of the near to the eye, Solution: For final image at infinity,
object. but the size
does f0 f0
M 5 1
not increase. f e and L f 0 fe fe
Terrestrial Telescope: The magnifying
power and length of a telescope for relaxed and 36 f 0 f e ii
eye will be
Solving these two equations , we have
f f
M 0 1 0 , L f f 4 f f 0 30cm and f e 6cm
fe fe 0 e
Page 448
RAY OPTICS PART – 3
ve 25 f0 f e 60 3
me 6 M
ue 25 fe 1 D 3 1 25 22.4
so
6
14 4 Now as by definition M ,
and hence, m m0 me 10 6 0
3
so the angular size of image
I
But as m 1
0
Page 449
RAY OPTICS PART – 3
2) Mirrors can be constructed with consider- i.e. intensity of scattered light varies inversely
able large diameters than lenses. as the fourth power of the wavelength of
3) Mirrors are free from chromatic and spheri- incident light. These conclusions were verified
cal aberrations experimentally by Tyndall.
Note: The latest telescope in India in Kavalur, Rayleigh established further that the rays do not
Tamilnadu. It is a 2.34 m diameter cassegrain undergo any change in wavelength on
telescope scattering.
Note: The largest telescope in the world are However, in case of scattering of light by
the pair of keck telescoep in Hawaii (USA) transparent media, the scattered light might
with a reflector of 10m in diameter. contain some frequencies or wavelengths,
Hubble space telescope is a cassegrain type which might not be present in the incident
telescope radiations. When some energy from incident
SCATTERING OF LIGHT: light is imparted to the scattering molecule, the
Most of the beautiful phenomena like ‘blue scattered light will have lesser energy, i.e.,
colour of Sky’. ‘white colour of Cloud’, ‘red smaller frequency and longer wavelength. On
colour of Sunrise and Sunset’, have been the contrary, when the scattering molecule is in
explained in terms of scattering of light. a higher energy state before the collision than
after the collision some energy is received from
As sunlight travels through the earth’s
the scattering molecule. The scattered
atmosphere, it gets scattered by the large
light will have higher energy, i.e., longer
number of molecules and numerous small
frequency and shorter wavelength.
objects present in the atmosphere. Scattering
represents basically change in the direction However, when there is no exchange of energy
of light. between the incident light and the scattering
molecule, there is no change in wavelength of
Lord Rayleigh was the first to study scattering
frequency on scattering, aspostulated by
of light from air molecules.
Rayleigh.
If A = amplitude of incident light,
Another important factor in scattering is relative
= wavelength of incident light,
size of the wavelength of light and the
V = volume of the scattering particle
scatterer (of typical size, say, x), For x<< ,
a = amplitude of scattered light at a distance r
Rayleigh scattering is valid, i.e., when size of
from the scattering particle, then according to
scatterer is much smaller than wavelength of
Rayleigh,
When size of scatterer is much smaller light, I s I / 4 . But for x>> , Rayleigh
than wavelength of light, then scattering is not valid and all wavelengths are
scattered nearly equally.
AV
a 2 Some applications of scattering of
r
light are :
As intensity of sctatterd light I s varies directly (a) Blue colour of sky is due to scattering of
sunlight. Light from the sun, while travelling
as square of amplitude of scattered light a , through earth’s atmosphere, gets scattered by
2
Page 450
RAY OPTICS PART – 3
light. As blue colour has a shorter wavelength red colour on the outside (making an angle of
than red, therefore, blue colour is scattered 420 and violet colour on the inside (making an
much more strongly. Hence the sky looks blue. angle of 400. The primary rainbow is formed
(b) White colour of clouds. The clouds are at by the rays which undergo one internal reflection
much lower height. They are seen due to and two refractions before emerging finally from
scattering of light from lower parts of the the water droplets. The outer of the two bows
atmosphere, which contains large dust is called secondary rainbow. It is not as bright
particles, water droplets, ice particles etc. in as the primary one. It subtends an angle of about
this case, size of scatterer x>> . Therefore, 51.50 at the eye of the observer. It also shows
all wavelengths are scattered nearly equally. All all the colours of the solar spectrum but in
colours scattered equally merge to give us the reverse order i.e. red on the inner edge making
sensation of white. Hence clouds generally an angle of 500 and violet on the outer edge
appear white. making an angle of 530. Secondary rainbow is
formed by the rays which have undergone two
(c) The sun looks reddish at the time of sun internal reflections and two refractions before
rise and sun set. At the time of sun rise and emerging finally from the water droplets.
sun set, the sun is near the horizon. The rays
from the sun have to travel a larger part of the
atmosphere.
As b r , and intensity of scattered light
1
, therefore, most of the blue light is
4
scattered away. Only red colour, which is least
scattered enters our eyes and appears to come
from the sun. hence the sun looks red, both at
the time of sun rise and sun set. Sunlight
(d) Danger signals are red. Because wavelength 1
red colour is large and intensity of scattered
Raindrops
light varies inversely as the fourth power of 2
wavelength. Therefore, red colour is least
scattered. It can be seen from maximum 400
distance. That is why danger signals are red. observer 420
RAINBOW:
(b)
A rainbow is a spectrum of sun’s light in
nature. It occurs in the form of concentric
coloured circular arcs in the sky, when the
sun shines on rain drops during or after a
shower.
The essential condition for observing a
rainbow is that the observer must stand
with his back towards the sun.
The common centre of concentric arcs lies on
the line joining the Sun to the eye of the observer.
Generally, there are two kinds of rainbows:
1. Primary rainbow,
2. Secondary rainbow
The inner of the two bows seen is the primary
rainbow. It is brighter of the two and subtends
an angle of 420 at the eye of the observer. It
shows all the colours of the solar spectrum with
Page 451
Page 452
WAVE OPTICS
WAVE OPTICS
INTRODUCTION HUYGEN’S WAVE THEORY OF LIGHT
By now, you are well versed with laws that This theory was enunciated by Huygen’s in a
govern the formation of images through lens, hypothetical medium known as luminiferous
mirrors and various optical instruments. At what ether. Ether is that imaginary medium which
you have read was based on the fact that light prevails in all space, in isotropic, Perfectly
travels in straight line. But from this chapter you elastic and massless. The different colours of
will get to know that light does not always light are due to different wave lengths of these
travel in straight line, indeed light is a wave. waves. The velocity of light in a medium is
Hence light is a wave and it exhibit interferes, constant but changes with change of medium.
diffracts and even undergo polarization. This This theory is valid for all types of waves.
new branch of physics that deals with the wave (i) The locus of all ether particles vibrating in same
nature of light is called “Wave Optics”. phase is known as wave front.
NATURE OF LIGHT NEWTON’S CORPUSCULAR (ii) Light travels in the medium in the form of
THEORY OF LIGHT: wavefront.
This theory was given by Newton. (iii) When light travels in a medium then the particles
Important Characteristics of the theory is as of medium start vibrating and consequently a
follows disturbance is created in the medium.
(i) Extremely minute, very light and elastic (iv) Every point on the wave front becomes the
particles are being constantly emitted by all source of secondary wavelets. It emits
luminous bodies (light sources) in all directions. secondary wavelets in all directions which travel
with the speed of light.
(ii) These corpuscles travel with the speed of light.
(v) The tangent plane to these secondary wavelets
(iii) The different colours of light are due to different
represents the new position of wave front.
size of these corpuscles.
THE PHENOMENA EXPLAINED BY THIS THEORY
iv) Reflection of light is due to force of repulsion
applied by particle at denser medium on the (i) Reflection, refraction, interference, diffraction,
corpuscles Polarisation and double refraction.
v) The laws of refraction were explained by (ii) Rectilinear Propagation of light.
assuming that the particle of denser medium (iii) Velocity of light in rarer medium being greater
attract the particle of light causing a bending at than that in denser medium.
the surface PHENOMENA NOT EXPLAINED BY THIS THEORY
THE PHENOMENA EXPLAINED BY THIS THEORY. (i) Photoelectric effect, Compton effect and
(i) Reflection and refraction of light. Raman effect.
(ii) Rectilinear propagation of light (ii) Backward propagation of light
(iii)Existence of energy in light due to force of attraction, which is against the
THE PHENOMENA NOT EXPLAINED BY THIS THEORY experimental observation by foucault
(i) interference, diffraction, polarisation, double GEOMETRICAL OPTICS
refraction and total internal reflection. 1. In geometrical optics, light is assumed to be
(ii) velocity of light being lesser in denser medium travelling in a straight line. This property is
than that in a rarer medium due to force of known as rectilinear propagation.
attraction. 2. By using rectilinear propagation of light, laws
(iii) Photoelectric effect and Compton effect. of reflection, refraction, total internal reflection
etc are explained geometrically.
Page 453
WAVE OPTICS
PHYSICAL OPTICS OR WAVE OPTICS: such points, which are equidistant from the point
1. In physical optics, light is considered as a wave source, is a sphere (a)
2. Huygen’s wave principle and principle of Note:
superposition are used to explain interference 1
and diffraction. Amplitude of light wave, A
r
3. Electromagnetic wave nature of light is used to 1
explain the concept of polarisation. Intensity of light wave , I 2
r
Condition for applicability of geometrical
optics and wave optics: When the size of
the object interacting with light, is much larger
than the wavelength of light, we can apply
geometrical optics.
When the wavelength of light is comparable to
or larger than the size of the object interacting
with light, we can apply wave optics.
If ‘b’ is the size of the object interacting with (a) (b) (c)
light, ‘l’ is the distance between the object and 2. CYLINDRICAL WAVE FRONT:
the screen and ‘ ’ is the wavelengh of light When the source of light is linear in shape (such
then, as a slit), a cylindrical wavefront is produced.
It is because, all the points, which are
i) The condition for applicability of geometrical equidistant from the linear source, lie on the
b2 surface of a cylinder (b).
optics is 1
l 1
ii) The condition for applicability of wave optics Amplitude of light wave, A ,
r
is
1
b2 b2 Intensity of light wave , I
1 or 1 r
l l 3. PLANE WAVE FRONT :
Note: The object interacting with light may be a A small part of a spherical or a cylindrical
mirror, a lens, a prism, an aperture (pin hole), a wavefront originating from a distant source will
slit and a straight edge. appear plane and hence it is called a plane
WAVE FRONT: wavefront (c).
According to wave theory of light, a source of Intensity of light wave , I r 0 ,
light sends out disturbance in all directions. In Amplitude of light wave, A r 0
a homogeneous medium, the disturbance HUYGEN’S PRINCIPLE:
reaches to all those particles of the medium in
Every point on the wave front becomes a source
phase, which are located at the same distance of secondary disturbance and generates
from the source of light and hence at any instant, wavelets which spread out in the medium with
all such particles must be vibrating in phase with the same velocity as that of light in the forward
each other. direction only.
The locus of all the particles of the medium, 1. The envelope of these secondary waves at any
which at any instant are vibrating in the same instant of time gives the position of the new
wave front at that instant.
phase, is called the wavefront.
2. The wave front in medium is always
Depending upon the shape of the source of light, perpendicular to the direction of wave
wavefront can be of the following types propagation.
1. SPHERICAL WAVE FRONT:
A spherical wavefront is produced by a point
source of light. It is because, the locus of all
Page 454
WAVE OPTICS
v1t vt sin i v1
2 ...(iii )
sin i sin r sin r v2
Now, if r < i (i.e., ray bends towards the
B normal)
1 i i sin r < sin i
i
A r D sin i v
r 1 1 1 v1 v2
r sin r v2
2
C i.e., speed of light in medium 1 is greater than
that in medium 2. This is exactly what we
AB is width of incident beam studied in Geometrical (Ray) optics. This
prediction is opposite to the prediction as per
CD is width of refracted beam
the Newton’s Corpuscular Theory. Also if 1
width of incident beam cos i
be the refractive index of light in medium 1,
width of refracted beam cos r
then
APPLICATIONS OF HUYGENS’
PRINCIPLE TO STUDY REFRACTION c c
1 v1
AND REFLECTION v1 1
REFRACTION FROM RARER TO DENSER MEDIUM : c c
Let v1 and v2 represents the speed of light in similarly, 2 v and v2
2 2
medium – 1 at medium – 2 respectively.
Consider a plane wavefront PQ – propagating sin i v1 2
in the direction P1P, incident on the medium hence, from e.q (iii), sin r v
boundary at point P at an angle of incidence i. 2 1
Let t be the time taken to travel from Q to B. 1 sin i 2 sin r Snell’s law of refraction
Also, since same phase is achieved if the wave
travels a distance equal to integral multiple of
the wavelength, hence
QB n1 v1t and PR n2 v2t
On dividing , we have
v sin i 2
1 1
2 v2 sin r 1
Also, v1 1 f and v2 2 f
The frequency of the wave does not change as
QB v1t it travels from one medium to another as the
frequency is source dependent.
From the point P , draw a sphere of radius v2t Illustration 1:
, let BR represent the forward tangent plane. It A plane wavefront is incident at an angle
is refracted wavefront at t. of 370 with horizontal a boundary of re-
PR v2t ; From fractive medium from air 1 to a me-
QB v1t vt 3
PQB ,sin i PB 1 ....(i ) dium of refractive index . Find the
PB PB sin i 2
PR v2 t angle of refracted wavefront with horizon-
From PRB , Sinr tal.
PB PB Solution :
vt It has been given that incident wavefront makes
PB 2 ...............(ii)
sin r 370 with horizontal. Hence incident ray makes
Equating (i) and (ii), we have 370 with normal as the ray is perpendicular to
the wavefront.
Page 455
WAVE OPTICS
Page 456
WAVE OPTICS
Note that in the above formulae v is the
v
(neglecting higher powers of ) component of the source velocity along the
c line joining the observer to the source relative
v to the observer. v is positive when source
f A f 1 moves relatively away from observer and v is
c negative when source moves towards the
BLUE SHIFT: When the distance between the source observer. The Doppler effect for the
and observer is decreasing (i.e. the source is
approaching the observer) then frequency of light is very important in astronomy. It is the
light appears to be increasing or wavelength basis for the measurements of the radial
appears to be decreasing i.e. the spectral line velocities of distance galaxies.
in electromagnetic spectrum gets displaced When galaxy is approaching,
towards blue end, hence it is known as blue Vradial
shift. A C
CASE-2: if the source or the observer recede from Vradial
each other When galaxy is moving away ,
C
cv
fR f Source of light moves Source of light moves
cv towards the stationary away the stationary
observer (v<<c) observer (v<<c)
(I) Apparent frequency (I) Apparent frequency
S O
v v
f A f 1 and fA f 1 and
c c
v Apparent wavelength Apparent wavelength
which again reduces to f R f 1 for v
c so A 1 v so A 1 v
c c
<< c, here f R represents frequency of
Doppler’s shift: Apparent Doppler’s shift: Apparent
recession. wavelength < actual wave wavelength < actual wave
The same relationship can be transformed in length, so spectrum of the length, so spectrum of the
terms of the wavelength. radiation from the source radiation from the source
of light shifts towards the of light shifts towards the
c violet and of spectrum. red and of spectrum.
This is called violet or blue This is called red shift.
f shift. v Doppler’s shift v
c
1 Doppler’s shift c
c c v
1 A 1 v Note: Doppler’s shift and time period of
A c c
rotation (T) of a star relates as
v 2r
A 1 ; r radius of star..
c c T
v v Applications of Doppler effect:
A ; (i) Determination of speed of moving bodies
c c (aeroplane, submarine etc) in RADAR and
RED SHIFT: When the distance between the source SONAR.
and observer is increasing (i.e. the source is (ii) Determination of the velocities of stars and
receding from the observer) then frequency of galaxies by spectral shift
light appears to be decreasing or wavelength (iii) Detemination of rotational motion of sun
appears to be increasing i.e. the spectral line in (iv) Explanation of width of spectral lines
electromagnetic spectrum gets displaced
towards red end, hence it is known as red shift. (v) Tracking of satellites.
(vi) In medical sciences in echo cardiogram,
A sonography etc.
Page 457
WAVE OPTICS
Illustration 3: of the medium undergo displacements due to
with what speed should a star move with all the waves simultaneously. The resultant wave
respect to us so that the beam at wave- is due to the resultant displacement of the
length 460.0 nm, is observed at 460.8nm ? particles.
Solution : Principle of superposition of waves states that
when two or more waves are simultaneously
v v impressed on the particles of the medium, the
Now we have, radial
v c resultant displacement of any particle is equal
to the sum of displacements of all the waves.
Now, 460.8 460.0 nm 0.8nm (or)
8 “When two or more waves overlap, the
c 3 10 0.8 nm resultant displacement at any point and at any
Now, vradial
460 nm instant is the vector sum of the instantaneous
displacements that would be produced at the
5.217 105 m / s 521.7 km / s point by individual waves, if each wave were
Since, v comes out to be positive, hence we present alone”.
can conclude that the star is moving away from
us. If y1 , y2 ,......... yn denote the displacements of
Illustration 4: ‘n’ waves meeting at a point, then the
resultant displacement is given by
A galaxy moving with speed 300 km/s
shows blue shift. At what wavelength so- y y1 y2 .... yn .
dium line at 589.0 nm will be observed ? A) SUPERPOSITION OF COHERENT WAVES:
Solution : Consider two waves travelling in space with
an angular frequency . Let the two waves
Here, vradial 300 km / s 3 105 m / s
arrive at some point simultaneously. Let y1 and
Negative sign comes because we observe blue
shift i.e., galaxy is approaching us. y2 represent the displacements of two waves
v at this point.
Now, radial
c y1 A1 sin(t 1 ) & y2 A2 sin(t 2 )
3 105 Then according to the principle of superposition
the resultant displacement at the point is given
589.0nm 3 108 by, y y1 y2 or
589.0 103 nm 0.589 nm y A1 sin(t 1 ) A2 sin(t 2 )
y2 y
Hence, the observed wavelength will be
(589.0-0.589)nm=588.411 nm
Illustration 5: A
A2
What speed should a galaxy move with re-
spect to us so that the sodium line at 589.0
nm is observed at 589.6 nm?
A1 y1
Solution :
v A1 (sin t cos 1 cos t sin 1 )
;
C
A2 (sin t cos 2 cos t sin 2 )
0.6 0.6 A cos .sin t A sin .cosAt sin(t )
v c 3 108
589.0 589.0 Where
3.06 105 ms 1 A cos A1 cos 1 A2 cos 2 .......(1)
Therefore. the galaxy is moving away from us
with speed 306 km/s. and Asin A1 sin 1 A2 sin 2 ........(2)
Principle of superposition of waves: Here A and are respectively the amplitude
If two or more waves meet at a place and initial phase of the resultant displacement
simultaneously in the same medium, the particles Squaring and adding equations (1) & (2), we
Page 458
WAVE OPTICS
get Amin
2 2
A A A 2 A1 A2 cos(1 2 )
1 2 (iv) if A1 A 2 A 0 (iv) if A1 = A2 = A0
A = 2A0 A= 0
A12 A22 2 A1 A2 cos .....(3)
(v) I = ( I1 + I 2 )2 (v) I = ( I1 - I 2 )2
Where 1 2 , phase difference between
(cos 1) (cos 1)
the two waves.
Dividing equation (2) by equation (1), we get (vi) if I1 = I 2 = I0 (vi) if I1 = I2 = I0
Ires = 4I0 Ires = 0
A sin 1 A2 sin 2
tan 1 ........(4) B) SUPERSPOSITION OF INCOHERENT WAVES:
A1 cos 1 A2 cos 2
Incoherent waves are the waves which do not
Since the intensity of a wave is proportional to maintain a constant phase difference. The phase
square of the amplitude, the resultant intensity
I of the wave from equation (3) may be written of the waves fluctuates irregularly with time and
as independently of each other. In case of light
waves the phase fluctuates randomly at a rate
I I1 I 2 2 I1 I 2 cos .........(5) of about 108 per second. Light detectors such
as human eye, photographic film etc, cannot
where I1 and I 2 be the intensities of the two respond to such rapid changes. The detected
waves. intensity is always the average intensity,
It can be seen that the amplitude (intensity) of averaged over a time interval which is very much
the resultant displacement varies with phase larger than the time of fluctuations. Thus
difference of the constituent displacements.
CASE I : I av I1 I 2 2 I1 I 2 cos . The
When 1 2 0, 2 , 4 ...... 2n average value of the cos over a large time
where n 0,1, 2,.......... interval will be zero and hence I av I1 I 2
cos 1 This implies that the superposition of incoherent
A A1 A2 from (3) waves gives uniform illumination at every point
and is simply equal to the sum of the intensities
and I I1 I 2 from (5) of the component waves.
Hence the resultant amplitude is the sum of the INTERFERENCE:
two individual amplitudes. This condition refers The variation in intensity occurs due to the
to the constructive interference. redistribution of the total energy of the
Case II: interfering waves is called interference.
When 1 2 ,3 ,5 ..... (2n 1) Interference of light is a wave phenomenon.
where n 1, 2,3,....... ; cos 1 The source of light emitting wave of same
frequency and travelling with either same phase
A | A1 A2 | and I | I1 I 2 | or constant phase difference are called
Hence the resultant amplitude is the difference Coherent Sources.
of the individual amplitudes and is referred to Ex: Two virtual sources derived from a single
as destructive interference. source can be used as Coherent Sources.
Constructive Destructive The source producing the light wave travelling
Interference Interference with rapid and random phase changes are
(i) 0 or 2n (i) (2n 1) called Incoherent Sources.
where n=0,1,2.....where n=1,2,3..... Ex: 1. Light emitted by two candles.
(ii) x 0 or n 2. Light emitted by two lamps.
(ii) x (2n 1) /2 CONDITIONS FOR SUSTAINED INTERFERENCE
where n=0,1,2.....where n=1,2,3..... The two sources must be coherent.
(iii) A = A1 + A 2 =Amax (iii) A =| A1 - A 2 | = Two sources must be narrow.
Page 459
WAVE OPTICS
Two sources must be close together. maximum. It means it is the fringe with zero
path difference between two waves on reaching
Note:The two sources must be monochromatic, the point P.
otherwise the fringes of different colours The bright fringe corresponding to n = 1 is called
overlap and hence interference cannot be first order bright fringe i.e., if the path difference
observed. between the two waves on reaching ‘P’ is .
YOUNG’S DOUBLE SLIT EXPERIMENT Similarly second order bright fringe n 2 is
Young with his experiment measured the most located where the path difference is 2 and
important characteristic of the light wave i.e so on.
wavelength ( )
2
Young’s experiment conclusively established the From I 4 I 0 cos
2
wave nature of light.
For maximum intensity cos 1
2
i.e. 0, , , 2 ..........
2
(or) Phase difference between the waves
2 n with n = 0, 1, 2, 3 .........
P
The corresponding path difference, x n
4, x 2 2nd secondary max
sin(
t+k
x)
3, x 3 / 2 2nd minima
Hence I max 4 I 0 .
y1=A
)
S1 y
, x / 2 1st minima
t
Dark fringes occur whenever the waves from
(
sin
2
, x / 2 1st minima
2
y
d
x
0, x 0 Central maxima S1 and S 2 interfere destructively. i.e., on
, x / 2 1st minima
reaching ‘P’ one wave with its crest and another
s in
S2
wave with its trough superimpose. Then the
x
In a similar way, the same result will be obtained Note: If A1 A2 a then Amin 0
for the dark fringes also.
If I1 I 2 I 0 then I min 0
D
Fringe width, 2
d I max
I1 I 2 A A2
1
2
Page 462
WAVE OPTICS
x S2 P S1P D 2 d 2 D P ( S1P t ) t. The optical path from S 2
to P S 2 P.
1/2
d2 d2 d2 To get central zero fringe at P, s1 p s2 p
D 1 2 D D 1 2 D
D 2D 2D S1P t t S2 P
2
d S 2 P S1P ( 1)t
x .........(1)
2D Since 1, this implies S 2 P S1 P hence
But for missing wavelengths, intensity will be the fringe pattern must shift towards the beam
zero. i.e., the corresponding path difference,
from S1.
x (2n 1) ...... (2) y
2 But S 2 P S1P d sin d , where ‘y’ is
From equations (1) and (2) D
the lateral shift.
d2
(2n 1) y
2D 2 ( 1)t d ; Lateral shift
D
d2 D
Missing wavelength, (2n 1) D ( y ) ( 1)t ( 1)t
d
By putting n 1, 2,3,..., the wavelengths at P (or) Thickness of sheet
are yd y
2 2 2
t
d d d ( 1) D ( 1)
, , ,.......
D 3D 5 D From the above it is clear that
In the above case, if bright fringes are to be a) For a given colour, shift is independent of
formed exactly opposite to S1 then order of the fringe i.e. shift in zero order
maximum = shift in 9th minima (or) shift in 6th
d2 d2 maxima = shift in 2nd minima. Since the refractive
n index depends on wavelength hence lateral
2D 2 Dn shift is different for different colours.
By putting n = 1, 2, 3 ,,, the possible b) The number of fringes shifted =
wavelengths at P are
lateral shift
d2 d2 d2 fringe width
, , ,....
2 D 4 D 6D
XIV) LATERAL DISPLACEMENT OF FRINGES:
y ( 1)t
n (or) n ( 1)t
To determine the thickness of a given thin sheet
of transparent material such as glass or mica, Therefore, number of fringes shifted is more
that transparent sheet is introduced in the path for shorter wavelength.
of one of the two interfering beams. The fringe c) If a transparent sheet of thickness ‘t’ and
pattern gets displaced towards the beam in
whose path the sheet is introduced. This shift is its relative refractive index r (w.r.t.
known as lateral displacement or lateral shift. surroundings) be introduced in one of the
beams of interference, then
t
P ( r 1)tD
S1 1) the lateral shift y
y d
d O ( r 1)t
2) the number of fringes shifted n
S2
D d) Due to the presence of transparent sheet,
T the phase difference between the interfering
The optical path from S1 to waves at a given po int is given by
Page 463
WAVE OPTICS
2 P a b
( 1)t . N
e) If YDSE is performed with two different i S
colours of light of wavelengths 1 & 2 but by i
Q r
placing the same transparent sheet in the path
of one of the interfering waves then t r M
n11 n2 2 .
R T
where n1 and n2 are the number of fringes
shifted with wavelengths 1 & 2 .
f) When two different transparent sheets of d
Q1 C
thickness t1 , t2 and refractive index 1 , 2 where i = angle of incidence and
are placed in the paths of two interfering r = angle of refraction
waves in YDSE, if the central bright fringe
position is not shifted, then i) PQ is incident ray, At Q it splits as refrected ray
(Qa) and refracted ray (QR) by the division of
(1 1)t1 ( 2 1)t2 . amplitude. QR undergoes multiple reflections
XV) At constant temperature, with increase of inside the film and refract out as Sb, Rc and
pressure the refractive index of the medium Td.
increases and hence fringe width decreases ii) The rays Qa and Sb interface forming a reflected
1 system.
P density ; d , iii) The rays Rc and Td interfere forming a
V
transmitted system.
XVI) ANGULAR FRINGE WIDTH
iv) Path difference between the rays Qa and
i) The ratio of fringe width to distance between QRSb.
source and screen is defined as angular fringe
(P.D) = QRS in medium - QN in air
width ( )
( P.D ) 2t cos r this is the path lag
S1 1st bright Due to reflection on film additional path lag of
/ 2 exists. (stoke’s theorem)
S d
O
Total path difference = 2 t cos r
S2 Central bright 2
D
ii) Angular Fringe width is v) Condition for maximum 2 t cos r n
2
tan
D d or 2 t cos r 2n 1 for all values of n is
2
iii) Angular fringe width in a medium of refractive equal to 1, 2, 3 ........... n.
| | vi) Condition for Minimium
index is but |
d
2 t cos r (2n 1) ; 2 t cos r n
2 2
| for values of n=0, 1, 2, 3 .........
d
n=0 gives the central minima and it is applicable
= angular fringe width in air for any wavelength and this explains the central
FORMATION OF COLOURS IN THIN FILMS : dark spot in Newtons rings.
a) INTERFERENCE DUE TO REFLECTED LIGHT For normal incidnet i = o = r
2 t n for dark 2 t 2n 1 for bright.
2
Page 464
WAVE OPTICS
b) Transmitted system:
I max (9 A2 A2 )2 (10)2 25
i) Interference of two rays Rc and Td. By
symmetry it can be concluded that the path I min (9 A2 A2 ) 2 (8) 2 16
difference between the rays is 2 t cos r . But Illustration 7:
there would not be any extra phase lag because Two slits are made one millimeter apart
either of the two rays suffers reflection at denser and the screen is placed one meter away.
surface. When blue-green light of wavelength 500
nm is used, the fringe separation is
ii) Condition for maximum: 2 t cos r n
Solution :
iii) Condition for minimum :
D
Fringe separation,
2 t cos r 2n 1 d
2
On careful examination of the conditions for Given, D 1m, 500 nm 5 107 m and
maxima and minima in both the cases the
wavelengths in reflected system are found in d 1mm 1 103 m
transmitted system. 1 5 107
Fringe separation, m
II 1103
Interference in reflected Interference in refracted 5 104 m 0.5 mm
light light Illustration 8:
Condition of constructive In YDSE, the two slits are separated by
Condition of constructive
interference (maximum interference (maximum 0.1 mm and they are 0.5 m from the screen.
intensity) intensity) The wavelengh of light used is 5000 Å .
What is the distance between 7th maxima
x 2t cos r 2n 1 and 11th minima on the screen?
2 x 2t cos r 2n
For normal incidence r=0 2 Solution :
For normal incidence Here, d = 0.1 mm = 10-4 m,
So 2t 2n 1 2t n
2
D 0.5 m, 5000 Å 5.0 107 m
Condition of destructive Condition of destructive
interference (minimum interference (minimum x ( X 11 )dark ( X 7 )bright
intensity) intensity)
(2 11 1) D 7 D
x 2t cos r 2n x 2t cos r 2 n 1
2 2 2d d
For normal incidence For normal incidence
7D 75107 0.5
2t 2n 1 x
2t n 2 2d 2104
Illustration: 6 8.75 10 3 m 8.75 mm
Light waves from two coherent sources Illustration 9:
having intensity ratio 81 : 1 produce
interference. Then, the ratio of maxima In Young’s double slit experiment inter-
and minima in the interference pattern will ference fringes 10 apart are produced on
be the screen, the slit separation is
Solution : ( 589 nm)
Solution :
I1 A12 81
Given, 2 D
I 2 A2 1 The fringe width,
d
A1 9 The angular separation of the fringes is given
A 1 or A1 9 A2 .... (1) by
2
I max ( A1 A2 )2
I ( A A )2
D d
min 1 2
From Eq. (i), we get 0
Given, 1 rad
180
Page 465
WAVE OPTICS
589 nm Illustration 13:
In a double slit experiment the angular
589 180 109
d 0.0337 mm width of a fringe is found to be 0.20 on a
screen placed 1m away. The wavelength
Illustration 10: of light used in 600 nm. What will be the
In Young’s double slit experiment, the angular width of the fringe if the entire
wavelength of red light is 7800 Å and that experimental apparatus is immersed in
of blue light is 5200 Å. The value of n for water? Take refractive index of water to
which nth bright band due to red light co- be 4/3.
incides with (n + 1)th bright band due to Solution :
blue light, is Angular fringe separation,
Solution :
nR R D nB B D ord ; In water, d
or d
d d 1 3
nR B 5200 2 or
4
nB R 7800 3 n 2 3 3 0 0
Therefore 2 of red coincides with 3rd of blue.
nd or 0.2 0.15
4 4
Illustration 11: Illustration 14:
Young’s double slit experiment is made in In a Young’s experiment, one of the slits is
a liquid. The 10th bright fringe in liquid covered with a transparent sheet of thick-
lies where 6th dark fringe lies in vacuum.
ness 3.6 103 cm due to which position of
The refractive index of the liquid is ap-
proximately central fringe shifts to a position origi-
nally occupied by 30th fringe.If 6000
Solution :
Å, then find the refractive index of the
D
Fringe width
d
. When the apparatus is sheet.
Solution:
immersed in a liquid, and hence is reduced
(refractive index) times. The position of 30th bright fringe,
D D 30 D
y30 Now position shift of central
10 (5.5) or10 (5.5) d
d d
fringe is
10
or or 1.8 30 D
5.5 y0 ; But we know,,
Illustration 12: d
In Young’s double slit experiment, how D 30 D D
y0 ( 1)t ; ( 1)t
many maxima can be obtained on a screen d d d
(including the central maximum) on both
30 30 (6000 1010 )
sides of the central fringe if 2000 Å ( 1) 0.5
t (3.6 105 )
and d 7000 Å?
Solution : 1.5
For maximum intensity on the screen Illustration 15:
The maximum intensity in the case of n
n
d sin n or sin d ; identical incoherent waves each of inten-
W W
(n)(2000) n sity 2 2 is 32 2 the value of n is
m m
(7000) 3.5 Solution :
maximum value of sin 1 I = n I0, 32 = n 2, n = 16
Illustration 16:
n 3, 2, 1, 0,1, 2,3 ; 7 maxima.
Page 466
WAVE OPTICS
Compare the intensities of two points lo- (1.52 1.40)10.4
(1 2 )t n, m
n
cated at respective distance and from
4 3 (10.4 0.12) 1248
the central maxima in a interference of m nm
n n
YDSE ( is the fringe width)
When n=1, =1248 nm is not present in
Solution : the given range
2 2 d 2 d D 1248
x = D 4d When n=2, 624nm
D 4 2
2 1248
When n=3, 416nm
I 4 I 0 cos 2 3
4 2 4
Similarly 1248
When n=4, 312nm is not
4
2 2 present in the given range.
I 4I 0 cos2 I0
3 23 416nm and 324 nm will form maxima
at point ‘O’
required ratio = 2 :1
Illustration 19:
Illustration 17:
In Young’s double slit experiment the y co-
In Young’s double slit experiment inten- ordinates of central maxima and 10th
sity at a point is (1/4) of the maximum in- maxima are 2 cm and 5 cm respectively.
tensity. Angular position of this point is When the YDSE apparatus is immersed in
Solution : a liquid of refractive index 1.5 the corre-
sponding y co-ordinates will be
I
I I max cos 2 ; max I max cos 2 Solution :
2 4 2
Fringe width . Therefore, and hence
1 2 2 will decrease 1.5 times when immersed in
cos or , .x
2 2 2 3 3 liquid. The distance between central maxima
where x d sin and 10th maxima is 3 cm in vacuum. When
immersed in liquid it will reduce to 2 cm. Position
of central maxima will not change while 10th
d sin ,sin , sin 1 maxima will be obtained at y = 4 cm.
3 3d 3d
Illustration 18: Illustration 20:
YDSE is carried with two thin sheets of In YDSE, bi-chromatic light of wave-
thickness 10.4 m each and refractive lengths 400 nm and 560 nm are used. The
distance between the slits is 0.1 mm and
index 1 1.52 and 2 1.40 covering the the distance between the plane of the slits
slits s1 and s2 respectively. If white light and the screen is 1m. The minimum dis-
of range 400nm to 780 nm is used then tance between two successive regions of
which wavelength will form maximum complete darkness is:
exactly as Solution :
point ‘O’ the centre of the screen? Let nth minima of 400 nm coincides with mth
minima of 560nm, then
400 560
(2n 1) (2m 1) or
2 2
O 2n 1 7 14
.....
2m 1 5 10
i.e., 4th minima of 400 nm coincides with 3rd
Screen minima of 560 nm. Location of this minima is,
Solution :
Page 467
WAVE OPTICS
(2 4 1)(1000)(400 10 9 )
the plane of the slits and slits are
Y1 14 mm illuminated by plane monochromatic light
2 0.1 of wavelength . P is a point on the screen
Next 11th minima of 400 nm will coincide with at a distance y from the central maximum.
8th minima of 560 nm. If by some special arrangement the slits
Location of this minima is, be moved symmetrically apart with relative
velocity v, estiame the number of firnges
(2 11 1) (1000)(400 109 ) crossing tth point p per unit time.
Y2 42 mm
2 0.1 Solution:
Required distance = Y2 Y1 28 mm . Let l = distance bweteen the slits
Illustration 21: dl yl
V path diff x
An interf erence is observed due to two co- dt D
herent sources S1 placed at origin and S2 dx y dl yv
placed at (0,3 , 0) . Here is the wave- .
dt D dt D
length of the sources. A detector D is moved
along the positive x-axis. Find x-coordi- since a change in optical path difference of
nates on the x-axis (excluding x = 0 and corresponds to one fringe, so the number of
x ) where maximum intensity is ob- dx l yv
served. fringes crossing point P per unit is
dt D
Solution : Illustration 23: In figure shown
At x = 0, path difference is 3 . Hence, third
order maxima will be obtained. At x , path S1
difference is zero. Hence, zero order maxima
is obtained. In between first and second order S2 O
maxim will be obtained.
S3
S 1O S 2 O S 3 O S 2 O , Intensity
4
at O due to any one of the slits is I 0 . What is
the intensity due to all three coherent sources
S1 , S2 and S3 at O ?
First order maxima: Solution: Phase difference corresponding to the
given path difference
S 2 P S1 P (or ) x 2 9 2 x
A g 5A 0
2A0
or x 2 9 2 x Squaring both sides,
we get x 2 9 2 x 2 2 2 x . Solving
this, we get x 4 . Second order maxima: 0
90
S 2 P S1 P 2 ; (or) x 9 x 2 (or)
2 2
A0
2 2
x 2 9 2 ( x 2 ) Squaring both sides, . x
we get 4 2
x 2 9 2 x 2 4 2 4 x Let A0 be the amplitude
5 due to either of the slits
Solving, we get x 1.25
4 Ag 5 A0 , I R 5I 0 as I A2
Hence, the desired x coordinates are, Illustration 24:
x 1.25 and x 4 . Two coherent light sources A and B with
Illustration 22: separation 2 are placed on the x-axis
In YDSE the screen is at a distance D from symmetrically about the origin. They emit
Page 468
WAVE OPTICS
light of wavelength . Obtain the posi- 3
(2 ) 2 D 2 D
tions of maxima on a circle of large ra- 2
dius, lying in the x-y plane and with cen- 2
tre at the origin. 3
(2 ) 2 D 2 D
Solution : 2
9 2 3
4 2 D 2 D2 2 D
4 2
9 7 7
3 D 4 D
4 4 12
For the farthest minima, S1 P S2 P
2
For P to have maximum intensity, d cos n 4 2 D 2 D
2
n 2 15
2 cos n cos 2 where n is integer 42 D2 D2 D D 4 / 4
4 4
For n 0, 900 , 2700 Illustration 26:
A ray of light of intensity I is incident on
n 1, 600 ,1200 , 2400 ,3000 a parallel glass slab at a point A as shown.
n 2, 00 ,1800 It undergoes partial reflection and refrac-
tion. At each reflection 20% of incident
So, positions of maxima are at energy is reflected. The rays AB and A’ B’
00 ,600 ,900 ,1200 ,1800 ,2400 , 2700 a n d undergo interference. The ratio Imax / Imin
is
3000 ; i.e., 8 positions will be obtained. B
Short cut : In d n then number of maximum I B'
2
S1
D P
Solution :
xmin (2n 1) The farthest minima has
2
path difference / 2 while nearest minima
has path difference (3 / 2) . For the 16 I 0 81
nearest minima. I2 , I max ( I1 I 2 ) 2 I0 ,
125 125
3
S1 P S 2 P ; [as maximum path I I
2 I min ( I1 I 2 ) 2 0 , max 81 .
difference is 2 ] 125 I min
Page 469
WAVE OPTICS
Illustration 27: Solution :
In a YDSE experiment if a slab whose Phase difference
refractive index can be varied is placed in
front of one of the slits, then the variation 2 2
of resultant intensity at mid-point of (d sin ) d tan
screen with ' ' will be best represented
by ( 1). Y D small
[Assume slits of equal width and there is dy 2
no absorption by slab] i.e., y
D
The required graph is a straight line
I0 Illustration 29:
I0
The graph between the path difference
1
versus phase difference is a /an
1
1) Straight line 2) Parabola
3) Sine curve 4) None of these
I0
I0 Solution :
Since, a path difference of corresponds to
1 1 phase difference of 2
Solution :
x ( 1)t ; For 1, x 0 x x , hence graph is a
2
I = maximum = I0 ; As increases path straight line.
difference x also increases.; For x 0 to Illustration 30:
Which of the following graphs represent
, the
2
intensity will decrease from I0 to zero. variation of the path difference ( l )
between
Then for x to , intensity will increase the interferring waves in a double slit
2 experiment with the ‘angular position’ ( )
from zero to I 0 . of
Hence option 3 is correct the point on the screen?
Illustration 28:
Which of the following graphs best
represent the variation of phase difference
between the interfering waves in a double
slit experiment with the distance y from
the central maximum?
Phase Phase
difference difference
(a) (b)
Solution :
L d sin
Phase Phase
difference difference
L versus is a sine curve, with | | 2
c (d)
Page 470
WAVE OPTICS
Illustration 31: The point source of light S is having wave-
A Young’s double slit experimental set up length equal to . The light is reaching
is completely submerged in a transparent screen only after reflection. For point P to
liquid. Which of the following graphs best be 2nd maxima, the value of would be
represent the variation of total number of ( D d & d )
fringes N observed on the screen with the
index of refraction of the liquid?
N N
(1) (2)
N N
(4)
12d 2 6d 2 3d 2 24d 2
(3) 1) 2) 3) 4)
Solution : D D D D
Solution :
If L is size of the screen and is the size
(8d ) 3d
Ld a. At P, x ;
of a fringe, then N where ' D
'D
Ld 24d 2
N N M For 2nd maxima, x 2 ; 2
D D
S1
The graph is a straight line, with positive 2d
slope and passing through the origin. P
Illustration 32: S
The graph between the shift of the 3d
interference pattern in a double slit
experiment with the thickness t of a glass 6d
O
slab is best represented by Central
maxima
S2
s s 2
12d
D
t (1) t Illustration 34:
(2)
Two coherent point sources S1 and S 2 vi-
brating in phase emit light of wavelength
s s
. The separation between them is 2 as
shown in figure. The first bright fringe is
t t formed at ‘P’ due to interference on a
(3) (4) screen placed at distance ‘D’ from
Solution :
S1 ( D ), then OP is
Shift S= ( 1)t , a straight line with positive
slope and passing through the origin.
correct answer is (4)
Illustration 33:
Consider the optical system shown in fig.
Page 471
WAVE OPTICS
the fringes disappear. Till this happens, the
fringe separation remains fixed.
e) Same as in (d). As the source slit width
increases, fringes pattern gets less and less
sharp. When the source slit is so wide that the
condition S / d is not satisfied, the
interference pattern disappears.
f) The interference patterns due to different
component colours of white light overlap
(incoherently). The central bright fringes for
different colours are at the same position.
1) 3 D 2) 1.5 D 3) 2 D 4) 2 D Therefore, the central fringe is white. For
Solution :
appoint P for which S 2 P S1P b / 2 , where
1
x d cos ; cos d 2 2 b ( 4000 A0 ) represents the wavelength for
the blue colour, the blue component will be
x absent and the fringe will appear red in colour.
600 tan 60 x 3D Slightly farther away where
D
Experimental applications of YDSE: r
S 2Q S1Q b where r 8000 Ao is
a) When screen is moved away from the plane of 2
the slits; the wavelength for the red colour, the fringe
b) When monochromatic source is replaced by will be predominantly blue.
another monochromatic source of shorter Thus, the fringe closed on either side of the
wavelength; central white fringe is red and the farthest will
c) When separation between the two slits is appear blue. After a few fringes, no clear fringe
increased; pattern is seen.
d) When source slit is moved closer to the double-
slit plane; DIFFRACTION :
e) When width of the source slit is increased; Rectilinear propagation of light:
f) When monochromatic source is replaced by a
source of white light?
Observations:
d
a) Angular separation of the fringes remains
constant ( / d ). The actual separation of the
fringes increases in proportion to the distance
of the screen from the plane of the two slits d
b) The separation of the fringes (and also angular
separation) decreases. See, however the Diffraction
condition mentioned in (d) below.
c) The separation of the fringes (and also angular
separation) decreases. See, however, the
condition mentioned in (d) below. d
d) Let S be the size of the source and d is its
distance from the plane of the two slits. For
interference fringes to be seen, the condition
S< / d should be satisfied; otherwise d Spherical
interference patterns produce by different parts wave fronts
of the source overlap and no fringes are seen. The bending of light around edges of an obstacle
Thus, as S decreases (i.e., the source slit is or the encroachment of light within geometrical
brought closer), the interference pattern gets shadow is known as “diffraction of light”
less and less sharp, and when the source is Diffraction is a characteristic property of a
brought too close for this condition to be valid, wave.
Page 472
WAVE OPTICS
Diffraction is an effect exhibited by all electro-
magnetic waves, water waves and sound waves
Diffraction takes place with very small moving to P
P
particles such as atoms, neutrons and electrons
A
which show wave like properties.
When light passes through a narrow aperture M1
O to P P
some light is found to be enchroached into M2 L
shadow regions.
B a sin
When slit width is larger, the encroachment of
light is small and negligible.
When slit width is comparable to wavelength T
of light the encroachment of light is more i) Condition for minimum intensity:
If the size of obstacle or aperture is comparable Divide the slit into two equal halves AO the
with the wavelength of light, light deviates from and OB each of size a/2. The path difference
rectilinear propagation near edges of obstacle between the secondary waves from A and O
or aperture and enchroaches into geometrical will be / 2. For every point ( M 1 ) in the
shadow. upper half OA, there is a corresponding point
Diffraction phenomenon is classified into two (M 2 ) in the lower half (OB) and the path
types, a) Fresnel diffraction b) Fraunhofer
diffraction difference M 1 and M 2 at p1 is / 2.
FRESNEL DIFFRACTION
The source or screen or both are at finite First order
minima
distances from diffracting device (obstacle or a/2
a/2
aperture)
In Fresnel diffraction, the effect at any point on
the screen is due to exposed wave front which D
may be spherical or cylindrical in shape. Position of ?rst order minima
Page 473
WAVE OPTICS
the remaining one-third of the slit contributes light, the central maximum is white and the rest
to the intensity at a point between two minima. of the diffraction bands are coloured.
for first order maxima INTENSITY CURVE OF FRAUNHOFER’S
a a 3 DIFFRACTION
path difference .
3 3 2a 2
I0 Intensity
First order
Z1 maxima
a/3
Z2
a/3 Z3
D 3 2 O 2 3
Position of ? rst order maxima
Diffraction angle ()
Intensity of maxima in Fraunhofer’s direction
3 3 is determined by
(or ) a sin . similarly for second
2a 2 2
5 sin / 2
order maxuma a sin 2 . In general, for I I0 where I 0=intensity of
2 /2
higher order maxima, we can write central maxima.
2
a sin n 2n 1where n= 1, 2, 3, ..... a sin (Total phase difference
2
The intensity decreases as we go to successive between waves from the top and bottom
maxima away from the centre, on either side. portions of the slit)
The width of central maxima is twice as that of For minima and for maxima
secondary maxima.
Illustration 35:
A parallel beam of light of wavelength 500
nm falls on a narrow slit and the result-
ing diffraction pattern is observed on
y screen 1 m away. It is observed that the
O
P first minimum is at a distance of 2.5 mm
from the centre of the screen. Find the
D width of the slit.
Solution :
T y 2.5 103
For first minima a sin , radian
D 1
y D Now, a sin n
a ( sin tan ) y
D a Since is very small, therefore sin .
2 D n 1 500 109
Width of central maxima w 2 y or a m
a 2.5 103
Note: If lens is placed close to the slit, then D = f.
Hence ‘f’ be the focal length of lens, then width 2 104 m 0.2 mm
2f Illustration 36:
of the central maximum w . A screen is placed 50 cm from a single slit,
a which is illuminated with 6000 Å light, If
Note: If this experiment is performed in liquid other distance between the first and third
than air, width of diffraction maxima will minima in the diffraction pattern is 3.00
1 mm, what is the width of the slit?
decrease and becomes times. With white Solution :
Page 474
WAVE OPTICS
In case of diffraction at single slit, the position
of minima is given by a sin n . Where a (a = width of each slit); 10 2
is the aperture size and for small : d a
d 1
sin ( y / D ) a 0.2 mm
5 5
y D T HE VALIDITY OF RAY OPTICS:
a n , i.e., y (n )
D a The distance of the screen from the slit, so that
spreading of light due to diffraction from the
D D centre of screen is just equal to size of the slit,
So that, y3 y1 (3 ) (2 ) and is called Fresnel distance. It is denoted by ZF.
a a
hence, The diffraction pattern of a slit consists of
0.50 (2 6 107 ) secondary maximum and minima on the two
a 2 104 m sides of the central maximum. Therefore, one
3 103 can say that on diffraction from a slit, light
0.2 mm spreads on the screen in the form of central
maximum. The angular position of first
Illustration 37: secondary minimum is called half angular width
In a single slit diffraction experiment first of the central maximum and it is given by
minimum for 1 660 nm coincides with
first maxima for wavelength 2 . Calculate (provided is small)
a
2 . If the screen is placed at a distance D from the
slit, then the linear spread of the central
Solution : maximum is given by
Position of minima in diffraction pattern is given
by; a sin n D
y D
a
For first minima of 1 , we have
It is, in fact, the distance of first secondary
1 minimum from the centre of the screen. It
a sin 1 (1)1 or sin 1 .....(i) follows that as the screen is moved away (D is
a increased), the linear size of the central
The first maxima approximately lies between
maximum i.e., spread distance, when D Z F ,
first and second minima. For wavelength 2
its position will be y = a (size of the slit) Setting this condition in
the above equation, we have
3 3
a sin 2 2 sin 2 2 ...... (ii) ZF a2
2 2a a or Z F
The two will coincide if, a
It follows that if screen is placed at a distance
1 2 or sin 1 sin 2
beyond Z F , the spreading of light due to
1 32 diffraction will be quite large as compared to
or the size of the slit. The above equation shows
a 2a
that the ray -optics is valid in the limit of
2 2 wavelength tending to zero.
2 1 660 nm 440 nm
3 3 Illustration 39:
Illustration 38: For what distance is ray optics a good ap-
Two slits are made one millimeter apart proximation when the aperture is 3 mm
and the screen is placed one meter away. wide and the wavelength is 500 nm?
What should the width of each slit be to Solution :
obtain 10 maxima of the double slit pat-
tern within the central maximum of the For distance Z Z F ,
single slit pattern. ray optics is the good appropriate
Fresnel distance
Solution : We have a (or)
a
Page 475
WAVE OPTICS
a 2 (3 103 )2 2 tan
ZF 18 m (b) Resolving power (RP) 1.22
5 107
RESOLVING POWER OF OPTICAL where 2 is the angle subtended by the
INSTRUMENTS diameter of the objective lens at the focus of
the microscope and is the refractive index
i) The ability of optical instrument to produce of the medium between the object and the
distinctly seprate images of two objects located objective lens.
very close to each other is called its resolving
power. When a parallel beam of light falls on a convex
lens, the image is a diffraction pattern, consisting
ii) Resolving power is normally defined as the of central bright region surrounded by
reciprocal of the smallest value (resolving limit) concentric dark and bright rings.
subtended or separated w.r.t to the objective
of optical instrument by two point objects, Note: The radius of the central bright region is
which can be distinguished as separate. 1.22 f 0.61 f
given by r0 Rayleigh’ss
iii) To a human eye, the diameter of iris is about 2a a
2mm the resolving limit for green light of criterion. . Here, f is the focal length of the
wavelength 5000 Ao is 1.22 / a lens and 2a is the diameter of the circular
aperture or the diameter of lens.
3 104 rad (or )1min . of arc.
O 2a 0.61f
a
f
Illustration 40:
d Eye
What is the approximate radius of the cen-
tral bright diffraction spot of light of wave-
length 0.5 m , if focal length of the
O lens is 20 cm and radius of aperture of the
D lens is 5 cm ?
Solution :
(A) Resolving power of telescope: 0.61 f
(a) Resolving power of telescope is defined as r0
the reciprocal of the smallest angular seperation a
(resolving limit) between two distant objects putting values,
whose images are just separated w.r.t to the 0.61 0.5 106 20 102
objective lens. r0
5 102
(b) Resolving power limit ( ) is
d 1.22 1.22 10 6 m
where a is aperture of lens
D a 1.22 m
(c) Resolving power of telescope Note 2: The argument we applied for telescope
1 a can be applied for objective lens of a
microscope. In the case, the object is placed
1.22
(d) The resolving power of telescope increases slightly beyond f, so that a real images is formed
with increase in thediameter of the objective v v
but the effect of spherical aberration becomes at a distance v. the magnification m
u f
appreciable. Hence in case of large telescope Image
objectives, the central position of the objective
is covered with a stop to minimize the spherical D 1.22f
aberration this will not effect on resolving power f D
Object
B) Resolving power of microscope: Object v
plane
(a) Resolving power of microscope is defined as Objective Image plane
the reciprocal of the smallest distance (resolving lens
limit) between two closed objects whose images From ABC , we can show,,
are just separated w.r.t to the objective lens.
Page 476
WAVE OPTICS
D is used. Such an arrangement is called “Oil
D
AC 2 2 tan immersion objective”.
tan f
BC F 1
Resolving power
where 2 is the angle subtended by the
diameter of the objective lens at focus of the Illustration 41:
microscope. Which light would produce more resolu-
The diffraction effect becomes important when tion the red light or the blue one ?
the separation between two points in a Solution :
microscope specimen is comparable to the Resolving power is larger when blue light is
wavelength of the light used to observe the used instead of the red light.
object. The image of a point object will again Illustration 42:
be a diffraction pattern whose size in the image
Assume that light of wavelength 6000 Å
v1.22 is coming from a star. What is the limit of
plane will be v
D resolution of a telescope whose objective
Two objects whose images are closer than this has a diameter of 100 inch?
distance will not be resolved, and consequently Solution :
they will be seem as one. The corresponding A 100 inch telescope implies that
minimum separation in the objects plane will a = 100 inch = 254 cm. Thus if,
be
v v 1.22
6000 Å 6 105 cm then,
d min 1.22
m Dm 2.9 107 radians
a
1.22 v 1.22 f
Illustration 43:
D m D Light of wavelength 589 nm is used to view
D an object under a m icroscope. The
Also, we had 2 tan aperature of the objective has a diameter
f of 0.900 cm. Find
1.22 0.61 (a) The limiting angle of resolution
d min
2 tan tan (b) What is the maximum limit of resolution for this
microscope using visible light of any wavelength
0.61 you desire
(For small , sin = tan )
sin (c) What effect would this have on the resolving
if the medium between the object and the power. If water ( 1.33) fill the space
objective lens is not air but a medium of between the object and objective.
refractive index , Solution :
0.61 (a) Here; 589nm 589 109 m
then d min
sin
a 0.900cm 0.900 10 2 m
The product sin is called “numerical ( O r )
aperture” and is often marked on specification
of the objective. The resolving power of the 589 109 m
1.22 1.22 rad
minimum separation of two points seen as a
2
0.900 10 m
distinguished .
1 sin 7.89 105 rad
Hence, Resolving power d 0.61 (b) To obtain the smallest angle corresponding to
min the maximum limit of resolution, we have to use
Now for a given wavelength, Resolving power the shortest wavelength ( 400nm) in the
is directly proportional to refractive index visible spectrum
Hence, resolving power can be increased by Limiting angle of resolution
choosing a medium of higher refractive index.
Usually an oil having a refractive index close
to that of the glass used to make objective lens
Page 477
WAVE OPTICS
400 109 m 5
1.22 2 rad 5.42 10 rad (4)
0.900 10 m
(c) Wavelength of light in water, Partially polarized light
a 589nm
w 443nm (5)
1.33
443 10 9 m 5
1.22 2 rad 6.00 10 rad Unpolarized light
0.900 10 m
Since in this case is less than that in case
(a), resolving power increases.
unpolarised light Polarised
POLARIZATION : light
Polarizer Analyzer
The properties of light, like interference and METHODS OF OBTAINING POLARISED LIGHT:
diffraction demonstrate the wave nature of light.
i) By Scattering
Both longitudinal and transverse waves can
exhibit interference and diffraction effects. ii) By reflection
The properties like polarization can be exhibited iii) By refraction
only by transverse waves. iv) By double refraction (or) Birefringence
The peculiar feature of polarized light is that iv) By selective absorption (dichroic crystals)
human eye cannot distinguish between polarised a) Polarisation by Scattering:
and unpolarised light. Z
As light is an electromagnetic wave, among its
electric and magnetic vectors only electric
vector is mainly responsible for optical effects. POLARISED
The electric vector of wave can be identified LIGHT
as a “light vector”
Ordinary light is unpolarised light in which X
O
electric vector is oriented randomly in all POLARISED
directions perpendicular to the direction of LIGHT UNPOLARISED
propagation of light. LIGHT
The phenomena of confining the vibrations of Y
electric vector to a particular direction When a beam of light passes through a medium,
perpendicular to the direction of propagation it gets scattered from the particles constituting
of light is called “Polarization”. Such polarised the medium, provided the size of the particles
light is called linearly polarised or plane is of the order of the wavelength of the light.
polarised light. The scattered light viewed in a direction
The plane in which vibrations are present is perpendicular to the direction of the beam of
called “plane of polarization.” light is found to be plane polarised.
b) Polarisation by reflection:
(1)
When the angle fo incidence of light is gradually
changed, at a particualr angle of incidence, the
(2) reflected light is completely polarised.
This angle of incidence is known as polarising
angle
Polarized light
( P ) or Brewster’s angle. If i P
(3) i) Reflected light is plane polarised
ii) Refracted light is partially polarised
Partially polarized light iii) The vibrations parallel to the reflecting surface
are present in the reflected light (dot component
of the light)
Page 478
WAVE OPTICS
iv) The vibrations perpendcular to the reflecting
surface (arrow component) are present in the incidence p . p 600
refracted light. A few dot components are also Using Brewster’s law
present in the refracted lihgt.
v) Angle between reflected and refracted light is tan p tan 600 3
90o. b) From Snell’s law
vi) If i P or i P both reflected and refracted sin i sin 600
3
light will be partially polarised. sin r sin r
Ordinary
light i = p Plane 3 1 1
polarized light or sin r , r 300 .
2 3 2
i
c) Polarisation by refraction:
air
glass
Partially
polarized light 32.5
Brewster’s Law:
The tangent of polarising angle is equal In this the principle is multiple refraction. A pile
to the refractive index of material of the of plates is used. Usually 15 to 20 microscope
reflecting surface. tan P cover glasses are used as a pile. They are
arranged in a brass tube making an angle of
P 57.5o 57o301 for glass air media
32.5o with the axis of the tube. Now the light
i) P tan 1 ( )
ray travelling parallel to the axis of the tube
1 incidents at polarising angle P 57.5o on the
ii) where C= critical angle
sin C plates.
Refracted ray from a plate is partically polarised.
iii) Tan P 1 ; sin P ;
If the ray undergoes multiple refractions then
2 1
the emergent ray is completely plane polarised
containing arrow components.
2 1 D) Double refraction (Birefringence):
1
cos P
2 1 PPL E
P
1
O
iv) Polarising angle depends on nature of L i
UP r1
r2
PL E O
material and colour of light. P
Illustration 44:
When light of a certain wavelength is in- i) Bertholinus discovered double refraction.
cident on a plane surface of a material at ii) When a light ray is refracted through a calcite
a glancing angle 300, the reflected light crystal, two refracted rays are observed, one
is found to be completely plane polarized is ordinary-ray (O-ray) and the other one is
determine. extraordinary ray (E-ray). This phenomenon is
a) refractive index of given material and known as double refraction.
b) angle of refraction. iii) Both O-ray and E-ray are plane polarised in
Solution : perpendicular planes.
a) Angle of incident light with the surface is 300. iv) If ‘i’ is the angle of incidence and r1 , r2 be the
The angle of incidence = 900 - 300 = 600. Since angles of refraction of O-ray and E-ray
reflected light is completely polarized, therefore respectively.
incidence takes place at polarizing angle of
Page 479
WAVE OPTICS
sin i sin i I0
0 & e I
sin r1 sin r2 2
where o = refractive index of O-ray Thus, if unpolarized light of intensity I 0 is
incident on a polarizer, the intensity of light
e = refractive index of E-ray.. I
E) Dichroism: transmitted through the polarizer is 0 . The
2
i) It is the property of unequal absorption of A0
ordinary and extraordinary rays by some amplitude of polarized light is .
crystals. 2
ii) When unpolarised light strikes the tourmaline EFFECT OF ANALYSER ON PLANE POLARIZED
crystal of certain thickness at right angles to LIGHT:
the optic axis, the O-ray is completely absorbed Malus states that “the intensity of the polarized
and only the E-ray emerges from the crystal. light transmitted through the analyser is
This property is called Dichroism. proportional to cosine square of the angle
POLARIOD: between the plane of transmission of analyser
Polaroid is an optical device used to produce and the plane of transmission of polarizer.” This
plane polarised light making use of the is known as Malus law.
phenomenon of “selective absorption”.
More recent type of polaroids are H-polaroids.
H-polaroids are prepared by stretching a film
of polyvinyl alcohol three to eight times to A0
original length.
EFFECT OF POLARIZER ON NATURAL LIGHT:
If one of waves of an unpolarized light of
intensity I 0 is incident on a polaroid and its Therefore the intensity of polarized light after
vibration amplitude A0 makes an angle with I 2
passing through analyser is I= 0 cos
the transmission axis, then the component of 2
vibration parallel to transmission axis will be Where I 0 is the intensity of unpolarized light.
A0 cos while perpendicular to it A0 sin . The amplitude of polarized light after passing
Now as polaroid will pass only those vibrations A
which are parallel to its transmission axis, the through analyser is A 0 cos .
2
intensity I of emergent light wave will be
Transmission axis I
Case (i) : If 00 axes are parallel then I 0
A0cos 2
0
Case (ii): If 90 axes are perpendicular, then
I 0.
Case (iii):If 1800 axes are parallel then
A0sin I
I 0
2
Case (iv): If 2700 axes are perpendicular
then I = 0 Thus for linearly polarized light we
I KA02 cos 2 (or) obtain two positions of maximum intensity and
two positions of minimum (zero) intensity, when
I I 0 cos2 [as I 0 KA02 ] In unpolarized light, we rotate the axis of analyser w.r.t polarizer by
all values of starting from 0 to 2 are equally an angle 2 . In the above cases if the polariser
probable, therefore is rotated with respect to analyser then there is
I 2 I no change in the outcoming intensity.
I I 0 cos2 I 0 cos2 d 0 Case (v) : In case of three polarizers P1, P2 and
2 0 2
Page 480
WAVE OPTICS
P3: If 1 is the angle between transmission Illustration 46:
Two polaroids are oriented with their
axes of P1 and P2, 2 is the angle between transmission axes making an angle of 30°
transmission axes of P2 and P3. Then the with each other. What fraction of incident
intensity of emerging light from P3 is un polarized light is transmitted?
I0 Solution :
I cos 2 1 cos 2 2 .
2 If unpolarized light is passed through a polariod
Case (vi): ‘n’ polarisers are arranged so that the P1 its intensity will become half.
first and the last ones are crossed, is the 1
angle between any two successive polarisers, So I1 I 0 with vibrations parallel to the axis
I is the intensity of incident light then intensity 2
of emerging lihgt is of P1 . Now this light will pass through the
I second Polaroid P2 (Analyser) whose axis is
I 1 cos x , where x=2(n-1)
2 inclined at an angle of 30° to the axis of P1 and
OPTICAL ACTIVITY: hence vibrations of I1 . So in accordance with
i) When plane polarised light passes through Malus law, the intensity of light emerging from
certan substances, the plane of polarisation of
light is rotated about the direction of P2 will be
propagation of light through a certain angle. This 2
phenomenon is called optical activity. 2 1 3 3
I1 I 2 cos 30 I 0 I0
ii) If the optically active substances rotate the plane 2 2 8
of polarisation clockwise then it is said to be
I2 3
Dextro-rotatory or right handed. So the fractional transmitted light I 8
iii) If the subsance rotates the plane of polarisation 0
anti-clockwise then it is called Laevo-rotatory Illustration 47:
or left handed. Unpolarized light of intensity 32 Wm -2
iv) Optical activity of subtance is measured with passes through three polarizers such that
the help of polarimeter. the transmission axis of the last polarizer
Illustration 45: is crossed with the first. If the intensity of
Unpolarized light falls on two polarizing the emerging light is 3 Wm -2, what is the
sheets placed one on top of the other. What angle between the transmission axes of the
must be the angle between the character- first two polarizers? At what angle will the
istic directions of the sheets if the inten- transmitted intensity be maximum?
sity of the tramsitted light is one third of Solution :
intensity of the incident beam?
If is the angle between the transmission axes
Solution :
of first polaroid P1 and second P2 while
Intensity of the light transmitted through the first between the transmission axes of second
polarizer I1 I 0 / 2, where I0 is the intensity polaroid P2 and third P3, then according to given
of the incident unpolarized light. Intensity of the problem.
light transmitted through the second polarizer
900 or (900 ).....(1)
is I 2 I1 cos 2 where is the angle between
the characteristic directions of the polarizer Now if I 0 is the intensity of unpolarized light
sheets. incident on polaroid P1 , the intensity of light
But I 2 I 0 / 3 (given) transmitted through it,
I0 I 1 1 W
I 2 I1 cos 2 cos 2 0 I1 I 0 (32) 16 2 ......(2)
2 3 2 2 m
Now as angle between transmission axes of
2 polaroids P1 and P2 is , in a accordance with
cos 2 2 / 3 cos 1
3 Malus law, intensity of light transmitted through
P2 will be
Page 481
WAVE OPTICS
Page 482
Page 483
ELECTROMAGNETIC WAVES
ELECTROMAGNETIC WAVES
INTRODUCTION wires which can be detected using a magnetic
According to Maxwell, an accelerated charge compass needle. Consider two loops l1 and l2
produces a sinusoidal time-varying magnetic field, parallel to the plates P and Q of the capacitor.
which in turn produces a sinusoidal time varying l1 is enclosing only the connecting wire attached
electric field. The two fields so produced are to the plate P of the capacitor and l2 lies in the
mutually perpendicular. They constitute electro region between the two plates of capacitor. For
magnetic waves which can propagate through the loop l1 , a current I is flowing through it, hence
empty space. Ampere’s circuital law for loop l1 gives
CONDUCTION CURRENT B.d l 0 I ....ii
Conduction current arises due to the flow of C 1
electrons through the connecting wires of a Since the loop l2 lies in the region between the
circuit, in a given closed path. plates of the capacitor, no current flows in this
When a capacitor is connected to a cell, it region. Hence Ampere’s circuital law for loop
starts storing the charge. During this period of
B.d l 0 ....iii
storage, there is conduction current. When the l2 gives C
2
capacitor becomes fully charged, the conduc-
The relations (ii) and (iii) continue to be true
tion current becomes zero in the circuit.
Conduction current exists when there is a flow even if two loops l1 and l2 are infinitesimally
of electrons at uniform rate in the connecting close to the plate P of the capacitor. In the
wires. other hand, as the loops l1 and l2 are
Displacement current: - According to infinitesimally close, it is expected that
Ampere’s circuital Law, the magnetic field B is
related to steady current I as B .d l .d l 0 I ...iv
B
l1 l2
B . dl 0 I ....(i) where I is the current
Thus, relation (iv) is in contradiction with
travelling through the surface bounded by closed
relations (ii) and (iii). This led Maxwell to point
loop.
out that Ampere’s circuital law as given by (i) is
In 1864, Maxwell showed that relation (i) is logically inconsistent.
logically inconsistent. He accounted for this Idea of Displacement Current : Maxwell
inconsistency as follows: Consider a parallel plate predicted that not only a current flowing in a
capacitor having plates P and Q being charged conductor produces magnetic field but also a
with battery B. time-varying electric field in a vacuum/free space
(or in a dielectric) produces a magnetic field. It
means a changing electric field gives rise to a
l1 l2 current which flows through a region so long as
the electric field is changing there. Maxwell also
predicted that this current produces the same
magnetic field as a conduction current can
produce. This current is known as ‘displacement
current’.
Thus, displacement current is that current which
comes into play in the region in which the electric
During charging, a current I flows through the
connecting wires which changes with time. This field and hence the electric flux is changing with
current will produce magnetic field around the time.
Page 484
ELECTROMAGNETIC WAVES
Maxwell defined this displacement current in From equations ( iii) and ( iv) I d I
space where electric field is changing with time
This idea led Maxwell to modify Ampere’s
dE circuital law in order to make the same logically
as I D 0 ... v consistent. He states Ampere circuital law in
dt
where E is the electric flux. the form, B.dl 0 I I D
Maxwell also found that conduction current I C
Page 485
ELECTROMAGNETIC WAVES
This law states that magnetic field can be The conduction current at the end of one time
produced by a conduction current as well as constant can be obtained by substituting t in
by displacement current. the expression i i0e t /
At any instant in a circuit, conduction current
is equal to displacement current. i i0 e t /t
SOURCES OF ELECTROMAGNETIC R R/4
i i
WAVES:
Electromagnetic waves are produced in the
following physical phenomena. +Q -Q
a) An electric charge at rest has an electric field
in the region around it but no magnetic field. i0 '
as i i .(say) ,
When a charge moves, it produces both elec- e
tric and magnetic fields. If the charge moves Where e is the base of natural logarithm.
with a constant velocity( in other words, if the If Q be the charge at the mentioned instant then,
current is not changing with time), the magnetic the electric field between the plates is
field will not change with time. As such it can- Q
E
not produce an electromagnetic wave. But if 0 0 (R)2
the charge is accelerated, both the magnetic
The electric flux through the specified area is
and eletric fields will change with space and Rate of change of electric flux is
time and an electromagnetic wave is produced.
d E 1 dQ 1(i ') i
Thus an accelerated charge emits an electro- 0
magnetic wave. dt 160 dt 160 16e0
b) An oscillating charge (as in the case of LC Illustration 2:
oscillations) emits an electromagnetic wave A parallel plate capacitor consists of two
circular plates of radius R = 0.1 m. They
which has the same frequency as that of the are separated by a distance d = 0.5 mm. if
oscillating charge. electric field between the capacitor plates
c) An electron while orbiting around its nucleus changes as
in a stable orbit does not emit an electromag- dE V
5 1013 . Find displacement
netic wave even though it has acceleration. It dt m s
will emit an electromagnetic wave ( -ray) only current between the plates.
Solution:
when it falls from an orbit of higher energy to
one of lower energy. Area of plates A r 2 3.14 (0.1) 2 m 2
d) Electromagnetic waves (i.e., X-rays) are dE V
5 1013
produced when fast moving electrons hit a tar- dt m s
get of high atomic number. dE dE dE
Id 0 A A
Illustration 1: dt dt dt
A circular parallel plate capacitor with 2
8.85 10 12 3.14 0.1 5 1013
plate radius R is charged by means of a
cell, at time t=0. The initial conduction I d 13.9 A
current is i0 . Consider a circular area of Illustration 3:
radius R / 4 coplanar with the capacitor What is the instantaneous displacement current
plates and located symmetrically between in space between plates of parallel plate
them. Find the time rate of electric flux capacitor of capacitor 1 F which is charging
change through this area after one time at rate of 1 0 6 V / S
constant, given i i0e t/ Solution:
Solution:
Page 487
ELECTROMAGNETIC WAVES
E
C0
r r n,
C med
(Refractive Index of medium)
o Wave Propagation
X 8) In vaccum, EM waves are of different
B wavelengths, but velocity is same.
Illustration 4: An electomagentic wave travels
Z along z-axis which of the following pairs
of space and time varying field would
Characteristics of E M waves : generate such a wave
1) E x , B y 2) E y , Bx 3) Ez , Bx
1) E and B are such that E B is always in the
direction of propagation of wave. 4) E y , Bz
2) EM waves are transverse in nature whose speed Solution: (1)
is same as that of speed of light
E x and By would generate a plane EM wave
3) The two fields E and B have same frequency
of oscillation and they are in phase with each travelling to z-direction. E , B and k from a right
other.
handed system k is along z-axis .
Page 488
ELECTROMAGNETIC WAVES
As iˆ ˆj kˆ Illustration 6:
Suppose that the electric field amplitude
E xiˆ By ˆj Ckˆ of an EM wave is E0 120 N / C and that
i.e E is along x-axis and B is along y-axis its frequency 50 MH Z . Determine
DIRECTION OF POLARIZATION OF
(a) B0 , , and K
ELECTROMAGNETIC WAVE :
In an electromagnetic wave electrical and (b) Find expressions for E and B
magnetic fields are perpendicular to each other. Solution:
Wave propagates in a direction perpendicular E0
i) C
to both electric and magnetic filed as given by
B0 we get
EB . E0 120
Direction of polarization is perpendicular to B0 8
4 107 T = 400nT
C 3 10
direction of oscillation.
Optical phenomena are mainly due to electric 2 v 2 50 106
field. ii)
3.14 108 rad / Sec
In the polarization of light, it is the electrical
field that gets polarized. C 3 108
iii) c v 6m
Therefore direction of polarization must be v 50 106
perpendicular to electric field and parallel to 2 2 2 3.14
iv) K 1.05m 1
magnetic field. 6 6
The direction of polarization is given by x and b) E E0 sin kx t
that of wave propagation by k .
= 120 sin 1.05 x 3.14 108 t
Here, The direction of polarization = x . This
must be parallel to electric field and B B0 sin kx t
perpendicular to magnetic field. X || E 400 10 9 sin 1.05 x 3.14 108 t
Illustration7:
The directionof wave propagation k . This
must be perpendicular to electric and magnetic In a plane electromagnetic wave, the
electric field oscillates sinusoidally at a
field.
frequency of 2 1010 Hz and amplitude 48
k EB V/m. Find the amplitude of oscillating
Illustration 5: magnetic field.
Electro magnetic waves travel in a Solution:
medium with speed of 2 108 m / sec . The E 48
B 16 108Wb / m 2
relative permeability of the medium is 1. c 3 108
Find relative permittivity of the medium. Illustration 8:
Solution: In the above problem, the wavelength of
Give C 2 108 m / sec, r 1 electromagnetic wave is
Speed of EM waves in medium Solution:
wave length
1 C0
Cmed c 3 108
r 0 r 0 r r 1.5 102 1.5cm
v 2 1010
8 2 Illustration 9:
c2
0
3 10 2.25 o
r 8 2
c2r
2 10 1 If the wavelength of light is 4000 A ,
then the number of waves in 1mm length
will be.
Page 489
ELECTROMAGNETIC WAVES
Solution: 1 B2
Number of waves dV is , u 0 E 2 dV dV
2 20
103 Energy density of EM wave is
0.25 104
4000 10 10 1 B2
u 2
PROPERTIES OF EM WAVES U U 2 0 E 2
1. Velocity: The velocity of electromagnetic dV 0
Thus u aV 0 E 0
1 E 4 40
c 0 3 108 m / s
0 0 B0 1
Now E 0 CB 0 and 0 0 2
C
1 1 1 1 1
In isot ropic medium v uE 0 E 02 0 C 2 B 02 0 B 02
; where 4 4 4 0 0
Page 490
ELECTROMAGNETIC WAVES
Page 491
ELECTROMAGNETIC WAVES
Frequency (Hz)
x
Wavelength
E y 66 cos 2 1011 t
c
x Gamma-rays 0.1 A
Bz 2.2 107 cos 2 1011 t 10
19
c 1A
0.1 nm
1018
Illustration 17: X-rays
1 nm
400 nm
Solution: 12
Thermal IR 100 m
700 nm
10
8 For IR 1000 m
6 10 1 1000 MHz 1 mm
6 108 ; k v 3 108 2m UHF
1011
1 cm
300 MHz 10
10
Radar
Illustration 18: 10 cm
9
Page 493
ELECTROMAGNETIC WAVES
Micro 0.1m to 1mm 109 to 1011 Hz Klystron valve or magnetron valve Point contact diodes
wave
Infrared 1mm to 700nm 1011 to 1014 Hz Vibration of atoms and molecules Thermopiles bolometer,
wave infrared photographic film
Visible rays 700 nm to 41014 to Electrons in atoms emit light when The eyes photo cells,
400 nm 81014 Hz they move from higher energy level to photographic film
a lower energy level
Ultraviolet 400 nm to 14 17
10 to 10 Hz Inner shell electrons in atoms moving Photo cells,
rays 1 nm
from higher energy level to a lower photographic film
energy level
X-rays 1nm to 16 21
Photographic film,
-3
10 nm 10 to 10 Hz X-ray tubes or inner shell electrons Geiger tubes, ionisation
chamber
Photographic film,
Gamma
-3
<10 nm 18 22 Radioactive decay of the nucleus Geiger tubes, ionisation
10 to 10 Hz
rays chamber
TYPICAL USES OF ELECTROMAGNETIC SPECTRUM waves are thermal and give off heat. Anything
1. Radio Waves that gives off heat radiates infrared waves. This
Radio waves have the longest wavelengths of all includes the human body.
the electromagnetic waves. They range from 4. Visible light
around a foot long to several miles long. Radio The visible light spectrum covers the wavelengths
waves are often used to transmit data and have that can be seen by the human eye. This is the
been used for all sorts of applications including range of wavelengths from 390 to 700 nm which
radio, satellites, radar, and computer networks. corresponds to the frequencies 430-790 THz.
2. Microwaves You can go here to learn more about the visible
Microwaves are shorter than radio waves with spectrum.
wavelengths measured in centimeters. We use 5. Ultraviolet
microwaves to cook food, transmit information, Ultraviolet waves have the next shortest wave-
and in radar that helps to predict the weather. length after visible light. It is ultraviolet rays from
Microwaves are useful in communication because the Sun that cause sunburns. We are protected
they can penetrate clouds, smoke, and light rain. from the Sun's ultraviolet rays by the ozone layer.
The universe is filled with cosmic microwave Some insects, such as bumblebees, can see ul-
background radiation that scientists believe are traviolet light. Ultraviolet light is used by power-
clues to the origin of the universe they call the ful telescopes like the Hubble Space Telescope
Big Bang. to see far away stars.
3. Infrared 6. X-rays
Between microwaves and visible light are infra- X-rays have even shorter wavelengths than ul-
red waves. Infrared waves are sometimes clas- traviolet rays. At this point in the electromag-
sified as "near" infrared and "far" infrared. Near netic spectrum, scientists begin to think of these
infrared waves are the waves that are closer to rays more as particles than waves. X-rays were
visible light in wavelength. These are the infrared discovered by German scientist Wilhelm Roent-
waves that are used in your TV remote to change gen. They can penetrate soft tissue like skin and
channels. Far infrared waves are further away muscle and are used to take X-ray pictures of
from visible light in wavelength. Far infrared bones in medicine.
Page 494
ELECTROMAGNETIC WAVES
7. Gamma rays constant CR .
As the wavelengths of electromagnetic waves
are shorter, their energy increases. Gamma o P OP=0.5
rays are the shortest waves in the spectrum
1M
and, as a result, have the most energy. Gamma
2V
rays are sometimes used in treating cancer and
in taking detailed images for diagnostic medi- Solution: Time costant CR
cine. Gamma rays are produced in high energy 109 106 103 s
nuclear explosions and supernovas. t
t
3
ACCORDING TO FFREQUENCY RANGE RADIO q CV 1 e 2 10 9 1 e10
WAVES ARE CLASSIFIED BELOW
1. Medium wave (frequency band) : The electric field in between the plates at time
t is
300 MHz to 3000 MHz.
2. Short wave (high Frequency band) : q q
E A 1 m2
3 MHz to 30 MHz A 0 0
3. V.H.F band (very high frequency) : Consider a circular loop of radius
30 MHz to 300 MHz 1
m parallel to the plates passing through P.
4. U.H.F. band (ultra high frequency) : 2
The magnetic field B at all points on the loop
300 MHz to 3000 KHz is along the loop and of the same value
5. Microwaves : 3000 MHz to 3 × 105 MHz 2
Illustration 19: 1 E q
Flu E EA ' E
In a plane E.M. wave the electric field 2 4 4 0
oscillates sinusoidally at a frequency dE 1 dq
3 × 1010 Hz and amplitude 50 Vm –1. displacement current id 0
dt 4 dt
(a) What is the wavelength of the wave t
(b) What is the amplitude of the oscillat- 1 d 9
3
10
id 2 10 1 e
ing magnetic field 4 dt at t = 10-3s
Solution:
0.5 106 e 1
Frequency 3 1010 H z, E 0 50V m 1 Applying Ampere-Maxwell law to the loop
(a)Wavelength,
B 2 r 0 ic id
c 3 108
10 2 m 1cm
3 1010 1
B 2 0 0 id 0 0.5 106 e 1
(b) Amplitude of oscillating magnetic field, 2
E0 50 B 0.5 106 e 10 ; 0.74 10 13 T
B0 1.67 10 7 T
c 3 108
Illustration 20: Illustration 21:
A parallel plate capacitor with circular A plane electromagnetic wave of fre-
plates of radius 1m has a capacitance of 1 quency 25MHz travels in free space along
nF. At t=0, it is connected for charging the x-direction. At a particular point in
inseries with a resistor R 1M across a space and time, E 6.3iV / m . What is
2V battery. Calculate the magnetic field at this point?
at a point P, halfway between the centre B
and the periphery of the plates after Solution:
t 103 s (the charge on the capacitor at The magnitude of B is
t
E 6.3V / m
time t is q CV 1 e where the time B 2.1108 T
C 3 108 m / s
Page 495
ELECTROMAGNETIC WAVES
Page 496